digest labor law case

127
Labor Law 1 A2010 - 186 - Disini SECTION 14: TERMINATION OF EMPLOYMENT A. GENERAL CONCEPT 14.01 SECURITY OF TENURE A. NATURE OF SECURITY OF TENURE SONZA V ABS-CBN BROADCASTING CORP [PAGE 42] ALHAMBRA INDUSTRIES V. NLRC (RUPISAN) 238 SCRA 232 BELLOSILLO; November 18, 1994 NATURE Special civil action in the Supreme Court. Certiorari FACTS - Alhambra employed Rupisan as salesman on 6-mo probationary basis. Alhambra made surprise audit, alleged violations were purportedly committed by him. He was placed under 1-mo preventive suspension. He protested. He alleges that charges against him had become academic when he was given clearance of all accountabilities. - A day before end of suspension, he was terminated. He sued Alhambra. - Labor Arbiter found that the termination was for just cause, but there was a violation of due process (failure to furnish copy of audit report). - Both parties appealed to NLRC which affirmed Arbiter’s findings. ISSUE WON NLRC committed grave abuse of discretion in sustaining finding of Labor Arbiter that Rupisan was illegally dismissed but directing his reinstatement so he could have explained HELD YES - Employment is no longer just an ordinary human acctivity. For most families the main source of their livelihood, employment has now leveled off with property rights which no one may be deprived of without due process of law. - Termination of employment is not anymore a mere cessation or severance of contractual relationship but an economic phenomenon affecting members of the family. This explains why under the broad principles of social justice the dismissal of employees is adequately protected by the laws of the state. - A termination without just cause entitles a worker to reinstatement regardless of whether he was accorded due process. On the other hand, termination of a worker for cause, even without procedural due process, does not warrant reinstatement, but the employer incurs liability for damages. - Since the Labor Arbiter found a valid ground for dismissal, it erred when it directed reinstatement. - To order reinstatement and compel the parties to start the procedure from step one would be circuitous because almost invariably that same issue of validity of the ground of dismissal would be brought back to the Labor Arbiter for adjudication. We laid down in Wenphil Corporation v. NLRC3 that an otherwise justly grounded termination without procedural due process would only sanction payment of damages - Standards of due process in judicial as well as administrative proceedings have long been established. In its bare minimum due process of law simply means giving notice and opportunity to be heard before judgment is rendered. - When the private respondent filed a complaint against petitioner, he was afforded the right to an investigation by the labor arbiter. - Although belatedly, private respondent was afforded due process before the labor arbiter wherein the just cause of his dismissal had been established. With such finding, it would be arbitrary and unfair to order his reinstatement with backwages. - It will be highly prejudicial to the interests of the employer to impose on him the services of an employee who has been shown to be guilty of the charges that warranted his dismissal from employment. Indeed, it will demoralize the rank and file. - However, the petitioner must nevertheless be held to account for failure to extend to private respondent his right to an investigation before causing his dismissal. MANILA ELECTRIC COMPANY V NLRC (LOMABAO, MASAYA) 186 SCRA 763 NARVASA; July 2, 1991 NATURE CERTIORARI FACTS - Jose Masaya made an unauthorized electric service connection which supplied electricity to the house of Antonio Sanchez (who paid the former Php 200 for making the said connection.) - Sanchez neither applied with Meralco for electric service nor made the requisite deposit for it.

Upload: john-carlo-patiag

Post on 22-Feb-2015

989 views

Category:

Documents


62 download

TRANSCRIPT

Labor Law 1 A2010 - 186 - DisiniSECTION 14: TERMINATION OF EMPLOYMENT

A. GENERAL CONCEPT

14.01 SECURITY OF TENURE

A. NATURE OF SECURITY OF TENURE

SONZA V ABS-CBN BROADCASTING CORP[PAGE 42]

ALHAMBRA INDUSTRIES V. NLRC (RUPISAN)238 SCRA 232

BELLOSILLO; November 18, 1994

NATURESpecial civil action in the Supreme Court. Certiorari

FACTS- Alhambra employed Rupisan as salesman on 6-mo probationary basis. Alhambra made surprise audit, alleged violations were purportedly committed by him. He was placed under 1-mo preventive suspension. He protested. He alleges that charges against him had become academic when he was given clearance of all accountabilities.- A day before end of suspension, he was terminated. He sued Alhambra.- Labor Arbiter found that the termination was for just cause, but there was a violation of due process (failure to furnish copy of audit report).- Both parties appealed to NLRC which affirmed Arbiter’s findings.

ISSUEWON NLRC committed grave abuse of discretion in sustaining finding of Labor Arbiter that Rupisan was illegally dismissed but directing his reinstatement so he could have explained

HELDYES- Employment is no longer just an ordinary human acctivity. For most families the main source of their livelihood, employment has now leveled off with property rights which no one may be deprived of without due process of law.- Termination of employment is not anymore a mere cessation or severance of contractual relationship but an economic phenomenon affecting members of the family. This explains why under the broad principles of social justice the dismissal of employees is adequately protected by the laws of the state.- A termination without just cause entitles a worker to reinstatement regardless of whether he was accorded due process. On the other hand, termination of a worker for cause, even without procedural due process, does not warrant reinstatement, but the employer incurs liability for damages.- Since the Labor Arbiter found a valid ground for dismissal, it erred when it directed reinstatement.- To order reinstatement and compel the parties to start the procedure from step one would be circuitous because almost invariably that same issue of validity of the ground of dismissal would be brought back to the Labor Arbiter for adjudication. We laid down in Wenphil Corporation v. NLRC3 that an otherwise justly grounded termination without procedural due process would only sanction payment of damages- Standards of due process in judicial as well as administrative proceedings have long been established. In its bare minimum due process of law simply means giving notice and opportunity to be heard before judgment is rendered.- When the private respondent filed a complaint against petitioner, he was afforded the right to an investigation by the labor arbiter.- Although belatedly, private respondent was afforded due process before the labor arbiter wherein the just cause of his dismissal had been established. With such finding, it would be arbitrary and unfair to order his reinstatement with backwages.- It will be highly prejudicial to the interests of the employer to impose on him the services of an employee who has been shown to be guilty of the charges that warranted his dismissal from employment. Indeed, it will demoralize the rank and file.- However, the petitioner must nevertheless be held to account for failure to extend to private respondent his right to an investigation before causing his dismissal.

MANILA ELECTRIC COMPANY V NLRC (LOMABAO, MASAYA)

186 SCRA 763

NARVASA; July 2, 1991

NATURECERTIORARI

FACTS- Jose Masaya made an unauthorized electric service connection which supplied electricity to the house of Antonio Sanchez (who paid the former Php 200 for making the said connection.)- Sanchez neither applied with Meralco for electric service nor made the requisite deposit for it.- This clandestine and illicit connection was eventually discovered by Meralco who then charged him (through a letter) with a violation of the Company Code on Employee Discipline, and thereafter conducted a formal investigation of the matter.- Those who gave testimony at that investigation were Jose Masaya himself, and Renato Repuyan, Meralco field investigator.- Prior to being interrogated about the illegal connection and in response to preliminary questions by the investigator, Masaya stated for the record that he had received the letter accusing him of misconduct, that he had a copy of the code of discipline and understood the nature of the precise charge against him, and that he did not need to be assisted by a lawyer or a representative of his Union because he said that what he was about to say was “pawing katotohanan lamang.”- Repuyan testified on the fact of the undenied and indisputable installation of the illegal electrical connection at the residence of Antonio Sanchez (his description of the manner of its accomplishment being substantially the same as Masaya's own), and also, the disclosures made to him by Sanchez's house helpers and the owner of the house- After the investigation, and on the basis of the results thereof, Meralco filed with the Ministry of Labor and Employment an application for clearance to terminate Masaya's services, serving copy on the latter.- Meralco also placed Masaya under preventive suspension. - A week later, Masaya filed a complaint for illegal dismissal against Meralco.- After the trial, LA Andres M. Lomabao rendered a decision in Masaya's favor; saying that the record of the investigation conducted by Meralco should not be accorded credence; that Meralco's contention that Masaya had "surreptitiously effected the direct connection of . . . electric service" was not credible, because Masaya "was employed as a bill collector, not as a lineman collector, hence, he does not know how to install electrical connection;" and that the money received by Masaya from Sanchez (P200 or P250) was not in consideration of any clandestine connection but was accepted as "representation expenses in following up Mr. Sanchez' application for installation of electric facilities . . . with the Engineer's Office at the City Hall of Manila. - NLRC affirmed the Arbiter's decision; said that since Meralco was charging Masaya of a criminal offense, it should prove beyond reasonable doubt (pbrd) said crime which it was not able to do as it was not shown that Masaya was given the opportunity to be heard by counsel or at least, a representative to confront his accuser; that based on the doctrine of PBRD, there is no causal connection between Masaya' s duties to the crime imputed to him, mere substantial evidence is insufficient to hold Masaya guilty of installing electrical connection let alone deprive him of his right to labor."

ISSUEWON the LA & the NLRC committed GABD in failing to take into consideration or excluding Masaya’s admissions in their prononcement that Masaya was illegally dismissed

HELDYESReasoningNLRC’S ERROR:- Masaya was in truth asked if he wished to be assisted by a lawyer or a representative of his Union, and his response was in the negative because, in his own words, " ang sasabihin ko naman dito ay pawang katotohanan lamang"- In administrative or quasi-judicial proceedings, PBRD is not required as basis for a judgment of the legality of an employer's dismissal of an employee, nor even preponderance of evidence, substantial evidence being sufficient. - LC: ”the rules of evidence prevailing in courts of law or equity shall not be controlling and it is the spirit and intention of this Code that the Commission and its members and the Labor Arbiters shall use every and all reasonable means to ascertain the facts in each case speedily and objectively and without regard to the technicalities of law or procedure, all in the interest of due process. . . .- SC: the ground for an employer's dismissal of an employee need be established only by substantial evidence.

- It is absolutely of no consequence that the misconduct with which an employee may be charged also constitutes a criminal offense-The proceedings being administrative, the quantum of proof is governed by the substantial evidence rule and not, as the respondent Commission seems to imagine, by the rule governing judgments in criminal actions.-The Court cannot close its eyes to the following facts of record, to wit:

1) the reality of the illegal electrical connection;2) the letter to Masaya accusing him of misconduct3) Masaya's acknowledgment that, having a copy of the company's code of discipline, he understood the nature of the accusation against him, and his declining to be

Labor Law 1 A2010 - 187 - Disiniassisted by a lawyer or a representative of his Union because, according to him, "ang sasabihin ko naman dito ay pawang katotohanan lamang;"4) his voluntary admission that it was he who had made the illegal electrical connection, describing the manner by which he had made it, and that he had received P250.00 from the occupant of the house, Antonio Sanchez; and5) his plea to the company for forgiveness for having made the illegal connection.

- on record: testimony regarding identification of Masaya by Antonio Sanchez' servants and by Castañeda, the owner of the house occupied by Sanchez. - nothing in the record to demonstrate that Masaya's admissions were made otherwise than voluntarily.- Such an offense is obviously of so serious a character as to merit the penalty of dismissal from employment, as stated in the Meralco Code on Employee Discipline:

SECTION 7. Dishonesty. —xxx xxx xxx3) Directly or indirectly tampering with electric meters or metering installation of the Company or the installation of any device, with the purpose of defrauding the Company.

-The Labor Code pronounces "fraud or willful breach by the employee of the trust reposed in him by his employer or duly authorized representative," or "serious misconduct" on the part of the employee to be lawful ground to terminate employment.Ratio And this Court has held that the "dismissal of a dishonest employee is as much in the interests of labor as it is of management. The labor force in any company is protected and the workers' security of tenure strengthened when pilferage of equipment, goods and products which endangers the viability of an employer and, therefore, the workers' continued employment is minimized or eliminated and consequently labor-management relations based on mutual trust and confidence are promoted." (*IN short: Tenurial Security is not an absolute right for the law provides that an employee may be dismissed for just cause. )Disposition Petition for certiorari is GRANTED, the decisions of the NLRC and LA are ANNULLED AND SET ASIDE, and the petitioner's termination of the employment of private respondent is AUTHORIZED and APPROVED

CITYTRUST BANKING CORPORATION V NLRC (RUIZ)258 SCRA 621

MENDOZA; July 11, 1996

NATURESpecial civil action in the Supreme Court. Certiorari

FACTS- Private respondent Ruiz was the internal auditor of petitioner Citytrust Banking Corporation. She was designated manager of the Quiapo branch of the bank, but she refused the appointment on the ground that it was a demotion. As a consequence, she was suspended and, upon clearance given by the Department of Labor, she was terminated on November 8, 1974.- Private respondent filed a complaint for illegal dismissal. She was ordered reinstated as branch manager, the NLRC urging her to accept the position, otherwise her refusal would be considered a ground for her loss of employment. Private respondent appealed to the Minister of Labor (now Secretary of Labor and Employment) but again she lost. Both parties then appealed to the Office of the President, which ordered petitioner to reinstate private respondent to her former position as internal auditor and to pay her backwages from the time her compensation was withheld up to the time of her reinstatement.- Petitioner moved for a reconsideration on the ground that the position of internal auditor had been abolished (although the position of resident inspector was created in its stead), and therefore in lieu of reinstatement, it should only be made to pay private respondent's separation pay. The Office of the President modified its decision and ordered petitioner to reinstate private respondent to a substantially equivalent position without loss of seniority rights and to grant her the benefits and privileges to which she would be entitled had she not been dismissed.- Subsequently, petitioner reinstated private respondent as manager of the Auditing Department. Private respondent accepted the appointment but questioned her reinstatement to that position on the ground that it was not substantially equivalent to the position of resident inspector (the position created in place of internal auditor). She also questioned the award of backwages as the report of the socio-economic analyst allegedly did not include backwages from April 1974 to June 1974 when she was on leave with pay and vacation and sick leave in 1974 and other fringe benefits to which she was entitled before her termination.- Labor Arbiter Apolinario N. Lumabao issued an order holding that the position of manager of the Auditing Department was not substantially equivalent to that of resident inspector. possible as it appears (that) the position is already filled up (,) to relocate complainant to a substantially equivalent position with all the emoluments and privileges of a Resident Inspector. Respondent is hereby further ordered to pay.- The NLRC affirmed the Labor Arbiter's order with modification by ordering the following to be added to the award: (a) Her vacation and sick leave privilege during the period of her separation in accordance with the disposition hereinbefore stated in the body of this Resolution, and (b)the normal increases which complainant would have received during

the period of her separation.- In connection with the computation of the award in her favor, private respondent sought the production of the bank's payrolls for 1974-1981. Her motion was opposed by petitioner which offered instead P74,344.00, the total amount of backwages as computed by the socio-economic analyst of the Department of Labor, plus P9,040.00 in transportation allowance and P1,050.00 mid-year bonus for 1974.- Private respondent refused the offer, hence the NLRC directed the analyst to compute the award on the basis of the payrolls from 1974 to 1981. Petitioner appealed to the NLRC en banc, but its petition was dismissed, on the ground that the order appealed from was interlocutory. - Petitioner filed a petition for Certiorari and Prohibition with this Court, assailing the dismissal of its appeal. The petition was at first dismissed for lack of merit. Petitioner's motion for reconsideration was also dismissed. On July 21, 1986 this Court modified its decision and petitioner was ordered to pay private respondent "backwages limited to three years without qualification or deduction at the salary rate of private respondent at the time of dismissal."- The Labor Arbiter issued an alias writ of execution after finding that the amount corresponded to the amount found due private respondent in the decision of the NLRC and the resolution of this Court, consisting of salary differentials and other fringe benefits which were not paid to her from the time that she was reinstated on August 14, 1978 as manager of the Auditing Department.- Petitioner moved to quash the alias writ of execution. As its motion was denied, it filed a petition for Injunction in the NLRC en banc to stop the implementation of the alias writ of execution and prayed for a recomputation of the monetary award pursuant to this Court's resolution of July 21, 1986. Its petition was, however, denied, as was its motion for reconsideration, in the resolutions of the NLRC. Hence, this petition.

ISSUEWON private respondent is entitled to only three years of backwages and no more

HELDNO- Private respondent is, in addition, entitled to reinstatement without loss of seniority rights. Art. 280 of the Labor Code provides:

ART. 280. Security of Tenure. — In cases of regular employment, an employer shall not terminate the services of an employee except for a just cause or when authorized by this title. An employee who is unjustly dismissed from work shall be entitled to reinstatement without loss of seniority rights and to his backwages computed from the time his compensation was withheld from him up to the time of his reinstatement. (emphasis supplied)

- Backwages are for earnings which a worker has lost due to his illegal dismissal. Private respondent was illegally dismissed from November 8, 1974 to August 13, 1978. In its May 28, 1985 Report, the socio-economic analyst computed private respondent's backwages for this period but he erroneously considered as backwages private respondent's salary differential from August 14, 1978 to October 31, 1984. On August 14, 1978, private respondent had already been reinstated, albeit to a lower paying position as manager of the Auditing Department. Hence the award of backwages should be up to August 13, 1978 only. What she was entitled to receive after that date was the difference between the salary of internal auditor (resident inspector) and that of manager of the Auditing Department to which she was actually appointed. This position, as already noted, was found to be not a substantially equivalent position to that of internal auditor or resident inspector.- The resolution of July 21, 1986 of this Court, which limited the award of backwages, referred to the backwages for the period November 8, 1974 to August 13, 1978 as component of the relief granted by law to those who are illegally dismissed. The Court at that time limited the award of backwages to three years without qualification and deduction to avoid delays incident to the determination of the earnings of the laid-off employees during the pendency of the case and of deducting them from the backwages later awarded.- The second component of the relief granted under then Art. 280 of the Labor Code was reinstatement either to their former position or if, this was not possible, to a substantially equivalent position. Reinstatement contemplates a restoration to a position from which one has been removed or separated so that the employee concerned may resume the functions of the position he already held. Private respondent was the internal auditor of petitioner at the time of her dismissal. Since this position had been replaced by the position of resident inspector, private respondent should have been appointed resident inspector. The position of manager of the Auditing Department to which she was appointed was not a substantially equivalent position, as found by the Labor Arbiter in his order of February 26, 1979 and later by the NLRC.- The order to reinstate an employee to a former position or to a substantially equivalent position is a positive mandate of the law with which strict compliance is required. This is an affirmation that those deprived of a recognized and protected interest should be made whole so that the employer will not profit from his misdeeds.- Since private respondent retired from the bank on March 1, 1991, reinstatement is now academic. She should therefore be paid the difference in pay of a resident inspector and a manager of the Auditing Department from August 14, 1978 up to March 1, 1991. Disposition Petition dismissed.

PHILIPS SEMICONDUCTORS V FADRIQUELA[PAGE 77]

Labor Law 1 A2010 - 188 - DisiniQUIJANO V BARTOLABAC

480 SCRA 204TINGA; January 27, 1999

FACTS - Quijano was employed by Mercury Drug Corporation as a warehouseman --- a clerical/rank and file position. He was dismissed, so he filed a complaint with the NLRC for illegal dismissal. The case reached the SC. In 1998, the SC ruled for his reinstatement to his old position or to a substantially similar position. The SC denied the company’s mfr, and came out with a resolution in 1999 for Quijano’s reinstatement. - What’s this case all about, then? The respondents in this case are the LA and the NLRC commissioner, respectively. Quijano filed a case against then for violation of Canon 1 and Rule 1.01 of the Code of Professional Responsibility. WHY? They gave out orders contrary to the resolution of the SC. The LA said to make him self-service attendant because accdg to mercury there were only 4 positions open. All 4 positions required college graduates, but LA said he thinks Quijano could handle the self-service attendant job. The NLRC commissioner said since there are no available positions, he should just be given separation pay.

ISSUEWON Bartolabac & Quimpo erred

HELDYES- The decision of the SC was already final and executory. They had no place to use discretion in executing a final and executory order of the Supreme Court. SUPREME. If the final & executory orders of the SC would be second-guessed by other bodies, then cases would never reach finality. The implementation of the final and executory decision is mandatory. (The court was disappointed in the IBP recommendation to dismiss the complaint against Bartolabac & Quimpo.)- The SC won’t compel to instantly restore the position of warehouseman if it had already been abolished. It ruled that Quijano should be reinstated to original or substantially similar position. They took notice of Mercury Drug’s nationwide operation. SC couldn’t believe that they wouldn’t have a position for Quijano.- Our Constitution mandates that no person shall be deprived of life, liberty, and property without due process of law. It should be borne in mind that employment is considered a property right and cannot be taken away from the employee without going through legal proceedings. In the instant case, respondents wittingly or unwittingly dispossessed complainant of his source of living by not implementing his reinstatement. In the process, respondents also run afoul of the public policy enshrined in the Constitution ensuring the protection of the rights of workers and the promotion of their welfare.Disposition Bartolabac & Quimpo suspended from the practice of law for 3 months for violation of Canon 1 and Rule 1.01 of CPR.

B. IMPORTANCE OF EMPLOYMENT

EMPLOYMENT

GONZALES V NLRC (ATENEO DE DAVAO UNIVERSITY)313 SCRA 169

BELLOSILLO; August 26, 1999

FACTS - Lorlene Gonzales was a Grade 6 teacher in Ateneo de Davao University from 1974 to 1993, when she was terminated. In 1991, the Grade School Headmaster sent her a letter informing her of 2 complaints from parents of her students for alleged use of corporal punishment. She demanded to know who the parents were because Ateneo wouldn’t tell her. When she found out that Ateneo was soliciting complaints from parents of her students, she demanded an investigation. - Ateneo sent her a notice of investigation, schedule, Committee composition, affidavits of the parents, and the rules of procedure. She refused to take part in the investigation unless the rules of procedure were revised. The committee, under advise of counsel, did not revise the rules, since it had been used for a different teacher in the past. The investigation went on, without her participation. In 1993, she was asked to tender her resignation, otherwise she would be considered resigned.

- Lorlene filed for illegal dismissal with the LA. The LA found that she was indeed illegally dismissed because although she was afforded due process, Ateneo failed to establish substantial evidence as to Lorlene’s guilt. It was established that she is a very good teacher, equipped with the appropriate educational qualifications, trainings, seminars and work experiences. Such fact was affirmed by her present and former students, their parents, colleagues and the former headmaster of the grade school. As a matter of fact, 6 out of the nine 9 students and their parents/guardians retracted and withdrew their statements.- NLRC reversed LA’s decision, saying the dismissal was valid and legal.

ISSUEWON dismissal was valid and legal HELDNO- In view of the foregoing, the conclusion of the NLRC is unwarranted. No due process – The committee refused to revise the rules of procedure. As a result, Lorlene wasn’t afforded a chance defend herself and to examine / cross-examine the accusers.Failure to prove by substantial evidence – The evidence of Ateneo didn’t measure up to the standard laid down in Ang Tibay v CIR: "substantial evidence is more than mere scintilla. It means such relevant evidence as a reasonable mind might accept as adequate to support a conclusion."Lorlene’s evidence – She was able to prove that she’s a competent and dedicated teacher of Ateneo for 17 years.- Employment is not merely a contractual relationship; it has assumed the nature of property right. It may spell the difference whether or not a family will have food on their table, roof over their heads and education for their children. It is for this reason that the State has taken up measures to protect employees from unjustified dismissals. It is also because of this that the right to security of tenure is not only a statutory right but, more so, a constitutional right.Disposition NLRC decision reversed and set aside. LA decision reinstated, affirmed and adopted.

C. STATE REGULATION - RATIONALE

RATIONALE

LLOSA-TAN V SILAHIS INTERNATIONAL HOTEL181 SCRA 738

PARAS; February 5, 1990

NATUREPetition for certiorari seeking to set aside the decision and resolutions of the NLRC

FACTS- The complainant was a front office cashier of Silahis International Hotel since November 2, 1976 until her questioned dismissal on October 30, 1982. - Since 1977, the Silahis International Hotel, had a standing corporate policy (Corporate Policy No. 014), which orders all cashiers of SMC and its affiliates to refuse the cashing of personal checks of employees and officials, endorsement by any executive of the Sulo Management Company, or Philippine Village Hotel or Silahis International Hotel or Sulo Hotel notwithstanding, because based on experience, a number of these checks unfortunately bounce to the detriment of SMC and its affiliates.- On August 22, 1982, while petitioner was on duty, she was approached by Mr. Gayondato, the general cashier of Puerto Azul Beach Resort—a sister company of Silahis International Hotel and nephew of the Executive Vice President, to encash two (2) US dollar checks with a combined value of US$1,200.00 or P10,389.60. - Although petitioner politely explained the existence of Policy No. 014 prohibiting such transactions, Gayondato persisted and assured that the presentation of aforesaid checks to the front office cashier was upon instructions of the Executive Vice President.

Labor Law 1 A2010 - 189 - Disini- Petitioner, eventually encashed the aforesaid checks, notwithstanding Corporate Policy No. 014. - Thereafter, the said checks bounced.- On October 1, 1982, respondent Vanessa Suatengco issued a memorandum to the petitioner requiring her to explain in writing why she should not be terminated for encashing the two (2) personal checks without proper authorization.- Despite petitioner's explanation, her services were terminated effective October 30, 1982.- Petitioner filed a complaint against respondents for illegal dismissal. - Labor Arbiter Virginia G. Son rendered a decision in favor of petitioner. - Hotel appealed the decision of the LA to the NLRC, and the NLRC rendered a decision setting aside the decision of the Labor Arbiter and dismissing the complaint for illegal dismissal for lack of merit - Petitioner’s 2 MFRs having been denied, recourse was made to the SC

ISSUEWON the acts of petitioner constitute gross negligence resulting in a valid ground for the termination of her employment

HELDNO- Gross negligence has been defined as the want of any or slight care or the utter disregard of consequences.- Admittedly, the encashment of the checks in question is a violation of Policy No. 014 of said hotel. But as found by the Labor Arbiter, it was established that: (a) complainant was not motivated by bad faith; (b) Policy No. 014 is not strictly or consistently enforced but has been relaxed repeatedly to meet business exigencies; and (c) complainant's encashment of the checks in question was not only with the knowledge but with clearance from her superiors who are more knowledgeable as to the circumstances under which the enforcement of the same may be relaxed.- Moreover, it cannot be said that complainant was precipitate or that she has acted in utter disregard of consequences. On the contrary, she refused to encash subject checks despite the request of Mr. Gayondato, the general cashier of Puerto Azul, but was persuaded only upon the assurances of the latter that such was the wish of the Executive Vice President and that said encashment was necessary to meet certain disbursements in Puerto Azul. In addition, she informed personally Mr. Samuel Grulla, Assistant Manager of the Silahis International Hotel, of said encashment, who also told her that such is "alright".- Finally, against the background of her previous experience when she refused to encash a similar check for Mr. Katte, the Food and Beverage Manager of Silahis International Hotel, and that she was reprimanded by the management of the Silahis International Hotel for her refusal, as well as threatened with suspension or dismissal from her job, coupled with the advice of Mr. Nestor Famatigan, Jr., Silahis International Hotel Comptroller, to use her discretion in handling similar requests in the future, it is not at all surprising that she opted to take subject course of action.- It is well settled that dismissal based on loss of trust and confidence arising from alleged misconduct of employee, is not to be used as a shield to dismiss an employee arbitrarily. Although the power to dismiss is a normal prerogative of the employer, the same is not without limitations. The right of the employer must not be exercised arbitrarily and without just cause. Otherwise, the constitutional guarantee of security of tenure of the workers would be rendered nugatory. While dismissing or laying off of an employee is a management's prerogative, it must nevertheless be done without abuse of discretion. Furthermore, the right of employer to freely select or discharge his employees is regulated by the State, because the preservation of the lives of the citizens is a basic duty of the State, more vital than the preservation of the corporate profit. In addition, security of tenure is a right of paramount value guaranteed by the Constitution and should not be denied on mere speculation. Protection for labor and social justice provisions of the Constitution and the labor laws and rules and regulations are interpreted in favor of the exercise of labor rights.Disposition The assailed decision of the NLRC is DISMISSED, and SET ASIDE and private respondent Silahis International Hotel is ordered to reinstate petitioner Anita Llosa-Tan to her former position or similar position without loss of seniority rights with full backwages beginning October 30, 1982 for a period of three (3) years therefrom.

D. COVERAGE

CONTRACT EMPLOYEE

LABAJO V ALEJANDRO165 SCRA 747

FELICIANO; September 26, 1988

NATUREPetition for certiorari with preliminary injunction to review NLRC resolution

FACTS

- The 6 private respondents had all been contracted by the petitioners to work as classroom teachers at the San Andres HS, a private learning institution situated in Maramag, Bukidnon. They then filed a complaint before the Ministry of Labor and Employment, alleging that they had each received a letter from petitioner Fr. Labajo, Director of the San Andres High School which contained: “Please be informed that your service at the San Andres High School will be terminated effective March 31, 1985.Thank you for all services you have rendered to the school.” Thus, their dismissal was without justifiable cause and violated their rights to due process and security of tenure. Petitioners’ Claims> It was admitted that they had not paid in full the employment benefits claimed by the teachers. It was alleged, however, that private respondents, prior to their acceptance of teaching jobs at the San Andres High School, "were already made aware that the school could not give them everything due them under existing laws" and, hence, were estopped from claiming such benefits. > At time of their dismissal, they were merely probationary employees of the San Andres HS whose services were terminated for just cause (upon expiration on 31 March 1985 of their respective contracts and before any of them had achieved regular or permanent status in their jobs.) * Labor Arbiter ruled in favor of the teachers. It held that they were not probationary employees, and that they could only be dismissed for cause and only after having been accorded due process. * NLRC affirmed Labor Arbiter’s decision.

ISSUEWON the respondents were illegally dismissed

HELDNORatio As probationary and contractual employees, private respondents enjoyed security of tenure, but only to a limited extent — i.e., they remained secure in their employment during the period of time their respective contracts of employment remained in effect. As petitioners were not under obligation to renew those contracts of employment, the separation of private respondents in this case cannot be said to have been without justifiable cause, much less illegal.Reasoning - Par 75 of the Manual of Regulations for Private Schools is applicable in this case: “Full-time teachers who have rendered three years of satisfactory service shall be considered permanent.” This 3-year period is the maximum period or upper limit of probationary employment allowed. Whether or not one has indeed attained permanent status in one's employment, before the passage of 3 years, is a matter of proof. - NONE of them had been able to accumulate at least 3 years of service with the San Andres HS at the time of their separation. - Private respondent AMAR argued that the 12 years of teaching experience he had accumulated prior to his acceptance of employment at San Andres qualified him as a regular employee thereof. This is not persuasive since it is the length of time Mr. Amar has been teaching at San Andres that is material in determining whether or not he in fact qualified as a regular employee.- Respondent ALEJANDRO asserted that her appointment as "Night Principal" — after having served a year thereat as a non-regular full-time teacher — amounted to a promotion which raised her status to that of a regular employee. This is also not persuasive because mere appointment as "Night Principal" is not, by itself and absent any additional evidence, sufficient proof that her employment status had in fact been upgraded from probationary to regular. - The contracts of employment entered into by the San Andres HS separately with each of the respondents stipulated, among others: (a) that employment of the individual concerned took effect at the beginning of the school year, or sometime in the month of June; and (b) that payment of that individual's salary would be made "every month for 10 months." We read these stipulations together to mean that such contracts each had an effective term of ten (10) months, i.e., from June until either March or April of the following year. New contracts for another period of ten months were negotiated between them at the beginning of each school year. It does not appear from the record or from the stipulations in those contracts, however, that renewal was obligatory upon either party. - Private respondents claimed that Fr. Labajo’s allegedly "unusual antedated letter of termination" did not sufficiently inform them of the reasons for their dismissal, nor did it satisfy the due process requirements in termination cases. These contentions ignore the fact that their employment was on a contractual basis and for a stipulated period of time.- The use of the word "terminated" was inept and unfortunate but need not preclude recognition of the real nature of that letter. Such letter was either a formal reminder that their contracts were due to expire OR advance notice that such contracts would no longer be renewed for the next school year OR both. Assuming that prior notice of expiration of the contractual term was necessary in this case, we consider that Fr. Labajo's letter substantially complied with that requirement. * Since the six (6) private respondents were not illegally dismissed, the twin remedies of reinstatement and backwages are not available to them. Dispositive NLRC Resolution is SET ASIDE, except for the portion directing petitioners to pay P52,173.67 in favor of private respondents.

PROBATIONARY EMPLOYEE

Labor Law 1 A2010 - 190 - DisiniSKILLWORD MANAGEMENT AND MARKETING

CORPORATION V NLRC (MANUEL)186 SCRA 465

MEDIALDEA; June 13, 1990

NATUREPetition for certiorari

FACTS- On June 24, 1983, Francisco Manuel was deployed to Saudi Arabia to work as driver by petitioner Skillworld Management and Marketing, a duly licensed recruitment agency operated by petitioners-spouses Serafin and Alicia Ramos. Upon his arrival in Jeddah, Manuel signed a 2-year employment contract with his foreign employer, petitioner Shary Limousine for a monthly basic salary of $300. 2 months later, Manuel was repatriated to the Philippines. Upon his arrival in the Philippines, Manuel confronted the Ramoses who promised to deploy him to other projects.- After the lapse of more than one year without being deployed to other projects of petitioners, Manuel filed a complaint with the POEA against petitioners for illegal dismissal. He alleged that while he was employed as driver of Shary Limousine in its branch at Jeddah he was stopped, and his driver's license sought for inspection, by Saudi Arabian police. He showed the police two documents given to him by his employer, Shary Limousine who made him believe that these pertained to a driver's temporary license. However, Manuel was informed that the documents were not valid for a drivers license. Together with eleven other drivers, they brought the matter before their superiors. Three days after bringing the matter to his superior, respondent was ordered to pack his things. He was taken to Riyadh and from there, repatriated to the Philippines. Upon respondent's arrival in the Philippines, he requested the Ministry of Foreign Affairs for a translation of what purported to be his driver's license. When translated it was only a certification of employment with Shary Limousine in its branch at Jeddah. - Petitioners alleged that Manuel’s dismissal was for a valid and just cause. Petitioners alleged that Manuel was dismissed because of disobedience, absenteeism, refusal to work and banding together to engage in concerted activities against the employer. - POEA rendered judgment in favor of Manuel, directing petitioners to pay him $6,900.00 or its peso equivalent. Upon appeal, the NLRC affirmed said decision. - According to petitioners, because of the probationary status of the employment of Manuel, he may be dismissed at any time. Furthermore, this agreement was contained in paragraph four (4) of the employment contract signed by Manuel.

ISSUEWON Manuel was illegally dismissed

HELDYES- There is no dispute that as a probationary employee, Manuel had but a limited tenure. Although on probationary basis, however, he still enjoys the constitutional protection on security of tenure. During his tenure of employment therefore, or before his contract expires, he cannot be removed except for cause as provided for by law.- The alleged causes for which private respondent was dismissed (disobedience, absenteeism, refusal to work, etc.) were not established. Respondent NLRC found that the purported temporary licenses to drive issued to Manuel and his co-drivers by their employer-the Shary Rent a Car/Limousine, turned out to be mere certifications to the effect that they are Filipino citizens who are holders of given passport numbers and that they were sent to work with the Shary Limousine Branch in Jeddah. It is for this reason that after being accosted twice at checkpoints by Saudi police, who informed complainant and his co-drivers that the alleged temporary licenses were not valid, they brought the matter first to their Lebanese superior and then to the Philippine Embassy. - - Further, records show that Manuel reported for work regularly and even rendered regular overtime services; that he did not even attempt to join a strike or any other form of mass action while working in Jeddah, because he knew that the laws in Jeddah are very strict and being a foreigner he did not have the courage to join much less lead a strike which is prohibited there; that he and his co-workers merely inquired from the Philippine Embassy why they were allowed to drive without licenses; and that their action prompted the Philippine Embassy to write their employer, which is perfectly in order as it was designed to protect them in foreign soil.

MANAGERIAL EMPLOYEE

INTERORIENT MARITIME ENTERPRISES INC V NLRC (TAYONG)

235 SCRA 268FELICIANO; August 11, 1994

NATUREPETITION for reviewof a decision of the National Labor Relations Commission

FACTS

- Captain Rizalino Tayong, a licensed Master Mariner with experience in commanding ocean-going vessels, was employed on 1989 by petitioners for 1 yr as stated in his employment contract. He assumed command of petitioners’ vessel at the port of Hongkong. His instructions were to replenish bunker and diesel fuel, to sail to South Africa and there to load 120,000 metric tons of coal. However, while in HK and unwarding cargo, he received a weather report that a storm would hit HK, so precautionary measures were taken to secure the safety of the vessel and its crew, considering that the vessel’s turbo-charger was leaking and the vessel was 14 yrs old. He also followed-up the requisition by the former captain for supplies of oxygen and acetylene necessary for the welding-repair of the turbo-charger and the economizer. -The vessel then sailed from HK for Singapore. Captain Tayong reported a water leak from M.E. Turbo Chapter No. 2 Exhaust gas casing so he was instructed to black off the cooling water and maintain reduced RPM unless authorized by the owners. However, the vessel stopped in mid-ocean for 6 hrs and 45 minutes due to a leaking economizer. He was instructed to shut down the economizer and use the auxiliary boiler instead. - The Chief Engineer reminded Captain Tayong that the oxygen and acetylene supplies had not been delivered. He then informed the shipowner that the departure of the vessel for South Africa may be affected because of the delay in the delivery of the supplies. The shipowner advised Captain Tayong to contact its technical director who would provide a solution for the supply of said oxygen and acetylene. The technical director recommended to Captain Tayong that by shutting off the water to the turbo charger and using the auxiliary boiler, there should be no further problem. Captain Tayong agreed to the recommendation of the technical director, but communicated his reservations regarding proceeding to South Africa without the requested supplies. So the shipowner advised him to wait for the supplies.- Finally, the vessel arrived at South Africa. However, Captain Tayong was instructed to turn-over his post to the new captain, and was repatriated to the Philippines after serving petitioners for around 2 wks. He was not informed of the charges against him, and was just sent a letter after arriving in the Philippines. He therefore instituted a complaint for illegal dismissal before the POEA, claiming his unpaid salary for the unexpired portion of the written employment contract, plus attorney’s fees. - POEA: dismissed complaint, there was valid cause for his untimely repatriation (the company alleged that due to Captain Tayong’s refusal to sail immediately to South Africa, the vessel was placed “off-hire” by the charterers, and the charterers refused to pay the charter hire or compensation corresponding to 12 hours, amounting to US $15,500.00.They fired Captain Tayong for lost of confidence; POEA believed that the Captain’s concern for the oxygen and acetylene was not legitimate as these supplies were not necessary or indispensable for running the vessel.)- NLRC: reversed and set aside POEA decision because Captain Tayong had not been afforded an opportunity to be heard and that no substantial evidenced was adduced to establish the basis for petitioners’ loss of trust or confidence. Captain had acted in accordance with his duties to maintain the seaworthiness of the vessel and to insure the safety of the ship and crew.

ISSUEWON Captain Tayong was arbitrarily dismissed and without cause as reasonably established in an appropriate investigation (whether or not Captain Tayong had reasonable grounds to believe that the safety of the vessel and the crew under his command or the possibility of substantial delay at sea required him to wait for the delivery of the supplies needed for the repair of the turbo-charger and the economizer before embarking on the long voyage from Singapore to South Africa)

HELDYESRatio It is well settled in this jurisdiction that confidential and managerial employees cannot be arbitrarily dismissed at any time, and without cause as reasonably established in an appropriate investigation. Such employees, too, are entitled to security of tenure, fair standards of employment and the protection of labor laws.Reasoning- Captain Tayong was denied any opportunity to defend himself. Petitioners curtly dismissed him from his command and summarily ordered his repatriation to the Philippines without informing him of the charge or charges against him, and much less giving him a chance to refute any such charge. In fact, it was only 2 months after his repatriation that Captain Tayong received a telegram dated 24 October 1989 from Inter-Orient requiring him to explain why he delayed sailing to South Africa.- NLRC’s conclusion was supported by substantial evidence: The official report of the technical director, which stated that a disruption in the normal functioning of the vessel’s turbo charger and economizer had prevented the full or regular operation of the vessel and that he was the one who recommended the reduction of RPM during the voyage to South Africa instead of waiting in Singapore for the supplies that would permit shipboard repair of the malfunctioning machinery and equipment, supported NLRC’s conclusion that Captain Tayong did not arbitrarily and maliciously delay the voyage to South Africa.- Captain Tayong's decision (arrived at after consultation with the vessel's Chief Engineer) to wait seven (7) hours in Singapore for the delivery on board the Oceanic Mindoro of the requisitioned supplies needed for the welding-repair, on board the ship, of the turbo-charger and the economizer equipment of the vessel, did not constitute merely arbitrary, capricious or grossly insubordinate behavior on his part. In the view of the NLRC, that decision of Captain Tayong did not constitute a legal basis for the summary

Labor Law 1 A2010 - 191 - Disinidismissal of Captain Tayong and for termination of his contract with petitioners prior to the expiration of the term thereof. Obiter- The captain of a vessel is a confidential and managerial employee within the meaning of the above doctrine. A master or captain, for purposes of maritime commerce, is one who has command of a vessel. A captain commonly performs three (3) distinct roles: (1) he is a general agent of the shipowner; (2) he is also commander and technical director of the vessel; and (3) he is a representative of the country under whose flag he navigates. Of these roles, by far the most important is the role performed by the captain as commander of the vessel; for such role (which, to our mind, is analogous to that of "Chief Executive Officer" [CEO] of a present-day corporate enterprise) has to do with the operation and preservation of the vessel during its voyage and the protection of the passengers (if any) and crew and cargo. In his role as general agent of the shipowner, the captain has authority to sign bills of lading, carry goods aboard and deal with the freight earned, agree upon rates and decide whether to take cargo. The ship captain, as agent of the shipowner, has legal authority to enter into contracts with respect to the vessel and the trading of the vessel, subject to applicable limitations established by statute, contract or instructions and regulations of the shipowner. To the captain is committed the governance, care and management of the vessel. Clearly, the captain is vested with both management and fiduciary functions.- Indeed, if the ship captain is convinced, as a reasonably prudent and competent mariner acting in good faith that the shipowner's or ship agent's instructions (insisted upon by radio or telefax from their officers thousand of miles away) will result, in the very specific circumstances facing him, in imposing unacceptable risks of loss or serious danger to ship or crew, he cannot casually seek absolution from his responsibility, if a marine casualty occurs, in such instructions. 23 - Compagnie de Commerce v. Hamburg: xxx where by the force of circumstances, a man has the duty cast upon him of taking some action for another, and under that obligation adopts a course which, to the judgment of a wise and prudent man, is apparently the best for the interest of the persons for whom he acts in a given emergency, it may properly be said of the course so taken that it was in a mercantile sense necessary to take it." - ON management prerogative: that prerogative is nevertheless not to be exercised, in the case at bar, at the cost of loss of Captain Tayong's rights under his contract with petitioner's and under Philippine law.Disposition petitioners having failed to show grave abuse of discretion amounting to loss or excess of jurisdiction on the part of the NLRC in rendering its assailed decision, the Petition for Certiorari is hereby DISMISSED, for lack of merit. Costs against petitioners

E. MANAGEMENT RIGHTS AND SECURITY OF TENURE

MANAGEMENT RIGHTS AND SECURITY OF TENURE

COLEGIO DE SAN JUAN DE LETRAN V ASSN OF EMPLOYEES AND FACULTY OF LETRAN

340 SCRA 587KAPUNAN; September 18, 2000

NATUREPetition for review on certiorari

FACTS- Private respondent Ambas, the newly elected president of the Association of Employees and Faculty of Letran (Union) wanted to continue the renegotiation of its CBA with petitioner Colegio de San Juan de Letran (Letran) for the last 2 years of the CBA’s 5 year lifetime. However, petitioner claimed the CBA was already prepared for signing by the parties. The CBA was submitted to a referendum by the union members, who rejected it.- Petitioner accused the union officers of bargaining in bad faith before the NLRC which decided in favor of petitioner but was later reversed on appeal with the NLRC. - The Union notified the National Conciliation and Mediation Board (NCMB) of its intention to strike on the grounds of petitioner’s refusal to bargain. Later, the parties agreed to disregard the unsigned CBA and start negotiating a new 5 year CBA for which the Union submitted its proposals. Ambas protested a recent changing of her schedule and petitioner sent the Union a letter dismissing Ambas for alleged insubordination after which the Union amended its notice of strike to include the said dismissal.- Both parties again discussed the ground rules for the CBA renegotiation but petitioner stopped the negotiations after purportedly receiving information that a new group of employees (ACEC) filed a petition for certification election, giving rise to the issue of majority representation of the employees.- The Union finally went on strike and the Sec. of Labor and Employment assumed jurisdiction, ordering those on strike to return to work and for petitioner to accept them under the same terms before the strike. All were readmitted except Ambas. The Sec. issued an order declaring petitioner guilty of unfair labor practice and directing the

reinstatement of Ambas with backwages. Letran’s MFR was denied and the CA affirmed the Sec.’s decision, hence this petition.

ISSUES1. WON petitioner is guilty of unfair labor practice by refusing to bargain with the union2. WON the termination of the Ambas amounts to an interference of the employee’s right to self-organization

HELD 1. YES- Petitioner is guilty of unfair labor practice by its stern refusal to bargain in good faith with respondent union. - Article 252 defines collective bargaining as the performance of a mutual obligation to meet and convene promptly and expeditiously in good faith for the purpose of negotiating an agreement. The Union, in sending its proposals during the 2nd CBA negotiations, kept up its end of the bargain while Letran devised ways and means to prevent the negotiation.- Letran also failed to make a timely reply to the Union’s proposals (no counter-proposal a month later), violating Article 250 which requires such a reply within 10 days upon receipt of a written notice of said proposals. Letran’s refusal to reply is an indication of bad faith, showing a lack of sincere desire to negotiate. - In a last ditch effort, Letran suspended the bargaining process on the ground that it allegedly received information that ACEC had filed a petition for certification election. The mere filing of a petition for certification election does not ipso facto justify the suspension of negotiations when there is no legitimate representation issue raised; also, such an action for intervention had already prescribed.2. YES- While we recognize the right of the employer to terminate the services of an employee for just cause, the dismissal of employees must be made within the parameters of law and pursuant to the tenets of equity and fair play and must be exercised in good faith. It must not amount to interfering with, restraining or coercing employees in the exercise of their right to self-organization as it would amount to unlawful labor practice under Article 248.-It would appear that Letran terminated Ambas in order to strip the union of a leader who would fight for her co-workers’ rights at the bargaining table and frustrate their desire to form a new CBA. The charge of insubordination was a mere ploy to give a color of legality to the action to dismiss her. Management may have the prerogative to discipline its employees for insubordination but when it interferes with employees’ right to self-organization, it amounts to union-busting which is a prohibited act. Disposition petition is DENIED for lack of merit

SAN MIGUEL BREWERY SALES FORCE UNION V OPLE

170 SCRA 25GRIÑO-AQUINO; February 8, 1989

FACTS- A collective bargaining agreement was entered into by petitioner San Miguel Corporation Sales Force Union and the private respondent, San Miguel Corporation. One provision of the CBA was “employees within the appropriate bargaining unit shall be entitled to a basic monthly compensation plus commission based on their respective sales."- Few months after the said CBA, the company introduced a marketing scheme known as the "Complementary Distribution System" (CDS) whereby its beer products were offered for sale directly to wholesalers through San Miguel's sales offices.- The labor union filed a complaint for unfair labor practice in the Ministry of Labor, with a notice of strike on the ground that the CDS was contrary to the existing marketing scheme whereby the Route Salesmen were assigned specific territories within which to sell their stocks of beer, and wholesalers had to buy beer products from them, not from the company. It was alleged that the new marketing scheme violates Section 1, Article IV of the collective bargaining agreement because the introduction of the CDS would reduce the take-home pay of the salesmen and their truck helpers for the company would be unfairly competing with them.

ISSUES1. WON the CDS violates the collective bargaining agreement2. WON it is an indirect way of busting the union

HELD1. NO- CDS is a valid exercise of management prerogatives:Ratio Except as limited by special laws, an employer is free to regulate, according to his own discretion and judgment, all aspects of employment, including hiring, work assignments, working methods, time, place and manner of work, tools to be used, processes to be followed, supervision of workers, working regulations, transfer of employees, work supervision, lay-off of workers and the discipline, dismissal and recall of work.

Labor Law 1 A2010 - 192 - Disini- So long as a company's management prerogatives are exercised in good faith for the advancement of the employer's interest and not for the purpose of defeating or circumventing the rights of the employees under special laws or under valid agreements, this Court will uphold them2. NORatio Nothing in the record as to suggest that the unilateral action of the employer in inaugurating the new sales scheme was designed to discourage union organization or diminish its influence, but rather it is undisputable that the establishment of such scheme was part of its overall plan to improve efficiency and economy and at the same time gain profit to the highest. While it may be admitted that the introduction of new sales plan somewhat disturbed the present set-up, the change however was too insignificant as to convince this Office to interpret that the innovation interferred with the worker's right to self-organization.

Reasoning- Petitioner failed to consider is the fact that corollary to the adoption of the assailed marketing technique is the effort of the company to compensate whatever loss the workers may suffer because of the new plan over and above than what has been provided in the collective bargaining agreement. To us, this is one indication that the action of the management is devoid of any anti-union hues."Disposition Dismissed

F. GUIDELINE ON IMPOSITION OF PENALTIES

VALIAO V CA[PAGE 11]

FARROL V CA (RCPI)325 SCRA 331

YNARES-SANTIAGA; February 10, 2000

FACTS - Wenifrado Farrol was the station cashier of RCPI Cotabato City Station. - There was a P50K cash shortage in the branch’s Peragram Petty Cash Funds. Farrol was required to explain the cash shortage. He paid to P25K to RCPI- He was then required to explain why he should not be dismissed. Petitioner wrote to the Field Auditor stating that the missing funds were used for the payment of the retirement benefits earlier referred by the Branch Manager and that he already paid P25k. After he made 2 more payments of the cash shortage, he was placed under preventive suspensions. He still made 2 payments of the balance. - RCPI then sent Farrol a letter informing him of the termination of his services for alleging that part of the cash shortage was used for payment of salaries and retirement benefits, disregard of policies involving statistical reports, malversation/misappropriation (which is a ground for dismissal), and loss of trust and confidence. - Unaware of the termination letter, he requested his reinstatement since his preventive suspension had expired. Ferrol even manifested his willingness to settle the case. RCPI informed him that his employment had already been terminated. The conflict was sent to the grievance committee. Two years later, it was submitted for voluntary arbitration. - VA ruled in favor of Farrol. RCPI filed a petition for certiorari before the CA which reversed VA decision. CA also dismissed MFR. - Farrol now filed a petition for review on certiorari on the ground that his dismissal was illegal because he was not afforded due process and that he cannot be held liable for the loss of trust and confidence reposed in him.

ISSUEWON he was illegally terminated

HELDYES- BOP resides on the employer to prove that there was valid cause for dismissal, and that he was afforded the opportunity to be heard and defend himself. - For the 1st notice, RCPI required petitioner to explain why he failed to account for the shortage. The 2nd notice was that informing Farrol of his termination. it does not clearly cite the reasons for dismissal, nor were there facts and circumstances in support thereof. - Even assuming there was a breach of trust and confidence, there was no evidence that Farrol was a managerial employee. The term “trust and confidence” is restricted to managerial employees. - RCPI alleges that under its rules, petitioner’s infarction is punishable by dismissal. However, employer’s rules cannot preclude the state from inquiring whether strict and rigid application or interpretation would be too harsh to the employee. This is Farrol’s 1st

offense, to which the Court holds that dismissal is too harsh and grossly disproportionate.Disposition CA is REVERSED and SET ASIDE and new one entered REINSTATING the decision of the Voluntary Arbitrator subject to the MODIFICATION that petitioner’s separation pay be recomputed to include the period within which backwages are due. For this purpose, this case is REMANDED to the Voluntary Arbitrator for proper computation of backwages, separation pay, 13th month pay, sick leave conversion and vacation leave conversion.

VH MANUFACTURING INC V NLRC (GAMIDO)322 SCRA 417

DE LEON; January 19, 2000

NATUREBefore us is a petition for certiorari

FACTS- Since November 5, 1985 Gamido was employed in VH Manufacturing’s business of manufacturing liquefied petroleum gas (LPG) cylinders. He served as a quality control inspector with the principal duty of inspecting LPG cylinders for any possible defects. His service with the company was abruptly interrupted on February 14, 1995, when he was served a notice of termination of his employment. - His dismissal stemmed from an incident on February 10, 1995 wherein VH’s company President, Alejandro Dy Juanco, allegedly caught private Gamido sleeping on the job. On that same day, private respondent was asked through a written notice from the petitioner’s Personnel Department to explain within twenty-four (24) hours why no disciplinary action should be taken against him for his violation of Company Rule 15-b which provides for a penalty of separation for sleeping during working hours. Without delay, private respondent replied in a letter which reads: "Sir, ipagpaumanhin po ninyo kung nakapikit ako sa aking puwesto dahil hinihintay ko po ang niliha hi Abreu para i quality pasensiya na po kung hindi ko po namalayan ang pagdaan ninyo dahil maingay po ang painting booth." Notwithstanding his foregoing reply, he was terminated. - Feeling aggrieved, he filed a complaint for illegal dismissal, praying for reinstatement to his position as quality control inspector. Labor Arbiter declared that Gamido’s dismissal is anchored on a valid and just cause. NLRC reversed the decision.

ISSUEWON Gamido’s dismissal was too harsh a penaltly for his violation of company rule 15-b

HELDYES- Basically, the reason cited for the dismissal of private respondent is sleeping on the job in violation of Company Rule 15-b. But according to Gamido, he was not sleeping on the job but was merely idle, waiting for the next cylinder to be checked.- In view of the gravity of the penalty of separation, as provided by the Company Rules and Regulation., in termination disputes, the burden of proof is always on the employer to prove that the dismissal was for a just and valid cause. What is at stake here is not only the job itself of the employee but also his regular income therefrom which is the means of livelihood of his family.- A thorough review of the record discloses that, contrary to the findings of the Labor Arbiter, petitioner’s claim that private respondent slept on the job was not substantiated by any convincing evidence other than the bare allegation of the officer.- Next, VH’s reliance on the authorities it cited that sleeping on the job is always a valid ground for dismissal, is misplaced. The authorities cited involved security guards whose duty necessitates that they be awake and watchful at all times inasmuch as their function, to use the words in Luzon Stevedoring Corp. v. Court of Industrial Relations, is "to protect the company from pilferage or loss." Accordingly, the doctrine laid down in those cases is not applicable to the case at bar.- Finally, while an employer enjoys a wide latitude of discretion in the promulgation of policies, rules and regulations on work-related activities of the employees, those directives, however, must always be fair and reasonable, and the corresponding penalties, when prescribed, must be commensurate to the offense involved and to the degree of the infraction. In the case at bar, the dismissal meted out on private respondent for allegedly sleeping on the job, under the attendant circumstances, appears to be too harsh a penalty, considering that he was being held liable for first time, after nine 9 of unblemished service, for an alleged offense which caused no prejudice to the employer, aside from absence of substantiation of the alleged offense. Neither was it shown that private respondent’s alleged negligence or neglect of duty, if any, was gross and habitual. Thus, reinstatement is just and proper.Disposition petition is hereby DISMISSED, and the challenged Decision and Order of public respondent NLRC are AFFIRMED.

Labor Law 1 A2010 - 193 - Disini

REYNO V MANILA ELECTRIC COMPANY434 SCRA 660

SANDOVAL-GUTIERREZ; July 22, 2004

NATUREPetition for review on certiorari under Rule 45 of the 1997 Rules of Civil Procedure

FACTS- Reyno was employed by MERALCO where he eventually occupied the position of Assistant Squad Leader of Squad 12 at the Inspection Department. Petitioner and his team of inspectors were in charge of monitoring and inspecting electric meters installed at the premises of respondent’s customers; ensuring the accuracy of the electric consumption recorded in these meters; and reporting and apprehending violators who use insidious schemes or devices to reduce their electric consumption deliberately.- Later, MERALCO implemented an incentive scheme aimed at encouraging its inspectors to perform their duties zealously. Under this incentive scheme, the inspector concerned shall be paid an additional 30-minute overtime pay for every submitted report of major violation/s committed by customers against respondent.- Roger Sacdalan, Senior Investigator of respondent’s Special Presidential Committee (SPC), received several complaints against Gilbert Villapa, Leader of Squad 12, about an illegal connection.- SPC conducted an investigation wherein members of Squad 12 were summoned to explain. However, they failed to establish Villapa’s involvement in such illegal connection. Instead, their declarations pointed to Reyno’s irregular performance of his duties. - This prompted SPC to conduct clarificatory hearing. But the hearing was cancelled for failure of Reyno’s counsel to appear despite notice. When the case was called for hearing as scheduled, his counsel again failed to appear. He then opted to proceed with the clarificatory hearing without the assistance of his counsel.- After evaluating the records on hand, the SPC found petitioner guilty of dishonesty, serious misconduct and willful breach of trust. Respondent then sent petitioner a notice terminating his services.- Reyno filed with the Labor Arbiter a complaint for illegal dismissal and payment of overtime pay, premium pay for holidays and rest days, damages and attorney’s fees.

ISSUES1. WON Reyno was deprived of his right to cross examine witnesses before the Labor Arbiter2. WON Reyno was illegally dismissed

HELD1. NO- His right to cross-examine the three witnesses, did not err as it was not required to apply strictly the Rules of Evidence. At any rate, MERALCO had valid reasons why it did not present those three witnesses during the proceedings before the Labor Arbiter2. NO- The standard of substantial evidence is satisfied where the employer, as in this case, has reasonable ground to believe that the employee is responsible for the misconduct and his participation therein renders him unworthy of trust and confidence demanded by his position. Reyno violated MERALCO’s Code of Employee Discipline and committed serious misconduct in the performance of his duties have been proved by the affidavits of petitioner’s own subordinates in Squad 12 of which he was the Assistant Squad Leader. Moreover, MERALCO had lost his trust and confidence in petitioner. Under Article 282 of the Labor Code, as amended, these are just causes for his dismissal from the service.- The longer an employee stays in the service of the company, the greater is his responsibility for knowledge and compliance with the norms of conduct and the code of discipline in the company.- An employee’s length of service with the company even aggravates his offense. He should have been more loyal to company from which he has derived his family bread and butter for seventeen (17) years.Disposition Petition is DENIED. The assailed Decision dated January 17, 2001 and Resolution dated May 3, 2001 of the Court of Appeals in CA-G.R. SP No. 53987 are hereby AFFIRMED.

FACTORS

ASSOCIATED LABOR UNION V NLRC[PAGE 181]

PHILIPPINE LONG DISTANCE TELEPHONE V NLRC (GABRIEL)303 SCRA 9

QUISUMBING; February 11, 1999

NATURE Appeal from the order of the NLRC

FACTS- Private respondent, Enrique Gabriel, was foreman of petitioner PLDT and was a supervisor with territorial responsibility for Camp Crame’s First to 20 th Avenue and portions of Project 4, all located in Quezon City. On two occasions (September 5, 1989 and October 16, 1989) he ordered Medel Mercado and Juancho Jocson to install two telephone lines each at Unit R, Facilities Center Building, located at Shaw Boulevard, Mandaluyong.- The ordered installations were investigated because (a) the Facilities Center Building had no entrance cable facilities or conduit wires for telephone connection, (b) Mandaluyong was not within Gabriel’s area of jurisdiction, and (c) installers Mercado and Jocson were not under his direct supervision.- During the investigation, Gabriel. while acknowledging responsibility for his action, claimed that his actuation was motivated by the desire to provide customer satisfaction. He also claimed that the telephones were installed after the documents of approval were issued by PLDT. He dismissed from service on September 3, 1990 on the ground that he committed grave misconduct, breach of trust, and violations of company rules and regulations. - Gabriel filed an illegal dismissal complaint with the Labor Arbiter on September 6, 1990. Said Arbiter affirmed the dismissal but the same was reversed by the NLRC and ordered PLDT to reinstate Gabriel to the position he held as at the time of the complained dismissal, with full backwages, benefits, and proportionate privileges. Hence the appeal.

ISSUEWON Gabriel is guilty of serious misconduct and/or breach of trust anent the irregular installation of the telephones

HELDNO- The facts of the case do not point to any misconduct or breach of trust on the part of Gabriel. There was also no provision in the written rule of PLDT which penalizes unwarranted installation of telephone lines with dismissal. In any case, the installations were approved by the company. There was also no evidence that Gabriel profited personally with the transaction. The dismissal of Gabriel is illegal.Reasoning- Dismissal is the ultimate penalty and should not be imposed if the employee has been in service for a considerable length of time and has not been the recipient of any disciplinary actions. Where a penalty less punitive would suffice, whatever missteps may have been committed by the worker ought not to be visited with a consequence so severe such as dismissal. This interpretation gives meaning and substance to the liberal and compassionate spirit of the law as provided for in Article 4 of the Labor Code which states that “all doubts in the implementation and interpretation of the provisions of the Labor Code including its implementing rules and regulations shall be resolved in favor of labor. - Gabriel is not entirely faultless. As a supervisor, he is required to act judiciously and to exercise his authority in harmony with PLDT’s policies. When he jeopardized the status of the rank and file employees whom he ordered to by-pass the standard operating procedures of the company, to the detriment of his employer, he was not entirely blameless. The irregularity attributable to him could not be disregarded. He must not be rewarded, in fairness to the employer’s own legitimate concerns such as company morale and discipline.Disposition the resolution f the NLRC is affirmed subject to the deletion of the other awards of unspecified “benefits and proportionate privileges”.

DISMISSAL AS PENALTY

CEBU FILVENEER CORPORATION V NLRC (VILLAFLOR)

Labor Law 1 A2010 - 194 - Disini286 SCRA 556

PUNO; February 24, 1998

FACTS - Villaflor was the chief accountant of CFC. The top execs were Italians: Cordaro (president), Kun (GM), Marinoni (Production manager). Guillermo was the accounting clerk of Villaflor. - Kun resigned from the company and asked for the liquidation of his investment: P125k. Two weeks later, he asked Guillermo for a blank check and a blank check voucher. Guillermo gave him. Three days later, Villlaflor noticed that a check voucher was missing. She asked Guillermo, who said that Mr. Kun has it. - Villaflor immediately informed Mr. Cordaro of what happened. She also wrote to the bank demanding the return of the encashed check. - Marinoni charged Villaflor of complicity in Kun’s irregular disbursement of company funds. Two days later, she was prevented entry to the office by the security guards. Her office drawer and safe were also forcibly opened upon order of Marinoni. Villaflor reported the incident to the PNP. - Marinoni suspended her for 30 days without pay for failure to come to work for half a day (the day she was prevented entry). The next day she was preventively suspended for 30 days pending investigation of her involvement in Kun’s booboo. The company also printed a newspaper ad for an accountant. - Villaflor filed for illegal dismissal with the LA. LA decided in her favor. NLRC affirmed.

ISSUEWON Villaflor was illegally dismissed

HELDYES- Due to its far reaching implications, our Labor Code decrees that an employee cannot be dismissed, except for the most serious causes. Article 282 enumerates the causes for which the employer may terminate an employee.- Company says it’s loss of trust. The SC said that Villaflor’s omission cannot be described as “willful” to justify dismissal. A breach is willful if it is done intentionally, knowingly and purposely. Petitioners merely proved the omission of the private respondent but there is no evidence whatsoever that it was done intentionally.- Company says she’s grossly or habitually negligent in the performance of her duties. The SC said that since she has not been remiss in the performance of her duties in the past, she can’t be charged with habitual negligence. Neither is her negligence gross in character. Gross negligence implies a want or absence of or failure to exercise slight care or diligence or the entire absence of care. It evinces a thoughtless disregard of consequences without exerting any effort to avoid them. She had not the slightest reason to distrust Kun because he was the GM and appears to have conducted himself well in the performance of his duties in the past. At most, it’s error of judgment, not gross negligence. Disposition NLRC decision affirmed.

GOLDEN THREAD KNITTING INDUSTIRES V NLRC (MACASPAC)304 SCRA 720

BELLOSILLO; March 11, 1999

NATUREPetition to review decision of NLRC

FACTS- several employees of Golden Thread Knitting Industries (GTK) were dismissed for different reasons. 2 employees were allegedly for slashing the company’s products (towels), 2 for redundancy, 1 for threatening the personnel manager and violating the company rules, and 1 for abandonment of work. - The laborers filed complaints for illegal dismissal. They allege that the company dismissed them in retaliation for establishing and being members of the Labor Union. GTK, on the other hand, contend that there were valid causes for the terminations. The dismissals were allegedly a result of the slashing of their products, rotation of work, which in turn was caused by the low demand for their products, and abandonment of work. WRT to the cases involving the slashing of their products and threats to the personnel manager, the dismissals were in effect a form of punishment. - The labor arbiter ruled partially in favor of GTK. He said that there was no showing that the dismissals were in retaliation for establishing a union. He, however, awarded separation pay to some employees.- NLRC, however, appreciated the evidence differently. It held that there was illegal dismissal and ordered reinstatement.

ISSUEWON there was illegal dismissal

HELDYES

Ratio Dismissal is the ultimate penalty that can be meted to an employee. It must therefore be based on a clear and not on an ambiguous or ambivalent ground.Reasoning- WRT to the case involving slashing of towels, the employees were not given procedural due process. There was no notice and hearing, only outright denial of their entry to the work premises by the security guards. The charges of serious misconduct were not sufficiently proved.- WRT to the employees dismissed for redundancy, there was also denial of procedural due process. Hearing and notice were not observed. Thus, although the characterization of an employee’s services is a management function, it must first be proved with evidence, which was not done in this case. the company cannot merely declare that it was overmanned. - WRT to the employee dismissed for disrespect, the SC believed the story version of the company (which essentially said that the personnel manager was threatened upon mere service of a suspension order to the employee), but ruled that the dismissal could not be upheld.

“the dismissal will not be upheld where it appears that the employee’s act of disrespect was provoked by the employer. xxx the employee hurled incentives at the personnel manager because she was provoked by the baseless suspension imposed on her. The penalty of dismissal must be commensurate with the act, conduct, or omission to the employee.”

- The dismissal was too harsh a penalty; a suspension of 1 week would have sufficed. “GTK exercised their authority to dismiss without due regard to the provisions of the Labor Code. The right to terminate should be utilized with extreme caution because its immediate effect is to put an end to an employee's present means of livelihood while its distant effect, upon a subsequent finding of illegal dismissal, is just as pernicious to the employer who will most likely be required to reinstate the subject employee and grant him full back wages and other benefits.

Disposition Decision AFFIRMED

CENTRAL PANGASINAN ELECTRIC COOP INC V MACARAEG

395 SCRA 720PUNO; January 22, 2003

NATUREPetition for review on certiorari

FACTS- De Vera was employed as teller and Geronima Macaraeg as cashier by Central Pangasinan Electric cooperative inc. They accommodated and encashed two hundred eleven crossed checks of Evelyn Joy Estrada (de Vera’s sister) amounting to P6,945,128.95 payable to the cooperative despite the absence of any transaction or any outstanding obligation with it. They credited the checks as part of their collection and deposited the same together with their cash collection to the coop’s account at the Rural Bank of Central Pangasinan.- The finance department noticed these checks which bounced (insufficient funds).De Vera and Macaraeg were confronted with the discovery. De Vera admitted that the checks were issued by her sister and that she encashed them from the money collected from petitioner’s customers.- De Vera testified and admitted that she encashed the checks of Evelyn Joy Estrada because the latter is her older sister. Macaraeg admitted that she knew of the accommodations given by respondent de Vera to her sister; that she allowed her subordinate to do it because respondent de Vera is her kumare, and that she knew that Mrs. Estrada’s checks were sufficiently funded. - On March 19, 1999, on the basis of the findings and recommendation of Atty. Fernandez (presided over the hearing), the General Manager issued to respondents separate notices of termination for “serious misconduct, and breach of trust and confidence reposed on them by management.”- Respondents questioned their dismissal before the National Conciliation and Mediation Board (NCMB),claiming that their dismissal was without just cause and in violation of the Collective Bargaining Agreement (CBA), which requires that the case should first be brought before a grievance committee. Eventually, the parties agreed to submit the case to a voluntary arbitrator for arbitration.- LA-ruled in favor of defendants and ordered their reinstatementCA-affirmed

ISSUES1. WON the procedure leading to the termination of respondents Maribeth de Vera and Geronima Macaraeg was in violation of the provisions of the CBA2. WON the respondents were validly dismissed

HELD1. Issue is moot and academic- The parties’ active participation in the voluntary arbitration proceedings, and their failure to insist that the case be remanded to the grievance machinery, shows a clear intention on their part to have the issue of respondents’ illegal dismissal directly resolved by the voluntary arbitrator.

Labor Law 1 A2010 - 195 - Disini2. YES- The respondents were validly dismissed. Article 282(c) of the Labor Code allows an employer to dismiss employees for willful breach of trust or loss of confidence. Proof beyond reasonable doubt of their misconduct is not required, it being sufficient that there is some basis for the same or that the employer has reasonable ground to believe that they are responsible for the misconduct and their participation therein rendered them unworthy of the trust and confidence demanded of their position.Reasoning- the acts of the respondents were clearly inimical to the financial interest of the petitioner. During the investigation, they admitted accommodating Evelyn Joy Estrada by encashing her checks from its funds for more than a year. They did so without petitioner’s knowledge, much less its permission. - there was willful breach of trust on the respondents’ part, as they took advantage of their highly sensitive positions to violate their duties.- the acts of the respondents caused damage to the petitioner. During those times the checks were illegally encashed, petitioner was not able to fully utilize the collections, primarily in servicing its debts. - it is not material that they did not “misappropriate any amount of money, nor incur any shortage relative to the funds in their possession.” The basic premise for dismissal on the ground of loss of confidence is that the employees concerned hold positions of trust. The betrayal of this trust is the essence of the offence for which an employee is penalized. - the respondents held positions of utmost trust and confidence. As teller and cashier, respectively, they are expected to possess a high degree of fidelity. They are entrusted with a considerable amount of cash. Respondent de Vera accepted payments from petitioner’s consumers while respondent Macaraeg received remittances for deposit at petitioner’s bank. They did not live up to their duties and obligations.

PHILIPS SEMICONDUCTORS V FADRIQUELA[PAGE 77]

G. RULES – MANAGERIALS AND RANK AND RANK FILE EMPLOYEES

SALVADOR V PHILIPPINE MINING SERVICE CORP395 SCRA 729

PUNO; January 22, 2003

FACTS- JOSE V. SALVADOR was first employed by respondent in 1981. He rose from the ranks and assumed the position of Plant Inspection Foreman in 1991. He was tasked to: (1) supervise plant equipment and facility inspection; (2) confirm actual defects; (3) establish inspection standards and frequency; (4) analyze troubles and recommend counter measures; and (5) prepare weekly/monthly inspection schedule.[3]- As early as March 1, 1985, respondent instituted the “shift boss” scheme whereby the foreman from the Plant Section and the foreman from the Mining Section rotate as shift boss throughout their night shift to oversee and supervise both the mining and plant operations. The shift boss was entrusted with the care, supervision and protection of the entire plant.- Aside from his employment with respondent, petitioner co-owned and managed LHO-TAB Enterprises, with his partner Ondo Alcantara. They were engaged in the manufacture and sale of hollow blocks. On September 29, 1997, petitioner’s employment relation with respondent was tainted with charges of pilferage and violation of company rules and policy, resulting to loss of confidence. Respondent’s evidence disclose that on September 29, 1997, at about 9:30 a.m., Koji Sawa, respondent’s Assistant Resident Manager for Administration, was on his way back to his office in the plant. He and his driver, Roberto Gresones, saw petitioner operating respondent’s payloader, scooping fine ore from the stockpile and loading it on his private cargo truck. As the truck was blocking the access road leading to the stockyard’s gate, Sawa’s car stopped near the stockpile and the driver blew the horn thrice. Petitioner did not hear him because of the noise emanating from his operation of the payloader. Sawa’s driver found a chance to pass through when the payloader maneuvered to get another scoop from the fine ore stockpile.- As it was contrary to respondent’s standard operating procedure for the plant foreman to operate the payloader, Sawa went to the administration office to check the delivery receipt covering the loading operation of petitioner that morning. However, sales-in-charge Eduardo Guangco was in the wharf, overseeing the loading of respondent’s product. Hence, it was only in the afternoon that Sawa was able to verify the delivery receipt covering petitioner’s loading transaction. The delivery receipt showed that it was dolomite spillage that was purchased by buyer Ondo Alcantara, not the fine ore that he saw petitioner loading on his truck. The receipt also showed it was not the respondent but Alcantara, the buyer, who was responsible for loading the spillage he purchased from the plant. - On the basis of the foregoing facts PMSC terminated Salvador for pilferage of company property. Labor Arbiter and NLRC ruled in favor of Salvador but CA reversed. Hence, this recourse.

ISSUES1. WON the charge of pilferage against petitioner was supported by substantial evidence to warrant his dismissal from the service2. WON the employer was well within its rights in imposing a harsh penalty considering the length of the employee’s service

HELD1. YESRatio The settled rule in administrative and quasi-judicial proceedings is that proof beyond reasonable doubt is not required in determining the legality of an employer’s dismissal of an employee, and not even a preponderance of evidence is necessary as substantial evidence is considered sufficient. Substantial evidence is more than a mere scintilla of evidence or relevant evidence as a reasonable mind might accept as adequate to support a conclusion, even if other minds, equally reasonable, might conceivably opine otherwise. Thus, substantial evidence is the least demanding in the hierarchy of evidence.Reasoning- The Labor Code provides that an employer may terminate the services of an employee for just cause and this must be supported by substantial evidence. In the case at bar, our evaluation of the evidence of both parties indubitably shows that petitioner’s dismissal for loss of trust and confidence was duly supported by substantial evidence.2. NORatio As a general rule, employers are allowed wider latitude of discretion in terminating the employment of managerial employees as they perform functions which require the employer’s full trust and confidence.Reasoning - To be sure, length of service is taken into consideration in imposing the penalty to be meted an erring employee. However, the case at bar involves dishonesty and pilferage by petitioner which resulted in respondent’s loss of confidence in him. Unlike other just causes for dismissal, trust in an employee, once lost is difficult, if not impossible, to regain. Moreover, petitioner was not an ordinary rank-and-file employee. He occupied a high position of responsibility. As foreman and shift boss, he had over-all control of the care, supervision and operations of respondent’s entire plant. It cannot be over-emphasized that there is no substitute for honesty for sensitive positions which call for utmost trust. Fairness dictates that respondent should not be allowed to continue with the employment of petitioner who has breached the confidence reposed on him. - In the case at bar, respondent has every right to dismiss petitioner, a managerial employee, for breach of trust and loss of confidence as a measure of self-preservation against acts patently inimical to its interests. Indeed, in cases of this nature, the fact that petitioner has been employed with the respondent for a long time, if to be considered at all, should be taken against him, as his act of pilferage reflects a regrettable lack of loyalty which he should have strengthened, instead of betrayed.Disposition The petition is DENIED.

CAOILE V NLRC (COCA-COLA BOTTLERS, PHILIPPINES INC)

299 SCRA 76QUISUMBING; November 24, 1998

NATURESpecial action for certiorari

FACTS- Private respondent CCBPI, through the local plant management, contracted the services of Mr. Redempto de Guzman for the installation of a Private Automatic Branch Exchange (PABX) housewiring in the plant premises for the sum of P65,000.00. Since the project fell under the direct supervision of petitioner, all cash advances by the contractor were coursed through him.- Mr. De Guzman, the contractor, requested for an initial cash advance of P10,000.00. Petitioner caused the preparation of the Payment Request Memo in the amount of P15,000.00 and the issuance of a check in the same amount. After securing the endorsement of the contractor, petitioner encashed the check with the plant teller Mr. Dominador S. Pila and handed over P10,000.00 to Mr. De Guzman while retaining the amount of P5,000.00 for himself. - The contractor requested for second and third cash advances in the amounts of P5,000.00 and P10,000.00 respectively. As in the first cash advance, petitioner caused the preparation of 2 checks in the amounts of P10,000.00 and P15,000.00 respectively.

Labor Law 1 A2010 - 196 - DisiniAfter securing the endorsements of the contractor the requested cash advances while retaining for himself the difference of P10,000.00.- After the project was completed, the contractor requested payment of the balance of the contract price in the amount of P25,000.00. Petitioner caused the issuance of a check in the amount of P24,350.00 (after deducting 1% of the total contact price by way of witholding tax). Petitioner secured the endorsement of the contractor, encashed the check with the teller, then handed over to the contractor only P19,350.00 while retaining fore himself the amount of P5,000.00.- Upon completion of an additional project requested of the contractor, petitioner caused the issuance a check, and after securing the endorsement of the contractor, petitioner encashed the check and delivered P8,000.00 to the contractor and retained P500.00 for himself.- Mr. de Guzman executed an affidavit exposing the fraudulent acts perpetrated by petitioner, which prompted the company to conduct an investigation. - Petitioner was served a Notice of investigation. During the investigation, petitioner admitted that the initials in the check vouchers were his but denied having encashed the checks and delivering the cash payments to the contractor. - It was established through the testimony of Mrs. Macasinag and Mr. Pila that petitioner personally withdrew the checks from the GM Secretary and had them encashed with the teller after Mr.de Guzman has endorsed the same.- Mr. Mariano A. Limjap, Senior VP and Administration Director issued a memo sustaining the findings and recommendation of the local plant management for the termination of complainant from his employ on the grounds of grave misconduct and dishonesty considering that his position as EDP Supervisor is bestowed with the highest trust and confidence by the respondent as may be seen from the description of his duties and responsibilities.- As a consequence of his dismissal, petitioner filed a compliant for illegal dismissal with damages - Labor Arbiter rendered a decision finding that petitioner was illegally dismissed - Private respondents appealed to NLRC which reversed the Labor Arbiter's decision. NLRC held that petitioner committed acts constituting a breach of trust and confidence reposed on him by his employer, thereby justifying his dismissal.

ISSUEWON the NLRC committed grave abuse of discretion amounting to lack or excess of jurisdiction in reversing and setting aside the Labor Arbiter's decision finding private respondents guilty of illegal dismissal

HELDNORatio Law and jurisprudence have long recognized the right of employers to dismiss employees by reason of loss of trust and confidence. As provided for in the Labor Code, "Art. 282. An employer may terminate an employment for any of the following causes: x x x (c) Fraud or willful breach of the trust reposed in him by his employer or his duly authorized representative. x x x." In the case of supervisors or personnel occupying positions of responsibility, this Court has repeatedly held that loss of trust and confidence justifies termination. Obviously, as a just cause provided by law, this ground for terminating employment, springs from the voluntary or willful act of the employee, or "by reason of some blameworthy act or omission on the part of the employee".

Reasoning - Loss of confidence as a just cause for termination of employment is premised from the fact that an employee concerned holds a position of trust and confidence. But, in order to constitute a just cause for dismissal, the act complained of must be "work-related" such as would show the employee concerned to be unfit to continue working for the employer.- it must be noted the recent decisions of this Court has distinguished the treatment of managerial employees from that of rank-and-file personnel, insofar as the application of the doctrine of loss of trust and confidence is concerned. Thus with respect to rank-and-file personnel, loss of trust and confidence as ground for valid dismissal requires proof of involvement in the alleged events in question, and that mere uncorroborated assertion and accusations by the employer will not be sufficient. But, as regards as a managerial employee, mere existence of a basis for believing that such employee has breached the trust of his employer would suffice for his dismissal. Hence, in the case of managerial employees, proof beyond reasonable doubt is not required, it being sufficient that there is some basis for such loss of confidence, such as when the employer has reasonable ground to believe that the employee concerned is responsible for the purported misconduct, and the nature of his participation therein renders him unworthy of the trust and confidence demanded by his position.- In the present case, petitioner is not an ordinary rank-and-file employee. He is the EDP Supervisor tasked to directly supervise the installation of the PABX housewiring project in respondent company's premises. He should have realized that such sensitive position requires the full trust and confidence of his employer. Corollary, he ought to know that his job requires that he keep the trust and confidence bestowed on him by his employer unsullied. Disposition Petition is DISMISSED for lack of merit.

G. TERMINATION OF EMPLOYMENT BY EMPLOYEE

14.02 CAUSES

A. JUST CAUSES

B. WITHOUT JUST CAUSE

C. RESIGNATION

DEFINITION

HABANA V NLRC (HOTEL NIKKO)298 SCRA 537

KAPUNAN; November 16, 1998

NATUREPetition for certiorari seeking reversal of NLRC decision which affirmed LA

FACTS- On March 16, 1989, petitioner Antonio Habana was employed by Hotel Nikko Manila Garden (Nikko) as Rooms Division Director (RDD). One of his tasks as RDD was to conduct regular and surprise inspection of all work areas to ensure quality of performance. In the course of his employment, petitioner encountered several problems: his frequent clashes with Dolores Samson (his Senior Rooms Mgr); frequent absence and tardiness; rampant violations of hotel rules due to his failure to effectively manage his own division; and complaints regarding the overall quality (or lack thereof) of service of Nikko. As a result, private respondent Mr. Okawa, who replaced private respondent Mr. Yokoo as the executive asst. for Sales, issued a memorandum instructing petitioner, along with 2 others, to conduct and report daily inspection of the guestrooms and public areas. Petitioner sent a memorandum of protest claiming that Mr. Okawa’s orders was a form of harassment to “ease him out of his position” and illustrated in detail the other forms of alleged harassment supposedly perpetrated by Mr. Okawa. He, however, manifested that he had no intention to resign. - But on May 2, 1990, petitioner went to the Hotel’s Comptroller asking for his severance pay of P120,000 plus accrued benefits of P11, 865.28. The check was not given to him until he submitted his resignation letter (part of standard procedure). He also executed an Affidavit of Quitclaim, along with his resignation. The very next day, however, respondents received a letter from petitioner (addressed to Mr. Okawa) who insisted that he was forced to resign because he could no longer endure Mr. Okawa’s acts of harassment against him. 2 weeks later, petitioner filed a complaint for illegal dismissal and damages against Hotel Nikko and its officers, including his direct superiors, Yokoo and Okawa. The LA dismissed the complaint finding that petitioner voluntarily resigned and that the alleged acts of harassment were non-existent. On appeal, the NLRC affirmed the LA’s decision likewise finding that petitioner voluntarily resigned as manifested by his act of negotiating for a huge amount of separation pay. When his MFR was dismissed, he came to the SC.

ISSUEWON the resignation was forced upon Habana or he did so voluntarily

HELDThe resignation was voluntary. Ratio Voluntary resignation is the voluntary act of an employee who “finds himself in a situation where he believes that personal reasons cannot be sacrificed in favor of the exigency of the service and he has no other choice but to disassociate himself from his employment.” Reasoning - In this case, petitioner was clearly having trouble performing his job, which undeniably carries immense responsibilities. Notable too was petitioner’s failure to see eye to eye with his immediate bosses, Mr. Yokoo and Mr. Okawa. Because of these difficulties, it was quite reasonable for petitioner to think of, and eventually, relinquishing his position voluntarily (and get a fat sum as severance pay in the bargain) instead of waiting to be fired. - Petitioner laments that he was completely stripped of his powers and functions as Director when Mr. Okawa tasked him with inspecting the hotel’s guest and public areas. Conducting these daily inspections, in effect, demoted him to a mere room inspector “one notch higher than a bellboy.” He claims that the humiliation he endured in going room to room, inspecting toilets and garbage areas, was all part of a malicious scheme to harass him out of his position. These orders were not borne out of mere whim and caprice. They were made in response to the complaints they were getting. Moreover, these measures executed by the hotel’s top management were legitimate exercise of management prerogatives.

Labor Law 1 A2010 - 197 - Disini- Petitioner asserts that private respondents coerced and intimidated him to resigning through their collective acts of harassment. Contrariwise, private respondents contend that it was petitioner who approached them indicating his desire to resign due to his difficulty in coping with his responsibilities and his differences with his immediate boss, Mr. Okawa. - Petitioner could not have been intimidated by private respondents to quit. In his memorandum, petitioner emphatically vowed not to resign despite private respondents’ alleged acts of harassment. Surprisingly, however, after only a few days he did quit alleging that he was forced and harassed to do so. If petitioner was adamant in his intention not to be coerced into leaving, how could he suddenly be forced to resign? Petitioner glaringly contradicted himself. His excuse is thus, unbelievable and unjustifiable. - Moreover, the issue in this case is factual in nature and firm is the principle that “factual findings of the NLRC, particularly when they coincide with those of the LA, are accorded respect, even finality, and will not be disturbed for as long as such findings are supported by substantial evidence.” We have painstaking reviewed the records of this case and we find no justifiable reason to overturn the findings of both the LA and the NLRC. Disposition Petition is DISMISSED

REQUISITES

AZCOR MANUFACTURING V NLRC (CAPULSO)303 SCRA 26

BELLOSILLO; February 11, 1999

NATUREPetition for certiorari

FACTS- Candido Capulso filed with the Labor Arbiter a complaint for constructive illegal dismissal and illegal deduction of P50.00 per day for the period April to September 1989. The evidence presented by Capulso showed that he worked for AZCOR as ceramics worker for more than two (2) years starting from 3 April 1989 to 1 June 1991. From April to September 1989 the amount of P50.00 was deducted from his salary without informing him of the reason therefor.- In the second week of February 1991, upon his doctor's recommendation, Capulso verbally requested to go on sick leave due to bronchial asthma. It appeared that his illness was directly caused by his job as ceramics worker where, for lack of the prescribed occupational safety gadgets, he inhaled and absorbed harmful ceramic dusts. His supervisor, Ms. Emily Apolinaria, approved his request. Later, on 1 June 1991, Capulso went back to petitioner AZCOR to resume his work after recuperating from his illness. He was not allowed to do so by his supervisors who informed him that only the owner, Arturo Zuluaga, could allow him to continue in his job. He returned five (5) times to AZCOR but when it became apparent that he would not be reinstated, he immediately filed the instant complaint for illegal dismissal.- Capulso presented the following documentary evidence in support of his claim: (a) His affidavit and testimony to prove that he was terminated without just cause and without due process; (b) Identification card issued by AZCOR which he continued to use even after his supposed employment by Filipinas Paso; (c) Certification of SSS premium payments; (d) SSS Member Assistance Form wherein he stated that he worked with AZCOR from March 1989 to April 1991; (e) Certification of Employee Contribution with SSS; and, (f) Payslips issued by AZCOR.- AZCOR alleged that Capulso was a former employee of AZCOR who resigned on 28 February 1990 as evidenced by a letter of resignation and joined Filipinas Paso on 1 March 1990 as shown by a contract of employment; in February 1991 Capulso allegedly informed his supervisor, Ms. Emilia Apolinaria, that he intended to go on terminal leave because he was not feeling well; on 1 March 1991 he submitted a letter of resignation addressed to the President of Filipinas Paso, Manuel Montilla; and, in the early part of June 1991 Capulso tried to apply for work again with Filipinas Paso but there was no vacancy.- Petitioners submitted the following documentary evidence: (a) Sworn Statement of Ms. Emilia Apolinaria and her actual testimony to prove that respondent indeed resigned voluntarily from AZCOR to transfer to Filipinas Paso, and thereafter, from Filipinas Paso due to failing health; (b) Contract of Employment between Filipinas Paso and respondent which took effect 1 March 1991; (c) Letter of resignation of respondent from AZCOR dated 28 February 1990, to take effect on the same date; (d) Undated letter of resignation of respondent addressed to Filipinas Paso to take effect 1 March 1991; (e) BIR Form No. W-4 filed 6 June 1990; (f) Individual Income Tax Return of respondent for 1990; and, (g) BIR Form 1701-B which was an alphabetical list of employees of Filipinas Paso for the year ending 31 December 1990.- Labor Arbiter rendered a decision dismissing the complaint for illegal dismissal for lack of merit, but ordered AZCOR and/or Arturo Zuluaga to refund to Capulso P200.00 representing the amount illegally deducted from his salary.- NLRC modified the Labor Arbiter's decision by: (a) declaring the dismissal of Capulso as illegal for lack of just and valid cause; (b) ordering petitioners to reinstate Capulso to his former or equivalent position without loss of seniority rights and without diminution of benefits; and, (c) ordering petitioners to jointly and solidarily pay Capulso his back wages computed from the time of his dismissal up to the date of his actual reinstatement.

- Petitioners' motion for reconsideration was denied by the NLRC. Meanwhile, during the pendency of the case before this Court, Capulso succumbed to asthma and heart disease, and died.- Petitioners insist that Capulso voluntarily resigned. They also contend that they could not be held jointly and severally liable for back wages since AZCOR and Filipinas Paso are separate and distinct corporations with different corporate personalities; and, the mere fact that the businesses of these corporations are interrelated and both owned and controlled by a single stockholder are not sufficient grounds to disregard their separate corporate entities.

ISSUEWON NLRC erred in finding that Capulso was illegally dismissed and in holding petitioners jointly and solidarily liable to Capulso for back wages HELDNO- On resignation, requisitesRatio To constitute a resignation, it must be unconditional and with the intent to operate as such. There must be an intention to relinquish a portion of the term of office accompanied by an act of relinquishment. - The fact that Capulso signified his desire to resume his work when he went back to petitioner AZCOR after recuperating from his illness, and actively pursued his case for illegal dismissal before the labor courts when he was refused admission by his employer, negated any intention on his part to relinquish his job at AZCOR.- a closer look at the subject resignation letters readily reveals the following: (a) the resignation letter allegedly tendered by Capulso to Filipinas Paso was identically worded with that supposedly addressed by him to AZCOR; (b) both were pre-drafted with blank spaces filled up with the purported dates of effectivity of his resignation; and, (c) it was written in English, a language which Capulso was not conversant with considering his low level of education. No other plausible explanation can be drawn from these circumstances than that the subject letters of resignation were prepared by a person or persons other than Capulso. And the fact that he categorically disowned the signatures therein and denied having executed them clearly indicates that the resignation letters were drafted without his consent and participation. - Even assuming for the sake of argument that the signatures were genuine, the resignation letters still cannot be given credence in the absence of any showing that Capulso was aware that what he was signing then were in fact resignation letters or that he fully understood the contents thereof. - On illegal dismissal> In illegal dismissal cases, the onus of proving that the dismissal of the employee was for a valid and authorized cause rests on the employer, and failure to discharge the same would mean that the dismissal is not justified and therefore illegal.- On joint and several liability> The doctrine that a corporation is a legal entity or a person in law distinct from the persons composing it is merely a legal fiction for purposes of convenience and to subserve the ends of justice. This fiction cannot be extended to a point beyond its reason and policy. Where, as in this case, the corporate fiction was used as a means to perpetrate a social injustice or as a vehicle to evade obligations or confuse the legitimate issues, it would be discarded and the two (2) corporations would be merged as one, the first being merely considered as the instrumentality, agency, conduit or adjunct of the other.Disposition petition is DISMISSED. NLRC Decision is MODIFIED. Petitioners AZCOR MANUFACTURING, INC., FILIPINAS PASO and ARTURO ZULUAGA are ORDERED to pay, jointly and solidarily, the heirs of private respondent Candido Capulso the amounts representing his back wages, inclusive of allowances and other benefits, and separation pay to be computed in accordance with law.

METRO TRANSIT ORG V NLRC (GARCIA)284 SCRA 308

BELLOSILLO; January 16 1998

FACTS- Garcia had been working for Metro Transit (METRO) for almost 8 years as station teller. On April 22 1992, he called up his immediate supervisor if he could go on LOA to go to Cebu to look for his wife and children who suddenly left home without his knowledge. After a few weeks of fruitless search he returned to Manila. - When he reported to the office on May 15 1992 Garcia was not allowed to resume work but was directed to proceed to the legal department of METRO where he would undergo investigation. He was asked by Pili about his absence from work. After he explained to Pili his predicament, Pili cut short the inquiry and informed him right away that it would be better for him to resign rather than be terminated for his absences. Still in a state of extreme agitation and weighed down by a serious family problem, Garcia at once prepared a resignation letter. Then he left again for the province to look for his family. But like his first attempt his effort came to naught. Soon after the Personnel Committee of METRO approved his resignation.- Garcia sought advice from his labor union and asked that the union intervene in his case by bringing the matter of his forced resignation before their grievance machinery for arbitration. METRO rejected Garcia's plea that he be not considered resigned from his

Labor Law 1 A2010 - 198 - Disiniemployment. Garcia filed a complaint for illegal dismissal. Labor Arbiter and NLRC ruled in favor of Garcia.- Petitioner: private respondent absented himself on 22 April 1992 without official leave and then later on freely and willingly relinquished his employment because he was establishing his own business.

ISSUE WON Garcia resigned from his employment

HELDNO- An examination of the circumstances surrounding the submission of the letter indicates that the resignation was made without proper discernment so that it could not have been intelligently and voluntarily done.- What Pili did as petitioner's representative was to advise Garcia, who at that time was thoroughly confused and bothered no end by a serious family problem, that he had better resign or face the prospect of an unceremonious termination from service for abandonment of work. At that precise moment, the employee could not be said to have fully understood what he was doing, i.e., writing his resignation letter, nor could have foreseen the consequences thereof, for it is established that as soon as he came out of the investigation office he prepared his resignation letter right then and there at a table nearby with no time for reflection. It is noteworthy that shortly thereafter he consulted his union president for help regarding his forced resignation. This does not indicate by any means a resignation that was knowingly and voluntarily done. On the contrary , it shows that his writing and handing in the resignation letter to petitioner were a knee-jerk reaction triggered by that singular moment when he was left with no alternative but to accede, having been literally forced into it by being presented with the more unpleasant fate of being terminated.- the voluntariness of complaint's resignation can hardly be believed if he was not forced by circumstances due to the following:- First he was already in the employ of respondent for almost eight years with a high paying job and benefits; Second, no offense or violation has been attributed to the complainant during his period of employment; Third, the filing of this instant complaint by the complainant for illegal dismissal negates or is inconsistent with abandonment and voluntary resignation. Lastly, there is no iota of evidence that complainant is indeed engaged in business, and belies the contents of his resignation.- Evidently the complainant was asked to make a choice whether to tender his resignation or be terminated for his absences which to our mind is anchored on justifiable grounds. Such compulsion to make an unnecessary choice placed undue and unjustifiable pressure on the employee who otherwise would not have thought of leaving his position as Station Teller if he had not been induced to do so. This being the case, the resignation filed by the complainant did not become effective.

VOLUNTARY RESIGNATION

PHIL WIRELESS INC V NLRC (LUCILA)310 SCRA 653

PARDO; July 20, 1999

NATUREPetition for certiorari to set aside a decision of the NLRC

FACTS- January 8, 1976 – Phil. Wireless Inc. (Pocketbell) hired respondent Doldwin Lucila as an operator/encoder. Three years later, Lucila was promoted as Head Technical and Maintenance Department of the Engineering Department. On September 11, 1987, he was promoted as Technical Services Supervisor and later on October 1, 1990, he became Project Management Superintendent.- December 8, 1990 – Lucila tendered his resignation.- December 3, 1991 – Lucila filed with the NLRC a complaint for illegal/constructive dismissal. - Lucila alleges that his “promotion” from Technical Services Supervisor to Project Management Superintendent was actually a demotion because it was demeaning, illusory and humiliating. He based it on the fact that he was not given a secretary/assistant and subordinates.- June 29, 1992 – Labor Arbiter Villarente declared that Lucila actually resigned and dismissed the complaint for lack merit.- June 15, 1993 – NLRC reversed the findings of the Labor Arbiter and ordered for Lucila’s reinstatement with payment of backwages or separation pay.

ISSUEWON Lucila was constructively dismissed

HELDNORatio Constructive dismissal is an involuntary resignation resorted to when continued employment is rendered impossible, unreasonable or unlikely; when there is a demotion

in rank and/or diminution in pay; or when a clear discrimination, insensibility or disdain by an employer becomes unbearable to the employee.Reasoning- In this case, the Court ruled that Lucila voluntarily resigned and was not pressured into doing so.- Voluntary resignation is defined as the act of en employee who finds himself in a situation where he believes that personal reasons cannot be sacrificed in favor of the exigency of the service and he has no other choice but to disassociate himself from his employment.- Lucila’s basis for his “demotion” is inadequate as the Court ruled that there is no demotion where there is no reduction in position rank or salary as a result of such transfer. Disposition The petition is hereby granted. The questioned decision of the NLRC is set aside and the decision of the Labor Arbiter is reinstated and affirmed. No costs.

PASCUA V NLRC287 SCRA 554

PANGANIBAN; March 13, 1998

NATUREReview on certiorari

FACTS- The complainants are among the employees of Henry Lao at the Tiongsan Super Bazaar. On August 7, 1991, Henry Lao received a telephone call who informed him that one of his sales ladies had just stolen a Karaoke, the previous night. There, said saleslady made a confession, that, there were others who were involved in the stealing of goods. She was required by Henry Lao to write down their names. Violeta Soriano and Susan Castillo were included in her list. The eighteen (18) sales ladies who admitted their guilt resigned. The remaining workers were placed under the watchful eyes of respondent.- On August 21, 1991, Lilia Pascua was caught repairing three (3) pairs of pants that belonged to Mrs. Manaois and allegedly were not bought at the Tiongsan Super Bazaar. Respondent scolded Lilia Pascua for this offense, because it is against the respondent’s policy that repair jobs of items not bought at the bazaar should not be accepted. She was given a warning, that this prohibition should be strictly followed. Lilia Pascua did not report for work the next day. She went to see the respondent’s bookkeeper for the computation of her separation pay. Respondent paid her separation pay.- On August 24, 1991, Victoria Santos was caught charging a meter of a cloth for the price of a yard. For this offense, she was suspended for a period of thirty (30) days. She never returned to work since then.- Mimi Macanlalay was employed on June 10, 1989. Previously, she worked for Mrs. Tan. On September 19, 1991, Mrs. Tan went to the Tiongsan Super Bazaar, and she saw Mimi Macanlalay working as a cashier. Mrs. Tan informed Mr. Lao, that Mimi Macanlalay was previously dismissed by her for dishonesty. Mimi Macanlalay later on “resigned”.- Violeta Soriano was employed on May 16, 1984. After the August 7, 1991 incident, she was assigned as a cashier. She was reverted back as a sales lady after a few weeks when Mr. Lao learned, that, she had some knowledge of the schemes of the resigned employees. On November 9, 1991, Mr. Lao required her to explain in writing, why she should not be the subject matter of a disciplinary action, for her failure to fill up her daily time record. Respondent reviewed her past records and found out that, she was the subject matter of a disciplinary action in the past. She was terminated [sic] on December 8, 1991.- Susan De Castro refused to receive her salary on November 18, 1991, because she insisted on receiving more than what is indicated in the payrolls. Respondent told her that if she is not satisfied with her salary, she can find employment elsewhere. She failed to report for work on the following day. In any case, respondent states, that, she can be dismissed for lack of trust and confidence, for her involvement in the pilferage of goods.”Petitioners filed at the Regional Arbitration Branch of the NLRC separate complaints against Henry Lao for “illegal dismissal and claims for violation of labor standards pertaining to payment of wages.” Subsequently, the labor arbiter ruled that the dismissals were illegal and awarded back wages and separation pay to petitioners.- The NLRC, which modified the appealed decision and found the termination of petitioners’ employment to be due either to voluntary resignation or dismissals with just cause.

ISSUES1. WON petitioners’ employment terminated because of resignation, abandonment or dismissal2. WON petitioners’ employment terminated in accordance with law

HELD1. ILLEGAL DISMISSAL (except for Santos).- Petitioner Pascua was aware of the close relationship between Henry Lao and Mrs. Manaois. Thus, Pascua feared that, if she turned down Mrs. Manaois’ request, she would be subjected to public scolding by Lao. Thus, accommodation of the said request may have been an act of disobedience of her employer’s order, but hardly an instance of the “wrongful and perverse attitude” that would warrant a penalty as grave as dismissal. That

Labor Law 1 A2010 - 199 - Disiniafter the incident, Henry Lao kept pushing me by my shoulders as he repeatedly told me in a loud manner, ‘pakuwenta mo na ang separation pay mo at hindi ka na rin makakabalik. Puntahan mo ang accountant.’ which made me nervous and afraid especially that he kept on pushing me even when I was already on top of the stairs; It is evident from the above that Petitioner Pascua was forced to resign -- an act which was tantamount to a dismissal, an illegal one at that.- The NLRC could not explain the contradictions in Petitioner De Castro’s case. If she had not been dismissed but was still an employee of private respondent, then why did she file this case for illegal dismissal? And even more perplexing: Why would the NLRC conclude that reinstatement was no longer possible because of the parties’ “respective imputations of charges against each other”? Furthermore, the labor arbiter’s finding that there was no evidence on record to establish her dismissal is refuted by the uncontested allegations of Petitioner De Castro.- Prior to her employment at Tiongsan Super Bazaar Petitioner Macanlalay had been a saleslady at Rommel’s which was owned by a certain Mrs. Tan. On September 20, 1991, while she was working as a cashier at Tiongsan, Mrs. Tan saw her; thereupon, Mrs. Tan reported to Henry Lao that Petitioner Macanlalay had previously been dismissed for alleged dishonesty. Petitioner was then called by Lao and unceremoniously told: “Kunin mo na ang separation pay mo. Pa total mo na sa accountant. At huwag ka ng magtrabajo dito.” Clearly, she did not resign; she was orally dismissed by Lao. It is this lack of clear, valid and legal cause, not to mention due process, that made her dismissal illegal, warranting reinstatement and the award of back wages.- The NLRC justified Petitioner Soriano’s dismissal by alleging that it was due to her failure to make regular entries in her daily time records. We believe, however, that this alleged “just cause” was convincingly disputed by Petitioner Soriano in her letter dated November 9, 1991.- We agree that Petitioner Santos voluntarily resigned. The labor arbiter did not find Petitioner Santos to have been illegally dismissed. Rather, after her suspension for “charging for a meter of cloth bought [at] the price of a yard,” she offered to resign. The solicitor general supports this by stating that “even the Labor Arbiter discovered this when he ruled that ‘there [was] no evidence on record to support Santos’ dismissal.’”2. NOReasoning- Basic is the doctrine that resignation must be voluntary and made with the intention of relinquishing the office, accompanied with an act of relinquishment. Based on the evidence on record, we are more than convinced that Petitioners Lilia Pascua, Mimi Macanlalay, Susan C. De Castro and Violeta Soriano did not voluntarily quit their jobs. Rather, they were forced to resign or were summarily dismissed without just cause. Petitioners -- except Victoria L. Santos -- forthwith took steps to protest their layoff and thus cannot, by any logic, be said to have abandoned their work.- In labor cases, the employer has the burden of proving that the dismissal was for a just cause; failure to show this, as in the instant case, would necessarily mean that the dismissal was unjustified and, therefore, illegal. To allow an employer to dismiss an employee based on mere allegations and generalities would place the employee at the mercy of his employer; and the right to security of tenure, which this Court is bound to protect, would be unduly emasculated. Considering the antecedents in the summary dismissals effected against Petitioners Pascua, Macanlalay, De Castro and Soriano, the causes asserted by private respondent are, at best, tenuous or conjectural; at worst, they are mere afterthoughts. - Under the Labor Code, as amended, the dismissal of an employee which the employer must validate has a twofold requirement: one is substantive, the other procedural. Not only must the dismissal be for a just or an authorized cause as provided by law (Articles 282, 283 and 284 of the Labor Code, as amended); the rudimentary requirements of due process -- the opportunity to be heard and to defend oneself -- must be observed as well.- Petitioners Pascua and Macanlalay’s acceptance of separation pay did not necessarily amount to estoppel; nor did it connote a waiver of their right to press for reinstatement, considering that such acceptance -- particularly by Petitioner Pascua who had to feed her four children -- was due to dire financial necessity.Disposition REVERSED.

AZCOR MANUFACTURING INC V NLRC[PAGE 197]

VALDEZ V NLRC (NELBUSCO INC)286 SCRA 87

REGALADO; February 9, 1998

NATURESpecial civil action for certiorari

FACTS- Sometime in December, 1986, petitioner was hired by private respondent as a bus driver on commission basis, with an average earning of P6,000.00 a month. On February 28, 1993, the airconditioning unit of the bus which petitioner was driving suffered a mechanical breakdown. Respondent company told him to wait until the airconditioning unit was repaired. Meanwhile, no other bus was assigned to petitioner to keep him gainfully employed.

- Thereafter, petitioner continued reporting to his employer's office for work, only to find out each time that the airconditioning unit had not been repaired. Several months elapsed but he was never called by respondent company to report for work. Later, petitioner found out that the bus formerly driven by him was plying an assigned route as an ordinary bus, with a newly-hired driver.- On June 15, 1993, petitioner filed a complaint against private respondent for illegal dismissal, with money claims for labor standard benefits, and for reimbursement of his bond and tire deposit. He claimed that the reason why respondent company did not allow him to drive again was due to his refusal to sign an undated company-prepared resignation letter and a blank affidavit of quitclaim and release.- Private respondent, on the other hand, admitted that it told petitioner to wait until the airconditioning unit of the bus was repaired. However, private respondent alleged that after the bus driven by the petitioner broke down due to his fault and negligence, the latter did not report for work. He supposedly informed the management later that he was voluntarily resigning from his employment in order to supervise the construction of his house. Consequent to his resignation, petitioner demanded the return of his cash bond and tire deposit. Respondent company required him to secure the necessary management clearance and other pertinent papers relative to his resignation. Instead of complying with those requirements, petitioner filed the instant complaint.

ISSUEWON petitioner was illegally dismissed because he did not voluntarily resigned as claimed by respondents

HELD- The reason for the stoppage of operation of the bus assigned to petitioner was the breakdown of the airconditioning unit, which is a valid reason for the suspension of its operation. However, such suspension regarding that particular bus should likewise last only for a reasonable period of time. The period of six months was more than enough for it to cause the repair thereof. Beyond that period, the stoppage of its operation was already legally unreasonable and economically prejudicial to herein petitioner who was not given a substitute vehicle to drive.- The so-called "floating status" of an employee should last only for a legally prescribed period of time. When that "floating status" of an employee lasts for more than six months, he may be considered to have been illegally dismissed from the service. Thus, he is entitled to the corresponding benefits for his separation, and this would apply to the two types of work suspension heretofore noted, that is, either of the entire business or of a specific component thereof.- It was not denied by private respondent that it tried to force private respondent to sign an undated company-prepared resignation letter and a blank undated affidavit of quitclaim and release which the latter validly refused to sign. Furthermore, the bus which petitioner used to drive was already plying a transportation route as an ordinary bus and was being driven by another person, without petitioner having been priorly offered the same alternative arrangement.- The other allegation of private respondent that petitioner voluntarily resigned from work obviously does not deserve any consideration. It would have been illogical for herein petitioner to resign and then file a complaint for illegal dismissal. Resignation is inconsistent with the filing of the said complaint.- Resignation is defined as the voluntary act of an employee who finds himself in a situation where he believes that personal reasons cannot be sacrificed in favor of the exigency of the service, and, that he has no other choice but to disassociate himself from his employment. Resignation is a formal pronouncement of relinquishment of an office. It must be made with the intention of relinquishing the office accompanied by an act of relinquishment. - The cardinal rule in termination cases is that the employer bears the burden of proof to show that the dismissal is for just cause, failing in which it would mean that the dismissal is not justified. This rule applies adversely against herein respondent company since it has utterly failed to discharge that onus by the requisite quantum of evidence.- Under Article 279 of the Labor Code, as amended, an employee who is unjustly dismissed from work shall be entitled to reinstatement without loss of seniority rights and other privileges and to his full back wages, inclusive of allowances, and to other benefits or their monetary equivalent computed from the time his compensation was withheld from him up to the time of his actual reinstatement.Disposition Decision of respondent National Labor Relations Commission is SET ASIDE and the decision of the Labor Arbiter REINSTATED

VALIDITY OF POLICY

MANILA BROADCASTING COMPANY V NLRC (OLAIREZ, BANGLOY)

294 SCRA 486MENDOZA; 1998

NATUREPetition for certiorari to set aside the decision of the National Labor Relations Commission, affirming the decision of the Labor Arbiter which found private respondent to have been illegally dismissed and which ordered him reinstated with damages.

Labor Law 1 A2010 - 200 - DisiniFACTS- Private respondent Samuel L. Bangloy was production supervisor and radio commentator of the DZJC-AM radio station in Laoag City. The radio station is owned by petitioner Manila Broadcasting Company.- On February 28, 1992, private respondent applied for leave of absence for 50 days, from March 24 to May 13, 1992, in order to “run for Board Member” in Ilocos Norte under the Kilusang Bagong Lipunan (KBL). He made his application pursuant to §11(b) of R.A. No. 6646 which provides:

Sec. 11(b) . . . Any mass media columnist, commentator, announcer, or personality who is a candidate for any elective public office shall take a leave of absence from his work as such during the campaign period.

- After a week, private respondent’s application was returned to him, together with a copy of an office memorandum of Eugene Jusi, Assistant Vice-President for Personnel and Administration, to Atty. Edgardo Montilla, Executive Vice-President and General Manager of the FJE Group of Companies, in which it was stated that as a matter of “company policy,” any employee who files a certificate of candidacy for any elective national or local office would be considered resigned from the company. - It would appear that private respondent nonetheless ran in the election but lost. On May 25, 1992, he tried to return to work, but was not allowed to do so by petitioner on the ground that his employment had been terminated.- Private respondent filed a complaint for illegal dismissal against petitioner before the Department of Labor and Employment.

ISSUES1. WON the company policy that any employee who files a certificate of candidacy for any elective national or local office would be considered resigned from the company valid2. WON the company policy was made known to employees before it was sought to be applied to private respondent

HELD1. YES- the policy is valid and justified.2. NO- There are a number of circumstances which raise some doubts whether the company policy was strictly enforced.Ratio Although §11(b) of R.A. No. 6646 does not require mass media commentators and announcers such as private respondent to resign from their radio or TV stations but only to go on leave for the duration of the campaign period, we think that the company may nevertheless validly require them to resign as a matter of policy.- The policy is justified on the following grounds:1) Working for the government and the company at the same time is clearly disadvantageous and prejudicial to the rights and interest not only of the company but the public as well. In the event an employee wins in an election, he cannot fully serve, as he is expected to do, the interest of his employer. The employee has to serve two (2) employers, obviously detrimental to the interest of both the government and the private employer.2) In the event the employee loses in the election, the impartiality and cold neutrality of an employee as broadcast personality is suspect, thus readily eroding and adversely affecting the confidence and trust of the listening public to employer’s station. These are valid reasons for petitioner. No law has been cited by private respondent prohibiting a rule such as that in question.Disposition Decision AFFIRMED

14.03 NO TERMINATION – PERFORMANCE OF MILITARY OR CIVIC DUTY

C. TERMINATION OF EMPLOYMENT BY EMPLOYER

1. PRELIMINARY MATTERS

14.04 BASIS OF RIGHT AND REQUIREMENTS

BASIS

GUTIERREZ V SINGER SEWING MACHINE411 SCRA 512

QUISUMBING; September 3, 2003

NATUREreview is the decision of the Court of Appeals

FACTS- Petitioner Mario Gutierrez was initially hired by Singer Sewing Machine Company as Audit Assistant on contractual basis in 1993. He became an Accounts Checker on probationary status on February 8, 1994. Thereafter, he acquired regular status as Asset Auditor on March 1, 1995, receiving a monthly salary of P4,455, until September 9, 1996, when he was dismissed from employment. Singer premised the petitioner’s termination on the following incidents:- On August 1, 1996, at around 3:15 p.m., Ms. Emelita Garcia, Personnel Supervisor of Singer, caught Gutierrez and three other Asset Auditors, watching a video tape inside the Asset/Legal Department Office. Despite Ms. Garcia’s reminder that it was no longer break time and that the other occupants of the room might be disturbed, Gutierrez and company ignored Ms. Garcia and continued to watch the video. The following day, August 2, 1996, Ms. Evangeline Que-Ilagan, Administration Manager of Singer, noticed a sign posted at the door of the Asset/Legal Department Office, which read “MAIPARIT TI UMISBO DITOY.” When she asked who placed the sign at the door, Gutierrez admitted responsibility. When Ms. Que-Ilagan asked what it meant, Gutierrez answered, “BAWAL ANG UMIHI DITO” (No Urinating Here). Ms. Que-Ilagan then asked if Gutierrez had seen anyone urinate at the door where the sign was posted and the latter replied in the negative. Ms. Que-Ilagan then asked why he placed such a sign, to which Gutierrez replied, “Gusto ko, eh” (It is my pleasure). She admonished him not to do the same thing again and requested him to remove the sign, but Gutierrez refused to do so.- Later that same day, August 2, 1996, Gutierrez personally explained his side to the Asset Manager, Mr. Leonardo Consunji, at the latter’s office. Gutierrez claimed that he only admitted to the posting of the sign in order to take the cudgels for a co-employee. He also explained that their use of the video equipment was upon the orders of their supervisor, Mr. Romeo C. Ninada. The latter wanted to test the quality of their video players. Mr. Consunji brought the matter to the attention of Mr. Ninada. The latter promptly issued a Memo dated August 6, 1996, requiring Gutierrez to explain his side. Gutierrez then informed Mr. Ninada that he had already discussed the matter with Mr. Consunji. In his letter to Mr. Consunji dated August 21, 1996, Mr. Ninada opined, “[T]he case does not deserve to be devoted with too much time and effort” as he considered it a “minor offense.”- Nevertheless, Mr. Consunji issued a Memo dated August 28, 1996, informing Gutierrez of the latter’s violation of company rules and regulations, specifically citing the following:> Part V-B.9 Use of Company’s time, materials, equipment and other assets for personal use or business; and > Part V-B.18 Acts of vandalism such as defacing or destroying Company documents and records; posting, altering or removing any printed matter, announcements or signs in the Bulletin Boards unless specifically authorized. - Under the Company Code of Discipline, these infractions were classified as 4 th Degree Offenses with the corresponding sanction of dismissal. In the same Memo, Gutierrez was directed to explain in writing why the aforesaid penalty should not be imposed on him. He was given until August 30, 1996, to comply with the directive. As Gutierrez insisted that he had previously verbally explained his side to Mr. Consunji, no written explanation was submitted by him. - On September 9, 1996, another Memo was issued by Mr. Consunji, worded as follows:> After a thorough investigation of the incident and after having found your explanations to be unsatisfactory and due to your refusal to comply with my memo to you dated August 28, 1996 which constitutes willful defiance or disregard of Company authority, the management deems it fitting and proper to impose upon you the penalty of dismissal effective immediately upon receipt hereof. - On September 19, 1996, petitioner filed a motion/request for reconsideration with Singer, but the latter stood pat on its decision to dismiss him. - Thus, petitioner filed the complaint for illegal dismissal with claims for damages before the Labor Arbiter, docketed as NLRC NCR Case No. 00-10-06201-96. In a decision dated August 13, 1997, Labor Arbiter Renato A. Bugarin dismissed the complaint for lack of merit.- Aggrieved, Singer filed a petition for certiorari with this Court, which in turn was referred, by resolution dated December 2, 1998, to the Court of Appeals.The Court of Appeals reversed the NLRC, thereby upholding and reinstating the decision of the Labor Arbiter. Gutierrez now comes to the Court via a petition for review on certiorari seeking to reverse and set aside the decision of the Court of Appeals, with a prayer for moral damages and attorney’s fees.

ISSUEWON the appellate court erred in reversing the NLRC which declared respondents guilty of illegal dismissal of the petitioner from his employment

HELD YESRatio We agree with the NLRC that petitioner’s dismissal from employment was unjustified and illegal. Petitioner’s dismissal was based on his alleged violation of two company rules and regulations, namely: (1) acts of vandalism; and (2) use of company’s time, materials, equipment and other assets for personal use/business. These acts were found by the Labor Arbiter to constitute serious misconduct or willful disobedience under paragraph (a) of Article 282 of the Labor Code. The Labor Arbiter characterized Gutierrez’ “undesirable or unreasonable behavior and unpleasant deportment with his

Labor Law 1 A2010 - 201 - Disinifellow employees, all the more his supervisors,” as within the scope of the analogous just causes for termination under paragraph (e) of the same article.- Singer averred that petitioner’s defiance of the reasonable rules and regulations being implemented by Singer was enough reason for his dismissal. Singer emphasized that the two violations of company rules and regulations on the two consecutive days, were manifestations that petitioner was “challenging the authorities of Singer.”In its impugned decision, however, the NLRC stated:- We agree with the complainant that the questioned poster contained an innocuous and harmless statement, which when translated in tagalog means “Bawal Umihi Dito” and that such posting cannot be interpreted as an act of vandalism. The affidavit of Ms. Ilagan, in relation with such poster, is not sufficient to establish complainant’s guilt of vandalism.…The complainant likewise justified his action in relation to his act of watching video films during office hours by arguing that he, together with four (4) other co-employees, were asked by their immediate supervisor, Mr. Romy Ninada to test the video tape player. Such claim was not denied by Mr. Ninada, who could have been easily required by the respondents to do so. Mr. Ninada was the logical officer to negate the claim of the complainant that he was authorized to test the quality of the VHS and CTV 143 to guarantee the excellency (sic) of respondent firm’s products.- Though no admission was made that the use of the video player was upon the orders of the immediate supervisor of Gutierrez, Mr. Ninada himself considered the same to be a minor infraction, not worth the time and effort of the company spent on the matter. - We might add that, as contended by petitioner, the act of posting the sign does not fall squarely within the scope of the cited company rules and regulations, Part V-B.18, on vandalism. The rule prohibits unauthorized posting “in the Bulletin Board,” while the present case involved posting of a sign at one of the office doors, a different matter. We must also stress that, even on the assumption that Gutierrez in fact committed the cited infractions, in our view they are not major violations but only minor ones which do not merit the supreme penalty of dismissal from employment. Time and again, this Court has underscored the need for restraint in the dismissal of workers:- Extreme caution should be exercised in terminating the services of a worker for his job may be the only lifeline on which he and his family depend for survival in these difficult times. That lifeline should not be cut off except for a serious, just and lawful cause, for, to a worker, the loss of his job may well mean the loss of hope for a decent life for him and his loved ones.- In the present case, the penalty of dismissal appears in our view unjustified, much too harsh and quite disproportionate to the alleged infractions. Not only were the alleged violations minor in nature, in this case the evidence adduced to prove them did not fairly show they fall exactly within the rules and regulations allegedly violated. Otherwise stated, the evidence did not square fully with the charges. That is why the Labor Arbiter found only “analogous” causes which, in our view do not sufficiently justify the extreme penalty of termination. - The penalty imposed on the erring employee ought to be proportionate to the offense, taking into account its nature and surrounding circumstances. In the application of labor laws, the courts and other agencies of the government are guided by the social justice mandate in our fundamental law. - To be lawful, the cause for termination must be a serious and grave malfeasance to justify the deprivation of a means of livelihood. This is merely in keeping with the spirit of our Constitution and laws which lean over backwards in favor of the working class, and mandate that every doubt must be resolved in their favor.- To conclude, the Court of Appeals erred in reversing the decision of the NLRC which declared respondents guilty of illegal dismissal.

MANILA TRADING AND SUPPLY CO INC V ZULUETA69 PHIL 485

LAUREL; January 30, 1940

NATUREPetition for Certiorari

FACTS- On July 7, 1938, the Secretary of Labor apprised the Court of Industrial Relations of a labor dispute existing between the petitioner company and its employees who were members of the Philippine Labor Union- A preliminary hearing was held after which, on August 6, 1938 the respondent court entered an order requiring the company, inter alia not to dismiss any of its employees and laborers except for good cause and with its permission. - Subsequently, on June 30, 1939, one of the gatekeepers of the petitioners, Filomeno Ramollo, was suspended for a breach of duty. The breach consisted in that as gatekeeper of the petitioner he permitted, contrary to instructions, one of the customers to pass thru the exit gate without paying for the work done on the car. Before this, it is also alleged that he refused to work in the setting up department of the company when ordered by his superior. - The Philippine Labor Union submitted a petition requesting the reinstatement of the suspended laborer, to which an answer was filed by the company. - In its order of July 28, 1939, the respondent court found that the laborer was guilty of the breach imputated to him, but, deciding that his suspension from June 30 to July 28, 1939 was a sufficient punishment, ordered his immediate reinstatement.

- The petitioner moved for reconsiderations, but the respondent Court of Industrial Relations, sitting in banc, denied the motion.

ISSUEWON the Court of Industrial Relations can order the readmission of a laborer who has been found derelict in the performance of his duties

HELDNO- The right of an employer to freely select or discharge his employees, is subject to regulation by the State. An employer cannot legally be compelled to continue with the employment of a person who admittedly was guilty of misfeasance or malfeasance towards his employer, and whose continuance in the service of the latter is patently inimical to his interest. The law, in protecting the rights of the laborer, authorizes neither oppression nor self-destruction of the employer. There may, of course, be cases where the suspension or dismissal of an employee is whimsical or unjustified or otherwise illegal scrutinized carefully and the proper authorities will go to the core of the controversy and not close their eyes to the real situation.Disposition Writ of Certiorari granted

AGABON V NLRC[PAGE 35]

PLDT V TOLENTINO438 SCRA 555

CORONA; September 21, 2004

FACTS - Arturo R. Tolentino Tolentino was employed in petitioner PLDT for 23 years. - He started in 1972 as an installer/helper and, at the time of his termination in 1995, was the division manager of the Project Support Division, Provincial Expansion Center, Meet Demand Group. - His division was in charge of the evaluation, recommendation and review of documents relating to provincial lot acquisitions. Sometime in 1995, Jonathan de Rivera, a supervisor directly under respondent Tolentino, was found to have entered into an “internal arrangement” with the sellers of a parcel of land which he recommended for acquisition under PLDT’s expansion program. Quirino Donato, the attorney-in-fact of the landowner, executed an affidavit disclosing his “internal arrangement” with de Rivera. - Donato’s affidavit revealed that all follow-up calls regarding the transaction were to be directed to the office of respondent and de Rivera. Upon being apprised of this “internal arrangement,” PLDT dismissed de Rivera. After he was dismissed, de Rivera submitted a sworn statement to PLDT implicating respondent as the person behind the anomalous “internal arrangement.” Respondent, in an affidavit, denied this and pointed out that his authority to approve real estate acquisitions was limited to land valued below P200,000.- Petitioner PLDT sent a notice of dismissal, effective October 27, 1995, to respondent Tolentino. Attached to this notice was a handwritten note from Nicanor E. Sacdalan, Vice-President of the Provincial Expansion Center, Meet Demand Group, giving respondent Tolentino the option to resign. Petitioner did not grant respondent’s request for a formal hearing but delayed the implementation of his dismissal. On December 4, 1995, petitioner informed respondent that his dismissal was already final and effective on December 5, 1995.- Respondent then filed a complaint for illegal dismissal, moral and exemplary damages and other monetary claims against petitioner PLDT in January, 1996. The labor arbiter found that petitioner PLDT failed to prove and substantiate the charges against respondent - On appeal, the NLRC reversed the labor arbiter’s decision on the ground that respondent was a managerial employee and that loss of trust and confidence was enough reason to dismiss him.- Respondent’s petition for certiorari was referred by this Court to the Court of Appeals which rendered the assailed decision reinstating the decision of the labor arbiter, that is, ordering respondent’s reinstatement.

ISSUEWON the Court of Appeals erred in ruling that the dismissal was not founded on clearly established facts sufficient to warrant separation from employment

HELD NO- The petition is without merit. PLDT’s basis for respondent’s dismissal was not enough to defeat respondent’s security of tenure.- There is no dispute over the fact that respondent was a managerial employee and therefore loss of trust and confidence was a ground for his valid dismissal. The mere existence of a basis for the loss of trust and confidence justifies the dismissal of the employee because:

[w]hen an employee accepts a promotion to a managerial position or to an office requiring full trust and confidence, she gives up some of the rigid guaranties available to ordinary workers. Infractions which if committed by others would be overlooked or

Labor Law 1 A2010 - 202 - Disinicondoned or penalties mitigated may be visited with more severe disciplinary action. A company’s resort to acts of self-defense would be more easily justified.

- Proof beyond reasonable doubt is not required provided there is a valid reason for the loss of trust and confidence, such as when the employer has a reasonable ground to believe that the managerial employee concerned is responsible for the purported misconduct and the nature of his participation renders him unworthy of the trust and confidence demanded by his position. - However, the right of the management to dismiss must be balanced against the managerial employee’s right to security of tenure which is not one of the guaranties he gives up. This Court has consistently ruled that managerial employees enjoy security of tenure and, although the standards for their dismissal are less stringent, the loss of trust and confidence must be substantial and founded on clearly established facts sufficient to warrant the managerial employee’s separation from the company. Substantial evidence is of critical importance and the burden rests on the employer to prove it. Due to its subjective nature, it can easily be concocted by an abusive employer and used as a subterfuge for causes which are improper, illegal or unjustified. - In the case at bar, this Court agrees with the Court of Appeals that the petitioner’s dismissal was not founded on clearly established facts sufficient to warrant separation from employment. The factual findings of the court a quo on the issue of whether there was sufficient basis for petitioner PLDT to dismiss respondent Tolentino are binding on this Court. In the exercise of the power of review, the factual determinations of the Court of Appeals are generally conclusive and binding on the Supreme Court. - The evidence relied upon by petitioner PLDT — de Rivera’s sworn statement and Donato’s affidavit — does not, in our view, establish respondent Tolentino’s complicity in the “internal arrangement” engineered by his subordinate de Rivera. - To be sure, respondent Tolentino was remiss in his duties as division manager for failing to discover the “internal arrangement” contrived by his subordinate. However, dismissal was not the proper sanction for such negligence. It was not commensurate to the lapse committed, especially in the light of respondent’s unblemished record of long and dedicated service to the company. In Hongkong Shanghai Bank Corporation vs. NLRC, we had occasion to rule that:

The penalty imposed must be commensurate to the depravity of the malfeasance, violation or crime being punished. A grave injustice is committed in the name of justice when the penalty imposed is grossly disproportionate to the wrong committed. [D]ismissal is the most severe penalty an employer can impose on an employee. It goes without saying that care must be taken, and due regard given to an employee’s circumstances, in the application of such punishment.

- Certainly, a great injustice will result if this Court upholds Tolentino’s dismissal.An employee illegally dismissed is entitled to full backwages and reinstatement pursuant to Article 279 of the Labor Code, as amended by RA 6715.

- Although a managerial employee, respondent should be reinstated to his former position or its equivalent without loss of seniority rights inasmuch as the alleged strained relations between the parties were not adequately proven by petitioner PLDT which had the burden of doing so. In Quijano vs. Mercury Drug Corporation, the Court ruled that strained relations are a factual issue which must be raised before the labor arbiter for the proper reception of evidence. In this case, petitioner PLDT only raised the issue of strained relations in its appeal from the labor arbiter’s decision. Thus, no competent evidence exists in the records to support PLDT’s assertion that a peaceful working relationship with respondent Tolentino was no longer possible. In fact, the records of the case show that PLDT, through VP Sacdalan, gave respondent Tolentino the option to resign. [18] Such a deferential act by management makes us doubt PLDT’s claim that its relations with respondent were “strained.” The option to resign would not have been given had animosity existed between them.- Furthermore, respondent was dismissed in December, 1995 when petitioner PLDT was still under the Cojuangco group. PLDT has since then passed to the ownership and control of its new owners, the First Pacific group which has absolutely nothing to do so with this controversy. Since there are no strained relations between the new management and respondent, reinstatement is feasible.Disposition The petition was denied.

PEREZ V MEDICAL CITY GENERAL HOSPITAL484 SCRA 138

AZCUNA; March 6, 2006

NATUREPetition for certiorari

FACTS- September 9, 1999:Prompted by reports of missing medicines and supplies in the Emergency Room/Trauma Room (ER/TR) and upon the suggestion of one of the Hospital’s staff nurses, Medical City General Hospital, opened 22 lockers of employees

assigned to the ER/TR. The Hospital found four lockers with items belonging to it. The employees corresponding to the lockers (Dominador Perez, Celine Campos, Lailanie Espiritu and Mateo Butardo) were directed to submit written explanations as to why these items were inside their lockers.- Perez, Campos and Butardo submitted their written explanations, while Espiritu opted to resign. An administrative hearing was held where the three employees who responded were represented by a union counsel. At the end of the proceedings, the charge against Butardo was dismissed while Perez and Campos, herein petitioners, were found to have violated category seven of the company rules, a serious infraction meriting dismissal. The Hospital offered them the opportunity to voluntarily resign with separation pay, under a clause provided in the Collective Bargaining Agreement. They refused and the Hospital dismissed them from the service.- January 19, 2000: petitioners filed a complaint for illegal dismissal with the NLRC.- Labor Arbiter found respondents guilty of illegal dismissal and ordered the reinstatement of petitioners with backwages and without loss of seniority rights. NLRC reversed the Labor Arbiter’s decision and dismissed the complaint. CA affirmed. Hence, this petitiom.- Petitioners maintain that they have sufficiently accounted for the presence of these items inside their lockers and that the evidence presented against them is insufficient to show that they are guilty of misappropriating company property. Moreover, assuming ex gratia argumenti that there was violation of company rules, the penalty of dismissal would be too harsh considering their long years of dedicated service to the Hospital.

ISSUES1. WON there was sufficient basis to hold that petitioners misappropriated hospital property2. WON dismissal was the appropriate penalty

HELD1. YES- The Supreme Court is not a trier of facts, and this rule applies with greater force in labor cases. Hence, the factual findings of the NLRC are generally accorded not only respect but even finality if supported by substantial evidence and especially when affirmed by the CA. However, a disharmony between the factual findings of the Labor Arbiter and the NLRC opens the door to a review by this Court.- Contrary to the position taken by the Labor Arbiter, the Hospital’s dismissal of petitioners did not rest on speculative inferences. Petitioners themselves have admitted that properties belonging to the Hospital were found inside their lockers. As to how these items got inside the lockers, petitioners acknowledged having placed them there against company rules. In view of these admissions, there is ample evidence to support a charge for pilferage unless petitioners can satisfactorily explain their possession. - It was made clear to all hospital staff that hospital equipment should only be kept in the supplies locker. 2. NO- The power to dismiss an employee is a recognized prerogative that is inherent in the employer’s right to freely manage and regulate his business. An employer cannot be expected to retain an employee whose lack of morals, respect and loyalty to his employer or regard for his employer’s rules and appreciation of the dignity and responsibility of his office has so plainly and completely been bared. An employer may not be compelled to continue to employ a person whose continuance in service will patently be inimical to his interest. The dismissal of an employee, in a way, is a measure of self-protection. - Nevertheless, whatever acknowledged right the employer has to discipline his employee, it is still subject to reasonable regulation by the State in the exercise of its police power. Thus, it is within the power of this Court not only to scrutinize the basis for dismissal but also to determine if the penalty is commensurate to the offense, notwithstanding the company rules.- In this case, the Court agrees with the Labor Arbiter that dismissal would not be proportionate to the gravity of the offense considering the circumstances present in this case. During Perez and Campos' long tenure (19 and 7 years, respectively) with the Hospital, it does not appear that they have been the subject of disciplinary sanctions and they have kept their records unblemished. Moreover, the Court also takes into account the fact that petitioners are not managerial or confidential employees in whom greater trust is placed by management and from whom greater fidelity to duty is correspondingly expected. - The reinstatement of petitioners is in line with the social justice mandate of the Constitution. Nevertheless, the Court does not countenance the wrongful act of pilferage but simply maintains that the extreme penalty of dismissal is not justified and a lesser penalty would suffice. Under the facts of this case, suspension would be adequate. Without making any doctrinal pronouncement on the length of the suspension in cases similar to this, the Court holds that considering petitioners’ non-employment since January 2000, they may be deemed to have already served their period of suspension. Consequently, the Labor Arbiter’s order of reinstatement is upheld, with the deletion of the award of backwages, so as not to put a premium on acts of dishonesty. Disposition Petition partially granted.

REQUIREMENTS

Labor Law 1 A2010 - 203 - DisiniSUBSTANTIVE AND PROCEDURAL DUE PROCESS

FUJITSU COMPUTER PRODUCTS OF THE PHILS V CA (DE GUZMAN, ALVAREZ)

454 SCRA 737CALLEJO SR; April 8, 2000

NATUREA petition for review assailing the Decision of the Court of Appeals in reversing the decision of the National Labor Relations Commission (NLRC).

FACTS- Petitioner Fujitsu Computer Products Corporation of the Philippines (FCPP) is a corporation organized and existing under Philippine laws engaged in the manufacture of hard disc drives, MR heads and other computer storage devices for export.- Respondent Victor de Guzman began working for FCPP on September 21, 1997 as Facilities Section Manager. As of 1999, he was also holding in a concurrent capacity the position of Coordinator ISO 14000 Secretariat. Allan Alvarez, on the other hand, was employed as a Senior Engineer on April 21, 1998. He was assigned at the Facilities Department under the supervision of respondent De Guzman.- The garbage and scrap materials of FCPP were collected and bought by the Saro’s Trucking Services and Enterprises (Saro’s). On January 15, 1999, respondent De Guzman as Facilities Section Manager, for and in behalf of FCPP, signed a Garbage Collection Agreement with Saro’s, and the latter’s signatory therein was its owner and general manager, Larry Manaig.- De Guzman served as middleman between Sta. Rosa Bible Baptist Church and Saro. The Church was looking for scrap metal, and was willing to buy the purlins at P3. The scrap metal was then delivered from FCPP to Sta. Rosa Bible Baptist Church.- Ernesto Espinosa, HRD and General Affairs Director of FCPP, received a disturbing report from Manaig. Manaig reported that respondent De Guzman had caused the “anomalous disposal of steel [purlins] owned by FCPP.” Two of Manaig’s employees, Roberto Pumarez and Ma. Theresa S. Felipe, executed written statements detailing how respondent De Guzman had ordered the steel purlins to be brought out. Thereafter, petitioner Espinosa sent a two-page Inter-Office Memorandum dated July 24, 1999 to respondent De Guzman, effectively placing him under preventive suspension.- On July 28, 1999, respondent Alvarez sent an e-mail message to his co-employees, expressing sympathy for the plight of respondent De Guzman. Respondent Alvarez used a different computer, but the event viewer system installed in the premises of petitioner FCPP was able to trace the e-mail message to him. Respondent Alvarez submitted a written Explanation dated September 29, 1999 where he apologized, readily admitted that he was the sender of the e-mail message in question, and claimed that he “acted alone with his own conviction.” He alleged, however, that he was only expressing his sentiments, and that he was led by his desire to help a friend in distress.- Respondent Alvarez was informed that his services were terminated on the ground of serious misconduct effective August 13, 1999. Respondent De Guzman’s employment was, thereafter, terminated effective August 23, 1999 through an Inter-Office Memorandum. - The respondents then filed a complaint for illegal dismissal against the petitioners with prayer for reinstatement, full backwages, damages and attorney’s fees before the NLRC. Labor Arbiter Antonio R. Macam ruled in favor of FCPP, stating that it was justified in terminating the employment of the respondents. According to the Labor Arbiter, respondent De Guzman, a managerial employee, was validly dismissed for loss of trust and confidence. Citing a number of cases,[24] the Labor Arbiter stressed that where an employee holds position of trust and confidence, the employer is given wider latitude of discretion in terminating his services for just cause.- The NLRC sustained the ruling of the Labor Arbiter and dismissed the respondents’ appeal for lack of merit. The NLRC also affirmed the Labor Arbiter’s finding that respondent De Guzman, a managerial employee who was routinely charged with the custody and care of the petitioner’s property, was validly dismissed on the ground of willful breach of trust and confidence. In so far as the dismissal of respondent Alvarez was concerned, the Commission held that the circumstances surrounding the sending of the clearly “malicious and premeditated e-mail message” constituted no less than serious misconduct. Hence, respondent Alvarez’s dismissal was also justified under the circumstances.- The CA reversed the ruling of the NLRC and held that the respondents were illegally dismissed. According to the appellate court, the non-payment of the scrap steel purlins by the Sta. Rosa Bible Baptist Church (Sta. Rosa) to Saro’s was not a valid cause for the dismissal of respondent De Guzman. Contrary to the findings of the Labor Arbiter, respondent De Guzman did not betray the trust reposed on him by his employer, as the transaction involving the sale of scrap steel purlins was between Sta. Rosa and Saro’s. Anent the dismissal of respondent Alvarez, the CA ruled that his act of “sympathizing and believing in the innocence of respondent De Guzman and expressing his views” was not of such grave character as to be considered serious misconduct which warranted the penalty of dismissal.

ISSUES1. WON De Guzman is guilty of breach of confidence, thus warranting dismissal2. WON Alvarez committed serious misconduct in sending the e-mail

HELD1. NO- De Guzman is not guilty of breach of confidence.Ratio To be a valid ground for dismissal, loss of trust and confidence must be based on a willful breach of trust and founded on clearly established facts. A breach is willful if it is done intentionally, knowingly and purposely, without justifiable excuse, as distinguished from an act done carelessly, thoughtlessly, heedlessly or inadvertently. It must rest on substantial grounds and not on the employer’s arbitrariness, whims, caprices or suspicion; otherwise, the employee would eternally remain at the mercy of the employer. In order to constitute a just cause for dismissal, the act complained of must be work-related and shows that the employee concerned is unfit to continue working for the employer.Reasoning- The term “trust and confidence” is restricted to managerial employees. In this case, it is undisputed that respondent De Guzman, as the Facilities Section Manager, occupied a position of responsibility, a position imbued with trust and confidence.- The Court had the occasion to reiterate in Nokom v. National Labor Relations Commission the guidelines for the application of the doctrine of loss of confidence:Loss of confidence should not be simulated;> It should not be used as a subterfuge for causes which are improper, illegal or unjustified;> It may not be arbitrarily asserted in the face of overwhelming evidence to the contrary; and> It must be genuine, not a mere afterthought to justify earlier action taken in bad faith.- The scrap metals, including the steel purlins, were already classified as scrap materials and ready for disposal. No less than the written statements of the witnesses for the petitioners confirm this.- No fraud or bad faith could be attributed to respondent De Guzman, as evinced by his readiness to disclose his participation in the transaction between Saro’s and Sta. Rosa.- Loss of trust and confidence as a just cause for termination of employment is premised on the fact that the employee concerned is invested with delicate matters, such as the handling or care and protection of the property and assets of the employer. After such scrap materials are weighed, loaded onto a truck and carried out of the company premises, the petitioner FCPP can no longer be considered the owner thereof, and ceases to exercise control over such property. In this case however, Saro’s, as the new owner of the scrap materials in question, including the steel purlins, was free to contract with anyone as it wished.- A condemnation of dishonesty and disloyalty cannot arise from suspicions spawned by speculative inferences. Because of its subjective nature, this Court has been very scrutinizing in cases of dismissal based on loss of trust and confidence because the same can easily be concocted by an abusive employer. Thus, when the breach of trust or loss of confidence theorized upon is not borne by clearly established facts, as in this case, such dismissal on the ground of loss of confidence cannot be allowed.2. NO- Alvarez did not commit serious misconduct in sending the e-mail.Ratio Misconduct has been defined as improper or wrong conduct. It is the transgression of some established and definite rule of action, a forbidden act, a dereliction of duty, willful in character, and implies wrongful intent and not mere error of judgment. The misconduct to be serious must be of such grave and aggravated character and not merely trivial and unimportant.

Reasoning- For misconduct or improper behavior to be a just cause for dismissal, (a) it must be serious; (b) must relate to the performance of the employee’s duties; and (c) must show that the employee has become unfit to continue working for the employer.- The Court finds that respondent Alvarez’s act of sending an e-mail message as an expression of sympathy for the plight of a superior can hardly be characterized as serious misconduct as to merit the penalty of dismissal.- There is no showing that the sending of such e-mail message had any bearing or relation on respondent Alvarez’s competence and proficiency in his job. To reiterate, in order to consider it a serious misconduct that would justify dismissal under the law, the act must have been done in relation to the performance of his duties as would show him to be unfit to continue working for his employer.Disposition Petition is denied. Decision of the CA is affirmed, with costs against the petitioners.

ARIOLA V PHILEX MINING CORP446 SCRA 514

CARPIO; August 9, 2005

NATUREPetition for review of the decision of the CA finding the retrenchment of the petitioners to be valid

FACTS- Petitioners are former supervisors of respondent Philex Mining Corp. Philex sustained financial losses in its operations and adopted several measures including reducing personnel through early voluntary retirement and retrenchment programs to save costs.

Labor Law 1 A2010 - 204 - DisiniThe labor union representing the rank-and-file employees and the union representing the supervisory employees signed a MOA with Philex prescribing the criteria for retrenchment.- Petitioners, with 6 other supervisors and 49 rank-and-file employees, received from Philex termination notices informing them of their retrenchment. Philex paid them separation pay, and all of them signed Deeds of Release and Quitclaim in Philex’s favor. Claiming that Philex dismissed them illegally, these supervisors and rank-and-file employees separately submitted for voluntary arbitration the legality of their separation from service.The rank-and-file employees’ case- The rank-and-file employees’ case was referred to Arbitrator Valdez. Valdez ruled in the employees’ favor, declared their dismissal illegal, and ordered their reinstatement. He held that Philex failed to prove its claim of financial losses and that the criteria for retrenchment in the rank-and-file’s MOA were arbitrary and inconsistent with the CBA then in force. The CA reversed Valdez’s finding on Philex’s financial condition and held that Philex had a valid reason to undertake retrenchment. Nevertheless, the appellate court affirmed Valdez’s ruling that Philex is liable for illegal dismissal because the criteria for retrenchment in the rank-and-file’s MOA were inequitable. Philex further appealed to this Court, which denied Philex’s petition.The supervisory employees’ case- The supervisors’ case was referred to Arbitrator Advincula, who issued an order to reinstate petitioners and their co-complainants, after Philex failed to timely file its Position Paper. On Philex’s motion, Advincula admitted Philex’s Position Paper and “Supplementary” Position Paper. He rendered judgment finding “sufficient basis or just cause” for Philex to undertake a retrenchment.Advincula also held that petitioners were barred from questioning their separation from service because they availed of the early retirement program and executed the Deeds of Release and Quitclaim releasing Philex from further liability. Petitioners appealed to the CA, which denied the petition for lack of merit. The appellate court no longer ruled on the validity of Philex’s retrenchment program because it treated its decision in the rank-and-file employees’ case as the law of the case on that issue.

ISSUES1. WON petitioners retired or whether Philex dismissed them from service2. WON petitioners’ dismissal was illegal

HELD1. NORatio If the intent to retire is not clearly established or if the retirement is involuntary, it is to be treated as a discharge.Reasoning- Although there is no dispute that petitioners received varied amounts denominated as “retirement gratuity,” the records show that Philex paid these amounts because of petitioners’ retrenchment. Under Philex’s Retirement Gratuity Plan, “retirement gratuity” is paid not only to retiring employees but also to those who, like petitioners, are dismissed for cause “beyond their control” such as retrenchment. Philex treated the “retirement gratuity” as petitioners’ basic separation pay as indicated in Deeds of Release and Quitclaims petitioners signed. Significantly, Philex paid petitioners such separation pay after notifying them of their retrenchment.Obiter - In the letter addressed to petitioner Biete, Roxas of Philex Retirement Trust informed Biete that he was entitled to receive “retirement gratuity” because his separation, as a result of the retrenchment program, is for cause beyond his control. Biete submitted Roxas’ letter to the CA after that court had rendered its decision. However, at that time, petitioners did not yet file their MFR. Considering the import of the letter, it was error for the CA not to have considered the letter in resolving petitioners’ MFR. There can be no denial of due process where the party claiming to be aggrieved is the one who is guilty of not disclosing to the court the vital document that contains the most conclusive evidence regarding the matter in dispute. Philex cannot feign ignorance of this letter.2. YESRatio A substantive defect invalidates a dismissal because the ground for dismissal is negated by such defect, rendering the dismissal without basis.Reasoning- Philex’s financial condition justified petitioners’ retrenchment. What Philex failed to do was implement its retrenchment program in a just and proper manner. Its failure to use a reasonable and fair standard in the computation of the supervisors’ demerits points is not merely a procedural but a substantive defect which invalidates petitioners’ dismissal. When the defect is procedural, the dismissal remains valid because the basis of the dismissal is not in any way affected by such defect.Disposition The petition is GRANTED. The decision of the CA is SET ASIDE. We ENTER another judgment finding petitioners to have been illegally dismissed and ordering Philex to reinstate petitioners with full backwages, provided that the amounts petitioners received shall be deducted therefrom. If reinstatement is no longer possible, Philex shall pay backwages as computed above plus separation pay.

PHILIPPINE NATIONAL BANK V CABANSAG460 SCRA 514

PANGANIBAN; June 21, 2005

NATUREPetition for review on certiorari

FACTS- Florence Cabansag arrived in Singapore as a tourist. She applied for the Singaore branch of PNB. At that time, PNB had 2 types of employees: 1) employees hired in Manila and assigned in Singapore 2) locally hired.- Ruben Tobias, the general manager of the bank, found her qualified and recommended her to the President of the bank in Manila. The latter approved- Cabansag then applied for an Employment pass with the Ministry of Manpower of the Government of Singapore. She was issued said pass.- On December 7, 1998, she was offered a temporary appointment, as Credit Officer, wherein she was to be on probation for 3 months. Cabansag accepted the position and assumed office. In the meantime, the Philippine Embassy in Singapore processed the employment contract of Florence O. Cabansag and, on March 8, 1999, she was issued by the Philippine Overseas Employment Administration, an ‘Overseas Employment Certificate,’ certifying that she was a bona fide contract worker for Singapore- On April 15, 1999, she was asked to resign. Tobias said that it was a cost cutting measure. He likewise said that the PNB branch would be transformed into a remittance office. Cabansag then asked Tobias that she be furnished with a ‘Formal Advice’ from the PNB Head Office in Manila. However, Tobias flatly refused. Cabansag did not submit any letter of resignation. - On April 16,1999, Tobias again demanded that she submit a resignation letter. She was warned that he will be dismissed if she does not. Cabansag asked for more time in order for her to look for another job. Cabansag said that she should be out by May15, 1999. - However, on April 19, 1999, Tobias again asked that Cabansag submit her letter of resignation. Cabansag refused. The next day she was terminated. - NLRC ruled in favor of Cabansag. CA affirmed.

ISSUES1. WON the NLRC has jurisdiction over the case at bar2. WON the arbitration of the NLRC in the National Capital Region is the most convenient venue or forum to hear and decide the instant controversy3. WON Cabansag was illegally dismissed

HELD1. YES- As enunciated in A217 of the Labor Code, labor arbiters clearly have original and exclusive jurisdiction over claims arising from employer-employee relations, including termination disputes involving all workers, among whom are overseas Filipino workers- When Cabansag obtained an employment pass from the Singapore Ministry of Manpower, it did not imply a waiver of one’s national labor laws. The permit only grants one a status as a worker in the issuing country. She also applied for an Overseas Employment Certificate from the POEA through the Philippine Embassy in Singapore. This entitles her to all benefits and processes under our statutes- Moreover, petitioner admits that it is a Philippine corporation doing business through a branch office in Singapore. Significantly, respondent’s employment by the Singapore branch office had to be approved by Benjamin P. Palma Gil,[19] the president of the bank whose principal offices were in Manila. This circumstance militates against petitioner’s contention that respondent was “locally hired”; and totally “governed by and subject to the laws, common practices and customs” of Singapore, not of the Philippines. Instead, with more reason does this fact reinforce the presumption that respondent falls under the legal definition of migrant worker. 2. YES- The law gives her two choices: (1) at the Regional Arbitration Branch (RAB) where she resides or (2) at the RAB where the principal office of her employer is situated3. YES- Cabansag was already a regular employee at the time she was terminated, since her 3 months probationary period has already ended. - The twin requirements of notice and hearing constitute the essential elements of procedural due process, and neither of these elements can be eliminated without running afoul of the constitutional guarantee- In dismissing employees, the employer must furnish them two written notices: 1) one to apprise them of the particular acts or omissions for which their dismissal is sought; and 2) the other to inform them of the decision to dismiss them. As to the requirement of a hearing, its essence lies simply in the opportunity to be heard.- Respondent was not notified of the specific act or omission for which her dismissal was being sought. Neither was she given any chance to be heard, as required by law. At any rate, even if she were given the opportunity to be heard, she could not have defended herself effectively, for she knew no cause to answer to- All that petitioner tendered to respondent was a notice of her employment termination effective the very same day, together with the equivalent of a one-month pay. This Court has already held that nothing in the law gives an employer the option to substitute the required prior notice and opportunity to be heard with the mere payment of 30 days’ salary.- Moreover, Articles 282,[26] 283[27] and 284[28] of the Labor Code provide the valid grounds or causes for an employee’s dismissal. The petitioner has not asserted any grounds as a valid reason for terminating the employment of respondent

Labor Law 1 A2010 - 205 - DisiniDisposition Petition denied

GENUINO ICE CO INC V MAGPANTAY493 SCRA 195

AUSTRIA-MARTINEZ; June 27, 2006

NATUREReview on certiorari

FACTS- Alfonso Magpantay (respondent) was employed as a machine operator with Genuino Ice Company, Inc. (petitioner). On November 18, 1996, respondent filed against petitioner a complaint for illegal dismissal with prayer for moral and exemplary damages. In his Position Paper, respondent alleged that he was dismissed from service effective immediately by virtue of a memorandum, after which he was not allowed anymore to enter the company premises. Respondent bewailed that his termination from employment was done without due process.Petitioner countered that he was not illegally dismissed, since the dismissal was based on a valid ground, i.e., he led an illegal strike at petitioner’s sister company, Genuino Agro Industrial Development Corporation, which lasted from November 18 to 22, 1995, resulting in big operation losses on the latter’s part. Petitioner also maintained that respondent’s dismissal was made after he was accorded due process.- Petitioner initially claimed that respondent’s acts were tantamount to serious misconduct or willful disobedience, gross and habitual neglect of duties, and breach of trust. Subsequently, petitioner amended its position paper to include insubordination among the grounds for his dismissal, since it came out during respondent’s cross-examination, and the matter was reported only after the new personnel manager assumed his position in August 1996.- Labor Arbiter of the National Labor Relations Commission (NLRC) dismissed the case for lack of merit finding that petitioner had valid cause to dismiss respondent. Labor Arbiter’s Decision affirmed. Motion for reconsideration of the NLRC Decision was denied. Special civil action for certiorari with the CA was filed. Petitioner filed its Comment, contending that the petition was filed out of time, considering that contrary to respondent’s claim that the NLRC Resolution dated August 31, 1999 was received on December 20, 1999, it was actually received on September 15, 1999, as shown in the registry return card. Petitioner also reiterated its arguments that respondent was dismissed for cause and with due process.- CA rendered the assailed Decision granting the petition and declaring respondent’s dismissal as illegal. Petitioner filed a motion for reconsideration which the CA denied. ISSUES1. WON the petition was filed by petitioner out of time2. WON he was illegally dismissed (and on what ground)3. WON there was due process under Section 2 (d), Rule 1, Book VI of the Omnibus Rules Implementing the Labor Code provides for the standards of due process

HELD1. NO- The New Rules of Procedure of the NLRC provides the rule for the service of notices and resolutions in NLRC cases, to wit:

Sec. 4. Service of notices and resolutions. – a) Notices or summons and copies of orders, resolutions or decisions shall be served on the parties to the case personally by the bailiff or the duly authorized public officer within three (3) days from receipt thereof by registered mail; Provided, that where a party is represented by counsel or authorized representative, service shall be made on such counsel or authorized representative;

- The presumption is that the decision was delivered to a person in his office, who was duly authorized to receive papers for him, in the absence of proof to the contrary. It is likewise a fundamental rule that unless the contrary is proven, official duty is presumed to have been performed regularly and judicial proceedings regularly conducted, which includes the presumption of regularity of service of summons and other notices. The registry return of the registered mail as having been received is prima facie proof of the facts indicated therein. Thus, it was necessary for respondent to rebut that legal presumption with competent and proper evidence. Records show that Ducut is not an employee of the FEU Legal Aid Bureau, but is connected with the Computer Services Department. The FEU Legal Aid Bureau has its own personnel which include Ms. dela Paz who is the one authorized to receive communications in behalf of the office. It has been ruled that a service of a copy of a decision on a person who is neither a clerk nor one in charge of the attorney’s office is invalid. The CA was correct in ruling that the reckoning period should be the date when respondent’s counsel actually received the NLRC Resolution dated August 31, 1999, which was on December 20, 1999. Petitioner, however, pointed out that a certain Ruby D.G. Sayat received a copy of their Motion for Reconsideration filed by registered mail on August 16, 2000. Respondent contended that at the time Sayat received the motion, she was then detailed at the office and was authorized to receive said pleading, and that it was an isolated and exceptional instance. On this matter, the FEU Acting Postmaster certified that Sayat is a permanent employee of the FEU Legal Aid Bureau. As such, she is authorized to receive communications in behalf of the office and need not possess an express authority to do so. More importantly, the Court has consistently frowned upon the dismissal of an appeal on

purely technical grounds. While the right to appeal is a statutory, not a natural right, it is, nonetheless, an essential part of our judicial system. Courts should proceed with caution so as not to deprive a party of the right to appeal, but rather, ensure amplest opportunity for the proper and just disposition of a cause, free from the constraints of technicalities. 2. NO, on the ground of habitual neglect of duties but YES on the ground of insubordination. The Court sustained the CA’s finding that respondent’s four-day absence does not amount to a habitual neglect of duty; however, the Court found that respondent was validly dismissed on ground of willful disobedience or insubordination.- FOR HABITUAL NEGLECT OF DUTY: Neglect of duty, to be a ground for dismissal, must be both gross and habitual. Gross negligence connotes want of care in the performance of one’s duties. Habitual neglect implies repeated failure to perform one’s duties for a period of time, depending upon the circumstances. On the other hand, fraud and willful neglect of duties imply bad faith on the part of the employee in failing to perform his job to the detriment of the employer and the latter’s business. Thus, the single or isolated act of negligence does not constitute a just cause for the dismissal of the employee. Thus, the Court agrees with the CA that respondent’s four-day absence is not tantamount to a gross and habitual neglect of duty. As aptly stated by the CA, “(W)hile he may be found by the labor courts to be grossly negligent of his duties, he has never been proven to be habitually absent in a span of seven (7) years as GICI’s employee. The factual circumstances and evidence do not clearly demonstrate that petitioner’s [respondent] absences contributed to the detriment of GICI’s operations and caused irreparable damage to the company.”- FOR INSUBORDINATION OR WILLFUL DISOBEDIENCE: On this point, the CA opined that petitioner included insubordination as a “mere after-thought.” It noted that petitioner seemed to be “irresolute” in stating the cause of respondent’s dismissal, as in its Position Paper, it originally relied on respondent’s four-day absence or participation in the illegal strike as a cause for dismissal but later on amended its Position Paper to include insubordination. Thus, the CA did not make any factual finding or conclusion in its Decision vis-à-vis petitioner’s allegation of respondent’s insubordination. While its perception may be true, it should not have deterred the CA from making any resolution on the matter. For one, respondent was able to argue against petitioner’s allegation of insubordination before the Labor Arbiter and the NLRC. For another, it was respondent himself who raised the subject before the CA, wherein he stated in his Petition. Further, the proceedings before the Labor Arbiter and the NLRC are non-litigious in nature. As such, the proceedings before it are not bound by the technical niceties of the law and procedure and the rules obtaining in courts of law, as dictated by Article 221 of the Labor Code:

ART. 221. Technical rules not binding and prior resort to amicable settlement. – In any proceeding before the Commission or any of the Labor Arbiters, the rules of evidence prevailing in courts of law or equity shall not be controlling and it is the spirit and intention of this Code that the Commission and its members and the Labor Arbiters shall use every and all reasonable means to ascertain the facts in each case speedily and objectively and without regard to technicalities of law or procedure, all in the interest of due process. This rule applies equally to both the employee and the employer. In the interest of due process, the Labor Code directs labor officials to use all reasonable means to ascertain the facts speedily and objectively, with little regard to technicalities or formalities. What is essential is that every litigant is given reasonable opportunity to appear and defend his right, introduce witnesses and relevant evidence in his favor, which undoubtedly, was done in this case. Willful disobedience, or insubordination as otherwise branded in this case, as a just cause for dismissal of an employee, necessitates the concurrence of at least two requisites: (1) the employee's assailed conduct must have been willful, that is, characterized by a wrongful and perverse attitude; and (2) the order violated must have been reasonable, lawful, made known to the employee and must pertain to the duties which he had been engaged to discharge. Company policies and regulations are generally valid and binding on the parties and must be complied with until finally revised or amended, unilaterally or preferably through negotiation, by competent authority. For misconduct or improper behavior to be a just cause for dismissal, the same must be related to the performance of the employee’s duties and must show that he has become unfit to continue working for the employer. In the case at bench, petitioner informed respondent, through a Memorandum dated November 14, 1995, that he was being transferred to its GMA, Cavite operations effective November 20, 1995.

- Due to his refusal to report to the Cavite plant, petitioner reiterated its order transferring respondent in its Memorandum dated November 24, 1995, where respondent was also warned that his failure to report to the Cavite plant will be considered as an absence without leave (AWOL) and insubordination. Respondent was required to comply with the order within 24 hours from receipt, otherwise, disciplinary action will be imposed on respondent. Respondent replied with a request that he remain in the Otis plant since a transfer to the Cavite plant will entail additional expenditure and travel time on his part. Petitioner again wrote respondent inviting him to appear before the Plant Level Investigation on December 11, 1995 for the latter to be able to clarify his reasons for refusing the transfer. Finally, petitioner issued its Memorandum dated December 12, 1995 informing respondent of its decision to terminate his services. The rule is that the transfer of an employee ordinarily lies within the ambit of the employer’s prerogatives. The employer exercises the prerogative to transfer an employee for valid reasons and according to the requirement of its business, provided the transfer does not result in demotion in rank or diminution of the employee’s salary, benefits and other privileges. In this case, petitioner’s order for respondent to transfer to the GMA, Cavite Plant is a reasonable and lawful order was made known to him and pertains to his duties as a machine operator. There was no demotion involved or diminution of salary, benefits and

Labor Law 1 A2010 - 206 - Disiniother privileges, and in fact, petitioner was even willing to provide respondent with monetary allowance to defray whatever additional expenses he may incur with the transfer. Such being the case, respondent cannot adamantly refuse to abide by the order of transfer without exposing himself to the risk of being dismissed. Hence, his dismissal was for just cause in accordance with Article 282 (a) of the Labor Code. Consequently, respondent is not entitled to reinstatement or separation pay and backwages.3. YES- Simply stated, the employer must furnish the employee a written notice containing a statement of the cause for termination and to afford said employee ample opportunity to be heard and defend himself with the assistance of his representative, if he so desires, and the employee must be notified in writing of the decision dismissing him, stating clearly the reasons therefor. - The CA found that petitioner failed to observe the twin requirements of notice and hearing, stating that its Memorandum dated December 13, 1995 does not squarely meet the standards of due process. The circumstances surrounding respondent’s dismissal, however, prove the contrary. The CA failed to take into account that prior to the Memorandum dated December 13, 1995, petitioner sent respondent several memoranda apprising him of the possible implications of his refusal to comply with the order of transfer. Thus, in its Memorandum dated November 24, 1995, petitioner notified respondent that his continued non-compliance with the order of transfer might bring about disciplinary action. Respondent replied to this memorandum, stating the reasons for his refusal, i.e., additional expenses, longer travel time, and union concerns. Petitioner sent another Memorandum on December 9, 1995, asking respondent to appear on December 11, 1995, for further clarification of his reasons for refusing the transfer. Despite the meeting, and since respondent, apparently, stubbornly refused to heed petitioner’s order, it was then that the Memorandum dated December 13, 1995 was issued to respondent informing him of the management’s decision to terminate his services. Clearly, respondent’s right to due process was not violated.Disposition petition is GRANTED. The CA Decision dated August 3, 2000 and Resolution dated March 16, 2001 are SET ASIDE, and the NLRC Decision dated June 30, 1999 is REINSTATED.

14.05 JUST CAUSES – SUBSTANTIVE DUE PROCESS – GROUNDS FOR TERMINATION

A. SERIOUS MISCONDUCT

DEFINITION AND ACTS

VALIAO V CA[PAGE 11]

VILLAMOR GOLF CLUB V PEHID472 SCRA 36

CALLEJO; October 4, 2005

NATUREPetition for review on certiorari of CA decision

FACTS- Rodolfo Pehid was employed by the Villamor Golf Club (VGC) as an attendant in the men’s locker room, and, thereafter, he became the Supervisor-in-Charge. His subordinates included Superal, Parilla, Mendoza, Velasquez, Casabon, Buenaventura and Modelo. Pehid and these employees agreed to establish a common fund from the tips they received from the customers, guests and members of the club for their mutual needs and benefits. Each member was to contribute the amount of P100 daily. The contributions of the employees had reached the aggregate amount of P17,990 based on the logbook maintained in the locker room. This agreement was not known to the VGC management.- An audit of the Locker Room Section of the golf club was conducted stating, among others, that based on the information relayed, there was an undeclared and unrecorded aggregate amount of P17,990 for the fund from May ‘98 to October ‘98. Further, not one in the said section admitted custody of such amount and there was no record that the money had been distributed among those employed in the locker room. In said report, Capuyan recommended that an investigation be conducted to determine the whereabouts of said amount and who was accountable therefor.- After the requisite formal investigation by the Administrative Board of Inquiry, Pehid received order that his employment was terminated. Based on its findings, Pehid committed gross misconduct in the performance of his duties in violation of Paragraph IV-E(d) of the VGC Rules and Regulations. He was also informed that he committed acts of dishonesty which caused and tend to cause prejudice to the club for misappropriating the common fund of P17,990.00 for his personal benefit.

- Pehid filed a complaint for illegal dismissal, unfair labor practice, separation pay/retirement benefits, damages and attorney’s fees against petitioners VGC. LA ruled in favor of Pehid saying that his dismissal was illegal. NLRC set aside and reversed the decision of LA. - CA set aside and reversed NLRC decision. The CA declared that Paragraph IV-E(a) and (d) of the VGC Rules1 expressly provide that the funds referred to therein are funds of the club and that the P17,990 did not form part of such fund but belonged to the locker room personnel. The CA also declared that the management of the VGC had no personal knowledge about the funds and, in fact, had not sanctioned its existence. Moreover, VGC was not prejudiced by the loss of the fund. Hence, this petition by VGC.Petitioners’ contentions: > That when confronted with the letter-complaint against him, Pehid admitted that his accountability arose from the proceeds of the sale of the golf club and golf shares entrusted to him, which he used for his personal needs without the knowledge of the persons concerned; > That there is substantial evidence that Pehid was the custodian of fund belonging to the members of the locker room and that his misappropriation of the same constituted gross misconduct; > That it is an act of manifest dishonesty within the context of Paragraph IV-E(d) of the Rules of Conduct of the club, in relation to A282(e) of the Labor Code, tending to prejudice the VGC> That, based on the substantial evidence Pehid misappropriated the fund as his co-employees in the locker room even positively identified him as the custodian thereof; and > that Pehid’s failure to account for and distribute the common fund which the locker personnel had established for their mutual aid and benefit is a manifest dishonesty falling within the scope of the proviso Respondent’s arguments:> That he was dismissed without just cause and due process of law; > that there was no basis or evidence to show that he had custody of the common fund which was used for his own benefit; > that he incurred the ire of his superiors for testifying in support of Tansiongco, a former Director of Personnel who was dismissed by VGC; and > that one of Tansiongco’s accusers was the brother of Velasquez, one of the locker boys who complained against him.

ISSUES1. WON CA decision is contrary to law and jurisprudence and therefore reversible2. WON the incident of the case shall fall within the provision of Article 282 paragraph (e) of the Labor Code

HELD1. NO- Company policies and regulations are, unless shown to be grossly oppressive or contrary to law, generally valid and binding and must be complied with by the parties unless finally revised or amended, unilaterally or preferably through negotiation. However, while an employee may be validly dismissed for violation of a reasonable rule or regulation adopted for the conduct of the company’s business, an act allegedly in breach thereof must clearly and convincingly fall within the express intendment of such order.- The CA was correct in ruling that the NLRC had overlooked and misapplied certain facts and circumstances of substance, which, if properly appreciated, would affect the disposition of the case. - There’s no doubt that funds alleged to have been embezzled by the petitioner, belonged to the personnel of respondent VGC and not to respondent VGC. Under the afore-quoted VGC rule (see footnote), the dishonesty of an employee to be a valid cause for dismissal must relate to or involve the misappropriation or malversation of the club funds, or cause or tend to cause prejudice to VGC. The substantial evidence on record indicates that the P17,990, which was accumulated from a portion of the tips given by the golfers from May 1998 to October 1998 and was allegedly misappropriated by the respondent as the purported custodian thereof, did not belong to VGC but to the forced savings of its locker room personnel. Hence, VGC was not prejudiced. So it is within law and jurisprudence that CA reversed NLRC ruling.2. NORatio The principle in statutory construction of ejusdem generis: Where general words follow an enumeration of persons or things, by words of a particular and specific meaning, such general words are not to be construed in their widest extent, but are to be held as applying only to persons or things of the same kind or class as those specifically mentioned.Reasoning- Based on the grounds of termination provided under A282 of the Labor Code and the VGC Rules and Regulations, the common denominator thereof to constitute gross misconduct as a ground for a valid termination of the employee, is that – it is committed in connection with the latter’s work or employment. In the instant case, as previously pointed out, the alleged petitioner’s misappropriation or malversation was committed, assuming it to be true, against the common funds of the Locker Room personnel, which

1 E. Dishonesty

1. The following shall constitute violation of this section.a) Misappropriation or malversation of Club funds.d) All other acts of dishonesty which cause or tend to cause prejudice to VGC

Labor Law 1 A2010 - 207 - Disinidid not belong nor sanctioned by respondent VGC. A fortiori, respondent VGC was not prejudiced or damaged by the loss or misappropriation thereof. Obiter- Important for our purposes in the outline: “Serious misconduct” as a valid cause for the dismissal of an employee is defined as improper or wrong conduct; the transgression of some established and definite rule of action, a forbidden act, a dereliction of duty, willful in character, and implies wrongful intent and not mere error in judgment. To be serious within the meaning and intendment of the law, the misconduct must be of such grave and aggravated character and not merely trivial or unimportant. However serious such misconduct, it must be in connection with the employee’s work to constitute just cause for his separation. The act complained of must be related to the performance of the employee’s duties such as would show him to be unfit to continue working for the employer.Disposition Petition is DENIED for lack of merit. CA decision AFFIRMED.

LAKPUE V BELGA473 SCRA 617

YNARES-SANTIAGO; October 20, 2005

FACTS- Petitioner Tropical Biological Phils., Inc. (Tropical), a subsidiary of Lakpue Group of Companies, hired on March 1, 1995 respondent Ma. Lourdes Belga (Belga) as bookkeeper and subsequently promoted as assistant cashier. On March 19, 2001, Belga brought her daughter to the Philippine General Hospital (PGH) for treatment of broncho-pneumonia. On her way to the hospital, Belga dropped by the house of Marylinda O. Vegafria, Technical Manager of Tropical, to hand over the documents she worked on over the weekend and to give notice of her emergency leave. - While at the PGH, Belga who was pregnant experienced labor pains and gave birth on the same day. On March 22, 2001, or two days after giving birth, Tropical summoned Belga to report for work but the latter replied that she could not comply because of her situation. On March 30, 2001, Tropical sent Belga another memorandum ordering her to report for work and also informing her of the clarificatory conference scheduled on April 2, 2001. Belga requested that the conference be moved to April 4, 2001 as her newborn was scheduled for check-up on April 2, 2001. When Belga attended the clarificatory conference on April 4, 2001, she was informed of her dismissal effective that day.

ISSUEWON Belga was illegally dismissed

HELDYES- Tropical terminated Belga on the following grounds: (1) Absence without official leave for 16 days; (2) Dishonesty, for deliberately concealing her pregnancy; (3) Insubordination, for her deliberate refusal to heed and comply with the memoranda sent by the Personnel Department on March 21 and 30, 2001 respectively- Tropical cites the following paragraphs of Article 282 of the Labor Code as legal basis for terminating Belga:

Article 282. Termination by employer. — An employer may terminate an employment for any of the following causes:(a) Serious misconduct or willful disobedience by the employee of the lawful orders of his employer or representative in connection with his work;…(c) Fraud or willful breach by the employee of the trust reposed in him by his employer or duly authorized representative

- We have defined misconduct as a transgression of some established and definite rule of action, a forbidden act, a dereliction of duty, willful in character, and implies wrongful intent and not mere error in judgment. Such misconduct, however serious, must, nevertheless, be in connection with the employee’s work to constitute just cause for his separation - Her absence for 16 days was justified considering that she had just delivered a child, which can hardly be considered a dereliction of duty or wrongful intent on the part of Belga.-Tropical harps on the alleged concealment by Belga of her pregnancy. This argument, however, begs the question as to how one can conceal a full-term pregnancy. We agree with respondent’s position that it can hardly escape notice how she grows bigger each day. While there may be instances where the pregnancy may be inconspicuous, it has not been sufficiently proven by Tropical that Belga’s case is such- The charge of disobedience for Belga’s failure to comply with the memoranda must likewise fail. Disobedience, as a just cause for termination, must be willful or intentional. In the instant case, the memoranda were given to Belga two days after she had given birth. It was thus physically impossible for Belga to report for work and explain her absence, as ordered- Tropical avers that Belga’s job as Treasury Assistant is a position of responsibility since she handles vital transactions for the company. It adds that the nature of Belga’s work and the character of her duties involved utmost trust and confidence. - In order to constitute a just cause for dismissal, the act complained of must be “work-related” such as would show the employee concerned to be unfit to continue working for

the employer. More importantly, the loss of trust and confidence must be based on the willful breach of the trust reposed in the employee by his employer. A breach of trust is willful if it is done intentionally, knowingly and purposely, without justifiable excuse, as distinguished from an act done carelessly, thoughtlessly, heedlessly or inadvertently- Belga was an assistant cashier whose primary function was to assist the cashier in such duties as preparation of deposit slips, provisional receipts, post-dated checks, etc. As correctly observed by the Court of Appeals, these functions are essentially clerical.

COCA-COLA BOTTLERS PHIL INC V KAPISANAN NG MALAYANG MANGGAGAWA SA COCA-COLA

452 SCRA 480CALLEJO; February 28, 2005

NATUREThis is a petition for review of the Resolution1 the Court of Appeals reversing the Resolution of the National Labor Relations Commission

FACTS- Petitioner Coca-Cola Bottlers Phil., Inc. is a domestic corporation engaged in the manufacture, sale and distribution of softdrinks.- On July 1, 1982, the petitioner hired Florentino Ramirez as "driver-helper" with the following duties: (a) as driver, he checks the truck’s oil, water, wheels, etc.; (b) as helper, he is charged of loading and unloading truck’s load; putting bottles in the coolers and displays company products to each outlet or customer’s store.2

- Ramirez became a member of the respondent Kapisanan ng Malayang Manggagawa Sales Force Union, the bargaining representative of the rank- and-file employees of the petitioner company. In 1996, he was the "shop steward" of the union at the company’s Batangas Sales Office. - Sometime in October 1996, it happened that the route salesman for Route M11 was unavailable to make his usual routes. Since Ramirez had been driving for the route salesman for so long, the petitioner company decided to assign him as temporary replacement of the regular route salesman for routes M11, AMC and LPR. - Thereafter, in a Letter dated December 5, 1996, the Officer-in-Charge of the Batangas Sales Office, Victor C. dela Cruz, informed the Officer-in-Charge of DSS-District 44, Rolando Manzanares, that a review of the copies of the invoices relating to the transactions of Ramirez in Rt. M11 revealed the following discrepancies: (a) the number of cases delivered to customers; (b) empty bottles retrieved from them, and (c) the amounts in Sales Invoices Nos. 3212215, 3288587, 3288763, 3288765 and 3288764- Ramirez received a Memorandum from District Office Nos. 44 and 45 requiring him to report to the said office starting December 5, 1996 until such time that he would be notified of the formal investigation of the charges against him. - During the formal investigation conducted by a panel of investigators on December 20, 1996, Ramirez was not represented by counsel. He also manifested that he was waiving his right to be represented by counsel when the members of the panel asked him about it. - Ramirez was then asked to explain the discrepancies subject of the charges - On February 11, 1997, Ramirez received a notice from the company informing him that his services were being terminated; his employment was terminated effective February 12, 1997.- On March 17, 1997, Ramirez and the union filed a Complaint for unfair labor practice and illegal dismissal against the company with the Arbitration Branch of the NLRC. - Ramirez likewise claimed that he was denied of his right to due process, based on the following grounds: Firstly, individual complainant was dismissed without having been first issued a "notice of dismissal" which supposedly should contain the charges against him, which would be made as basis for his termination. Secondly, individual complainant was dismissed without affording him an ample opportunity to defend himself, as he was not notified in advance of the subject of the administrative investigation. Thirdly, individual complainant was terminated without just and valid cause, and in gross violation of his right to due process. Lastly, individual complainant was terminated by respondents in utter bad faith, as the decision on the said termination was arrived at, without any just and valid cause. Simply put, respondents simply acted oppressively, malevolently, and with grave abuse of prerogatives.7

- Petitioner company alleged that the dismissal of Ramirez was based on the facts unearthed during the formal investigation, and that he was guilty of serious misconduct, a valid ground for termination of employment. Even if he was occupying the position of route driver/helper, he was nevertheless performing the functions and duties of a route salesman, and, as such, he not only committed fraud, but also willfully breached the trust and confidence reposed on him by the petitioner company. - According to the petitioner company, considering the sanctions imposed on Ramirez for prior breaches of company rules, his dismissal from employment was with basis. The petitioner company also insisted that Ramirez was accorded his right to due process: he was notified of the charges against him, was subjected to a formal investigation during

Labor Law 1 A2010 - 208 - Disiniwhich he was allowed to explain the discrepancies, and was notified of the outcome thereof, as well as the bases of the termination of his employment. - On July 31, 1998, the Labor Arbiter rendered judgment dismissing the complaint for lack of merit. The LA found that based on the evidence, there was a justifiable basis for the dismissal of Ramirez. According to the LA, it was of no moment that the official designation of Ramirez was "driver-helper," since he committed the infractions while he was performing the functions of an "acting salesman." The LA further found that due process had been complied with.9

- Ramirez appealed the decision to the NLRC- On September 20, 1999, the NLRC rendered a Resolution affirming the decision of the LA.- Upon the denial of his motion for reconsideration, Ramirez filed a petition for certiorari under Rule 65 of the Rules of Court with the Court of Appeals- In a Decision dated October 25, 2000, the CA dismissed the petition. It ruled that the petitioner’s designation at the time of the infraction was of no moment; when he agreed to be an "acting salesman" for Route M11, AMC and LPR, he actually performed the duties of a salesman, and in so doing, assumed the responsibilities of the position. The CA further ratiocinated that notwithstanding Ramirez’s lack of training, he had assumed and performed the duties of a salesman; hence, he was obligated to do so with due care, dedication, and with due regard to the exercise of the degree of diligence to prevent the commission of any serious error, mistake or blunder on his part.- The petitioner filed a motion for the reconsideration of the decision- This time, the CA found merit in petitioner’s cause

ISSUEWON respondent Florentino Ramirez was dismissed by the petitioner without just or valid cause

HELD- with just cause, but too severe penalty- The respondent, by his acts and omissions, committed irregularities in the performance of his duties. However the penalty imposed on respondent by the petitioner company was too severe. In order to effect a valid dismissal of an employee, the law requires that there be just and valid cause as provided in Article 282 and that the employee was afforded an opportunity to be heard and to defend himself. Pursuant to Article 282 of the Labor Code, an employee’s services can be terminated for the following just causes:

(a) Serious misconduct or willful disobedience by the employee of the lawful orders of his employer or representative in connection with his work;(b) Gross and habitual neglect by the employee of his duties;(c) Fraud or willful breach by the employee of the trust reposed in him by his employer or duly-authorized representative.(d) Commission of a crime or offense by the employee against the person of his employer or any immediate member of his family or his duty-authorized representative; and (e) Other causes analogous to the foregoing.

- In termination disputes, the burden of proof is always on the employer to prove that the dismissal was for a just and valid cause. Considering the nature of the charges and the penalties therefore, the petitioner is bound to adduce clear and convincing evidence to prove the same.- It is recognized that company policies and regulations, unless shown to be grossly oppressive or contrary to law, are generally valid and binding on the parties and must be complied with until finally revised or amended, unilaterally or preferably through negotiation, by competent authority. The Court has upheld a company’s management prerogatives so long as they are exercised in good faith for the advancement of the employer’s interest and not for the purpose of defeating or circumventing the rights of the employees under special laws or under valid agreements. For misconduct or improper behavior to be a just cause for dismissal, the same must be related to the performance of the employee’s duties and must show that he has become unfit to continue working for the employer.- In cases when an employer may dismiss an employee on the ground of willful disobedience, there must be concurrence of at least two requisites: (1) the employee’s assailed conduct must have been willful or intentional, the willfulness being characterized by a wrongful and perverse attitude; and (2) the order violated must have been reasonable, lawful, made known to the employee and must pertain to the duties which he had been engaged to discharge. - That the individual petitioner has not been specifically trained as salesman is undisputed. In acting as a salesman, he was tasked with a duty involving trust and specialized skills for which he was never trained. His alleged failure to comply strictly with all the procedures, of which he was unfamiliar, was to be expected. Yet Ramirez was penalized as a full-fledge salesman, not as a driver-helper who was forced to perform the functions of acting salesman or perhaps risk being charged with insubordination. Then it was not just any penalty meted out to him, as if there is only one punishment possible for him: the supreme sanction of dismissal.- Perhaps, individual petitioner should first have been given a mere warning, then a reprimand or even a suspension, but certainly not outright dismissal from employment. One must keep in mind that a worker’s employment is property in the constitutional sense, and he cannot be deprived thereof without due process and unless it was commensurate to his acts and degree of moral depravity.

- In order to validly dismiss an employee on the ground of loss of trust and confidence under Article 282 of the Labor Code of the Philippines, the following guidelines must be followed:

1. The loss of confidence must not be simulated;2. It should not be used as a subterfuge for causes which are illegal, improper or unjustified; 3. It may not be arbitrarily asserted in the face of overwhelming evidence to the contrary;4. It must be genuine, not a mere afterthought, to justify earlier action taken in bad faith; and5. The employee involved holds a position of trust and confidence.

- Considering the factual backdrop in this case, we find and so rule that for his infractions, the respondent should be meted a suspension of two (2) months.Disposition PARTIALLY GRANTED

GENUINO ICE CO INC V MAGPANTAY[PAGE 206]

PREMIERE DEV’T BANK V MANTAL485 SCRA 234

YNARES-SANTIAGO; March 23, 2006

NATUREPetition for review on certiorari seeking to annul and set aside the Decision of the Court of Appeals in CA-G.R. SP No. 80975 dated January 17, 2005 and its Resolution dated April 7, 2005 holding the petitioner Premiere Development Bank liable for illegal suspension and illegal dismissal, ordering it to reinstate respondent Elsie Escudero Mantal to her former position and to pay her full backwages from date of suspension and dismissal until actual reinstatement, half month salary and half month 13th month pay, as well as attorney’s fees.

FACTS - Respondent is a regular employee of petitioner’s Cubao branch, serving as accounting clerk since July 17, 1996. On November 24, 2000, the branch manager, Rosario Detalla, instructed respondent: "Elsie, baka may mag-confirm sa Bank Guarantee ng GIA Fuel, sabihin mo OKAY NA, may kulang pa lang dokumento."- Later that day, Emmie Crisostomo of Filpride Energy Corporation inquired whether GIA Fuel and Lubricant Dealer has a credit line or maintains an account with petitioner Bank which respondent confirmed after checking the files on the computer. Crisostomo also inquired if the bank guarantee signed by Detalla is in order, and likewise respondent replied in the affirmative. However, upon verification from petitioner’s head office, Crisostomo was informed that the bank guarantee was spurious.- On the same day, respondent was summoned to the head office and was required to write down what she knew about the subject bank guarantee. Respondent also received a memorandum placing her under preventive suspension effective immediately for a period of 30 days. During the investigation, Detalla admitted issuing the falsified bank guarantee.- On December 21, 2000, Detalla tendered her irrevocable letter of resignation. Respondent was asked to execute a resignation letter on December 22, 2000, but she declined. The following day, respondent received a Notice of Termination dated December 22, 2000.- Respondent filed a complaint for illegal suspension, illegal dismissal, unpaid salary and 13th month pay, moral and exemplary damages. The Labor Arbiter rendered a decision holding petitioner liable for illegal suspension and illegal dismissal and ordering the reinstatement of respondent to her former position, with full backwages, half month salary and half month 13th month pay, and attorney’s fees. NLRC reversed the labor arbiter’s decision, and dismissed the complaint for lack of merit. The motion for reconsideration having been denied, respondent appealed to the Court of Appeals which affirmed the Labor Arbiter.

ISSUEWON respondent was validly suspended and dismissed from her position as accounting clerk

HELD NORatio Misconduct is improper or wrongful conduct. It is the transgression of some established and definite rule of action, a forbidden act, a dereliction of duty, willful in character, and implies wrongful intent and not mere error in judgment. Under Article 282 of the Labor Code, the misconduct, to be a just cause for termination, must be of such grave and aggravated character, not merely of a trivial or unimportant nature. For serious misconduct to warrant the dismissal of an employee, it (1) must be serious; (2) must relate to the performance of the employee’s duty; and (3) must show that the employee has become unfit to continue working for the employer. Reasoning - Respondent did what was expected of her as an employee of the bank. Before answering the telephone inquiry, respondent verified the existence of the GIA Fuel and Lubricant Dealer account through the bank computer. If ever she was negligent, it would

Labor Law 1 A2010 - 209 - Disinionly constitute a single or isolated act which is not a just cause for the dismissal of the respondent from her employment.In addition, although respondent’s position as accounting clerk involves a high degree of responsibility requiring trust and confidence, carrying with it the duty to observe proper company procedures in the fulfillment of her job as it relates closely to the financial interests of the company, the charge against her is not reasonably connected to her job of opening of savings, current and/or time deposits and the payment of withdrawals. The duty and ultimately, the responsibility of approving transactions relating to bank guarantees lie with the branch manager and the management personnel of the petitioner’s head office. Thus, in Metropolitan Bank and Trust Company v. Barrientos, the Court held that respondent therein was not liable of misconduct for allowing the opening of fictitious accounts, because he was merely a cashier and had no authority to approve new accounts and had no way of knowing the anomalous transactions. Disposition petition is DENIED. The Decision of the Court of Appeals in CA-G.R. SP No. 80975 dated January 17, 2005 finding petitioner guilty of illegal dismissal and ordering the reinstatement of respondent to her former position, with full backwages, inclusive of allowances and to the other benefits or their monetary equivalent from the time her compensation was withheld up to her actual reinstatement, plus attorney’s fees, and the Resolution dated April 7, 2005 denying the motion for reconsideration, are AFFIRMED.

MOLINA V PACIFIC PLANS INC484 SCRA 498

CALLEJO; March 10, 2006

NATURE Petitions for Review on Certiorari assailing the decision and resolution of the CA reversing the decision of the NLRC.

FACTS - The accident occurred on July 9, 1912.- Because of injuries, plaintiff spent 10 days in the hospital. The first 4-5 days he couldn’t leave his bed. After being discharged, he received medical attention from a private practitioner for several days. - Plaintiff testified that he had down no work since the accident, that his earning capacity was P50/month - He described himself as being well at the end of July; the trial took place September 19- Plaintiff sold distillery products and had about 20 regular customers who purchased in small quantities, necessitating regular, frequent deliveries- It took him about 4 years to build up the business he had at the time of the accident, and since the accident, he only kept 4 of his regular customers. - The lower court refused to allow him any compensation for injury to his business due to his enforced absence therefrom.

ISSUE How to determine the amount of damages to award plaintiff

HELD- The judgment of the lower court is set aside, and the plaintiff is awarded the following damages; ten pesos for medical expenses; one hundred pesos for the two months of his enforced absence from his business; and two hundred and fifty pesos for the damage done to his business in the way of loss of profits, or a total of three hundred and sixty pesos. No costs will be allowed in this instance. Reasoning- Actions for damages such as the case at bar are based upon article 1902 of the Civil Code: "A person who, by act or omission, causes damage to another where there is fault or negligence shall be obliged to repair the damage so done." Of this article, the supreme court of Spain, in considering the indemnity imposed by it, said: "It is undisputed that said reparation, to be efficacious and substantial, must rationally include the generic idea of complete indemnity, such as is defined and explained in article 1106 of the said (Civil) Code." - Art 1106. Indemnity for losses and damages includes not only the amount of the loss which may have been suffered, but also that of the profit which the creditor may have failed to realize, reserving the provisions contained in the following articles.- Art 1107. The losses and damages for which a debtor in good faith is liable, are those foreseen or which may have been case is will gradually increase. The injury to plaintiff's business begins where these profits leave off, and, as a corollary, there is where defendant's liability begins. Upon this basis, we fix the damages to plaintiff's business at P250. - Before us is a Petition for Review on Certiorari assailing the Decision and Resolution of the Court of Appeals (CA) in CA-G.R. SP No. 81298 reversing the Decision of the National Labor Relations Commission (NLRC) in NLRC-NCR (South) Case No. 30-07-03393-01.Pacific Plans, Inc. (PPI) is a domestic corporation engaged in the business of selling pre-need plans, such as educational, pension, and memorial plans. It maintains regional offices throughout the Philippines. At the time material to this case, Metro Manila regional offices were divided into two sales divisions - the South Sales Division and the North Sales Division. Metro Manila VI was part of the North Sales Division. Among the corporate officers of PPI were Geoffrey Martinez, Executive Vice-President for Finance;

Luciano Abia, Senior Assistant Vice-President, Metro Manila Marketing Division; and Atty. Manuel Reyes, the Head of the Legal Department. Roy Padiernos then occupied the position of Regional Manager of Metro Manila VI. - PPI solicited subscribers and buyers of its pre-need plans through clusters of sales associates. One of them was Ruth Padiernos, wife of Roy Padiernos. Sometime in October 1994, PPI hired Agripino Molina as Regional Manager of Metro Manila VI, replacing Roy Padiernos who was promoted as First Vice-President for Marketing Operations. As Regional Manager, Molina performed both administrative and marketing functions, whose duties and responsibilities included the following:

a. formulating and recommending short and long range marketing plans for the Region and executing approved plans;b. generating new and conserving existing pre-need plan businesses;c. motivating, training, and developing a dedicated and effective counselor force;d. conducting researches to determine sales potentials and share of the market, pricing, and profitability of Company's products, competition and the directing of product development for the Region;e. hiring and terminating counselors, unit managers or group managers in accordance with policies previously laid out;f. recommending the creation of additional positions or termination of services of any employee within the Region;g. recommending promotions or changes in salaries of personnel within the Region and lateral shifts of supervisor, their assistants, understudies of positions of equal rank;h. training and developing understudies for each position within the Region to provide immediate replacement whenever vacated;i. changing methods and procedures not affecting the other Regions, provided, however, that radical changes should first be cleared with [the] superior;j. controlling the operations of the Region and establishing a system of periodic work reporting;k. coordinating the Region’s activities with those of the other Regions;l. keeping [the] superior informed of [the] Region's activities and specially of [the] decision on matters for which he may be held responsible;m. realizing the Company’s objective for service, growth, and profit;n. establishing and maintaining harmonious and dignified relationship with plan holders, counselors, employees, the public, government instrumentalities, other pre-need plan companies; [and]o. further enhancing the prestige of the Company and maintaining its position of leadership in its field.

- Since Metro Manila VI was consistently on top in terms of nationwide sales and productivity, Molina was promoted Assistant Vice-President with the same functions as those of a regional manager of the same sales region.- Caritas Health Shield, Inc. (Caritas for brevity), a health maintenance organization (HMO) engaged in selling health and hospitalization plans, was established on December 16, 1998. Geoffrey Martinez resigned as Executive Vice-President of PPI and became the President and Chief Executive Officer of Caritas. Among the incorporators and members of the Board of Directors were Luciano Abia and Atty. Manuel Reyes. Molina was hired as Assistant Vice-President and Marketing Head of Area 10. His wife, Fe Molina, was the head of a sales agency of Caritas.- In the meantime, from February 2000, there was a considerable decrease in the sales output production of PPI’s Metro Manila Region VI. - On March 21, 2000, Molina received a Memorandum from PPI, through its Senior Assistant Vice-President for Human Relations, Patricio A. Picazo, informing him that, based on written reports, he committed the following: 1) recruiting and pirating activities in favor of Caritas, in particular, initiating talks and enticing associates to join Caritas, and a number of associates have already signed up; 2) he called for a meeting with his associates sometime in November 1999, and solicited contributions from them for the bill but later asked for reimbursement from the company; and 3) acts of misdemeanor on several occasions, such as coming to the office under the influence of liquor, initiating a smear campaign against PPI, and other acts inimical to the company’s interest. Molina was also required to submit, on March 23, 2000, a written explanation why he should not be held administratively liable for said acts which, it opined, might constitute conduct unbecoming of an officer, conflict of interest, and breach of trust and confidence. Molina was also informed that he was preventively suspended pending formal investigation effective immediately until April 24, 2000. - In a letter addressed to Picazo dated March 22, 2000, Molina categorically denied the acts attributed to him. He, however, requested that he be furnished with copies of the alleged written reports to enable him to prepare the required written explanation. However, instead of acceding to the request of copies of the written reports, Picazo wrote a letter dated April 3, 2000, citing the particulars of the charges against Molina, thus:

I. Conflict of Interest1. Recruiting and pirating activities in favor of Caritas Health Shield, Inc.* You have acted as conduit for Caritas in recruiting/pirating Mr. Restie Acosta on March 04, 2000 and Ms. Eppie Acosta on March 06, 2000.*Your failure to stop and/or tolerating your wife's activities in recruiting for Caritas Ms. Lennie Gatmaitan who belongs to Ms. Celeste Villena, a PPI GA.

II. Misappropriation of Funds1. Solicitation of associates' personal funds in the amount of P200.00 per person, to which 12 persons contributed for a total P2,400.00, for payment of official function during the meeting held at Barrio Fiesta last November 27, 1999. Amount

Labor Law 1 A2010 - 210 - Disinisolicited was subsequently reimbursed from the company but not returned to the associates concerned.

III. Dereliction of Duties1. You failed to prevent associates from leaving the company in favor of competitors, thus causing demoralization among your sales associates.2. You even encouraged associates to transfer to Caritas.

IV. Conduct unbecoming of a Company Officer1. Often reporting to office under the influence of liquor.2. Sowing intrigue in the case of Vilma del Rosario which almost caused her early retirement from the company and transfer to Caritas.3. Sowing intrigues between Mr. Roy Padiernos and Mr. Abia.4. Showing disrespect to immediate superior, Mr. Roy Padiernos, by shouting at him and walking out in one of the meetings called by him after the retirement of Atty. Haceta.

- During the investigation the following day, April 4, 2000, Molina reiterated his request to be provided with a copy of the written reports. Picazo denied the request in a Memorandum dated April 6, 2000, and reiterated his order for Molina to submit his written explanation on April 11, 2000, and to address his concerns during the investigation scheduled on April 14, 2000. Molina failed to submit any written explanation. On April 24, 2000, PPI issued a Memorandum advising Molina that he would be reinstated in the payroll effective April 25, 2000 without requiring him to report for work during the pendency of his investigation. - Molina filed a "Motion to Dismiss Complaints and Motion for Full Reinstatement" on May 2, 2000. He asserted that the charges should be dismissed since he was compelled to prepare a written explanation on the basis of "summarized specific acts," denying him the right to be informed of the exact charges and to confront those who made written reports against him. As to the issue of reinstatement, he alleged that he should be allowed to report for work, conformably with Rule XIV, Section 4 of the Implementing Rules of the Labor Code. - On May 11, 2000, Picazo wrote Molina that his motion to dismiss the charges would be resolved after the investigation. He was warned that his non-appearance at the investigation would be considered a waiver of his right to be heard. - On the same day, May 11, 2000, Abia issued an inter-office Memorandum announcing the appointment of Sercy F. Picache as the Officer-In-Charge (OIC) for Metro VI and XVI effective May 6, 2000. - Molina and his counsel attended the May 19, 2000 investigation and filed a Motion to Suspend Proceedings, praying that the administrative investigation be deferred until the resolution of the "prejudicial" issues raised in his previous motion. - When Picazo failed to respond, Molina filed, on June 1, 2000, a complaint for damages with a prayer for a temporary restraining order and preliminary injunction based on Article 19 of the New Civil Code. PPI filed a Motion to Dismiss, maintaining that the courts have no jurisdiction over matters arising from employee-employer relationship. The trial court denied the motion as well as PPI’s motion for reconsideration. - Meanwhile, in letter dated June 13, 2000, Molina was notified of the termination of administrative investigation. PPI considered his failure to submit a written explanation as a waiver of his right to be heard, and as such, the investigating committee had evaluated the evidence at hand and submitted its recommendations to the "higher management" for decision. Also, it confirmed the denial of his Motion to Suspend Proceedings - On June 23, 2000, the trial court issued an Order granting Molina's prayer for temporary restraining order, which was later made permanent per its Order dated July 12, 2000. The motion for reconsideration filed by PPI on July 26, 2000 was likewise denied. Thereafter, it filed a petition for certiorari before the CA, assailing the writ of preliminary injunction issued by the RTC and its order denying the motion to dismiss the complaint. On July 16, 2001, the CA rendered judgment in favor of PPI and nullified the writ of preliminary injunction issued by the RTC as well as the order denying the motion of PPI for the dismissal of the complaint. - On July 30, 2001, PPI resolved to dismiss Molina from employment on its finding that the latter violated its standard operating procedure. - Molina forthwith filed a complaint with the NLRC against PPI and Alfredo C. Antonio, Patricio A. Picazo, and Certerio B. Uy, in their capacity as President, Senior Assistant Vice-President of Human Resources Development, and Division Head, respectively, for illegal dismissal and illegal suspension with claim for monetary benefits.- In his Position Paper, Molina principally argued that he was denied the right to due process due to the failure of PPI to furnish him a copy of the written reports of the sales associates and co-employees, the basis of the accusations against him. Since an OIC for his position was already appointed even before all his pending motions were resolved, he surmised that there were really no such reports, and that the alleged accusations were merely concocted in order to replace him with someone close to Picazo. Molina maintained that since he was denied the opportunity to dispute the authenticity and substantive contents of the reports, his alleged violations of company rules and policies were hearsay and, therefore, lacked probative value. Besides, the termination of his employment was made without the 30-day prior notice; his dismissal from employment took effect immediately, only six days after PPI received the CA decision decreeing that the NLRC has the rightful jurisdiction over the case. Thus, he prayed for the following relief:

1. Total Money Claimsa) Salary with (overriding) commission from March 21 to April 24, 2000 - suspended w/o pay - P45,000.00 (P25,000[.00] mo. salary & P20,000[.00] [overriding])b) Unpaid (overriding) commission from April 25, 2000 to present - P400,000[.00]

c) Unpaid salary from August 1, 2001 to present - P125,000[.00]d) One mo. salary for every yr. of service in lieu of reinstatement - 7 years = P175,000.00

2. Leave Credits - P100,000.00 for 7 years3. Profit Bonus for Year 2000 & 2001 - P400,000.004. Moral Damages - P300,000.005. Exemplary Damages - P500,000.006. Actual Damages - for lifetime medical attendance and medicines at 16 more years life expectancy - P1,249,384.007. Attorney's Fees - P300,000.008. Amount debited from complainant's ATM [as partial payment for hospitalization expenses incurred by him which PPI had advanced] - P12,000.009. Retention of complainant's car, as additional penalty for illegal dismissal.

- For its part, PPI stressed that Caritas was its competitor in the pre-need plans business, and that Molina and his wife recruited and enticed some of the sales associates of PPI to work for Caritas, in violation of its policy against conflict of interest. Some of these sales associates were the spouses Eppie and Restie Acosta, Lenita Gatmaitan, Lolita Casaje, Lydia Magalso, Lydia San Miguel, and Alice Halili, and including Vilma del Rosario, the secretary of Roy Padiernos. PPI, likewise, averred that Molina had the habit of coming to the office under the influence of liquor; he constantly shouted to lady employees and solicited money from his sales associates in connection with an official company function without returning the same after PPI reimbursed him for the expenses incurred; disseminated intrigues and created divisiveness among the employees and PPI’s senior officers; and disrespected Padiernos, his superior, by shouting at him during one of the meetings with other senior officers, and walked out of the meeting afterwards. Supporting its claims that Molina committed breach of trust, serious misconduct, fraud, and gross neglect of duty by reason thereof, PPI appended to its position paper the statements/affidavits of Marivic Uy, Ruth and Roy Padiernos, Eppie and Restie Acosta, Celeste Villena, and Vilma del Rosario. - On the claim of Molina that he was denied due process, PPI averred that he was given sufficient opportunity to present his personal submissions before finally issuing the notice of dismissal but Molina persistently refused to submit his explanation. PPI further argued that he was not entitled to the payment of 13th and 14th month salaries, overriding commission, profit bonus, actual, moral or exemplary damages, and attorney’s fees. PPI maintained that, under Article 217(a) of the Labor Code, as amended, and the ruling of this Court in Bañez v. Valdevilla, Molina should be held liable for P1,000,000 as moral damages and an amount not less than P428,400.00 for the salary he received during the time when the restraining order/ writ of injunction was erroneously enforced. - In his Reply, Molina averred that the affidavits submitted by PPI were antedated since he was never furnished copies of said reports/affidavits despite demands. PPI even failed to present the reports/affidavits before the RTC where his complaint for damages against PPI and its officers was pending. He and Roy Padiernos had been at odds because the latter appointed his brother and wife as agency manager and group manager of PPI to which he objected. Molina averred that the P200.00 collected from each of the employees of PPI during their luncheon meeting was a voluntary contribution, and that they spent P4,000.00, more than the amount collected from the employees. He contended that he had no motive to recruit sales associates or employees of PPI to be employed by Caritas because the depletion of sales associates would diminish his effectiveness as an area manager, including his overriding commission, profit bonus and fringe benefits. He admitted that he may have raised his voice in the heat of arguing a point during meetings, but averred that it should not be considered as disrespect or misdemeanor.- Molina further emphasized that Caritas was not a competitor of PPI, as the former was engaged in selling health care and is supervised by the Department of Health (DOH), while the latter is into the business of selling pre-need plans and supervised by the Securities and Exchange Commission (SEC). Finally, he averred that the so-called "associates" of PPI were not actually employees but "independent journeymen" who derived income on commission basis, free to engage in any kind of selling activities not in direct competition with PPI.- Molina admitted having had drinking sessions with Certerio Uy, Ilustre Acosta and Reynaldo Villena, who provided the hard liquor and pulutan, but only after office hours. He claimed that his officemates mistook him for being drunk when he went to his office even after office hours because of his "mestizo complexion."- In its response, PPI averred that, based on the sales data, the acts of Molina caused demoralization of the sales associates, resulting in a sudden decrease of the region's output from P343,009,643.00 in 1998 to P263,099,773.00 in 1999, and P228,752,090.00 in 2000. PPI insisted that he should be held liable for not less than P507,348.00, P2,000,000, and P1,000,000 as actual, moral and exemplary damages, and attorney's fees, respectively, and P273,600.00 which was the balance on his car plan agreement with PPI. - In his Rejoinder and Sur-Rejoinder Molina submitted the affidavit of Geoffrey Martinez, who belied the reports of Uy, Villena, Del Rosario, and the spouses Padiernos and Acosta. He also appended the affidavits of Natividad Gatchalian, San Miguel, Gatmaitan, and Magalso, who all disputed, in one way or another, Molina's alleged violations. To counter the imputations of conflict of interest, Molina also alleged that Abia and Atty. Reyes were incorporators of Caritas, and that Villena had in her possession a license to sell Caritas products. With regard to the declining sales output of his region, Molina attributed the same to the Asian regional crisis that hit the Philippines sometime in 1997. He noted, however, that the same records revealed that despite the financial bane, Metro

Labor Law 1 A2010 - 211 - DisiniVI still managed to be on top from 1998 up to 2000 in terms of its sales relative to the other regions.- Molina denied any liability for the car plan, claiming that he already settled the obligation when PPI demanded full payment as, in fact, all the papers related thereto, including the Release of Mortgage, were already in his possession.- In its Sur-Rejoinder, PPI stressed its claim that Caritas was a business competitor, as may be inferred from the benefits available under its health care agreement and the pre-need contract of PPI. Particularly with regard to the pension plan contract, it noted the following similarities: (a) Caritas also provides Term Life Insurance, Accidental Death Insurance, Credit Life Insurance, and Waiver of Installment Due to Disability; (b) there are similarities in the provisions on contract price, grace period, cancellation, reinstatement, and transfer and termination; and (c) unlike other health care programs that provide a one-year coverage, renewable every year thereafter, Caritas offers a continuous five year coverage and sells the same in units payable in five-year installment basis, with maturity period and guaranteed return of investment in the form of Full-Term Medical Expense Fund computed at P10,000.00 for every unit purchased with increment of 10% yearly after the maturity period, which may be withdrawn in cash by its member. It stressed that this was similar to the pension program offered by PPI which was also sold in per unit basis, payable by installment in certain number of years or lump sum payment, and upon maturity also gives P10,000.00 pension benefit per unit purchased by the plan holder. With respect to the alleged interest of Atty. Reyes with Caritas, PPI adduced in evidence a Deed of Sale to prove that as early as February 1999 he had already divested his stockholdings in Caritas. - On November 18, 2002, Labor Arbiter Roma C. Asinas rendered a Decision dismissing the complaint and the counterclaims for lack of merit. The labor arbiter ruled that Molina was lawfully dismissed from his employment for serious misconduct in office and fraud or willful breach of trust and confidence. It declared that Molina’s mere denial of the charges against him did not overthrow the overwhelming evidence against him tending to show that he committed the allegations against him. Moreover, his wife was then an agency manager of Caritas, and some PPI sales associates were with Caritas because they were recruited by Molina. The labor arbiter also ruled that other employees of respondent attested to the fact that they were being recruited and enticed by the complainant to join Caritas. This act of pirating constituted serious misconduct in office, fraud or willful breach of trust and confidence, which are just causes for termination of employment under Article 282 of the Labor Code, as amended. As such, PPI could not legally be compelled to continue Molina’s employment due to breach of trust. - The labor arbiter likewise held that Molina was afforded his right to due process, but that he refused to give an answer to the charges leveled against him, and instead insisted that he be furnished a copy of the alleged reports against him. Since he was given ample opportunity to answer the charges and explain his side during the investigation, and a formal or trial-type hearing is not at all times essential, Molina’s right to due process was not violated. The labor arbiter stressed that the requirements of due process are satisfied where the parties are afforded fair and reasonable opportunity to explain their side of the controversy at hand. - Molina appealed the decision to the NLRC, which rendered judgment in his favor. The NLRC reversed the decision of the Labor Arbiter and ordered Molina’s immediate reinstatement to his former position as Assistant Vice President without demotion in rank and salary; and the payment of his backwages from August 1, 2001 up to his actual reinstatement, and other accrued monetary benefits. However, the NLRC denied all other claims for damages. - According to the NLRC, the charges of coming to the office under the influence of liquor and making PPI reimburse the expenses already paid by Molina's co-employees were not supported by the records. The "loss of trust and confidence" had no factual basis since the alleged acts of Molina did not result to any loss in favor of PPI. - Anent Molina’s recruitment activities, the NLRC ratiocinated that PPI failed to show that Caritas was a competitor of PPI. Caritas caters to the health care needs of its clients while PPI to the pre-need (pension, educational, and memorial) requirements of its plan holders. Any similarity between PPI and Caritas’ extra features like term life insurance, accidental death insurance, credit life insurance, and waiver of installment due to disability, did not ipso facto make Caritas a competitor of PPI. Thus, there was no conflict of interest in Molina’s act of trying to recruit counselors for Caritas to help his wife. Moreover, PPI failed to establish that recruiting for Caritas affected Molina’s decisions in the performance of his duties with PPI. According to the NLRC, the drop in the sales and productivity of complainant’s area of responsibility may be due to market forces and depressed economic condition at that time; absent any clear and convincing proof, it cannot be attributed to the alleged acts of Molina which constituted willful breach of trust or confidence. - PPI filed a motion for reconsideration, and appended a Letter dated June 13, 2002 from the SEC to Caritas, indicating that its HMO Plan was similar to the previous plans offered by pre-need companies, hence, under the regulatory suspension of the SEC; another letter of SEC ordering Caritas to immediately desist from selling its HMO plan with the full term medial expense fund; and the letter of Caritas, through counsel, endorsing the objectionable features of the HMO plan. - The NLRC, however, was not persuaded, and resolved to deny PPI’s motion in its Order dated September 30, 2003. On November 19, 2003, the NLRC declared its Decision final and executory as of November 14, 2003. - PPI filed a Petition for Certiorari with the CA for the nullification of the decision and resolution of the NLRC and the reinstatement of the decision of the Labor Arbiter. - On August 13, 2004, the CA rendered a decision reversing the Decision and Resolution of the NLRC, and reinstating the November 18, 2002 Decision of the Labor Arbiter. Later,

the CA denied Molina’s Motion for Reconsideration in its Resolution dated September 27, 2004. - The issues for resolution are the following: whether the decision of the NLRC was already final and executory when PPI filed its petition for certiorari in the CA; and whether the NLRC committed grave abuse of discretion amounting to excess or lack of jurisdiction in issuing the assailed decision and resolution.- On the first issue, we find and so hold that the decision of the NLRC had become final and executory when PPI filed its Petition for Certiorari in the CA. PPI received a copy of the NLRC Decision on July 11, 2003 and filed the Motion for Reconsideration thereof on July 18, 2003, which motion was denied on September 30, 2003. Under Rule VII, Section 2 of the NLRC Omnibus Rules of Procedure, the decision of the NLRC becomes final and executory after ten (10) calendar days from receipt of the same. PPI received a copy of the NLRC decision on November 30, 2003; hence, such decision became final and executory on December 3, 2003. Nonetheless, the Court ruled in St. Martin Funeral Home v. NLRC that, although the 10-day period for finality of the NLRC decision may have elapsed as contemplated in the last paragraph of Section 223 of the Labor Code, the CA may still take cognizance of and resolve a petition for certiorari for the nullification of the decision of the NLRC on jurisdictional and due process considerations. Indeed, the remedy of the aggrieved party from an adverse decision of the NLRC is to timely file a motion for reconsideration as a precondition for any further or subsequent remedy, and if the motion is denied, such party may file a special civil action in accordance with law and jurisprudence considering that these matters are inseparable in resolving the main issue of whether the NLRC committed grave abuse of discretion.- The Labor Arbiter and the NLRC act in quasi-judicial capacity in resolving cases after hearing and on appeal, respectively. On the presumption that they have already acquired expertise in their jurisdiction, which is confined on specific matters, their findings of facts are oftentimes accorded not only with respect but even finality if supported by substantial evidence. However, in spite of the statutory provision making "final" the decision of the NLRC, the Court has taken cognizance of petitions challenging such decision where there is a clear showing that there is want of jurisdiction, grave abuse of discretion, violation of due process, denial of substantial justice, or erroneous interpretation of law. - In this case, the Labor Arbiter declared that there is substantial evidence on record warranting the dismissal of petitioner as Assistant Vice President for serious misconduct in office, fraud or willful breach of trust and confidence. The NLRC disagreed with the Labor Arbiter and reversed the latter’s findings. The CA, for its part, concurred with the findings of the Labor Arbiter. In view of the discordance between the findings of the Labor Arbiter and the CA on one hand, and the NLRC on the other, there is a need for the Court to review the factual findings and the conclusions based on the said findings. As this Court held in Diamond Motors Corporation v. Court of Appeals: - A disharmony between the factual findings of the Labor Arbiter and the National Labor Relations Commission opens the door to a review thereof by this Court. Factual findings of administrative agencies are not infallible and will be set aside when they fail the test of arbitrariness. Moreover, when the findings of the National Labor Relations Commission contradict those of the labor arbiter, this Court, in the exercise of its equity jurisdiction, may look into the records of the case and reexamine the questioned findings - Article 282 of the Labor Code of the Philippines provides:

Art. 282. Termination by employer. – An employer may terminate an employment for any of the following causes:a. Serious misconduct or willful disobedience by the employee of the lawful orders of his employer or representative in connection with his work;b. Gross and habitual neglect by the employee of his duties;c. Fraud or willful breach by the employee of his duties of the trust reposed in him by his employer or duly authorized representative;d. Commission of a crime or offense by the employee against the person of his employer or any immediate member of his family or his duly authorized representative; and e. Other causes analogous to the foregoing.

- Misconduct has been defined as improper or wrong conduct; the transgression of some established and definite rule of action; a forbidden act, a dereliction of duty, unlawful in character and implies wrongful intent and not mere error of judgment. The misconduct to be serious must be of such grave and aggravated character and not merely trivial and unimportant. Such misconduct, however, serious, must nevertheless, be in connection with the employee’s work to constitute just cause for his separation. - The loss of trust and confidence, in turn, must be based on the willful breach of the trust reposed in the employee by his employer. Ordinary breach will not suffice. A breach of trust is willful if it is done intentionally, knowingly and purposely without justifiable excuse, as distinguished from an act done carelessly, thoughtlessly, heedlessly or inadvertently. The Court has laid down the guidelines for the application of the doctrine for loss of confidence, thus:

1. the loss of confidence must not be simulated;2. it should not be used as a subterfuge for causes which are illegal, improper or unjustified;3. it may not be arbitrarily asserted in the face of overwhelming evidence to the contrary;4. it must be genuine, not a mere afterthought, to justify earlier action taken in bad faith; and5. the employee involved holds a position of trust and confidence.

In Samson v. Court of Appeals, the Court enumerated the conditions for one to be considered a managerial employee:

Labor Law 1 A2010 - 212 - Disini(1) Their primary duty consists of the management of the establishment in which they are employed or of a department or subdivision thereof;(2) They customarily and regularly direct the work of two or more employees therein;(3) They have the authority to hire or fire other employees of lower rank; or their suggestions and recommendations as to the hiring and firing and as to the promotion or any other change of status of other employees are given particular weight.

- As a general rule, employers are allowed wide latitude of discretion in terminating the employment of managerial personnel. The mere existence of a basis for believing that such employee has breached the trust and confidence of his employer would suffice for his dismissal. - In this case, petitioner was not a mere employee of respondent. He was the Assistant Vice-President with the same functions of a regional manager of the same sales region, Metro Manila VI. Taking into account his job description, he was one of the top managers of the respondent, tasked to perform key and sensitive functions in the interest of his employer and, thus, bound by the more exacting work ethic.- We find, however, that the charge of misappropriation of funds was not proven with substantial evidence. As gleaned from the handwritten statement of Ilustre Acosta, the General Manager of the Springs and Blessings General Agency under Metro Manila VI, it appears that, aside from him and petitioner, there were 10 other attendees during the luncheon conference on November 27, 1999 at the Barrio Fiesta, Cubao, Quezon City. Petitioner received the amount of only P2,386.00 from respondent to pay for the cost of the luncheon for the conference, based on Petty Cash Voucher signed by petitioner,74 but the conferees spent more than P4,000.00. Upon petitioner’s suggestion, the conferees agreed to contribute P200.00 each, or the total amount of P2000.00 to answer for the difference. Petitioner had no obligation to return the contributions of the conferees, nor was he liable for said amount. Significantly, except for Ilustre Acosta, the other attendees in the conference never complained against petitioner or requested him to return their respective contributions of P200.00.- Regarding the charge that the petitioner peddled false and malicious informations against Abia and Padiernos, Abia has not executed any affidavit to confirm paragraph 9 of the affidavit of Roy Padiernos. As admitted by del Rosario, the informations allegedly relayed to her by the petitioner pertaining to Roy Padiernos were confirmed by Zita Domingo. - The petitioner does not deny having had a heated exchange of words with Roy Padiernos in the course of a meeting. However, such incident does not constitute proof that the petitioner thereby showed disrespect to Roy Padiernos, nor a valid cause for petitioner’s dismissal. It does happen that in the course of exchange of views during a meeting, participants may become so assertive to the point of being overbearing or unyielding and in the process lose their temper, on their sincere belief of being right. There is no evidence on record that petitioner committed the same or similar acts thereafter.- To prove its charge of conduct unbecoming of a company officer, more specifically of drinking alcoholic beverages within the premises of the company during office hours or going to work drunk, respondent relied on the statement/affidavit of Celeste Villena, the Agency Manager of the Wondrous and Miraculous General Agency under Metro Manila VI; and Marivic Uy, the General Manager of the D’MBP General Agency under Metro Manila VI. Both claimed that they always saw petitioner drunk during office hours, most especially during cut-offs when many sales counselors were present. Petitioner admitted having had drinking sessions with Certerio Uy, the husband of Marivic Uy, Ilustre Acosta and Reynaldo Villena, the husband of Celeste Villena, and who, according to petitioner, provided the hard liquor and the pulutan. He, however, denied reporting to office drunk and insisted that he reported for work sober.- We are inclined to give credence to petitioner’s claim, noting that in her handwritten letter-report to Norman Gonzales dated March 10, 2000, Villena made no mention of the petitioner going to office drunk. It was only in her affidavit dated January 16, 2002 that Villena made such declaration. Villena did not explain her failure to report the matter to Gonzales on March 10, 2000, and why she made the charge for the first time in her Affidavit dated January 16, 2002. Uy is the wife of no less than Certerio Uy, the Senior Vice-President of the Manila North Sales Division of respondent. If petitioner’s "drinking problem" had any ring of truth, she should have immediately reported the matter to her husband or to other officials concerned. Uy’s unexplained silence until March 10, 2000 thus renders her report implausible.- Respondent avers that petitioner served directly as agent of Caritas, a business competitor of the respondent, when he connived with his wife in recruiting Sales Associates of the Metro Sales Division VI to transfer to Caritas as sales associates. Respondent claims that, by his acts, petitioner failed to dedicate his full time on the job with respondent and prevented said sales associates from doing the same. Aside from violating its policy against conflict of interest, petitioner’s acts adversely affected his decisions in the performance of his duties and obligations to respondent. - Loyalty of an employee to his employer consists of certain very basic and common sense obligations. An employee must not, while employed, act contrary to the employer’s interest. The scope of the duty of loyalty that an employee owes to his employer may vary with the nature of their relationship. Employees occupying a position of trust and confidence owe a higher duty than those performing low-level tasks. Assisting an employee’s competitor can even constitute a breach of the employee’s duty of loyalty. An employee’s self-dealing may breach that duty. However, it has been ruled that- A reality of contemporary life is that many families will consist of two wage earners, one wage earner with two jobs, or both. For some employees, particularly those earning low or modest incomes, second sources of income are an economic necessity. For them, a second job or "moonlighting" is the only way to make ends meet. Conversely, employers

need the assurance that employees will not disserve them by furthering their own interests or those of competitors at the employers’ expense. - A slight assistance to a direct competitor could constitute a breach of the employee’s duty of loyalty. However, when competition is indirect or minimal, the employer may be required to show that the employee received substantial assistance from the competitor. If an employee usurped a corporate opportunity or secretly profited from a competitive activity, the employer may receive the value of the lost opportunity or the secret profit. - An employee’s skill, aptitude, and other subjective knowledge obtained in the course of employment are not the property of his employer. However, an employee occupying a managerial position or office is obliged to protect the trade secret of his employer consisting of formula, process, device or compilation which it uses in its business and gives it an opportunity to obtain an advantage over competitors who do not know of such trade secret. However, the rule does not apply to a matter of public knowledge or of general knowledge within the industry. Moreover, an employer has a protectible interest in the customer relationships of its former employee established and/or nurtured while employed by the employer, and is entitled to protect itself from the risk that a former employee might appropriate customers by taking unfair advantage of the contract developed while working for the employer. While acting as an agent of his employer, an employee owes the duty of fidelity and loyalty. Being a fiduciary, he cannot act inconsistently with his agency or trust. He cannot solicit his employer’s customers or co-employees for himself or for a business competitor of his employer. If such employee or officer connives with and induces another to betray his employer in favor of a business competitor of his employer, he is held accountable for his mischief. - In this case, we are not persuaded that Caritas is the business competitor of respondent. The evidence on record shows that while Abia, the Senior Vice-President of respondent’s Metro Manila Marketing, is one of the incorporators of Caritas and is even a member of the Board of Directors, respondent did not dismiss him from employment. The Head of the Legal Division of the respondent, Atty. Reyes, was also an incorporator of Caritas and a member of its Board of Directors, and although he appears to have sold his shares to Herminigildo C. Belen for P127,312.34, he only did so on March 7, 1999. There is no evidence on record whether the transfer of such shares of stocks has already been reflected in the books of Caritas. Celeste Villena, one of the Sales Associates of respondent, is herself licensed by Caritas to sell plans for the latter. Villena has likewise not been prohibited from selling pre-need plans for Caritas. Fe Molina, who is the head of a sales agency of Caritas, is also a sales agency head of respondent. Petitioner, his wife, and Villena were not charged nor meted any sanction by the respondent for conflict of interest. Petitioner was the Assistant Vice-President, Marketing Head, Area 10, of Caritas, and for a while, without any protest from respondent. If Caritas is a business competitor of the respondent, it should have meted sanctions not only on petitioner but also on Abia, Reyes, Fe Molina and Villena as well.- The truth of the matter is that, as averred by Caritas President Geoffrey Martinez, Caritas is engaged in health care and hospitalization package, whereas respondent sells educational, pension, and pre-need plans. Caritas is an HMO and is directly supervised by the DOH, while respondent is under the supervision of the SEC. The so-called sales associates of the respondent are non-salaried employees and are paid on commission basis only. Their commissions are based on their individual initiative and industry. That the contracts executed by the beneficiaries of both corporations have similar provisions regarding contract price, grace period, cancellation, reinstatement, transfer and termination, do not constitute proof that Caritas and respondent are business competitors. There is also no proof that the two corporations compete with each other in the same or similar business; in fact, the business of Caritas and that of the respondent complement each other.- Respondent relied on the declarations of Ruth Padiernos, Spouses Eppie and Ilustre Acosta, Celeste Villena, and Marivic Uy to prove its charge that Fe Molina pirated sales associates working for respondent and that petitioner tolerated the actuations of his wife and even connived with her.- The Court finds, however, that the evidence adduced by respondent insufficient to warrant the petitioner’s dismissal from employment.- Ruth Padiernos, wife of Roy Padiernos, averred in her written statement dated March 8, 2000, that as far back as July 1999, she had a conference with her husband and Abia where she reported that petitioner connived with his wife in pirating sales associates. She was assured that something would be done to arrest the problem.90 However, Ruth Padiernos failed to name any such sales associate who was recruited by Fe Molina. There is likewise no evidence that Abia ever confronted petitioner relative to the charge. Roy Padiernos confronted petitioner, but the latter denied the charge. Since then, no further action was taken against the petitioner by respondent, until the letter of Picazo dated March 21, 2000 was sent to him. Roy Padiernos did not explain why he executed his affidavit regarding the matter almost three years later, only on January 18, 2002. In an Affidavit dated January 18, 2002, it was made to appear that Ruth Padiernos claimed that petitioner’s wife, the Unit Manager of the Ark Group under Metro Manila Sales Group VI and also an Agency Manager of Caritas, recruited sales associates under respondent to work for Caritas, and that petitioner did the same; and that she (Padiernos) learned that almost all the productive Sales Associates in Metro Manila VI were already connected with Caritas, using "different names." Although notarized, the affidavit has no probative weight because it was unsigned. - Celeste Villena, for her part, declared in her handwritten statement dated March 10, 2000 that Fe Molina recruited Lenie Gatmaitan to join Caritas and that she confronted petitioner.92 In her Affidavit dated January 16, 2002, she alleged that petitioner and his wife, Fe Molina, recruited Gatmaitan to join Caritas. However, the signature of the notary public does not appear in said affidavit. For his part, Ilustre Acosta, averred in his

Labor Law 1 A2010 - 213 - Disinihandwritten statement dated March 11, 2000, that on March 4, 2000, petitioner informed him that Geoffrey Martinez called petitioner to inquire if petitioner would have no objection for him (Ilustre) to be with Caritas and that petitioner replied that he had no objection if that was Ilustre’s decision. Ilustre maintained this claim in his Affidavit dated January 16, 2002. Eppie Acosta, the wife of Ilustre Acosta, averred in her handwritten statement of March 12, 2000, that on March 6, 2000, petitioner commented about their low sales production, and she retorted that he was the cause, hence, may have grudges against him. Petitioner replied that he and his wife did not interfere with each other’s business dealings, and that petitioner even declared "Mare, for all you know, ikaw na lang ang hindi nag-ca-Caritas." She reiterated her claim in her affidavit dated January 16, 2000. Marivic Uy averred that the wife of petitioner had been pirating sales associates of respondent since 1999 to join Caritas and that she tried to recruit Morena Siasoco, one of the Group Managers. Petitioner failed to stop his wife, but rather tolerated her actuations. She reiterated her claim in her Affidavit dated January 16, 2002 - However, there is no evidence on record to prove that respondent expressly prohibited its Sales Associates from selling for Caritas. Neither is there evidence on record to prove that Caritas prohibited its sales associates from selling pre-need plans of respondent.- Respondent likewise failed to present the affidavits of Siasoco, Casaje, Magalso, San Miguel and Halili. In contrast to the evidence of respondent, Gatchalian, San Miguel, Siasoco, and Gatmaitan executed their respective affidavits declaring that neither petitioner nor his wife ever recruited them.99 They admitted that they sold plans for Caritas, but without any prodding from petitioner and his wife. Geoffrey Martinez declared, in his affidavit, that Siasoco, San Miguel, Casaje, Magalso, and Halili joined Caritas voluntarily and individually, through him, and he was not aware that petitioner and his wife recommended them to Caritas. Lenita Gatmaitan called him and inquired if she could join Caritas, and he replied in the affirmative. He never called petitioner concerning Ilustre Acosta; on the contrary, it was the latter who called to inquire if he was entitled to a discount if he purchased a Caritas health plan. He talked to Vilma Del Rosario and convinced her to apply as Branch Manager of Caritas, which she did, but backed out later on.Disposition IN LIGHT OF ALL THE FOREGOING, the instant petition is hereby GRANTED. The August 13, 2004 Decision and September 27, 2004 Resolution of the Court of Appeals are REVERSED AND SET ASIDE. The decision and resolution of the NLRC are reinstated.

WILLFUL DISOBEDIENCE

MICRO SALES OPERATION NETWORK V NLRC472 SCRA 328

QUISUMBING; October 11, 2005

NATUREFor review on certiorari of the Resolutions the CA dismissing petitioners’ special civil action for certiorari against the NLRC Resolution, which affirmed the Labor Arbiter’s Decision finding petitioners herein liable for illegal dismissal.

FACTS- Micro Sales Operation Network is a domestic corporation engaged in local transportation of goods by land. Petitioner Willy Bendol was the company’s operations manager at the time of the controversy. - Private respondents Larry Hermosa, Leonardo de Castro, and Ramil Basinillo were employed by the company as driver, warehouseman, and helper, respectively. Hermosa was hired on November 17, 1997, de Castro on February 1, 1996, and Basinillo on February 4, 1998.- Hermosa failed to promptly surrender the ignition key of the company’s vehicle after discharging his duties. Such failure was allegedly contrary to the company’s standard operating procedure. Thus, he was asked to explain within 24 hours why disciplinary action should not be meted on him. He explained that he kept the ignition key because the vehicle was stalled when its battery broke down. Unsatisfied with Hermosa’s explanation, the company dismissed him on January 9, 1999.- LA found that private respondents were illegally dismissed. NLRC affirmed the Labor Arbiter’s decision. It also denied petitioners’ motion for reconsideration.CA dismissed the petition for being defective in form.

ISSUES1. WON the private respondents were unjustly dismissed2. WON there was willful disobedience on the part of the private respondents, justifying their dismissal

HELD1. YES- Hermosa was unjustly dismissed2. NO- For willful disobedience to be a valid cause for dismissal, the following twin elements must concur: (1) the employee's assailed conduct must have been willful, that is, characterized by a wrongful and perverse attitude; and (2) the order violated must have been reasonable, lawful, made known to the employee and must pertain to the duties which he had been engaged to discharge.

- Both elements are lacking. We find no hint of perverse attitude in Hermosa’s written explanation. On the contrary, it appears that the alleged company procedure for leaving the ignition key of the company’s vehicles within office premises was not even made known to him. Petitioners failed to prove Hermosa willfully disobeyed the said company procedure. At any rate, dismissal was too harsh a penalty for the omission imputed to him.Disposition NLRC Resolution affirming the Labor Arbiter’s Decision, finding petitioners liable for illegal dismissal, is AFFIRMED.

BASCON V CA (METRO CEBU COMMUNITY HOSPITAL)

422 SCRA 122QUISUMBING; February 5, 2004

FACTS- ELIZABETH BASCON and NOEMI COLE, petitioners, were employees of Metro Cebu Community Hospital, Inc. (MCCH) and members of the Nagkahiusang Mamumuo sa Metro Cebu Community Hospital (NAMA-MCCH), a labor union of MCCH employees. Bascon had been employed as a nurse by MCCH since May 1984. At the time of her termination from employment in April 1996, she already held the position of Head Nurse. Cole had been working as a nursing aide with MCCH since August 1974. Both were dismissed for allegedly participating in an illegal strike.- The controversy arose from an intra-union conflict between the NAMA-MCCH and the National Labor Federation (NFL), the mother federation of NAMA-MCCH. In November 1995, NAMA-MCCH asked MCCH to renew their CBA, which was set to expire on December 31, 1995. NFL, however, opposed this move. Mindful of the apparent intra-union dispute, MCCH decided to defer the CBA negotiations until there was a determination as to which union had the right to negotiate a new CBA. Believing that their union was the certified CBA agent, NAMA-MCCH staged a series of mass actions inside MCCH’s premises starting February 27, 1996. The DOLE in Region 7 issued two certifications stating that NAMA-MCCH was not a registered labor organization. This finding, however, did not deter NAMA-MCCH from filing a notice of strike with the Region 7 Office of the National Conciliation and Mediation Board (NCMB). Said notice was, however, disregarded by the NCMB for want of legal personality of the union.- MCCH notified the petitioners that they were to be investigated for their activities in the mass actions. Petitioners, however, denied receiving said notices. In a notice dated April 8, 1996, MCCH ordered petitioners to desist from participating in the mass actions conducted in the hospital premises with a warning that non-compliance would result in the imposition of disciplinary measures. Petitioners again claimed they did not receive said order. Bascon and Cole were then served notices terminating their employment effective April 12, 1996 and April 19, 1996, respectively.- The Labor Arbiter found the termination to be valid and legal. The Labor Arbiter held that petitioners were justly dismissed because they actually participated in the illegal mass action. It also concluded that petitioners received the notices of hearing, but deliberately refused to attend the scheduled investigation.- The NLRC reversed the ruling and ordered the reinstatement of petitioners with full back wages. First, it found that petitioners merely wore armbands for union identity, per instruction of their union officials. Said wearing of armbands while nursing patients, is a constitutional right, which cannot be curtailed if peacefully carried out. Second, it ruled that the placards complained of by MCCH did not contain scurrilous, indecent or libelous remarks. Finally, it concluded that, in a belated but crude attempt to camouflage the illegal dismissal of petitioners, MCCH merely fabricated the notices allegedly sent to petitioners. On the charge of gross insubordination, it ruled that petitioners were not guilty, because the elements had not been sufficiently proven, to wit: (1) reasonableness and lawfulness of the order or directive, (2) sufficiency of knowledge on the part of the employee of such order, and (3) the connection of the order with the duties which the employee had been engaged to discharge.- MCCH filed a special civil action for certiorari before the CA. The CA granted the petition but ordered payment of separation pay.

ISSUES1. WON petitioners were validly terminated for allegedly participating in an illegal strike 2. WON petitioners were validly terminated for gross insubordination to the order to stop wearing armbands and putting up [of] placards

HELD1. NORatio While a union officer can be terminated for mere participation in an illegal strike, an ordinary striking employee must have participated in the commission of illegal acts during the strike. There must be proof that they committed illegal acts during the strike. But proof beyond reasonable doubt is not required. Substantial evidence, which may justify the imposition of the penalty of dismissal, may suffice.Reasoning - Article 264 (a) of the Labor Code provides in part that:

Any union officer who knowingly participates in illegal strike and any worker or union officer who knowingly participates in the commission of illegal acts during a strike may be declared to have lost his employment status…

Labor Law 1 A2010 - 214 - Disini- The CA found that petitioners’ actual participation in the illegal strike was limited to wearing armbands and putting up placards. There was no finding that the armbands or the placards contained offensive words or symbols. Thus, neither such wearing of armbands nor said putting up of placards can be construed as an illegal act. In fact, per se, they are within the mantle of constitutional protection under freedom of speech. Evidence shows that various illegal acts were committed by unidentified union members in the course of the protracted mass action. And we commiserate with MCCH, patients, and third parties for the damage they suffered. But we cannot hold petitioners responsible for acts they did not commit. The law, obviously solicitous of the welfare of the common worker, requires, before termination may be considered, that an ordinary union member must have knowingly participated in the commission of illegal acts during a strike.2. NORatio Willful disobedience of the employer’s lawful orders, as a just cause for dismissal of an employee, envisages the concurrence of at least two requisites: (1) the employee's assailed conduct must have been willful, that is, characterized by a wrongful and perverse attitude; and (2) the order violated must have been reasonable, lawful, made known to the employee and must pertain to the duties which he had been engaged to discharge.

Reasoning - Article 282 of the Labor Code provides in part:

An employer may terminate an employment for any of the following causes: (a) Serious misconduct or willful disobedience by the employee of the lawful orders of his employer or representative in connection with his work.

- We find lacking the element of willfulness characterized by a perverse mental attitude on the part of petitioners in disobeying their employer’s order as to warrant the ultimate penalty of dismissal. Wearing armbands and putting up placards to express one’s views without violating the rights of third parties, are legal per se and even constitutionally protected. Thus, MCCH could have done well to respect petitioners’ right to freedom of speech instead of threatening them with disciplinary action and eventually terminating them.- Neither are we convinced that petitioners’ exercise of the right to freedom of speech should be taken in conjunction with the illegal acts committed by other union members in the course of the series of mass actions. It bears stressing that said illegal acts were committed by other union members after petitioners were already terminated, not during the time that the latter wore armbands and put up placards.- Finally, even if willful disobedience may be properly appreciated, still, the penalty of dismissal is too harsh. Not every case of willful disobedience by an employee of a lawful work-connected order of the employer may be penalized with dismissal. There must be reasonable proportionality between, on the one hand, the willful disobedience by the employee and, on the other hand, the penalty imposed. In this case, evidence is wanting on the depravity of conduct and willfulness of the disobedience on the part of petitioners, as contemplated by law. Wearing armbands to signify union membership and putting up placards to express their views cannot be of such great dimension as to warrant the extreme penalty of dismissal, especially considering their long years of service and the fact that they have not been subject of any disciplinary action in the course of their employment with MCCH.Disposition Petition is GRANTED. The Decision of the CA is REVERSED. MCCH is hereby ordered to reinstate petitioners without loss of seniority rights and other privileges and to pay them full back wages, inclusive of allowances, and other benefits computed from the time they were dismissed up to the time of their actual reinstatement.

R TRANSPORT CORP V EJANELRA[PAGE 55]

B. GROSS AND HABITUAL NEGLECT OF DUTIES

REQUISITES

JUDY PHILIPIINES V NLRC289 SCRA 755

MARTINEZ; April 29, 1998

NATURESpecial civil action for certiorari to annul NLRC decision

FACTS- Virginia Antiola was employed as an assorter of baby infant dresses by Judy Philippines, Inc. in its export business. She was directed by her supervisor, to sort out baby infant dresses pursuant to an instruction sheet.

- Petitioner required Antiola to explain in writing why she should not be meted disciplinary sanctions for her erroneous assortment and packaging of 2,680 dozens of infant wear. She admitted her error and asked for forgiveness. Antiola’s supervisor and the packer also received a memo requiring them to explain why they should not be penalized. Both submitted their explanations.- Petitioner found Antiola guilty of negligence and she was dismissed from employment. The supervisor was suspended for one month on the ground of negligence through command responsibility. The packer was found innocent on the ground that when she undertook the packing of the infant wear, the same were already sealed in black plastic bags and could no longer be checked. - The National Federation of Labor Union (NAFLU), in behalf of Antiola, filed a complaint for unfair labor practice and illegal dismissal against Judy Philippines, Inc. They alleged that the dismissal was unjustified because the infant wear erroneously assorted by Antiola should not have been shipped to the buyer had the company’s supervisor and the buyer’s quality comptroller exercised due diligence in the performance of their duties in ensuring that the goods were properly assorted. - Labor arbiter held that the dismissal was lawful on the ground of fault and negligence causing an irreparable damage to the goodwill of the petitioners’ business, especially considering that the latter is an export oriented entity- NLRC held that to qualify as a valid cause for dismissal under Art. 282(b) of the Labor Code, neglect must not only be gross, it should be ‘Gross and habitual neglect’ in character. NLRC ordered petitioner to reinstate Antiola, with one year backwages

ISSUES1. WON the appeal before the NLRC had been seasonably made2. WON the offense committed by Antiola constitute a just cause for dismissal under article 282 of the labor code.

HELD1. YES- Under Article 223 of the Labor Code, as amended, the period to appeal to the Commission is ten calendar days, to wit:

Article 223. Appeal. - Decisions, awards, or orders of the Labor Arbiter are final and executory unless appealed to the Commission by any or both parties within ten (10) calendar days from receipt of such decisions, awards or orders.

- It is admitted that Antiola received the labor arbiter’s decision on May 2, 1990. She filed her appeal on May 14, 1990, a Monday.- In subsequent cases, We ruled that if the tenth day to perfect an appeal from the decision of the Labor Arbiter to the NLRC falls on a Saturday, the appeal shall be made on the next working day as embodied in Section 1, Rule VI of the NLRC Rules of Procedure promulgated on January 14, 1992. This conclusion recognizes the fact that on Saturdays the offices of NLRC and certain post offices are closed.- Even assuming arguendo that the appeal was filed beyond the period allowed by law, technical rules of procedure in labor cases are not to be strictly applied if the result would be detrimental to the working man.2. NO- Gross negligence implies a want or absence of or failure to exercise slight care or diligence, or the entire absence of care. It evinces a thoughtless disregard of consequences without exerting any effort to avoid them.- “Article 282 (b) of the Labor Code requires that xxx such neglect must not only be gross, it should be ‘ Gross and habitual neglect ’ in character .” - The employer’s obligation to give his workers just compensation and treatment carries with it the corollary right to expect from the workers adequate work, diligence and good conduct. - Considering however that private respondent worked with the company for 4 years with no known previous bad record, the ends of social and compassionate justice would be better served if she was merely suspended from work rather than terminated. - Petitioner should be reinstated but not awarded backwages. RA 6715, which provides that an illegally dismissed employee is entitled to full backwages, inclusive of allowances, and to his other benefits or their monetary equivalent computed from the time his compensation was withheld from him up to the time of his actual reinstatement, has no retroactive effect. Disposition NLRC decision AFFIRMED but MODIFIED in that petitioner. is ordered to pay private respondent Virginia Antiola backwages for a period of three years, without qualification or deduction.

CHAVEZ V NLRC[PAGE 59]

CHALLENGE SOCKS CORP V CA (NLRC, ANTONIO ET AL)

474 SCRA 356YNARES-SANTIAGO; November 8, 2005

NATURECERTIORARI under RULE 45

Labor Law 1 A2010 - 215 - DisiniFACTS- CHALLENGE SOCKS CORP (CSC) hired Elvie Buguat as knitting operator.- In the course of her employment, she incurred absences and tardiness without prior approval and had been neglectful of her duties. - May 25, 1998: she failed to check the socks she was working on causing excess use of yarn and damage to the socks’ design. - She was suspended for 5 days and warned that a repetition of the same act would mean dismissal from the service.- February 2, 1999: she committed the same infraction and was given a warning.- Despite the previous warnings, Buguat continued to be habitually absent and inattentive to her task. - March 1, 1999: she again failed to properly count the bundle of socks assigned to her. - March 2, 1999: CSC terminated her services on grounds of habitual absenteeism without prior leave, tardiness and neglect of work. - Thereafter, Buguat filed a complaint for illegal dismissal.[8]

- LA: Buguat was illegally dismissed; ordered CSC to reinstate her without loss of seniority rights and benefits, but w/o backwages; ruled that mistake in counting bundles of socks is tolerable and should be punished by suspension only. - NLRC: adopted the findings of LA. Denied CSC's Appeal and MR. - CA: reversed and set aside LA’s and NLRC’s decisions; CSC was ordered to pay BUGUAT full backwages; remanded to the Regional LA for the computation of the backwages.- CA also noted that petitioner failed to comply with the twin-notice requirement in terminating an employee hence, the dismissal was considered ineffectual.

ISSUEWON Buguat’s termination is valid

HELDYESReasoning- One of the just causes for terminating an employment under Article 282 of the Labor Code is gross and habitual neglect by the employee of her duties. This cause includes gross inefficiency, negligence and carelessness. Such just causes is derived from the right of the employer to select and engage his employees. - As a knitting operator, Elvie was required to check the socks she was working on and to count the bundles of socks she had to pack to be forwarded to the Looping Section.- Her repeated commission of the same offense could be considered willful disobedience. Elvie, despite the suspension and warning, continued to disregard the company rules and regulations…. - Habitual neglect implies repeated failure to perform one’s duties for a period of time. Buguat’s repeated acts of absences without leave and her frequent tardiness reflect her indifferent attitude to and lack of motivation in her work. Her repeated and habitual infractions, committed despite several warnings, constitute gross misconduct. Habitual absenteeism without leave constitute gross negligence and is sufficient to justify termination of an employee. - Her repeated negligence is not tolerable; neither should it merit the penalty of suspension only. - The record of an employee is a relevant consideration in determining the penalty that should be meted out. - An employee’s past misconduct and present behavior must be taken together in determining the proper imposable penalty. The totality of infractions or the number of violations committed during the period of employment shall be considered in determining the penalty to be imposed upon an erring employee. The offenses committed by him should not be taken singly and separately but in their totality. Fitness for continued employment cannot be compartmentalized into tight little cubicles of aspects of character, conduct, and ability separate and independent of each other.- It is the totality, not the compartmentalization, of such company infractions that Buguat had consistently committed which justified her dismissal. - Terminating an employment is one of petitioner’s prerogatives. - Management has the prerogative to discipline its employees and to impose appropriate penalties on erring workers pursuant to company rules and regulations. - The Court has upheld a company’s management prerogatives so long as they are exercised in good faith for the advancement of the employer’s interest and not for the purpose of defeating or circumventing the rights of the employees under special laws or under valid agreements. - In the case at bar, petitioner exercised in good faith its management prerogative as there is no dispute that Buguat had been habitually absent, tardy and neglectful of her work, to the damage and prejudice of the company. Her dismissal was therefore proper. - The law imposes many obligations on the employer such as providing just compensation to workers, observance of the procedural requirements of notice and hearing in the termination of employment. On the other hand, the law also recognizes the right of the employer to expect from its workers not only good performance, adequate work and diligence, but also good conduct and loyalty. The employer may not be compelled to continue to employ such persons whose continuance in the service will patently be inimical to his interests. - The employer has the burden of proving that the dismissed worker has been served two notices : ( 1) one to apprise him of the particular acts or omissions for which his dismissal is sought, and (2) the other to inform him of his employer’s decision to dismiss him.

- A review of the records shows that private respondent was served a written termination notice on the very day she was actually dismissed from the service. It was not shown that CSC notified Elvie in advance of the charge or charges against her nor was she given an opportunity to refute the charges made against her. - Agabon v. National Labor Relations Commission: Upheld as valid the dismissal for just cause even if there was no compliance with the requirements of procedural due process. While the procedural infirmity cannot be cured, it should not invalidate the dismissal. However, the employer should be held liable for non-compliance with the procedural requirements of due process.Disposition CA’S DECISION IS AFFIRMED; backwages is DELETED; Nominal damages (for violation of Buguat’s statutory due process) in the amount of P30,000.00.

GROSS AND HABITUAL NEGLIGENCE DEFINED

VALIAO V CA[PAGE 11]

REYES V MAXIM’S TEA HOUSE398 SCRA 288

QUISUMBING; February 27, 2003

NATUREPeition for review on certiorari of a decision of the Court of Appeals

FACTS- Respondent Maxim's Tea House (hereinafter Maxim's for brevity) had employed Reyes as a driver since October 1995. He was assigned to its M.H. del Pilar Street, Ermita, Manila branch. His working hours were from 5:00 P.M. to 3:00 A.M., and among his duties was to fetch and bring to their respective homes the employees of Maxim's after the restaurant closed for the day.- In the wee hours of the morning of September 27, 1997, petitioner was driving a Mitsubishi L300 van and was sent to fetch some employees of Savannah Moon, a ballroom dancing establishment in Libis, Quezon City. Petitioner complied and took his usual route along Julia Vargas Street in Pasig City. He was headed towards Meralco Avenue at a cruising speed of 50 to 60 kilometers per hour, when he noticed a ten-wheeler truck coming his way at full speed despite the fact that the latter's lane had a red signal light on. Petitioner maneuvered to avoid a collision, but nonetheless the van he was driving struck the truck. As a result, petitioner and seven of his passengers sustained physical injuries and both vehicles were damaged- The management of Maxim's required petitioner to submit, within forty-eight hours, a written explanation as to what happened that early morning of September 27, 1997. He complied but his employer found his explanation unsatisfactory and as a result he was preventively suspended for thirty (30) days. Subsequently, Maxim's terminated petitioner for cause.- Feeling that the vehicular accident was neither a just nor a valid cause for the severance of his employment, petitioner filed a complaint for illegal dismissal docketed as NLRC NCR Case No. 00-12-08773-97. In his decision, the Labor Arbiter found that petitioner was grossly negligent in failing to avoid the collision. Instead of filing the requisite pleading for appeal, petitioner filed a "Motion for Partial Reconsideration" with the NLRC. The NLRC opted to treat petitioner's motion as an appeal. The NLRC reversed the decision of the Labor Arbiter on the ground that there was no negligence on petitioner's part. Respondents moved for reconsideration of the foregoing decision, but said motion was denied by the Commission in its resolution- Respondents then filed a special civil action for certiorari with the Court of Appeals, The appellate court decided in favor of the employer and its manager. Hence, the instant case.

ISSUEWON petitioner’s dismissal from employment is valid and legal

HELDNO- The issue of whether a party is negligent is a question of fact. As a rule, the Supreme Court is not a trier of facts and this applies with greater force in labor cases. But where the findings of the NLRC and the Labor Arbiter are contradictory, as in this case, the reviewing court may delve into the records and examine for itself the questioned findings. - Under the Labor Code, gross negligence is a valid ground for an employer to terminate an employee. Gross negligence is negligence characterized by want of even slight care, acting or omitting to act in a situation where there is a duty to act, not inadvertently but willfully and intentionally with a conscious indifference to consequences insofar as other persons may be affected. In this case, however, there is no substantial basis to support a finding that petitioner committed gross negligence.- In sustaining the Labor Arbiter's finding that petitioner was grossly negligent, the appellate court stressed that the cited episode was the second vehicular accident involving petitioner, and as such it "may clearly reflect against [his] attitudinal character as a driver." The Court notes, however, that the Commission found that in the first vehicular accident involving petitioner "he was the victim of the reckless and negligent act

Labor Law 1 A2010 - 216 - Disiniof a fellow driver." An imputation of habitual negligence cannot be drawn against petitioner, since the earlier accident was not of his own making.

The test to determine the existence of negligence is as follows: Did petitioner in doing the alleged negligent act use that reasonable care and caution which an ordinarily prudent person would use in the same situation? It is not disputed that petitioner tried to turn left to avoid a collision. To put it otherwise, petitioner did not insist on his right of way, notwithstanding the green light in his lane. Still, the collision took place as the ten-wheeler careened on the wrong lane. Clearly, petitioner exerted reasonable effort under the circumstances to avoid injury not only to himself but also to his passengers and the van he was driving. To hold that petitioner was grossly negligent under the circumstances goes against the factual circumstances shown. It appears to us he was more a victim of a vehicular accident rather than its cause. - There being no clear showing that petitioner was culpable for gross negligence, petitioner's dismissal is illegal. Disposition Petition granted.

CEBU FILVENEER CORPORATION V NLRC [PAGE 194]

CITIBANK NA V GATCHALIAN240 SCRA 212

PUNO; January 18, 1995

FACTS - Petitioner bank received thirty-one (31) applications from alleged APBCI employees for the issuance of Citibank credit cards, popularly known as Mastercard.- A Citibank employee verified by phone the data which appeared on the application forms. It was Florence Verendia, as secretary of the APBCI General Manager, who answered the check calls. The applications were then approved and the corresponding new and unsigned credit cards were issued. Petitioner bank's policy is for new and unsigned credit cards to be released only to the cardholders concerned or their duly authorized representatives. However, a Citibank employee may himself take delivery of new and unsigned credit cards after accomplishing a Card Pull-Out Request Form wherein the employee assumes the responsibility of delivering the same to the cardholder concerned.- Supnad (an employee of bank) and Verendia, conspired together to get the fictitious cards. They got seven cards from bank employee Llonillo. As a result, the two (Supnad and Verendia) used the cards in commercial establishments causing injury to the bank in the amount of 200k.- the Bank found out about this and conducted an investigation- Investigation resulted in the decision to terminate Llonilla and to file charges against Verendia and Supnad-the labor arbiter ruled that Llonilla be reinstated based on evidence that what Llonilla did was not gross negligence

ISSUEWON Llonilla’s negligence was gross

HELDYES- Gross negligence implies a want or absence of or failure to exercise slight care or diligence, or the entire absence of care. It evinces a thoughtless disregard of consequences without exerting any effort to avoid them. The evidence on record succinctly established the gross negligence of respondent Llonillo. She admitted that the first time she was asked by Verendia to pick up one of the newly approved and unused credit cards, she immediately acceded. Yet at that time, she had not personally met nor previously seen Verendia. When asked how she came to know to whom she would give the card, respondent Llonillo responded that Verendia described herself over the phone and that was how she was able to identify Verendia when she first met her. Thus, on the basis of a mere description over the telephone, respondent Llonillo delivered the credit cards to Verendia.- Furthermore, not only is her negligence gross, it was also habitual it being found out that she picked up the newly approved credit cards on five (5) separate occasions and delivered the same to Verendia and the latter's messenger. Certainly, these repetitive acts and omissions bespeak of habituality.- Company says she’s grossly or habitually negligent in the performance of her duties. The SC said that since she has not been remiss in the performance of her duties in the past, she can’t be charged with habitual negligence. Neither is her negligence gross in character. Gross negligence implies a want or absence of or failure to exercise slight care or diligence or the entire absence of care. It evinces a thoughtless disregard of consequences without exerting any effort to avoid them. She had not the slightest reason to distrust Kun because he was the GM and appears to have conducted himself well in the performance of his duties in the past. At most, it’s error of judgment, not gross negligence. Disposition NLRC decision affirmed.

CHUA V NLRC (SCHERING-PLOUGH CORP ET AL)453 SCRA 244

MELO; March 11, 2005

NATUREPetition for review on certiorari of a decision and resolution of the CA

FACTS- On June 1, 1995, Dennis Chua was hired as a Professional Medical Representative by Schering-Plough Corporation (SPC), and thereafter became a regular employee on December 1, 1995.- As a Professional Medical Representative, he was tasked to promote SPC and its products to physicians, hospitals, paramedics, including trade and government outlets in his assigned territory. - One of the petitioner’s duties was to submit a Daily Coverage Report (DCR) every Monday, or at least to mail the same to the Field Operations Manager. Furthermore, he was required to have “call cards” signed by any of the eighty (80) doctors under his coverage to show that he indeed visited them and handed out promotional items. This system enabled the SPC to know how many doctors the petitioner had visited in a week and the number of call cards he was required to submit.- Respondent Roberto Z. Tada, Field Operations Manager of the corporation for the Bicol Region, noticed that the petitioner filed his DCRs late, and in batches at that. Specifically, a batch of DCRs up to January 10, 1997 was filed only on March 13, 1997, while another batch was filed only on March 18, 1997. The petitioner also failed to submit the DCRs for the period covering February 10, 1997 to April 7, 1997. Respondent Tada also found some discrepancies in the DCRs submitted by the petitioner.- On April 6, 1997, respondent Tada confronted the petitioner regarding the said discrepancies, to which Tada merely replied, “Pagbigyan mo na lang ako, boss. Tulungan mo na lang ako, boss.- On April 8, 1997, Tada went to the petitioner’s residence and confiscated all the paraphernalia used by the latter for his fieldwork, including the call cards and medicine samples. The car assigned to the respondent was likewise confiscated.- On April 9, 1997, the petitioner filed an application for a “three-day sick leave,” but indicated therein that he was going on leave only for two (2) days, from April 10 to 11, 1997. However, after the lapse of his applied leave of absence, the petitioner failed to report for work.- On April 15, 1997, the petitioner had already filed a complaint for illegal dismissal with the National Labor Relations Commission (NLRC) against the SPC, Epitacio Titong, Jr. (as President and General Manager), Danny T. Yu (as Division Manager) and Roberto Z. Tada (as Field Operations Manager- On April 16, 1997, the petitioner received a telegram from the SPC instructing him to report to the office on April 18, 1997 and to see respondent Danny T. Yu who was the Division Manager. The petitioner, however, failed to comply.- On April 18, 1997, respondent Tada sent a Memorandum to the petitioner requiring the latter to explain the late submission of DCRs, insufficiency of the information on the call cards, etc. - The same letter informed the petitioner that he was under preventive suspension effective April 11, 1997 while the case was under investigation.- On May 8, 1997, while the case for illegal dismissal was pending resolution before the arbitration branch of the NLRC, the SPC sent another letter to the petitioner, informing him that his employment was terminated. - On September 30, 1998, Labor Arbiter Ramon Valentin C. Reyes rendered a Decision declaring the petitioner’s dismissal from employment as illegal. The Labor Arbiter held that the SPC failed to establish any ground for the petitioner’s dismissal and ordered the SPC to reinstate him. - SPC appealed the decision of the Labor Arbiter to the NLRC.- On October 19, 1999, the NLRC issued a Resolution, finding respondent to have validly dismissed complainant.- The petitioner filed a motion for reconsideration of the said resolution, but the same was dismissed. - The petitioner sought relief from the CA, which affirmed, in toto, the resolution of the NLRC, and consequently denied the petitioner’s MFR

ISSUEWON petitioner’s dismissal form employment was illegal

HELDNO- The petitioner’s termination from employment was anchored on the following: (a) gross and habitual neglect; (b) serious misconduct; and (c) willful disobedience to the lawful orders of the employer. Thus, it all boils down to the filing of the requisite DCRs due every Monday. As found by both the NLRC and the CA, the petitioner failed to file the DCRs on time on several occasions, and instead filed them in batches. Furthermore, the petitioner failed to submit the DCRs for February 10, 1997 to April 7, 1997. Considering that about ninety percent (90%) of the petitioner’s work as a medical representative entails fieldwork, such DCRs were vital to his job; the DCRs were the primary basis upon which the petitioner’s employer could track his accomplishments and work progress. Without the said DCRs, the employer would have no basis to determine if the petitioner was actually performing his assigned tasks or not.

Labor Law 1 A2010 - 217 - Disini- In the same light, the petitioner also failed to submit several doctors’ call cards, and submitted others which were incomplete; that is, undated although signed by the doctors. It must be stressed that the said call cards were also vital to the petitioner’s fieldwork. The requirement of asking the doctors to affix their signatures in the call cards, the date of the visit, as well as the samples and promotional items, if any, given to the doctors, enabled the SPC to verify whether such doctors were indeed visited by the petitioner.- Gross negligence under Article 282 of the Labor Code, as amended, connotes want of care in the performance of one’s duties, while habitual neglect implies repeated failure to perform one’s duties for a period of time, depending upon the circumstances. Clearly, the petitioner’s repeated failure to submit the DCRs on time, as well as the failure to submit the doctors’ call cards constitute habitual neglect of duties. Needless to state, the foregoing clearly indicates that the employer had a just cause in terminating the petitioner’s employment.***But because there was a violation of the petitioner’s statutory right to two notices prior to the termination of his employment for a just cause, he is entitled to nominal damages of P30,000.00, absent sufficient evidence to support an award for actual or moral damages. (In line with the ruling in Agabon)Disposition The decision of the Court of Appeals is affirmed with modification that petitioner is entitled to above stated award for nominal damages..

GENUINO ICE CO INC V MAGPANTAY[PAGE 206]

PREMIER DEVT BANK V MANTAL[PAGE 210]

SIMPLE NEGLIGENCE

PAGUIO TRANSPORT CORP V NLRC (MELCHOR)294 SCRA 657

PANGANIBAN; August 28, 1998

NATURE Petition for review of NLRC decision

FACTS - Complainant Wilfredo Melchor was hired by respondent company as a taxi driver under the "boundary system.” He was to drive the taxi unit assigned to him on a 24-hour schedule per trip every two 2 days, for which he used to earn an average income from P500 to P700 per trip, exclusive of the P650 boundary and other deductions.- He was involved in a vehicular accident along Quirino Ave when he accidentally bumped a car. He was allegedly advised to stop working and have a rest. When reported for work, he was told that his service was no longer needed.- He then filed complaint for illegal dismissal.- Paguio maintained that Melchor was not illegally dismissed since there was no employer-employee relationship. (no control, no payment of compensation) Even if EER existed, complainant's termination arose out of a valid cause since he was already involved in 3 accidents.- NLRC ruling: there was illegal dismissal

ISSUES1. WON an employer-employee relationship exists2. WON dismissal was for a just cause3. WON Melchor was afforded due process4. WON ‘doctrine of strained relations’ applies

HELD 1. YESRatio The relationship of taxi owners and taxi drivers is the same as that between jeepney owners and jeepney drivers under the "boundary system." This relationship is that of employer-employee and not of lessor-lessee. The fact that the drivers do not receive fixed wages but get only the excess of that so-called boundary they pay to the owner/operator is not sufficient to withdraw the relationship between them from that of employer and employee. Reasoning - He was considered an employee because he was engaged to perform activities which were usually necessary or desirable in the usual trade of the employer. - This is different from lease of chattels, wherein the lessor loses complete control over the chattel leased. In the case of jeepney owners/operators and jeepney drivers, the former exercise supervision and control over the latter.2. NORatio Employer has the burden of proving that the dismissal of an employee is for a just cause. The failure of the employer to discharge this burden means that the dismissal is not justified and that the employee is entitled to reinstatement and backwages.

- Mere involvement in an accident, absent any showing of fault or recklessness on the part of an employee, is not a valid ground for dismissal.3. NORatio The twin requirements of notice and hearing are essential elements of due process. The employer must furnish the worker two written notices: (1) one to apprise him of the particular acts or omissions for which his dismissal is sought and (2) the other to inform him of his employer's decision to dismiss him. The essence of due process lies simply in an opportunity to be heard, and not always and indispensably in an actual hearing.4. NORatio Strained relations must be demonstrated as a fact. - The doctrine on "strained relations" cannot be applied indiscriminately since every labor dispute almost invariably results in "strained relations"; otherwise, reinstatement can never be possible simply because some hostility is engendered between the parties as a result of their disagreement. Reasoning - Paguio’s allegation that private respondent was incompetent and reckless in his manner of driving, which led to his involvement in three vehicular accidents, is not supported by the records. No evidence was properly submitted by petitioner to prove or give credence to his assertions.

C. FRAUD – WILLFUL BREACH OF TRUST

SANTOS V SAN MIGUEL CORPORATION399 SCRA 172

SANDOVAL-GUTIERREZ; March 14, 2003

NATUREPetition for review on certiorari

FACTS- Petitioner Carmelita Santos was appointed Finance Director of respondent SMC’s Beer Division for Luzon Operations. On September 15, 1987, SMC issued a Memorandum prohibiting the encashment of personal checks at respondent's Plants and Sales Offices. Thereafter, SMC noticed that petitioner encashed her 3 personal checks in various Metro Manila Sales Offices.SMC commenced an audit investigation. Petitioner received from respondent an inter-office memorandum requiring her to explain in writing why no disciplinary action should be taken against her in view of her unauthorized encashment of her 3 personal checks at respondent's sales offices.- Petitioner admitted that she encashed three personal checks at respondent's sales offices but claimed that such act was not irregular since all personnel in respondent's Beer Division were allowed to encash their personal checks at any sales office upon clearance from the region management concerned. She stated that her encashment of personal checks had prior clearance. She further clarified that only two of the three checks she encashed were dishonored for insufficiency of funds, but she promptly funded the checks upon receipt of notice of such dishonor, thereby causing no damage to respondent.- Meanwhile, the audit results revealed that, aside from petitioner's reported encashment of 3 personal checks, she had previously encashed 50 personal checks in varying amounts, which were not endorsed by the Sales Operations Manager or the Region Finance Officer. Additionally, petitioner encashed 2 other personal checks. After receiving such report, SMC formed an Investigating Panel to conduct a full-blown investigation.- The Investigating Panel found the encashment by petitioner of her personal checks with the region/sales offices as highly irregular transactions to the detriment of the Company. It recommended that Santos be terminated from employment. - In a memorandum, SMC adopted the findings of the Investigating Panel and informed petitioner of her termination from employment for abuse of position as Finance Director, engaging in highly irregular transactions to the detriment of the company and employer's loss of trust and confidence. - The complaint filed by petitioner against SMC for illegal dismissal was dismissed by the Labor Arbiter for lack of merit. The NLRC reversed the Labor Arbiter’s decision. Upon an MR filed by SMC, the NLRC dismissed the complaint filed by Santos. Hence, this recourse.

ISSUEWON SMC dismissed the petitioner from employment without just cause

HELD NO- Under the Labor Code, a valid dismissal from employment requires that: (1) the dismissal must be for any of the causes expressed in Article 282 of the Labor Code and (2) the employee must be given an opportunity to be heard and to defend himself.Article 282(c) of the same Code provides that "willful breach by the employee of the trust reposed in him by his employer" is a cause for the termination of employment by an employer. This ground should be duly established. Substantial evidence is sufficient as long as such loss of confidence is well-founded or if the employer has reasonable ground

Labor Law 1 A2010 - 218 - Disinito believe that the employee concerned is responsible for the misconduct and her act rendered her unworthy of the trust and confidence demanded of her position. It must be shown, though, that the employee concerned holds a position of trust. The betrayal of this trust is the essence of the offense for which an employee is penalized.- Petitioner argues that her position as Finance Director of respondent's Beer Division is not one of trust but one that is merely functional and advisory in nature. She possesses no administrative control over the plants and region finance officers, including cashiers. She reports to two superiors. Petitioner's argument is misplaced. As Finance Director, she is in charge of the custody, handling, care and protection of respondent's funds. The encashment of her personal checks and her private use of such funds, albeit for short periods of time, are contrary to the fiduciary nature of her duties.- Moreover, petitioner has functional control over all the plant and region finance officers, including cashiers, within the Luzon Operations Area. In fact, she is the highest ranking managerial employee for the finance section of the Luzon Beer Division Operations. Obviously, her position is a factor in abetting the encashment of her personal checks.- Indeed, there is substantial ground for respondent's loss of confidence in petitioner. She does not deny encashing her personal checks at respondent's sales offices and diverting for her own private use the latter's resources. The audit investigation accounted for all the checks she encashed, some of which were dishonored for insufficiency of funds. The Investigating Panel concluded that petitioner not only encashed her personal checks at respondent's sales offices, but also used company funds to temporarily satisfy her insufficient accounts. This Court has held that misappropriation of company funds, although the shortages had been fully restituted, is a valid ground to terminate the services of an employee of the company for loss of trust and confidence.- Petitioner contends that there is a prolonged practice of other payroll personnel, including persons in managerial levels, who encashed personal checks but remained unpunished by respondent. She asserts that her administrative superiors even encouraged her to encash her checks at the nearest sales office since her appearance at the bank for encashment would entail undue digression from her daily work routine.- Prolonged practice of encashing personal checks among respondent's payroll personnel does not excuse or justify petitioner's misdeeds. Her willful and deliberate acts were in gross violation of respondent's policy against encashment of personal checks of its personnel, embodied in its Memorandum. She cannot feign ignorance of such memorandum as she is duty-bound to keep abreast of company policies related to financial matters within the corporation. Equally unmeritorious are her claims that the acts complained of are regular, being with the knowledge and consent of her superiors, Francisco Gomez de Liano and Ben Jarmalala, and that she is being charged because she resisted the sexual advances of her superior. Suffice it to state that she could have proved these matters during the investigation had she attended the proceedings.

LAKPUE DRUG INC V BELGA[PAGE 208]

LOSS OF CONFIDENCE – REQUISITES

JARDINE DAVIES INC V NLRC (REYES)311 SCRA 289

QUISUMBING; July 28, 1999

FACTS- Petitioner, the exclusive distributor of “Union 76” lubricating oil, engaged the services of a private investigation agency due to reports that petitioner’s products, particularly Union 76, were being illegally manufactured, blended, packed and distributed. Upon confirmation of the investigator through a surveillance report and having secured a search warrant, petitioner seized some of the fake items found in the apartment of private respondent, a former sales representative of petitioner. - a criminal complaint for unfair competition violating Article 189 of the RPC (repealed by Section 239 of the Intellectual Property Code) was filed against Reyes along with administrative charges for serious misconduct inimical to the interest of petitioner. He was advised to go on an indefinite leave which later led to his termination. - the materials seized were released in view of a petition filed by Reyes’ younger brother Donato, who convinced the court that the materials belonged to him and that he was legally engaged in the business of general merchandising (Lubrix Conglomerate) reselling oil and lubricant products to the public.- with that, Reyes sued petitioner for illegal dismissal but the complaint was dismissed by the Labor Arbiter as he thought otherwise. Upon appeal with the NLRC, the decision was reversed on the ground that there was no cogent reason for petitioner to lose trust and confidence in private respondent, there being “no shadow of an act amounting to serious misconduct, fraud or breach of trust.”- petitioner’s MFR was denied, hence this petition.

ISSUEWON there was reason for petitioner to lose trust and confidence in private respondent and justify his dismissal

HELDNO

Ratio The right of an employer to dismiss employees on account of loss of trust of confidence must not be exercised arbitrarily and without showing just cause, so as not to render the employee’s constitutional right to security of tenure nugatory. Reasoning- Article 282 provides that an employer may terminate an employment for fraud or willful breach by the employee of the trust reposed in him by his employer. It is settled that loss of confidence as a just cause for termination must be premised on the fact that an employee concerned holds a position of trust and confidence, as in this case. And in order to constitute just cause, the act complained of must be work-related. Proof beyond reasonable doubt is not required, so long as there is some basis for the loss of confidence, but basis thereof must still be clearly and convincingly established, arising from particular proven facts which the employer bears to prove.- in the instant case, the surveillance report of the private investigator was unreliable as the conclusions therein were mere deductions not supported by substantial corroborating evidence. Petitioner also failed to controvert proof presented by private respondent that the reselling of the oil was in support of petitioner’s marketing policy. It was also odd that petitioner’s agents did not submit the alleged fake merchandise to be tested in their labs, virtually affirming the articles were genuine, having been purchased from petitioner’s dealers. - another confirmation that petitioner lacked basis for its distrust of private respondent was the release of the seized articles, with Donato even presenting receipts to prove they were purchased from authorized dealers. - Considering this, private respondent was illegally dismissed. As such, he is entitled to backwages. Since he was terminated before the effectivity of RA 6715, he is entitled to only 3 years of backwages, and not full backwages as would be granted now. Because the antagonism and imputations of the criminal act strained the parties’ relationship, reinstatement would not be feasible. Instead, a more equitable disposition would be an award of separation pay. Disposition instant petition is DENIED for lack of merit

PLDT V TOLENTINO[PAGE 202]

DELA CRUZ V NLRC[PAGE 100]

PHILIPPINE NATIONAL CONSTRUCTION

CORPORATION V MATIAS458 SCRA 148

PANGANIBAN; May 6, 2005

FACTS- Rolando Matias was employed by Construction and Development Corporation of the Philippines (CDCP) as Chief Accountant and Administrative Officer. During his employment with the company, various parcels of land situated at Don Carlos Bukidnon were placed in the names of certain employees as trustees for the purpose of owning vast tracts of land more than the limit a corporation can own which were primarily intended for CDCP agricultural businesses. By internal arrangement documents transferring back the properties to the corporation were executed. A piece of land was registered in the name of Matias.- CDCP was later converted a government owned or controlled corporation, and the name of CDCP was changed to Philippine National Construction Corporation (PNCC). Under a new set up, PNCC offered a retrenchment program and on December 31, 1984 Matias availed of the said program.- Sometime in 1985, the Conjuangco Farms owned by Mr. Danding Conjuangco acquired CDCP Farms Corporation wh[ich] took over the operations of said farms. Not long after, or in 1989, CDCP Farms Corporation ceased to operate.- In July 1992, two former CDCP employees, namely Reynaldo Tac-an and Luciano Tadena went to the house of Matias and brought with them duly accomplished documents and Special Power of Attorney for his signature and informed him that the lands in Bukidnon under his name with all the others were invaded by squatters, and that the said land were covered by the Comprehensive Agrarian Reform Program (CARP) where Matias’ name was included in the list of landowners. Matias reluctantly signed the document and after six months, he signed an acknowledgment receipt of P100,000.00. - The original title registered in the name of Matias was cancelled and a new title was issued. The transfer of said parcel of land was made possible because Rolando Matias and Elena Esmeralda Matias received manager’s checks from the Land Bank of the Philippines in the amount of P102,355.96 and P219.22 and bond worth P203,478.48 as payment of Land Transfer Acquisition.- On August 12, 1996, Matias was rehired by PNCC as Project Controller in Zambales PMMA Project. - Not long after, Mr.Alday, Head of the Realty Management Group of PNCC invited Matias to his office and showed him a listing of parcels of land in the name of different persons with the corresponding status including the latter’s name. On the basis of the listing, Mr. Alday told Matias that the transfer of the property registered in the latter’s

Labor Law 1 A2010 - 219 - Disininame was not yet consummated by the LBP and then requested Matias to execute a Deed of Assignment in favor of PNCC pertaining to the said property, which Matias did and guaranteed in writing that the ‘parcel of land is free from any lien or encumbrance.’- On April 20, 1998, a memorandum was issued to Matias by PNCC directing the former to explain in writing why none of the following actions, falsification, estafa, dishonesty, and breach of trust and confidence, should be taken against him in connection with the Deed of Assignment. PNCC alleges that respondent fraudulently breached its trust and confidence when, without its knowledge and consent, he disposed of the Bukidnon property; though actually belonging to petitioner, that property had purportedly been merely placed in trust under his name. Thereafter, he assigned the same property to petitioner, allegedly despite his full knowledge that the title had already been transferred -- with his active planning and participation -- to the Republic of the Philippines . - In due time, Matias submitted his written explanation. However, he was later advised that he was terminated from the service on the ground of loss of trust and confidence. Hence, Matias filed a complaint for illegal dismissal and money claims against PNCC alleging that the dismissal on the ground of loss of trust and confidence was without basis.”

ISSUEWON the dismissal of Matias on the ground of loss of trust and confidence was without basis

HELDYESRatio: To constitute a valid cause to terminate employment, loss of trust and confidence must be proven clearly and convincingly by substantial evidence. To be a just cause for terminating employment, loss of confidence must be directly to the duties of the employee to show that he or she is woefully unfit to continue working for the employer.Reasoning- Undeniably, the position of project controller -- the position of respondent at the time of his dismissal -- required trust and confidence, for it related to the handling of business expenditures or finances. However, his act allegedly constituting breach of trust and confidence was not in any way related to his official functions and responsibilities as controller. In fact, the questioned act pertained to an unlawful scheme deliberately engaged in by petitioner in order to evade a constitutional and legal mandate.- It has oft been held that loss of confidence should not be used “as a subterfuge for causes which are illegal, improper and unjustified. It must be genuine, not a mere afterthought to justify an earlier action taken in bad faith.” Be it remembered that at stake here are the sole means of livelihood, the name and the reputation of the employee. Thus, petitioner must prove an actual breach of duty founded on clearly established facts sufficient to warrant his loss of employment.- We stress once more that the right of an employer to dismiss an employee on account of loss of trust and confidence must not be exercised whimsically. To countenance an arbitrary exercise of that prerogative is to negate the employee’s constitutional right to security of tenure. In other words, the employer must clearly and convincingly prove by substantial evidence the facts and incidents upon which loss of confidence in the employee may be fairly made to rest; otherwise, the latter’s dismissal will be rendered illegal.

CRUZ V CA (NLRC, CITYTRUST BANK)494 SCRA 226

AUSTRIA-MARTINEZ; July 12, 2006

NATURE Special civil action for certiorari under Rule 65 PROC seeking to annul CA decision affirming NLRC decision and resolution .

FACTS - Felix Cruz was an employee of Ciytrust Banking Corporation. He held a confidential position of Micro Technical Support Officer, whose duties include: evaluating and recommending requests for Micro Computers received by the bidding committee, further evaluating and accepting of bids done by the Technical Commitee. He was recognized with awards and citations due to his good performance. - There were feedbacks and informations that there were irregularities in the bidding process and purchase of the computers. A special investigation was conducted which found that there were unauthorized and unreported commissions and rebated given out by one of its computer suppliers (MECO) for purchases made by Citytrust.- Citytrust sent a show cause memorandm to Cruz placing him under preventive suspension and directing him to appear in an administrative hearing by the Ad Hoc Committee. The committee found him guilty of fraud, serious misconduct, gross dishonesty and serious violation of the bank policies. For the resultant loss of confidence, Citytrust terminated Cruz from employment. - Cruz filed before the Labor Arbiter an action for illegal dismissal and damages for being denied due process and hastily dismissed. LA decision favored Cruz. - Citytrust appealed to the NLRC, setting aside LA decision and dismissing the case fro lack of merit. Cruz filed MFR, but was denied for lack of merit. - Cruz filed petition for Certiorari with SC, which was referred to the CA for appropriate action and disposition.

- CA dismissed the petition. It held that although the signature of the petitioner does not appear in the check vouchers, other pieces of evidence prove that he benefited from the proceeds of the checks issued and that there is substantial evidence to hold the petitioner liable for soliciting; that his acts constituted a willful breach of the supplier’s trust and confidence; that the dismissal was the result of a thorough investigation and hearing.

ISSUES1. WON CA committed grave abuse of discretion2. WON he denied due process

HELD1. NO- Petitioner failed to prove such. - Petitioner was dismissed on the ground, among others, of loss of trust and confidence. Loss of trust and confidence, as a valid ground for dismissal, must be substantiated by evidence. - WRT to rank-and-file personnel, loss of trust and confidence requires proof of involvement in the alleged events in question. But as regards a managerial employee, the mere existence of a basis fro believing that such employee has breached the trust of his employer would suffice for his dismissal. Proof beyond reasonable doubt is not required, it being sufficient that there is some basis for such loss of confidence such when the employer has reasonable ground to believe that the employee concerned id responsible for the purported misconduct, and the nature of his participation renders him unworthy of the trust and confidence demanded by his position. - Art 282 ( c) LC states that the loss of trust and confidence must be based on willful breach. It should be done intentionally, knowingly and purposely without justifiable excuse. It must not be indiscriminately used as a shield by the employer against a claim that the dismissal of an employee was arbitrary. And, in order to constitute a just cause for dismissal, the act complained of must be work-related and shows that the employee concerned is unfit to continue working for the employer. In addition, loss of confidence is premised on the fact that the employee concerned holds a position of responsibility, trust and confidence or that the employee concerned is entrusted with confidence with respect to delicate matters. The betrayal of this trust is the essence of the offense for which an employee is penalized- Cruz’s job entails the observance of proper company procedures. His functions are also extended to all branches nationwide, involving high degree of responsibility requiring a substantial amount of trust and confidence. - Petitioner’s acceptance of commissions and rebates from MECO, without knowledge and consent from Citytrust, and without said rebates being reported and turned over to the latter, are acts which can be considered willful breach of the trust and confidence reposed by Citytrust on him.- An employer cannot be compelled to retain an employee who s guilty of acts inimical to the interests of the employer. 2. NO, he was not denied due process.- The basic requirement of notice and hearing in termination cases is for the employer to inform the employee of the specific charges against him and to hear his side and defenses. This does not mean a full adversarial proceeding. The parties may be heard through pleadings, written explanations, position papers, memorandum or oral argument. In all of these instances, the employer plays an active role by providing the employee with the opportunity to present his side and answer the charges in substantial compliance with due process.- The fact alone that he was not able to confront the witnesses against him during the investigation conducted by Citytrust does not mean that he was denied his right to due process. What is frowned upon is the absolute lack of notice and hearing.- Citytrust complied with the first requirement of notice when it informed petitioner through a letter of the charges against him, directing him to explain in writing why his employment should not be terminated and to appear in a hearing to be conducted by the company to give him further opportunity to explain his side. Citytrust also complied with the second requirement of notice when it sent a memorandum informing him of his dismissal from employment and the reasons therefore. Dispositon instant petition is DISMISSED for lack of merit.

BREACH OF TRUST – LOSS OF CONFIDENCE

CENTRAL PANGASINAN ELEC CORP V MACARAEG[PAGE 195]

POSITION, TRUST AND CONFIDENCE

SANTOS V SAN MIGUEL CORP[PAGE 219]

PANDAY V NLRC (LUZON MAHOGANY TIMBER INDUSTRIES INC)

209 SCRA 122

Labor Law 1 A2010 - 220 - DisiniGUTIERREZ; May 20, 1992

NATUREPetition seeking the review of the order rendered by the NLRC authorizing the separation from the service of Panday to the payment by the private respondent of separation pay equivalent to one-half month salary for every year of service. It likewise ordered the payment of the complainant's 13th month pay for 1977 but dismissed his claim for living allowance for lack of merit.

FACTS- Panday was hired by Luzon Mahogany Timber Industries since Aug. 23, 1973. Sometime in Dec. 1977, Panday was called by Martin Gaw, the owner-manager who instructed him to cut off the living allowance of the employees. Panday requested that a memorandum to this effect be made so that he would not be blamed by the workers. Gaw got angry and shouted "what for is the memorandum? I am telling you to do so." He then butted, "Ano ba talaga Naning ang ibig mong sabihin? Sa tuwing magsasalita ka, panay ka "policy" ng companya at panay ka records". - From the time of that incident, Panday was deprived of free light. He was no longer given any accounting work. His per diem was abruptly cut off. All that was left for him to do was the simple clerical job of registering or paying SSS premiums. Still complainant continued to bear it out. In 1979, however he was totally divested of all his duties and he was compelled to approach Manager Martin Gaw to clear up matters. Gaw referred him to Mr. Gerry Lumban who was supposed to give him some work to do. It turned out, however, that no such instructions were given to said Mr. Lumban. - In 1979, Panday filed a request for vacation leave with pay for 15 days from April 14-30, 1979. On that same day he brought his son to Manila for medical treatment and stayed there up to the end of the month. Upon his return to Isabela, he asked for his salaries only to learn that his application for leave was disapproved.- Hence, he filed this case for illegal dismissal, non-payment of 13th month pay for 1977, emergency allowance under P.D. 525 since 1975 up to 1977 and unpaid wages for April 16-30, 1979. - The Office of the Minister found and ruled that Panday was constructively dismissed from the service. Luzon Timber was ordered to comment on the petition but failed to so. Thus, the actual findings are affirmed. The only complaint of Panday which remains is his claim that Deputy Minister Vicente Leogardo, Jr. should have ordered his reinstatement with backwages.

ISSUEWON Panday’s prayer for reinstatement should have been granted

HELDNO- Panday, as branch accountant occupied a position involving trust and confidence and in the light of the estranged relation between the complainant and the respondent that may not permit the full restoration of an employment relationship based on trust and confidence, we have to allow termination of the employer-employee relationship but upon the payment of separation pay equivalent to one-half (1/2) month for every year of service rendered.- The case of Lepanto Consolidated Mining Co. v. Court of Appeals provides a definition of a "position of trust and confidence". It is one where a person is "entrusted with confidence on delicate matters, or with the custody, handling, or care and protection of the employer's property"- A few examples were given by the Court in the case of Globe-Mackay Cable and Radio Corporation v. National Labor Relations Commission and Imelda Salazar, G.R. No. 82511, March 3, 1992, to illustrate the principle:

- where the employee is a Vice-President for Marketing and as such, enjoys the full trust and confidence of top management- or is the Officer-In-Charge of the extension office of the bank where he works - or is an organizer of a union who was in a position to sabotage the union's efforts to organize the workers in commercial and industrial establishments - or is warehouseman of a non-profit organization whose primary purpose is to facilitate and maximize voluntary gifts by foreign individuals and organizations to the Philippines - or is a manager of its Energy Equipment Sales- Credit and Collection Supervisor (Tabacalera Insurance Co. v. National Labor Relations Commission)

- If the respondent had been a laborer, clerk or other rank-and-file employee, there would be no problem in ordering her reinstatement with facility. An officer in such a key position as Vice President for Marketing(or as Chief Accountant as in the present case) can work effectively only if she enjoys the full trust and confidence of top management. - The case of Metro Drug Corp. v. National, Labor Relations Commission, aptly describes the difference in treatment between the positions of trust on one hand and mere clerical positions on the other. It states:

Managerial personnel and other employees occupying positions of trust and confidence are entitled to security of tenure, fair standards of employment, and the protection of labor laws. However, the rules on termination of employment, penalties for infractions, and resort to concerted action are not necessarily the same as those for ordinary employees.

A special and unique employment relationship exists between a corporation and its cashiers. More than most key positions, that of cashier calls f or the utmost trust and confidence. . . .When an employee accepts a promotion to a managerial position or to an office requiring full trust and confidence she gives up some of the rigid guaranties available to ordinary workers. Infractions which if committed by others would be overlooked or condoned or penalties mitigated may be visited with more severe disciplinary action. A company's resort to acts of self-defense would be more easily justified. It would be most unfair to require an employer to continue employing as its cashier a person whom it reasonably believes is no longer capable of giving full and wholehearted trustworthiness in the stewardship of company funds.

- Reinstatement in the present case is no longer possible not only because of the strained relationship between the employee and the employer but also because of the length of time that has passed from the date the incident occurred to its resolution. Instead of reinstating the employee, this Court has in several cases awarded separation pay although the employee was found to be illegally dismissed.

The following reasons have been advanced by the Court for denying reinstatement- reinstatement can no longer be effected in view of the long passage of time - because of the realities of the situation - that it would be inimical to the employer's interest- that reinstatement may no longer be feasible - that it will not serve the best interests of the parties involved - that the company would be prejudiced and by the workers' continued employment- that it will not serve any prudent purpose as when supervening facts have transpired which make execution on that score unjust or inequitable

Disposition the prayer for reinstatement is DENIED but the order rendered by Deputy Minister Vicente Leogardo, Jr. dated May 29, 1984 is modified to cover five (5) years backwages. The order is AFFIRMED in other respects.

CRUZ V COCA-COLA BOTTLERS PHILS INC460 SCRA 340

YNARES-SANTIAGO; June 15, 2005

FACTS- Cruz has been working for respondent company’s plant in Calamba, Laguna, as a driver/helper since June 1983. At times, however, Cruz gets designated as Acting Salesman for company’s soft drinks and other beverages. On July 25, 1998, petitioner was assigned as acting salesman of Route DA1, covering the small barangays. Together with his helper, Mr. Pablito Aguila, Cruz loaded their truck with CCBPI products. After the required verification and confirmation of the products loaded on the truck by the Checker and the guard at the gate, Cruz proceeded to leave the plant vicinity.- After gate inspection, however, Cruz drove back inside the plant on the pretext of refueling. While waiting in line to refuel, Cruz allegedly asked Aguila to load an additional thirty cases of assorted canned soft drinks as “plus load”. Aguila reminded him about the required documents but he merely stated “Ayos na” and continued with the refueling of the truck.- On his second exit from the plant premises, Cruz did not slow down for the mandatory inspection even as the security guards at the gate flagged him down. - One of the guards pursued the truck and when he caught up with petitioner at the Walter Mart Shopping Mall in Barangay Real, Calamba, the latter could not produce the proper documents for the extra thirty cases loaded on his truck. He was then directed to return to the plant and unload the products. At this point, it was confirmed that Cruz did not actually secure any paper for the added products nor did he follow the established procedure before taking out the extra cases.- Cruz admitted the incident but alleged that he forgot to secure the requisite documents for the products. On August 5, 1998, an investigation was conducted on the alleged violations committed by petitioner. On August 19, 1998, respondent company terminated the services of petitioner effective upon receipt of the memorandum.

ISSUEWON Cruz was validly dismissed

HELDYES- The Labor Arbiter, the NLRC and the Court of Appeals were unanimous in their findings that petitioner was guilty of dishonest acts but differed only on the propriety of the penalty imposed upon petitioner.- After a careful evaluation of the evidence on record of this case, we found no compelling reason to disturb the unanimous findings of the Court of Appeals, the NLRC and the Labor Arbiter. - Several factors militate against petitioner’s claim of good faith. Petitioner’s length of service, which spans almost fifteen years, works against his favor in this case. We have held that the longer an employee stays in the service of the company, the greater is his responsibility for knowledge and compliance with the norms of conduct and the code of

Labor Law 1 A2010 - 221 - Disinidiscipline in the company. Considering that petitioner has worked at respondent company for a long period of time, one expects that securing the LOGP or TGP would be automatic for him.- Faced with the overwhelming evidence presented by respondents on one hand and the mere general denial of petitioner on the other, the invocation of the protective mantle of the law in favor of labor cannot be upheld in this case. This principle cannot be adopted where there is clear and convincing evidence of the truth. While this court endeavors to live up to its mandate that the workingman’s welfare should be the primordial and paramount consideration, it cannot do so if it will be at the expense of justice and will result in the oppression or self-destruction of the employer. The interests of both the employers and employees are intended to be protected and not one of them is given undue preference. - Termination of employment by reason of loss of confidence is governed by Article 282(c) of the Labor Code, which provides that an employer can terminate the employment of the employee concerned for “fraud or willful breach by an employee of the trust reposed in him by his employer or duly authorized representative.” Loss of confidence, as a just cause for termination of employment, is premised on the fact that the employee concerned holds a position of responsibility, trust and confidence. He must be invested with confidence on delicate matters such as the custody, handling, care and protection of the employer’s property and/or funds. - Admittedly, the company rules violated by Cruz are punishable, for the first offense, with the penalty of suspension. However, company has presented evidence showing that Cruz has a record of other violations from as far back as 1986. To be sure, the nature of petitioner’s offenses is downright inimical to the interests of respondent company. By virtue of his job, Cruz is entrusted with the property and funds, which belong to respondent company. His actions on that fateful day highlight, not only his consistent and deliberate defiance of company rules and regulation, but also his duplicity in handling respondent company’s properties. It would appear that company had tolerated his work ethic far too long. We therefore find that it was justified in terminating petitioner after the flagrant dishonesty he committed.Disposition Instant petition is DENIED. Dismissal of petitioner is declared valid but respondent company is ORDERED to pay petitioner the amount of P20,000.00 as nominal damages for non-compliance with statutory due process.

GUIDELINES

VITARICH CORP V NLRC (RECODO)307 SCRA 509

BELLOSILLO; May 20, 1999

NATURE Special Civil action in the SC. Certiorari

FACTS- Private respondent, Isagani Recodo, started working at Vitarich as an Accounting clerk. He gradually moved up the organization ladder until he was made Sales Manager for Western Visayas in 1988. He was dismissed in October 15, 1992 for alleged violation of a memorandum dated August 4, 1992 and also for violation of company policies relating to credit extensions and cash advances. He was also terminated for loss of trust and confidence.- Apparently, his new boss, Onofre Sebastian, was under pressure from senior management to address and correct all the problems he had inherited from his predecessor. The problems included high account receivable level in the sales territory of Recodo. The two had a meeting sometime middle of July to address the problems, including the A/R level of one Rex Cordova. - The August 4 Memorandum referred to contains instructions to Recodo to ground salesmen with thirty say overdue A/R so that the levels of said A/R can be regularized. Apparently, Recodo received the said memo garbled and had to verify its contents on September 5, 1992. In the meantime, he postponed the grounding of Cordova until August 20 to bring about the desired reduction. The reduction hoped for in fact happened when Cordova’s A/R went down from Pesos 800,000 to just Pesos 250,000. Huffing and puffing, Sebastian was asked to explain why he should not be terminated for failure to ground Cordova in compliance with the August 4 memo of Sebastian.- Recodo complied with the order to explain and an investigation was conducted by the Head of Personnel, a certain Enriquez. In his report, Enriquez found that there was no defensible ground for terminating Recodo’s services in the absence of documented warnings given to Recodo to justify any loss of trust and confidence in him. Nonetheless, Recodo was terminated on October 15, 1992.- Private respondent filed a complaint for illegal termination, non-payment of managerial bonus, and for moral and exemplary damages. The Labor Arbiter ruled illegal dismissal. The NLRC initially overturned the ruling but on appeal by Recodo, the finding of the Labor Arbiter was upheld. Hence this action. ISSUEWON the NLRC committed a grave abuse of discretion in finding in favor of Recode

HELDNO

. In rectifying its previous appreciation and assessment of Recodo’s dismissal, the NLRC did not commit any abuse of discretion. A careful scrutiny of the records reveal that the decision of the Labor Arbiter is suffused with established facts and a correct understanding of them. Reasoning - While it may be true that there was a delay on the part of Recodo in implementing his superior’s order with regard Cordova’s grounding, the question is whether the delay constitutes disobedience and whether this disobedience was willful to merit loss of confidence. The SC, in AHS Philippines, Inc. vs. CA, explained that “willful disobedience of the employer’s lawful orders, as a just cause for dismissal of an employee, envisages the concurrence of at least two requisites:

a. the employee’s assailed conduct must be willful or intentional, the willfulness being characterized by a wrongful and perverse attitude;b. the order violated must have been reasonable, lawful, made known to the employee and must pertain to the duties which he had been engaged to discharge.

- In the case at bar, the non-compliance by Recodo was not an open defiance but “as one of the discretions which he had to take under the circumstances in his capacity as sales manager. As it turned out, the result both Recodo and Sebastian hoped for was achieved by not immediately grounding Cordova. - While an employer is allowed wide latitude to dismiss employees on loss of trust and confidence, still the loss thereof must have some basis and must be proved by the employer otherwise the social justice policy of the labor lawsand the constitution will be for naught. The guidelines for the application of the doctrine of loss of confidence are:

a. loss of confidence should not be simulatedb. it dhould not be used as subterfuge for causes which are improper, illegal, or unjustifiedc. it should not be arbitrarily asserted in the face of overwhelming evidence to the contraryd. it must be genuine, not a mere afterthought to justify earlier action taken in bad faith

Disposition the resolution f the NLRC is affirmed with the modification that corresponding back wages of respondent be forthwith updated and released to him.

COCA-COLA BOTTLERS PHIL INC V KAPISANAN NG MALAYANG MANGGAGAWA SA COCA-COLA

[PAGE 209]

WILLFUL BREACH

ATLAS CONSOLIDATED MINING & DEVELOPMENT CORP V NLRC (VILLACENCIO)

290 SCRA 479PUNO; May 21, 1998

NATUREpetition for certiorari under Rule 65 of the Revised Rules of Court of Decision dated December 27, 1994 of NLRC which ordered the payment of separation pay and backwages to private respondent Isabelo O. Villacencio, and its Resolution dated August 18, 1995 denying petitioner's Motion for Reconsideration.

FACTS- private respondent Isabelo O. Villacencio worked with petitioner ACMDC from January 23, 1970 to February 2, 1990. He started as an ordinary laborer/helper in the Mill Department. In 1973, he became supervisor of the Tailings Disposal Department. In 1982, he was elevated as a junior staff of the department. Finally, he was promoted general foreman of the Tailings Disposal and Water Supply Department with a monthly salary of P7,440.00. He held this position until his services were terminated on February 2, 1990. - As general foreman, Villacencio was the second-to-the-highest man in the department which has a field office located in Magdugo, Toledo City. Under Villacencio were some fifty nine (59) workers whom he supervised through regular field inspections. Villacencio was assigned a service jeep and a service motorcycle which he used alternately. He was given the privilege to withdraw the necessary fuel/gasoline for the vehicles at the Transport Department located inside the main compound of ACMDC. - September 8, 1989 - Engineer Sanchez of the Services Division wrote a memorandum requesting that Villacencio be investigated for alleged anomalies at the Magdugo Tailings Field Office. Villacencio was charged before the Special Investigation Board with acts of malfeasance consisting of:

1. withdrawal of company-owned gasoline for the refueling of his personal jeep;2. use of company personnel on company time as well as company-owned materials for the assembly of a jeep not belonging to the company; and 3. granting of authority to non-company personnel to withdraw company-owned stocks.

- January 1990 - He was summoned and investigations were conducted. the Special Investigation Board found Villacencio guilty of the charge of withdrawing on various dates a total of 192 liters of company-owned gasoline which he used to refuel his private jeep

Labor Law 1 A2010 - 222 - Disiniand of the charge of using company personnel on company time in the assembly of his jeep. The third charge was dismissed for insufficiency of evidence. Villacencio was dismissed from work on February 2, 1990. - February 19, 1990 - Villavicencio lodged a complaint against ACMDC before the Regional Arbitration Cebu City, for illegal dismissal with prayer for reinstatement and backwages plus damages. The case was assigned to Labor Arbiter Reynoso A. Belarmino.- Meanwhile, ACMDC initiated a criminal complaint against Villacencio for the misappropriation of 192 liters of gasoline amounting to P1,086.72. An Information for Estafa was filed against Villacencio before the Municipal Trial Court of Toledo City. After trial, he was found guilty and sentenced to prision correccional as maximum, and to pay ACMDC the amount of P1,086.72 for the misappropriated gasoline.- Villacencio appealed his conviction to RTC Toledo City. For failure of the prosecution to establish the guilt of Villacencio beyond reasonable doubt, the appellate court acquitted him- August 9, 1993 - Labor Arbiter Belarmino rendered a Decision dismissing Villacencio's complaint of illegal dismissal for lack of merit.- December 27, 1994 - NLRC reversed the Labor Arbiter's decision. - Both parties filed their respective Motion for Reconsideration. ACMDC's motion assailed the public respondent's decision for allegedly misapprehending the Labor Arbiter's decision. On the other hand, Villacencio's motion prayed for reinstatement and award of backwages in addition to separation pay.- August 18, 1995 – NLRC rendered a Resolution granting Villacencio's prayer for backwages and denying ACMDC's motion.

ISSUES1. WON NLRC acted with grave abuse of discretion amounting to lack of jurisdiction in reversing the Decision of the Labor Arbiter and holding Villavicencio’s dismissal illegal2. WON there is willful breach of trust

HELD1. NO- In illegal dismissal cases, the employer bears the burden of proof to show that the dismissal is for a just or authorized cause. The charges against private respondent are: (1) withdrawal of 192 liters of gasoline from company stocks for his private use; and (2) knowingly allowing company personnel to work on company time in the assembly of a privately-owned jeep. To prove the first charge, petitioner presented the Tenders Logbook showing the unsigned entries of gasoline withdrawals allegedly made by Villavicencio . Wilfredo Caba and Bienvenido Villacencio also testified that Villavicencio refused to sign the entries when requested to do so.(1) The evidence for the Villavicencio shows that during his more than twenty (20)-year stint with petitioner, he received several awards and commendations for his contribution in the areas of production, services and smooth operation of his department. The management recognized his ability in handling his subordinates and in protecting company assets in relation to his assigned duties. As a stickler for company rules, he never held back on issuing warnings, admonitions and even suspensions against erring subordinates. Consequently, he earned the ire of some of his subordinates. Among them were Wilfredo Caba., June Climaco, Felix Gonzales and Bienvenido Villacencio. In sum, Villavicencio’s position is that the logbook entries do not prove that he received the 192 liters of gasoline since his signature does not appear therein and that the witnesses presented by the petitioner to explain the absence of his signature in the logbook entries were motivated by vengeance since he offended their feelings when he disciplined them and denied their requests for promotion.- The Standard Guidelines of ACMDC require that all withdrawals of consumable items and the borrowing of company materials and equipments should be recorded in the Tender's Logbook by the tender on duty and should be signed by the withdrawing party. The tender on duty is also required to immediately report to his supervisor any discrepancy, error or irregularity. Needless to stress, the best evidence of any withdrawal is the Tender's Logbook. In the case at bar, the gasoline withdrawal entries were made by tenders Caba and Villacencio. Villavicencio’s signature does not appear in the logbook, thus, there is no proof that he actually withdrew and received the gasoline. (2) The Authorization to Work Overtime dated May 14, 1989 indicates that A. Saavedra, A. Sepada and V. Rago were among those authorized to work overtime 'to assist in emergency repair of busted 280 CIC Tailings Line' on that date. The same does not show or affirm petitioner's contention that said workers were not actually authorized or did not actually perform the required work but were at the Magdugo Field Office working on private respondent's personal jeep. On the contrary, the Authorization to Work Overtime appears regular on its face, as in fact, the same bears the imprimatur indicated by the signature not only of private respondent alone but of three (3) other officers: the Supervisor, J.V. Climaco, Jr., the Department Head, J. N. Tecson, and the Division Manager, C. N. Sanchez. If ever there was an irregularity, these officers would likewise have to be answerable to the company, instead of letting private respondent bear the burden alone. 2. NO- We reject the ruling of the Labor Arbiter that since Villavicencio neglected to inspect the logbook and thus failed to discover the irregularity, he committed breach of trust. Ratio Settled is the rule that under Article 283(c) of the Labor Code, the breach of trust must be willful. A breach is willful if it is done intentionally, knowingly and purposely, without justifiable excuse, as distinguished from an act done carelessly, thoughtlessly, heedlessly or inadvertently. It must rest on substantial grounds and not on the employer's

arbitrariness, whims, caprices or suspicion; otherwise, the employee would eternally remain at the mercy of the employer. It should be genuine and not simulated; nor should it appear as a mere afterthought to justify earlier action taken in bad faith or a subterfuge for causes which are improper, illegal or unjustified. It has never been intended to afford an occasion for abuse because of its subjective nature. Private respondent explained that he failed to inspect the logbook for about two (2) months before its disappearance because he was preoccupied with some emergency works brought about by a storm. With the foregoing explanation, it cannot be said that Villavicencio’s failure was willful.Disposition the assailed Decision and Resolution of public respondent NLRC are AFFIRMED.

COVERAGE

FUJITSU COMPUTER PRODUCTS CORP V CA[PAGE 204]

PROOF

RAMATEK PHILS V DE LOS REYES474 SCRA 129

CARPIO; October 25, 2005

NATUREpetition for review resolutions of CA (denying appeals for being filed out of time)

FACTS- Anelia de los Reyes was employed by Ramatek as a comptroller. Subsequently, Ramatek entered into a sub-contracting agreement with Sicar Micro-Electronics Corp, of which Anelia’s husband Nestor was a major stockholder, treasurer, and COO. - Some time after, Sicar filed a civil action for damages against the Ramatek officials for the unilateral termination of their contract. Later, the chairman of the board of directors of Ramatek informed Anelia that she should file a leave of absence while the case was ongoing. Afterwards, the chairman emailed Anelia, requesting her to tender her voluntary resignation from the company. The email said in part: “IT IS WITH GREAT REGRET THAT I MUST INFORM YOU OF MY REACTION TO THE SICAR AFFAIR. YOUR CONNECTION IN THIS MATTER HAS CAUSED ME TO LOSE MY FAITH AND TRUST IN YOU. IT IS A MAJOR CONFLICT OF INTEREST SITUATION.”- In a letter dated a month later, the company required Anelia to explain within 72 hours some of her allegedly questionable transactions. Such included awards of work bids to bidders who did not give the lowest bids, purchase of equipment not at the lowest prices, and failure to submit company documents despite demand. - Anelia did not answer (she failed to claim the letter sent by Ramatek through registered mail) nor did she appear in the administrative investigation. Ramatek, soon after, terminated Anelia’s employment for committing anomalies amounting to breach of trust and confidence. Anelia filed for illegal suspension and illegal dismissal.LA ruled in favor of Anelia. NLRC affirmed. Appeal to CA (by Ramatek) was denied for being filed out of time.

ISSUES1. WON the appeal was filed out of time2. WON dismissal based on loss of trust and confidence was valid

HELD1. NORatio The latest amendment to Rule 65, ROC allows filing of an appeal within 60 days after the notice of denial of a motion for reconsideration. Reasoning - The amended rule now reads: - Sec. 4. When and where petition filed. – The petition shall be filed not later than sixty (60) days from notice of the judgment, order or resolution. In case a motion for reconsideration or new trial is timely filed, whether such motion is required or not, the sixty (60) day period shall be counted from notice of the denial of the said motion.- In the present case, the petition filed in the Court of Appeals was indeed filed beyond the 60-day period if computed from the time the notice of judgment was received and interrupted only by the filing of the motion for reconsideration. However, if the 60-day period is reckoned from the receipt of the notice denying the motion for reconsideration, as provided under Circular No. 56-2000, then the petition for certiorari was filed on time.2. NORatio That the dismissal was based on loss of trust and confidence was not sufficiently proven by evidence. Ramatek’s evidence are insubstantial and inadequate to support a conclusion that Anelia engaged in anomalous transactions. Since the company had the burden of proving the same, said dismissal cannot be held valid. Reasoning- The SC upholds the findings of the Labor Arbiter that Anelia was able to prove that the charges against her were false and baseless.

Labor Law 1 A2010 - 223 - Disini“Despite the gravity of the charges, there is nothing competent in the records to substantiate the same. Xxx Ramatek has the burden to prove just cause, but it failed to undertake the burden. On the other hand, complainant explained to the satisfaction of this Office that the charges against her are utterly false and baseless.”

- Ramatek having failed to substantiate their charges against Anelia with competent and credible evidence, this Office perceives that the primordial inspiration for her dismissal was the filing by her husband of a civil suit against the company officials, a matter which respondents cannot legally use against complainant to deprive her of her tenurial rights. This is because the suit was not filed by Anelia against Ramatek or its officials but by her husband. There is no showing that the filing of the suit was a joint decision by the couple or was instigated by complainant as to charge complainant with disloyalty or a conflict of interests. Moreover, it appears that Anelia’s husband was merely asserting and exercising his right to seek redress in the courts, a matter which Ramatek should not begrudge Anelia about. Finally, the case was amicably settled by the parties such that there can be no rational justification for respondents to dismiss Anelia just because a plaintiff in the civil suit happened to be her husband.- Loss of confidence as a ground for dismissal does not require proof beyond reasonable doubt. The law requires only that there be at least some basis to justify it. Thus, there must be some evidence to substantiate the claim and form a legal basis for loss of confidence. The employer cannot exercise arbitrarily and without just cause the right to dismiss an employee for loss of trust and confidence.Disposition Resolutions of CA set aside. Decision of NLRC AFFIRMED.

LACK OF DAMAGE

CADIZ V CA (PHILIPPINE COMMERCIAL BANK [EQUITABLE PCIBANK])

474 SCRA 232TINGA; October 25, 2005

NATURECertiorari

FACTS- Cadiz, Bongkingki and Gloria were employed as signature verifier, bookkeeper, and foreign currency denomination clerk/bookkeeper-reliever, respectively, in the main office branch (MOB) of Philippine Commercial International Bank (respondent bank).- Cadiz reserved S/A No. 1083-4 in July 1987 as reflected on respondent bank’s “new account register.”- Foreign denominated checks payable to other payees were diverted into the said account.- The various deposit slips, covering the said checks, did not bear the machine validation of any of the tellers-in-charge.- Petitioner Cadiz agreed to pay Alqueza the equivalent amount of $600.00 but it was made to appear that Alfiscar paid the said amount.- In view of these findings, petitioners were served with show-cause memoranda asking them to explain the lapses. - Finding their explanations unsatisfactory, petitioners were terminated from employment. LA-adjudged that petitioners were illegally dismissed and ordered their reinstatement and payment of backwages. - NLRC-reversed- CA-affirmed reversal by NLRC

ISSUES1. WON petitioners were validly dismissed (with just cause and were afforded due process)2. WON petitioners should be relieved of any liability considering that respondent bank did not suffer a pecuniary loss

HELD1. YES- Petitioners had surreptitiously diverted funds deposited by depositors to S/A No. 1083-4 which was under their control and disposition. - Their behavior in the course of the discharge of their duties is clearly malfeasant, and constitutes ground for their termination on account of just cause. - respondent bank complied with the two-notice rule prescribed in Article 277(b) of the Labor Code. Petitioners were given all avenues to present their side and disprove the allegations of respondent bank. An informal meeting was held between the branch manager of MOB, the three petitioners and Mr. Gener, the Vice-President of the PCIB Employees Union. 2. NO- In University of the East v. NLRC the court held that lack of material or pecuniary damages would not in any way mitigate a person’s liability nor obliterate the loss of trust and confidence. - In the case of Etcuban v. Sulpicio Lines, this Court definitively ruled that:

. . . Whether or not the respondent bank was financially prejudiced is immaterial. Also, what matters is not the amount involved, be it paltry or gargantuan; rather the

fraudulent scheme in which the petitioner was involved, which constitutes a clear betrayal of trust and confidence. . . .

D. COMMISSION OF A CRIME

E. ANALOGOUS CAUSES

QUARELSOME – BOSSY

CATHEDRAL SCHOOL OF TECHNOLOGY V NLRC (VALLEJERA)214 SCRA 551

October 13, 1992

NATUREPetition for certiorari of a decision of NLRC.

FACTS- Starting as an aspirant to the Congregation of the Religious of Virgin Mary (RVM), VALLEJERA worked on a volunteer basis as a library aide of CST, an educational institution run by the RVM sisters. Eventually she became a regular employee of CST, again as library aide.- It was around such regular employment, however, that trouble developed. The sisters began receiving complaints from students and employees about VALLEJERA's difficult personality and sour disposition at work. On one occasion, VALLEJERA was summoned to the Office of the Directress by SISTER APOLINARIA, shortly after the resignation of the school's Chief Librarian on account of irreconcilable differences with VALLEJERA, for the purpose of clarifying the matter. SR APOLINARIA also informed VALLEJERA of the negative reports received by her office regarding the latter's frictional working relationship with co-workers and students and reminded VALLEJERA about the proper behavior in the interest of peace and harmony in the school library. VALLEJERA resented the observations about her actuations and was completely unreceptive to the advice given by her superior. She reacted violently to SR APOLINARIA and angrily offered to resign, repeatedly saying, "OK, I will resign. I will resign." Thereafter, without waiting to be dismissed from the meeting, she stormed out of the office.- On separate occasions thereafter, CST and SR APOLINARIA (PETITIONERS, for brevity) sent people to convince VALLEJERA to settle her differences with the former. VALLEJERA remained adamant in her refusal to submit to authority. Eventually, SR APOLINARIA, by letter, informed VALLEJERA to look for another job as the school had decided to accept her resignation. VALLEJERA filed a complaint for illegal dismissal. An issue arose as to whether there was lawful cause for her dismissal.

ISSUEWON there was there lawful cause for VALLEJERA’s dismissal

HELDYESRatio The reason for which VALLEJERA’s services were terminated, namely, her unreasonable behavior and unpleasant deportment in dealing with the people she closely works with, is analogous to the other "just causes" enumerated under ART.282, Labor Code.- PETITIONERS' averments on VALLEJERA’s disagreeable character as "quarrelsome, bossy, unreasonable and very difficult to deal with," are supported by testimonies of several co-employees and students of CST. In fact, her overbearing personality caused the chief librarian to resign, Furthermore, the complaints about her objectionable behavior were confirmed by her reproachable actuations during her meeting with SR APOLINARIA, when VALLEJERA, upon being advised of the need to improve her working relations with others, obstreperously reacted and unceremoniously walked out on her superior, and arrogantly refused to subsequently clear up matters or to apologize therefor. To make matters worse, she ignored the persons sent by PETITIONERS to intervene in an effort to bring the matter to a peaceful resolution. The conduct she exhibited on that occasion smacks of sheer disrespect and defiance of authority and assumes the proportion of serious misconduct or insubordination, any of which constitutes just cause for dismissal from employment. - As CST is run by a religious order, it is but expected that good behavior and proper department, especially among the ranks of its own employees, are major considerations in the fulfillment of its mission. Under the circumstances, the sisters cannot be faulted for deciding to terminate VALLEJERA whose presence "has become more a burden rather than a joy" and had proved to be disruptive of the harmonious atmosphere of the school.Disposition NLRC decision that VALLEJERA was illegally dismissed, SET ASIDE.

HEAVYLIFT MANILA INC V CA (GALAY, NLRC) 473 SCRA 541

Labor Law 1 A2010 - 224 - DisiniQUISUMBING; October 20, 2005

NATUREA petition for certiorari

FACTS- Petitioner Heavylift, a maritime agency, thru a letter signed by Josephine Evangelio, Admin. and Finance Manager of Heavylift, informed respondent Ma. Dottie Galay, Heavylift Insurance and Provisions Assistant, of her low performance rating and the negative feedback from her team members regarding her work attitude. The letter also notified her that she was being relieved of her other functions except the development of the new Access program.- Subsequently, Galay was terminated for alleged loss of confidence. - Thereafter, she filed with the Labor Arbiter a complaint for illegal dismissal and nonpayment of service incentive leave and 13th month pay against petitioners.- Petitioners alleged that Galay had an attitude problem and did not get along with her co-employees for which she was constantly warned to improve. Petitioners aver that Galay’s attitude resulted to the decline in the company’s efficiency and productivity. - The Labor Arbiter found that Galay was illegally terminated for petitioners’ failure to prove that she violated any company regulation, and for failure to give the proper notice as required by law.- Petitioner appealed to the NLRC. The latter, however, denied the appeal for lack of merit and affirmed the decision of the Labor Arbiter. - CA denied the motion for lack of justifying circumstances, and because the attached board resolution was issued after the petition was filed (petitioners failed to: state the full names and actual addresses of all the petitioners; attach the copies of all pleadings and supporting documents; properly verify the petition; and certify against forum-shopping) ISSUES1. WON petitioners were denied due process with the CA’s dismissal of the petition on technical grounds2. WON “attitude problem” is a valid ground for the termination of an employee. 3. If issue 2 is in the affirmative, WON this was sufficiently proved4. WON the procedural requirements for an effectual dismissal were present5. WON the awards of service incentive pay and 13th month pay were proper

HELD1. YESRatio The Rules of Court are designed for the proper and prompt disposition of cases. In not a few instances, we relaxed the rigid application of the rules to afford the parties opportunity to fully ventilate their cases on the merits. In that way, the ends of justice would be better served. Additionally, verification of a pleading is a formal, not a jurisdictional requisite. It is intended to secure an assurance that what are alleged in the pleading are true and correct and not the product of the imagination or a matter of speculation, and that the pleading is filed in good faith.2. YESRatio An employee who cannot get along with his co-employees is detrimental to the company for he can upset and strain the working environment. Without the necessary teamwork and synergy, the organization cannot function well. Thus, management has the prerogative to take the necessary action to correct the situation and protect its organization. When personal differences between employees and management affect the work environment, the peace of the company is affected. Thus, an employee’s attitude problem is a valid ground for his termination. It is a situation analogous to loss of trust and confidence that must be duly proved by the employer. Similarly, compliance with the twin requirement of notice and hearing must also be proven by the employer.3. NORatio We are not convinced that in the present case, petitioners have shown sufficiently clear and convincing evidence to justify Galay’s termination. Though they are correct in saying that in this case, proof beyond reasonable doubt is not required, still there must be substantial evidence to support the termination on the ground of attitude. The mere mention of negative feedback from her team members, and the letter, are not proof of her attitude problem. Likewise, her failure to refute petitioners’ allegations of her negative attitude does not amount to admission. Technical rules of procedure are not binding in labor cases. Besides, the burden of proof is not on the employee but on the employer who must affirmatively show adequate evidence that the dismissal was for justifiable cause.4. NORatio The letter did not constitute the required notice. It did not inform her of the specific acts complained of and their corresponding penalty. Additionally, the letter never gave respondent Galay an opportunity to explain herself, hence denying her due process.5. YESRatio Apropos the award of service incentive pay and 13th month pay, we find that they were properly prayed for by Galay. These were subsumed in the complaint and under the position paper’s general prayer of “such other relief as are just and equitable under the law”. Disposition Decision of the Labor Arbiter and the Resolution of the NLRC are hereby affirmed. PROBABLE CAUSE

STANDARD ELECTRIC MANUFACTURING CORP V STANDARD ELECTRIC EMPLOYEES UNION

CALLEJO; August 25, 2005

NATUREPetition for review on certiorari to review the CA decision annulling the NLRC Resolution which affirmed the LA decision

FACTS- Rogelio Javier, a radio machine operator, employee of Standard Electric Manufacturing Corp. (SEMC) and member of the Standard Electric Employees Union (Union), failed to report for work and failed to report the reason for his absence. This failure to report for work and failure to report the reason therefor happened several times until he was later found to have been arrested and detained for the charge of rape. - Javier informed SEMC (through a letter and through his counsel) that he was detained for the charge of rape which is why he failed to report for work. He requested that SEMC defer the implementation of its intention to dismiss him. The SEMC denied his request and issued a Memorandum terminating his employment for having been absent without leave (AWOL) for more than 15 days and for committing rape. - Javier, after the RTC granted his demurrer to evidence and ordered his release from jail, reported for work but the SEMC refused to accept him back. A grievance meeting between the Union, Javier and the SEMC was later held, but the SEMC refused to re-admit Javier. The Union and Javier then filed a complaint for illegal dismissal against SEMC before the NLRC, alleging that since his detention for rape was non-existent, the termination of his employment was illegal. SEMC averred that Javier’s prolonged absences caused irreparable damage to its orderly operation and that it could not afford to wait for Javier’s indefinite return from detention, if at all. - The LA dismissed the complaint but ordered SEMC to pay Javier P71, 760 as separation pay. On appeal, the NLRC affirmed the LA’s ruling (held that Javier was given a chance to explain his side), and later denied a subsequent MFR. Javier and the Union then filed a petition for certiorari with the CA, which reversed the findings of both the LA and the NLRC and ordered the reinstatement of Javier to his former position. The appellate court cited Magtoto v NLRC and City Govt of Makati v Civil Service as precedents and declared that it was not Javier’s intention to abandon his job; his incarceration reasonably justified his failure to report for work and negated the theory that he was on AWOL. The CA also held that Javier could not be terminated on the ground of commission of a crime, as he was acquitted of the rape charges. Hence, despite the fact that Javier was allegedly afforded the opportunity to explain his side (the basis of the LA and NLRC decisions), the same was unnecessary since there was no just or authorized cause for the dismissal. The MFR by SEMC was denied by the CA, hence, this recourse.

ISSUEWON the CA erred in holding that the termination was illegal

HELDNO- The CA was correct in holding that the termination was illegal and correctly applied the Magtoto case. Ratio Separation from employment founded on a false or non-existent cause is illegalReasoning - In the Magtoto case, Alejandro JONAS Magtoto was arrested. He was charged with violation of Arts 136 and 138 of the RPC. Although Magtoto informed his employer and pleaded that he be considered “on leave” until released, his employer denied the request. About seven months after his arrest, Magtoto was released after the City Fiscal dismissed the criminal charges for lack of evidence. On the same date, he informed his employer of his intent to start working again but the employer rejected the offer. In ruling that his termination was illegal, the SC ruled:

The employer tries to distance itself from the detention by stressing that the petitioner was dismissed due to prolonged absence. However, Mr. Magtoto could not report for work because he was in a prison cell. The detention cannot be divorced from prolonged absence. One caused the other. Since the causes for the detention, which in turn gave the employer a ground to dismiss the petitioner, proved to be non-existent, we rule that the termination was illegal and reinstatement is warranted.

- Respondent Javier was dismissed by the petitioner for: (a) being AWOL from July 31, 1995 up to January 30, 1996; and (b) committing rape. However, on demurrer to evidence, Javier was acquitted of the charge. With Javier’s acquittal, the cause of his dismissal from his employment turned out to be non-existent. - A non-existent cause for dismissal was explained in Pepito v. Secretary of Labor (96 SCRA 454):

“... A distinction, however, should be made between a dismissal without cause and a dismissal for a false or non-existent cause. In the former, it is the intention of the employer to dismiss his employee for no cause whatsoever, in which case the Termination Pay Law would apply. In the latter case, the employer does not intend to dismiss the employee but for a specific cause which turns out to be false or non-existent. Hence, absent the reason which gave rise to his separation from employment, there is no intention on the part of the employer to dismiss the employee concerned. Consequently, reinstatement is in order. And this is the situation here.

Labor Law 1 A2010 - 225 - DisiniPetitioner was separated because of his alleged involvement in the pilferage in question. However, he was absolved from any responsibility therefor by the court. The cause for his dismissal having been proved non-existent or false, his reinstatement is warranted. It would be unjust and unreasonable for the Company to dismiss petitioner after the latter had proven himself innocent of the cause for which he was dismissed.”

- The petitioner acted with precipitate haste in terminating respondent Javier’s employment on the ground that he had raped the complainant therein. Respondent Javier had yet to be tried for the said charge. In fine, the petitioner prejudged him, and preempted the ruling of the RTC. Petitioner had, in effect, adjudged Javier guilty without due process of law. While it may be true that after the preliminary investigation of the complaint, probable cause for rape was found and respondent Javier had to be detained, these cannot be made as legal bases for the immediate termination of his employment.Disposition petition DISMISSED for lack of merit. CA decision is AFFIRMED with MODIFICATION. Petitioner is ordered to reinstate Rogelio Javier to his former position or, if no longer possible, a substantially equivalent position without loss of seniority rights and other privileges appurtenant thereto, with full backwages from the time it refused to allow his reinstatement on May 24, 1996 until actually reinstated; or, if reinstatement is no longer feasible, to pay him separation pay equivalent to one (1) month salary for every year of service.

CONVICTION – MORAL TURPITUDE

IRRI V NLRC (MICOSA)221 SCRA 760

NOCON; May 12, 1993

NATUREPetition for certiorari

FACTS- International Rice Research Institute (IRRI) is an international organization recognized by the Philippine government and accorded privileges, rights and immunities normally granted to organizations of universal character. In 1977, it hired Nestor Micosa, who thereby became bound by IRRI Employment Policy and Regulations, the Miscellaneous Provisions of which states:

"C. Conviction and Previous Separation.X X X'2. An employer who has been convicted of a (sic) criminal offense involving moral turpitude may be dismissed from the service.'"

- On February 6, 1987, Micosa stabbed to death Reynaldo Ortega inside a beer house in Laguna. He was accused of homicide. During the pendency of the criminal case, Micosa voluntarily applied for inclusion in IRRI's Special Separation Program. However, IRRI's Director General expressed deep regret that he had to disapprove Micosa's application for separation because of IRRI's desire to retain the skills and talents that persons like him possess.- Trial court found Micosa guilty of homicide, but appreciated in his favor the mitigating circumstances of incomplete self-defense and voluntary surrender, and no aggravating circumstance. Subsequently, Micosa applied for suspension of his sentence under the Probation Law.- On February 8, 1990, IRRI's Director General personally wrote Micosa that his appointment as laborer was confirmed, making him a regular core employee whose appointment was for an indefinite period and who "may not be terminated except for justifiable causes as defined by the pertinent provisions of the Philippine Labor Code." - On March 30, 1990, IRRI’s HR head wrote Micosa urging him to resign from employment in view of his conviction in the case for homicide.- Laguna Parole and Probation Office No. II wrote IRRI informing the latter that said office found Micosa's application for probation meritorious as he was evaluated "to possess desirable social antecedents in his life." - Micosa informed IRRI that he had no intention of resigning from his job.- IRRI’s HR head replied to Micosa's letter insisting that the crime for which he was convicted involves moral turpitude and informing him that he is thereby charged of violating Section I-AA, Par VII, C-2 of the Institute's Personnel Manual (quoted above).- Micosa explained to IRRI that the slaying of Ortega arose out of his act of defending himself from unlawful aggression; that his conviction did not involve moral turpitude and that he opted not to appeal his conviction so that he could avail of the benefits of probation, which the trial court granted to him.- Micosa sought the assistance of IRRI's Grievance Committee who recommended to the Director General, his continued employment. However, IRRI issued a notice to Micosa that the latter's employment was to terminate effective May 25, 1990. - Micosa then filed a case for illegal dismissal. Labor Arbiter found the termination was illegal and ordered his reinstatement with full backwages from the date of his dismissal up to actual reinstatement. NLRC affirmed decision.Petitioner’s claims:> Micosa's conviction of homicide, which is a crime involving moral turpitude, is a valid ground for his dismissal under the Miscellaneous Provisions of IRRI's Employment Policy Regulations. IRRI has the prerogative to issue rules and regulations including those

concerning employee discipline and that its employees are bound by the aforesaid personnel manual- While IRRI admits that Micosa's interests — in his employment and means of livelihood — are adversely affected; that a convict should not be discriminated against in society and that he should be given the same opportunities as those granted to other fellow citizens, it claims that one's right is deemed superior than that of another. It believes that it has a superior right to maintain a very high degree or standard not only to forestall any internal problem hampering operations but also to prevent even the smallest possibility that said problems could occur considering that it is an international organization with concomitant obligation to the host country to avoid creating disturbance or give occasion for such disturbance.

ISSUEWON a conviction of a crime involving moral turpitude is a ground for dismissal from employment

HELDNO, it is not one of the causes enumerated in the Labor Code.- Article 282 of the Labor Code enumerates the just causes wherein an employer may terminate an employment. Conviction of a crime involving moral turpitude is not one of these justifiable causes. Article 282 (c) or (d) may not be applied by analogy. Analogous causes must have an element similar to those found in the specific just cause enumerated under Article 282.- Under Article 282 (c) fraud or willful breach by the employees of the trust reposed in him by his employer or duly authorized representative refers to any fault or culpability on the part of the employee in the discharge of his duty rendering him absolutely unworthy of the trust and confidence demanded by his position. The breach of trust must be related to the performance of the employee's function. - Commission of a crime by the employee under Article 282 (d) refers to an offense against the person of his employer or any immediate member of his family or his duly authorized representative.- The commission of the crime of homicide was outside the perimeter of the IRRI complex, thus, the conviction of Micosa for homicide was not work-related, his misdeed having no relation to his position as laborer and was not directed or committed against IRRI or its authorized agent.- IRRI failed to show how the dismissal of Micosa would be in consideration of the safety and welfare of its employees, its reputation and standing in the community and its special obligations to its host country. Micosa's service record is unblemished. IRRI's Director General even expressed his confidence in him when he disapproved his application for special separation and decided to promote him to the status of a regular core employee, with the commensurate increases in benefits. In addition, the employees at IRRI's Grievance Committee interceded favorably in behalf of Micosa when they recommended his retention despite his conviction showing that the very employees which IRRI sought to protect did not believe that they were placing their very own lives in danger with Micosa's retention.- Likewise, Micosa, although found guilty as charged, was also found worthy of probation. This means that there existed no undue risk that Micosa will commit another crime during his period of probation and that his being placed on probation would be to the benefit of society as a whole. - Even under IRRI's Employment Policy and Regulations, the dismissal of Micosa on the ground of his conviction for homicide cannot be sustained. The miscellaneous provisions of said personnel manual mentions of conviction of a crime involving moral turpitude as a ground for dismissal. IRRI simply assumed that conviction of the crime of homicide is conviction of a crime involving moral turpitude. - Moral turpitude has been defined in Can v. Galing citing In Re Basa and Tak Ng v. Republic as everything which is done contrary to justice, modesty, or good morals; an act of baseness, vileness or depravity in the private and social duties which a man owes his fellowmen, or to society in general, contrary to justice, honesty, modesty or good morals. As to what crime involves moral turpitude, is for the Supreme Court to determine. The conclusion of IRRI that conviction of the crime of homicide involves moral turpitude is unwarranted considering that the said crime which resulted from an act of incomplete self-defense from an unlawful aggression by the victim has not been so classified as involving moral turpitude.- The facts of the incident show that Micosa's intention was not to slay the victim but only to defend his person. The appreciation in his favor of the mitigating circumstances of self-defense and voluntary surrender, plus the total absence of any aggravating circumstance demonstrate that Micosa's character and intentions were not inherently vile, immoral or unjust.- Corollary issue: WON conviction of homicide involves moral turpitudeHomicide may or may not involve moral turpitude depending on the degree of the crime. Moral turpitude is not involved in every criminal act and is not shown by every known and intentional violation of statute, but whether any particular conviction involves moral turpitude may be a question of fact and frequently depends on all the surrounding circumstances. Moral turpitude is somewhat a vague and indefinite term, the meaning of which must be left to the process of judicial inclusion or exclusion as the cases are reached. Disposition petition is DISMISSED for lack of merit.

OANIA V NLRC (PHILEX MINING)244 SCRA 668

Labor Law 1 A2010 - 226 - DisiniROMERO; June 1, 1995

FACTS- Alfredo Oania, a welder, and Aurelio Caluza and Santiago Biay, miners, were employed by Philex Mining Corporation (Philex) in Benguet. They were accused of mauling their co-worker, Felipe Malong, at the gasoline area within the company compound. - Philex conducted investigation regarding the incident. After a formal hearing wherein petitioners were duly notified and accorded the opportunity to be heard, the company arrived at the decision to terminate their employment on the ground that petitioners violated Art I, par 1 of the company rules and regulations2 - Malong instituted a criminal complaint (frustrated murder) vs. petitioners. But later, Malong desisted from pursuing the criminal case because he said his conscience bothered him.- With Malong's affidavit of desistance, petitioners sought reconsideration of their dismissal from employment. Philex refused. Petitioners filed complaints for illegal dismissal before the labor arbiter.- Labor Arbiter: The termination of employment of petitioners was not justified was based on findings that there was no proof that the mauling of Malong was "caused by a dispute involving their employment" with private respondent (which, the Labor Arbiter believed, was the only dispute clearly prohibited by the company rule).- Petitioners had been illegally dismissed from employment. Philex to reinstate them to their former positions or substantially equivalent positions and to pay each of them one year's backwages.- NLRC: Reversed. “there is prima facie evidence that the complainants injured physically a co- employee under circumstance(s) which constitute an infraction of specific company rules; and that the respondent had valid cause to terminate their employment."

ISSUES1. WON the mauling comes under Art 1 of the company rules and regulations2. WON there was illegal dismissal

HELD1. YES- The provision in question obviously covers situations where any company employee inflicts or attempts to inflict physical harm or injury upon any person. There are two separate instances contemplated here. The first part of the sentence conceives of a situation wherein such injury was done "on the job site on company time or property," regardless of the reason. What is material is the venue. The second half of the sentence deals with a situation where an employee attempts to inflict or actually inflicts bodily injury upon another "anywhere at anytime," regardless of the venue, as long as it arose in connection with a dispute "involving one's employment." The site matters not; what is crucial in the subject matter, i.e. it should have something to do with the employee's job. Clearly, the commas in the sentence may be dispensed with without sacrificing the intent behind the provision.2. YES- Violation of a company rule prohibiting the infliction of harm or physical injury against any person under the particular circumstances provided for in the same rule may be deemed analogous to "serious misconduct" stated in Art. 282 (a). (H)owever, there is no substantial evidence definitely pointing to petitioners as the perpetrators of the mauling of Malong. What is an established fact is that, after investigation, private respondent dismissed them and, thereafter, a criminal complaint was filed against petitioners. It is of record that Malong desisted from suing the perpetrators before the regular courts. In criminal cases, an affidavit of desistance may create serious doubts as to be the liability of the accused- On the issue of the legality of the dismissal, two requisites must concur to constitute a valid dismissal: (a) the dismissal must be for any of the causes expressed in Art. 282 of the Labor Code, and (2) the employee must be accorded due process, basic of which are the opportunity to be heard and to defend himself.

LIM V NLRC (PEPSI-COLA FAR EAST TRADE DEV’T)259 SCRA 485

DAVIDE JR; July 26, 1996

NATUREPetition for certiorari

FACTS- Pepsi is a manufacturer of concentrates sold to Pepsi-Cola Bottlers Co. Inc. Petitioner Sixta Lim had been employed with the Pepsi Group since January 1, 1981, working as a secretary for Pepsi Bottling Co. Pepsi employed Lim as a secretary on June 15, 1983. - At the time of her dismissal she was a staff accountant.

> She assisted and worked closely with the Plant Accountant to carry out the accounting department's tasks necessary to ensure an accurate, timely, and coordinated compilation of data for each accounting transaction.

2 "Inflicting or attempting to inflict bodily injury on the job-site on company time or property for any reason, or

attempting to inflict or inflicting bodily injury anywhere at anytime, in any dispute involving one's employment”

> Her work involved cost accounting production, cost accounting financial reporting, payroll reporting, statutory reporting and preparation of daily trade accounts receivable reports, petty cash fund custodianship, and check preparation.

- Pepsi regularly evaluated its employees' performance using following ratings: Marginal (obviously well below the acceptable level for the position), Fair Below (shows noticeable need for improvement), Commendable (fully meeting the performance requirements of the position), Superior (noticeably better than required performance) and Distinguished Outstanding (obviously far above an acceptable job). - Lim’s overall performance appraisals rated as follows: (a) "S" (Superior) as of May 1, 1984; (b) "C" (Commendable) for the period for December 1, 1987 to August 31,1988; and (c) "U' (C minus), quantified as 81.10% for the period from September 1, 1988 to May 31, 1989.- In 1989, Pepsi changed its rating scale to: Significantly Above Target (SA, exceeds position requirements by a wide margin; exceptional), Above Target (AT, usually exceeds position requirements), On Target (OT, meets and sometimes exceeds position requirements), Below Target (BT, meets some or many but not all position requirements) and Significantly Below Target (SB, below position requirements by a wide margin; unacceptable).- July 1, 1989 to December 31, 1989 – Lim received an overall rating of BT.

> This was heavily influenced by her ratings in production reporting which made up 40% of her final rating. Her supervisor noted several discrepancies which could have been avoided had Sixta been more diligent in her work.> In cost accounting and financial reporting (20% of the rating), Lim also was given a BT. Her supervisor noted that she did not seem to be aware of the importance of the reports she issued and her work always needed to be reviewed. She also needed a systematic workplan.> For the remaining 60%, she was given an OT. Overall, she was given a BT.

- Lim questioned the change in the rating style as well as the ratings and appraisals given to her by her supervisors. She asserted her previous positive ratings and expressed disbelief over the sudden decline of her ratings. Pepsi conducted another evaluation and Lim’s overall rating was a BT. Following that evaluation, she was given a report which outlined the areas where she could improve. - Lim then wrote Mr. Mihara of Pepsi Co. in Japan and Mihara replied, saying that he would discuss the matters with her upon arrival in the Philippines. Pepsi, however, did not wait for Mihara and offered to pay Lim’s termination benefits if she resigned.- Lim refused to do so and on May 6, 1991, she was informed that she was terminated as an employee of Pepsi. On May 14, 1991, she filed a complaint for illegal dismissal with the Labor Arbiter. The Labor Arbiter decided matters in her favor, ordering Pepsi to reinstate Lim to her former position or to pay her separation pay, 13 th month and backwages.- The NLRC reversed the Labor Arbiter’s ruling.Petitioners’ Claim> Lim’s BT performance appraisal was sufficient ground to dismiss her under Article 282 (b) of the Labor Code.Respondents’ Comments> Lim argues her alleged inefficiency was not among the just causes prescribed by law for the dismissal of an employee and even assuming that such dismissal was justified, she was still entitled to separation benefits of P268,000.00 in accordance with company policy plus damages and attorney's fees.

ISSUEWON Lim’s alleged “gross inefficiency” was an adequate ground for her dismissal

HELDNORatio "Gross inefficiency" is closely related to "gross neglect," for both involve specific acts of omission on the part of the employee resulting in damage to the employer or to his business. The Court has ruled that failure to observe prescribed standards of work, or to fulfill reasonable work assignments due to inefficiency may constitute just cause for dismissal.Reasoning- Pepsi had not characterized as "gross inefficiency" whatever failures, shortcomings, or deficiencies may have been attributable to the petitioner.- Lim obtained an unfavorable rating, but not to the extent, under the company's standards, to warrant even a probationary measure which is given to the lowest rating of Significantly Below Target (SB).- In Pepsi's brochure entitled Managing Performance for the 90's, a BT rating does not merit dismissal from the service; as a matter of fact, the lower rating - Significantly Below Target (SB) - is not even a ground for termination of employment, but may only justify putting the employee "on probation, telling the said employee that improvement is necessary.- If the company truly found the petitioner's "inefficiency" to be of such a gross character, then it should have rated her even lower than SB, since the latter only requires that the employee be put on probation.- Pepsi also violated the petitioner's right to due process. Prior to the issuance of her termination letter, Pepsi never called Lim’s attention to any alleged "gross inefficiency" on her part. Likewise, she was never warned of possible disciplinary action due to any alleged "gross inefficiency." The evaluation report merely indicated her areas for improvement.

Labor Law 1 A2010 - 227 - DisiniDisposition the instant petition is GRANTED. Private Respondent Pepsi-Cola Far East Trade Development Co., Inc. is ordered to reinstate petitioner Sixta C. Lim to her position as Staff Accountant without loss of seniority rights, and to pay her (a) backwages from the time she was illegally dismissed until she was effectively reinstated, less whatever she may have received through payroll reinstatement and whatever amount she may have earned from employment elsewhere during the period of her illegal dismissal, and (b) other monetary benefits that may be due her from the date of her illegal dismissal until such effective reinstatement.

F. OTHER – JUST CAUSES CLAIMED BY EMPLOYER

1. ABANDONMENT

DEFINED

NUEVA ECIJA ELECTRIC COOP (NEECO) II V NLRC461 SCRA 169

CHICO-NAZARIO; June 23, 2005

NATUREPetition for review

FACTS- Petitioner NEECO II staunchly asserts that since its new GM assumed office on 01 March 1995, the GM never saw private respondent Eduardo Cairlan report for work prompting the former to issue a memorandum dated 22 November 1995, which required private respondent to explain in writing why he was not reporting for duty. Private respondent was likewise directed in the said memo to report to its main office at Calipahan, Talavera, Nueva Ecija. For failure of the private respondent to comply with the said memorandum, Mr. dela Cruz directed a certain “Mr. Marcelo” to conduct an investigation on the whereabouts of the petitioner. It was then that NEECO II uncovered that private respondent was at that time already working with the Provincial Government of Nueva Ecija as driver allegedly under an assumed name of “Eduardo Caimay.” For these reasons, petitioner contended that it was left with no other alternative but to terminate private respondent’s services.- Petitioner’s GM terminated private respondent’s services on ground of abandonment. Immediately thereafter, private respondent talked with the GM regarding this matter and the latter promised him that the issue would be brought to the attention of NEECO’s Board of Directors for appropriate action. But nothing came out of the GM’s promise prompting private respondent to institute a Complaint for illegal dismissal with prayer for reinstatement and payment of backwages since the NEECO’s Board of Directors did not act upon his termination. - The Labor Arbiter rendered a Decision declaring that private respondent was illegally dismissed on the following grounds: First, petitioner’s assertion that it required private respondent to explain in writing why he was not reporting for duty as driver assigned at Quezon Service Center merited scant consideration since a copy of the alleged memorandum dated 22 November 1995, purportedly as its Annex “A,” was nowhere to be found in the record of the case. Second, petitioner’s contention that private respondent Cairlan was later discovered to be working with the Provincial Government of Nueva Ecija under an assumed name of Eduardo Caimay remained unsubstantiated as petitioner failed to adduce independent evidence that said “Eduardo Caimay” and private respondent Eduardo Cairlan are one and the same person. Third, the Labor Arbiter held that the private respondent was denied his right to due process since the letter of termination dated 15 January 1996 stated that said termination is retroactively effected on 1 January 1996. Finally, according to the Labor Arbiter, petitioner failed to corroborate its claim that private respondent was guilty of dereliction of duty.Public respondent NLRC dismissed for lack of merit. The NLRC affirmed in toto the decision of Labor Arbiter. Hence this petition.

ISSUES1. WON petitioner was accorded due process2. WON petitioner is guilty of illegally dismissing private respondent

HELD1. YESRatio The rules of evidence prevailing in courts of law or equity shall not be controlling and it is the spirit and intention of this Code that the Commission and its members and the Labor Arbiters shall use every and all reasonable means to ascertain the facts in each case speedily and objectively and without regard to technicalities of law or procedure, all in the interest of due process.Reasoning - The Labor Arbiter shall motu proprio determine whether there is need for a formal trial or hearing.- Under Section 4, Rule V of the New Rules of Procedure of the NLRC, the Labor Arbiter is given the latitude to determine the necessity for a formal hearing or investigation, once the position papers and other documentary evidence of the parties have been submitted

before him. The parties may ask for a hearing but such hearing is not a matter of right of the parties. The Labor Arbiter, in the exercise of his discretion, may deny such request and proceed to decide the case on the basis of the position papers and other documents brought before him without resorting to technical rules of evidence as observed in regular courts of justice.- In the present case, a scrupulous study of the records reveals that the Labor Arbiter did not abuse his discretion conferred upon him by the Rules in not conducting a formal hearing. On this, the findings of the Court of Appeals, consistent with that of the NLRC and the Labor Arbiter, ought to be sustained.2. YESRatio Abandonment3 is the deliberate and unjustified refusal of an employee to resume his employment; it is a form of neglect of duty; hence, a just cause for termination of employment by the employer under Article 282 of the Labor Code, which enumerates the just causes for termination by the employer: i.e., (a) serious misconduct or willful disobedience by the employee of the lawful orders of his employer or the latter’s representative in connection with the employee’s work; (b) gross and habitual neglect by the employee of his duties; (c) fraud or willful breach by the employee of the trust reposed in him by his employer or his duly authorized representative; (d) commission of a crime or offense by the employee against the person of his employer or any immediate member of his family or his duly authorized representative; and (e) other analogous causes.Reasoning - Private respondent’s alleged abandonment of work through his employment with the Provincial Government of Nueva Ecija was not clearly established and proven. The evidence submitted by petitioner to buttress its allegation that private respondent abandoned his work consists merely of indexes of payments to employees under the name Eduardo Caimay without any further evidence showing that Eduardo Caimay and private respondent Eduardo Cairlan is one and the same person. The best evidence that could have established the allegation that Eduardo Caimay and private respondent Eduardo Cairlan is one and the same person is Eduardo Caimay’s Personal Data Sheet which definitely would have the pertinent personal information about him and a picture that would identify him and not a testimony of a representative from the Provincial Government of Nueva Ecija, as adverted to by petitioner to justify its motion for a trial type hearing.- Worse, private respondent received his notice of termination only on 15 January 1996 which termination is effective as early as 01 January 1996, all in gross violation of the requirements provided for by law.- Further negating petitioner’s contention of abandonment, as noted by the Labor Arbiter, is private respondent’s letter dated 04 March 1996 addressed to Mr. Danilo dela Cruz reiterating the former’s plea for reconsideration of his dismissal. This letter depicts private respondent’s fervor and yearning to continue working with petitioner – the very antithesis of abandonmentDisposition AFFIRMED.

GABUAY V OVERSEA PAPER SUPPLY INC436 SCRA 514

CALLEJO; August 13, 2004

NATUREPetition for review of the decision of the Court of Appeals

FACTS- The respondent Oversea Paper Supply, Inc. is a domestic corporation engaged in the business of selling paper products. On different dates, the respondent corporation hired the petitioners for the positions machine operators, driver and helpers.- On April 7, 1999, the respondent corporation’s sales and operations manager, James C. Tan, required all employees to fill up and submit their bio-data not later than April 17, 1999 so that their 201 files could be updated. All the employees complied except for the petitioners. Petitioners William Lacambra and Rodolfo Gabuay even failed to report for work starting April 19 and 21, 1999, respectively.- Thereafter, the respondent corporation required petitioners to explain why they refused to submit their updated bio-data and requiring each of them to (a) return to work, and (b) explain why they were absent. Despite the receipt of such notices, the petitioners, except for Reynante Lacambra, did not reoport back to work.- On April 21, 1999, petitioner Rodolfo Gabuay filed a complaint for illegal dismissal, payment of separation pay, accumulated vacation and sick leave, and reinstatement with full backwages before the arbitration branch of the National Labor Relations Commission (NLRC). On April 26, 1999, petitioners William Lacambra, Reynante Lacambra, Rolando Vicente and Tomacito Tabuli filed a similar complaint.- The petitioners alleged that they were barred from reporting for work after they refused to fill up their bio-data for the respondent corporation. They also claimed that they were not paid vacation and sick leave benefits; that their 13th month pay for 1996 to 1998 was underpaid; and, that the respondents violated their right to security of tenure and payment of separation pay.

3 The elements of abandonment are: (a) failure to report for work or absence without valid or justifiable reason; and (b)

a clear intention to sever the employer-employee relationship, with the second element as the more determinative factor manifested by some overt acts (Tomas Lao Construction v. NLRC, 278 SCRA 716 [1997]).

Labor Law 1 A2010 - 228 - DisiniISSUEWON the petitioners were legally dismissed by reason of abandonment of work

HELD- As correctly ruled by the Labor Arbiter, the NLRC and the CA, the petitioners were not illegally dismissed. Even after the petitioners received notices from the respondent corporation requiring them to report for work and to explain their unauthorized absences and failure to submit their updated bio-data, they still failed to report for work. It can then be inferred that the petitioners had abandoned their work. Indeed, the factors considered for finding a valid abandonment are present in the case at bar: the petitioners’ failure to report for work or absence was without valid or justifiable cause, and their refusal to report for work notwithstanding their receipt of letters requiring them to return to work, show their clear intention to sever the employer-employee relationship.- Consistent with the finding that the petitioners abandoned their work, the award of financial assistance in the form of separation pay should be deleted. Separation pay is defined as the amount that an employee receives at the time of his severance and is designed to provide the employee with the wherewithal during the period that he is looking for another employment. Under the Labor Code, the award of separation pay is sanctioned when termination was due to an authorized cause, i.e., (a) installation of labor saving device, redundancy, retrenchment to prevent losses, closure or cessation of business operations not due to serious business losses or financial reverses; and, (b) disease prejudicial to the health of the employee and his fellow employees.- Separation pay is, likewise, awarded in lieu of reinstatement if it can be shown that the reinstatement of the employee is no longer feasible, as when the relationship between employer and employee has become strained. In some cases, it is awarded as a measure of social justice. In the present case, the petitioners were not dismissed, either legally or illegally; the petitioners abandoned their jobs. They failed to return to work despite the respondents’ directive requiring them to do so. There is, thus, no room for the award of financial assistance in the form of separation pay. To sustain the claim for separation pay under the circumstances herein established would be to reward the petitioners for abandoning their work.Disposition Petition denied

REQUISITES

LEONARDO V NLRC (REYNALDO'S MKTG CORP)333 SCRA 589

DE LEON JR; June 16, 2000

NATUREPetitions for certiorari seeking the annulment of a Decision of the public respondent, NLRC.

FACTS- Petitioner AURELIO FUERTE was originally employed by private respondent REYNALDO'S MARKETING CORPORATION on August 11, 1981 as a muffler specialist, receiving P45.00 per day. He was appointed as supervisor in 1988and his compensation was increased.- DANILO LEONARDO was hired by private respondent on March 4, 1988 as an auto-aircon mechanic. - FUERTE alleges that on January 3, 1992, he was instructed to report at private respondent's main office where he was informed by the company's personnel manager that he would be transferred to its Sucat plant due to his failure to meet his sales quota, and for that reason, his supervisor's allowance would be withdrawn. - For a short time, FUERTE reported for work at the Sucat plant; however, he protested his transfer, subsequently filing a complaint for illegal termination.- LEONARDO abandoned his post following an investigation wherein he was asked to explain an incident of alleged "sideline" work which occurred on April 22, 1991. It would appear that late in the evening of the day in question, the driver of a red Corolla arrived at the shop looking for LEONARDO. The driver said that, as prearranged, he was to pick up LEONARDO who would perform a private service on the vehicle. When reports of the "sideline" work reached management, it confronted LEONARDO and asked for an explanation. According to private respondent, LEONARDO gave contradictory excuses, eventually claiming that the unauthorized service was for an aunt. - When pressed to present his aunt, it was then that LEONARDO stopped reporting for work. He filed a complaint for illegal dismissal some ten months after his termination.

ISSUES1. WON the demotion of Fuerte by the private respondent is proper2. WON Fuerte's action constitutes abandonment3. WON the dismissal of Leornado is justified

HELD1. YES- Private respondent's justification is well-illustrated in the record. Complainant Fuerte's failure to meet his sales quota which caused his demotion and the subsequent withdrawal of his allowance is fully supported by Exhibit "4" of respondents' position

paper showing that his performance for the months of July 1991 to November 1991 is below par.Reasoning- FUERTE nonetheless decries his transfer as being violative of his security of tenure, the clear implication being that he was constructively dismissed. We have held that an employer acts well within its rights in transferring an employee as it sees fit provided that there is no demotion in rank or diminution in pay. 11 The two circumstances are deemed badges of bad faith, and thus constitutive of constructive dismissal. In this regard, constructive dismissal is defined in the following manner:

an involuntary resignation resorted to when continued employment becomes impossible, unreasonable, or unlikely; when there is a demotion in rank or diminution in pay; or when a clear discrimination, insensibility or disdain by an employer becomes unbearable to the employee. 12

- However, this arrangement appears to us to be an allowable exercise of company rights. An employer is entitled to impose productivity standards for its workers, and in fact, non-compliance may be visited with a penalty even more severe than demotion. Thus, the practice of a company in laying off workers because they failed to make the work quota has been recognized in this jurisdiction. - In the case at bar, the petitioners' failure to meet the sales quota assigned to each of them constitute a just cause of their dismissal, regardless of the permanent or probationary status of their employment. Failure to observe prescribed standards of work, or to fulfill reasonable work assignments due to inefficiency may constitute just cause for dismissal. Such inefficiency is understood to mean failure to attain work goals or work quotas, either by failing to complete the same within the allotted reasonable period, or by producing unsatisfactory results. This management prerogative of requiring standards may be availed of so long as they are exercised in good faith for the advancement of the employer's interest.2. NO- his actions do not constitute abandonment. The filing of a complaint for illegal dismissal, as in this case, is inconsistent with a charge of abandonment. Ratio To constitute abandonment there must be (1) failure to report for work or absence without valid or justifiable reason; and (2) a clear intention, as manifested by some overt acts, to sever the employer-employee relationship.3. YES- He was not terminated by the company but Leonardo abandoned his position in light of the pending investigation against him. Abandonment is a valid ground for dismissal. - He protests that he was never accorded due process. This begs the question, for he was never terminated; he only became the subject of an investigation in which he was apparently loath to participate. As testified to by Merlin P. Orallo, the personnel manager, he was given a memorandum asking him to explain the incident in question, but he refused to receive it. In an analogous instance, we held that an employee's refusal to sign the minutes of an investigation cannot negate the fact that he was accorded due process.Disposition Petition dismissed.

R.P. DINGLASAN CONSTRUCTION INC V ATIENZA433 SCRA 263

PUNO; June 29 2004

NATURESpecial Civil Action in the Supreme Court. Certiorari

FACTS- This is an appeal from the decision and resolution of the Court of Appeals, dated January 17, 2001 and October 30, 2002, respectively, upholding the finding of constructive dismissal against petitioner.- Petitioner R.P. Dinglasan Construction, Inc. provided janitorial services to Pilipinas Shell Refinery Corporation (Shell Corporation) in Batangas City. Private respondents Mariano Atienza and Santiago Asi served as petitioner’s janitors assigned with Shell Corporation since 1962 and 1973, respectively. Private respondents claim that on July 7, 1994, petitioner called for a meeting and informed private respondents and three (3) other employees that their employment with Shell Corporation would be terminated effective July 15, 1994. They were told that petitioner lost the bidding for janitorial services with Shell. Petitioner notified respondents that they may reapply as helpers and redeployed in other companies where petitioner had subsisting contracts but they would receive only a minimum wage. Private respondents refused as the offer would be a form of demotion --- they would lose their seniority status and would not be guaranteed to work at regular hours.- In December 1994, private respondents filed a complaint against petitioner for non-payment of salary with the district office of the Department of Labor and Employment (DOLE) in Batangas City. In February 1995, during the conciliation proceedings with the DOLE, petitioner sent notices to respondents informing them that they would be reinstated with Shell Corporation as soon as they submit their barangay clearance, medical certificate, picture and information sheet as per the new identification badge requirements of Shell Corporation. Thereafter, petitioner again met with private respondents, who were then accompanied by the barangay captain and a councilor, and the latter confirmed to the former their willingness to be reinstated. Private respondents duly submitted the documents required for their reinstatement.

Labor Law 1 A2010 - 229 - Disini- In May 1995, respondents demanded the payment of their backwages starting from July 15, 1994. On June 1, 1995, petitioner notified private respondents that they have been declared absent without leave (AWOL) as they allegedly failed to signify their intention to return to work and submit the badge requirements for their reinstatement. On June 13, 1995, private respondents wrote petitioner and insisted that they had complied with the badge requirements. Accompanied by the barangay officials, private respondents attempted to meet with the officers of petitioner but the latter refused to dialogue with them. As proof of their compliance with the Shell requirements, private respondents submitted to the DOLE their x-ray results, dated May 17 and 19, 1995 and their barangay certification, dated May 13, 1995. The case was eventually referred to the National Labor Relations Commission (NLRC) for compulsory arbitration. Private respondents amended their complaint charging petitioner with illegal dismissal and non-payment of 13 th month pay, with a claim for payment of attorney’s fees and litigation expenses, and a prayer for reinstatement with payment of full backwages from July 15, 1994.- Petitioner gave a different version of the incident. It allegedly informed respondents and the other affected employees that they would be deployed to petitioner’s other principal companies but that their work would be different. Except for private respondents, all the affected employees accepted its offer of redeployment and reported back to work. Respondents failed to submit a resignation letter to signify their intention not to return to work. Thereafter, during the pendency of the labor case, petitioner in two (2) separate notices, informed private respondents that they could be reinstated at Shell Corporation with no diminution in their salary provided that they submit the documents for the new identification badge requirement of Shell Corporation. Private respondents, however, refused to return to work until they were paid their backwages. Consequently, petitioner was constrained to consider them as having abandoned their work and to terminate their employment on September 19, 1995. Petitioner, thus, justified the dismissal of private respondents on the grounds of gross and habitual neglect of duties and abandonment of work. On September 3, 1998, labor arbiter Andres Zavalla rendered a decision finding that private respondents were illegally dismissed from service and ordering their reinstatement.- On appeal, the decision of the labor arbiter was affirmed by the NLRC. Without moving for reconsideration, petitioner immediately filed a petition for certiorari before the Court of Appeals but petitioner suffered the same fate. On the procedural aspect, the Court of Appeals ruled that the petition could not prosper as petitioner failed to move for a reconsideration of the NLRC decision. On the substantive issues, the appellate court upheld the findings of the labor arbiter and the NLRC that: (1) private respondents were constructively dismissed as petitioner’s offer of reassignment involved a diminution in pay and demotion in rank that made their continued employment unacceptable; and, (2) private respondents could not be considered to have abandoned their work.- As petitioner’s motion for reconsideration was denied, petitioner filed this appeal

ISSUES1. WON the respondents’ dismissal is justified2. WON the Court of Appeals, contrary to existing law, erred in dismissing the petition for certiorari and affirming the decision of the NLRC insofar as the monetary award is concerned

HELD1. Ratio In an illegal dismissal case, the onus probandi rests on the employer to prove that its dismissal of an employee is for a valid cause. In the case at bar, petitioner failed to discharge its burden. It failed to establish that private respondents deliberately and unjustifiably refused to resume their employment without any intention of returning to work.- To constitute abandonment of work, two (2) requisites must concur: first, the employee must have failed to report for work or must have been absent without justifiable reason; and second, there must have been a clear intention on the part of the employee to sever the employer-employee relationship as manifested by overt acts. Abandonment as a just ground for dismissal requires deliberate, unjustified refusal of the employee to resume his employment. Mere absence or failure to report for work, after notice to return, is not enough to amount to abandonment.Reasoning - In the case at bar, the evidence of private respondents negates petitioner’s theory that they abandoned their work. Firstly, private respondents reported back to petitioner’s office a number of times expressing their desire to continue working for petitioner without demotion in rank or diminution of salary. This fact was established by the corroborating testimony of barangay councilman Valentin Clerigo who, together with the barangay captain, accompanied private respondents to petitioner’s office at least ten (10) times to negotiate their redeployment on more acceptable terms. Secondly, in seeking reinstatement, private respondents also sought the intervention of the DOLE to arbitrate the labor issue between the parties. Thirdly, private respondents submitted the barangay clearances and x-ray results required from them by petitioner for their reinstatement as witnessed by the barangay officials. Lastly, the records would bear that private respondents lost no time and sought their reinstatement by filing an illegal dismissal case against petitioner, which act is clearly inconsistent with a desire to sever employer-employee relations and abandon their work. All these overt acts on the part of private respondents negate petitioner’s claim of abandonment of work and prove beyond doubt their steadfast desire to continue their employment with petitioner and be reinstated to their former position. Moreover, petitioner failed to explain why it waited for 14 months from the time private respondents allegedly did not return to work before it dismissed them for being AWOL.

- We hold that private respondents were constructively dismissed by petitioner. Constructive dismissal is defined as quitting when continued employment is rendered impossible, unreasonable or unlikely as the offer of employment involves a demotion in rank and diminution of pay. In the case at bar, petitioner committed constructive dismissal when it offered to reassign private respondents to another company but with no guaranteed working hours and payment of only the minimum wage. The terms of the redeployment thus became unacceptable for private respondents and foreclosed any choice but to reject petitioner’s offer, involving as it does a demotion in status and diminution in pay. Thereafter, for six (6) months, private respondents were in a floating status. Interestingly, it was only after private respondents filed a complaint with the DOLE that petitioner backtracked in its position and offered to reinstate private respondents to their former job in Shell Corporation with no diminution in salary. Eventually, however, petitioner unilaterally withdrew its offer of reinstatement, refused to meet with the private respondents and instead decided to dismiss them from service.2. On the second issue, petitioner cannot impugn for the first time the computation of the monetary award granted by the labor arbiter to private respondents. Doctrine The settled rule is that issues not raised or ventilated in the court a quo cannot be raised for the first time on appeal as to do so would be offensive to the basic rules of fair play and justice. The computation of monetary award granted to private respondents is a factual issue that should have been posed at the arbitration level when the award was first granted by the labor arbiter who received and evaluated the evidence of both parties, or, at the latest, raised by petitioner in its appeal with the NLRC. - Petitioner omitted to do any of these. All throughout the proceedings below, from the labor arbiter to the NLRC, and even in its petition before the Court of Appeals, petitioner repeatedly pounded only on the sole issue of the validity of its dismissal of private respondents. Thus, at this late stage of the proceedings, it cannot ask the Court to review the bases and verify the correctness of the labor arbiter’s computation of the monetary award which it never assailed below. A first-hand evaluation of the evidence of the parties upon which the monetary award is based belongs to the labor arbiter. This Court is not a trier of facts and factual issues are improper in a petition for review on certiorari. Likewise, the Court notes that in seeking reinstatement and payment of their monetary claims, private respondents have traversed a long and difficult path. This case has passed the DOLE, the labor arbiter, the NLRC, the Court of Appeals and now this Court, with the finding of illegal dismissal having been consistently affirmed in each stage. Private respondents had been rendering janitorial services as early as 1962 and, at the time of their dismissal, were receiving a measly P4,000.00 monthly salary. It is time to put a period to private respondents’ travail. If there is anything that frustrates the search for justice by the poor, it is the endless search for it.

CHAVEZ V NLRC[PAGE 59]

FLOREN HOTEL V NLRC (CALIMLIM, RICO, ET AL)458 SCRA 128

QUISUMBING; May 6, 2005

FACTS- At the time of their termination, private respondents Roderick A. Calimlim, Ronald T. Rico and Jun A. Abalos were working in the hotel as room boys, private respondent Lito F. Bautista as front desk man, and private respondent Gloria B. Lopez as waitress. They all started working for the hotel in 1993, except for Jun A. Abalos who started only in 1995.- In the afternoon of June 6, 1998, petitioner Dely Lim randomly inspected the hotel rooms to check if they had been properly cleaned. When she entered Room 301, she found private respondent Lito F. Bautista sleeping half-naked with the air-conditioning on. Lim immediately called the attention of the hotel’s acting supervisor, Diosdado Aquino, who had supervision over Bautista. Lim admonished Aquino for not supervising Bautista more closely, considering that it was Bautista’s third offense of the same nature.When she entered Room 303, she saw private respondents Calimlim and Rico drinking beer, with four bottles in front of them. They had taken these bottles of beer from the hotel’s coffee shop. Like Bautista, they had switched on the air conditioning in Room 303.- That same afternoon, Dely Lim prepared a memorandum for Bautista, citing the latter for the following incidents: (1) sleeping in the hotel rooms; (2) entertaining a brother-in-law for extended hours during duty hours; (3) use of hotel funds for payment of SSS loan without management consent; (4) unauthorized use of hotel’s air-con; and (5) failure to pay cash advance in the amount of P4,000. - Dely Lim tried to give Bautista a copy of the memorandum but Bautista refused to receive it. Bautista then went on absence without leave. Calimlim and Rico, embarrassed by the incident, went home. When they returned to work the next day, they were served with a notice of suspension for one week.- Like Bautista, they refused to receive the notice of suspension, but opted to serve the penalty. Upon their return on June 15, 1998, they saw a memorandum dated June 13, 1998 on the bulletin board announcing (a) the suspension as room boys of Calimlim and Rico, or alternately, (b) returning to work on probation as janitors for the following reasons: unsatisfactory work, having a drinking spree inside the hotel’s rooms, cheating on the Daily Time Record, being absent without valid reason, leaving work during duty

Labor Law 1 A2010 - 230 - Disinitime, tardiness, and sleeping on the job. The memorandum also included Calimlim and Rico’s new work schedule.- Calimlim and Rico submitted handwritten apologies and pleaded for another chance, before they went AWOL- On June 25, 1998, Calimlim, Rico and Bautista filed separate complaints, for illegal dismissal and money claims, before the Labor Arbiter. Abalos and Lopez later also filed separate complaints for underpayment of wages, non-payment of their 13 th month pay, and service incentive leave pay. On July 7, 1998, after they stopped working, Abalos and Lopez amended their complaints. They claimed that petitioners orally dismissed them when they refused to withdraw their complaints.- Petitioners alleged that they did not dismiss private respondents but that private respondents had abandoned their jobs. - Private respondents filed a manifestation and motion dated November 24, 1998, praying that petitioners be ordered to reinstate them to their former positions since after all, according to petitioners, they were not dismissed.- Petitioners opposed the motion and argued that private respondents cannot be reinstated since they were not illegally dismissed but they had abandoned their jobs and management simply considered them dismissed for abandonment.- On March 19, 1999, the Labor Arbiter dismissed the complaints but ordered petitioners to pay private respondents their proportionate 13th month pay, and service incentive leave pay. He likewise ordered petitioners to pay Calimlim and Rico indemnity. - The Labor Arbiter found that Calimlim, Rico, and Bautista did not report for work and they did not show any order of dismissal, thus constructively, they abandoned their work and were not illegally dismissed. The Labor Arbiter also ruled that Calimlim and Rico’s demotion and reassignment were valid exercises of management prerogatives. The reassignment was intended to enable management to supervise them more closely and, in any event, did not involve a diminution of wages. The Labor Arbiter, however, held petitioners liable for indemnity to Calimlim and Rico for not observing the twin notices rule.- Private respondents appealed to the National Labor Relations Commission- On March 22, 2000, the NLRC rendered its decision. It reversed the decision of the Labor Arbiter and ordered the hotel management to immediately reinstate complainants-appellants to their former positions without loss of seniority rights, with full backwages and other benefits until they are actually reinstated. In the event that reinstatement was no longer possible, the respondent-appellees should pay herein private respondents their separation pay in addition to the payment of their full backwages; their incentive leave pay and their 13th month pay, together with P1,000 to each of them as indemnity. - Later, the NLRC also denied petitioners’ motion for reconsideration. The petitioners appealed to the Court of Appeals. - On September 10, 2002, the Court of Appeals decided the petition as follows: (1) The Court declares that the private respondents Roderick A. Calimlim and Jose Abalos [should be Ronald T. Rico] were illegally dismissed by petitioner Floren Hotel/Ligaya Chu who is ORDERED to reinstate them to their former positions without loss of [seniority] rights, with full backwages and other benefits until they are actually reinstated; but if reinstatement is no longer possible, Floren Hotel/Ligaya Chu shall pay their separation pay in addition to their backwages. (2) Declaring private respondents Lito Bautista, Jun Abalos and Gloria Lopez to have abandoned their employment, and, therefore, not entitled to either backwages nor separation pay; and (3) ORDERING Floren Hotel/Ligaya Chu to pay all the private respondents their 13 th month pay and incentive leave pay as computed in the Decision of the Labor Arbiter

ISSUES1. WON the Court of Appeals erred in giving due course to the petition for certiorari filed before the appellate court2. WON the private respondents were illegally dismissed3 WON the Court of Appeals erred in ordering petitioners to pay Calimlim and Rico indemnity of P1,5004 WON the appellate court erred in ordering petitioners to pay all of private respondents their proportionate 13th month pay and incentive leave pay

HELD1. NO- Acceptance of a petition for certiorari as well as the grant of due course thereto is addressed to the sound discretion of the court. Section 1, Rule 65 of the Rules of Court in relation to Section 3, Rule 46 of the same rules does not specify the precise documents, pleadings or parts of the records that should be appended to the petition other than the judgment, final order, or resolution being assailed. The Rules only state that such documents, pleadings or records should be relevant or pertinent to the assailed resolution, judgment or orders.2. YES- Petitioners claimed that all five private respondents were guilty of abandoning their jobs. Thus, it was incumbent upon petitioners to show that the two requirements for a valid dismissal on the ground of abandonment existed in this case. Specifically, petitioners needed to present, for each private respondent, evidence not only of the failure to report for work or that absence was without valid or justifiable reasons, but also of some overt act showing the private respondent’s loss of interest to continue working in his or her job. - It was true that private respondents abandoned their jobs, then petitioners should have served them with a notice of termination on the ground of abandonment as required

under Sec. 2, Rule XIV, Book V, Rules and Regulation Implementing the Labor Code, in effect at that time. Said Section 2 provided that:

Notice of Dismissal. Any employer who seeks to dismiss a worker shall furnish him a written notice stating the particular acts or omission constituting the grounds for his dismissal. In cases of abandonment of work, the notice shall be served at the worker’s last known address.

- But petitioners failed to comply with the foregoing requirement, thereby bolstering further private respondents’ claim that they did not abandon their work but were illegally dismissed. - None of the private respondents in this case had any intention to sever their working relationship. Just days after they were dismissed, private respondents Calimlim, Rico, Bautista, Abalos and Lopez filed complaints to protest their dismissals. The well-established rule is that an employee who takes steps to protest his layoff cannot be said to have abandoned his work. That private respondents all desired to work in the hotel is further shown by the fact that during the proceedings before the Labor Arbiter, shortly after private respondents received petitioners’ position paper where the latter averred that private respondents were never terminated, private respondents filed a manifestation and motion asking that petitioners be ordered to allow them back to work. This is nothing if not an unequivocal expression of eagerness to resume working.3. YES (should have reinstated)- Article 279 of the Labor Code gives to Calimlim and Rico the right to reinstatement without loss of seniority rights and other privileges or separation pay in case reinstatement is no longer possible, and to his full backwages, inclusive of allowances and other benefits. It was thus error for the Court of Appeals to affirm the NLRC decision to award Calimlim and Rico indemnity in addition to the measure of damages provided in Article 279. The award of indemnity is a penalty awarded only when the dismissal was for just or authorized cause but where the twin-notice requirement was not observed. 4. NO- Petitioners did not question the propriety of the award of proportionate 13 th month pay and service incentive leave in the Court of Appeals. They assailed the NLRC decision on only one ground: “Respondent NLRC committed grave abuse of discretion in reversing the Labor Arbiter’s decision insofar as it relates to the issues of illegal dismissal.” Hence, the correctness of the cited award in the NLRC ruling was never brought before the appellate court and is deemed to have been admitted by petitioners. It cannot therefore be raised anymore in this petition. The decision of the NLRC as regards the award of 13 th

month pay and service incentive leave pay became binding on petitioners because the failure to question it before the Court of Appeals amounts to an acceptance of the ruling. In any event, the award appears to us amply supported by evidence and in accord with law.Disposition Assailed decision MODIFIED

INFERENCE

HDA. DAPDAP V NLRC (BARRIENTOS JR) 285 SCRA 9

BELLOSILLO; January 26, 1998

FACTS- Nine workers of Hda. Dapdap I, a sugar farm in Victorias, Negros Occidental, filed a complaint for illegal dismissal against its owner Magdalena Fermin alleging that they had been working in the farm since 1977 but were unjustly terminated, without notice and without any valid ground, on 27 January 1992. - The only reason for their dismissal was their refusal to return the 6-hectare lot given to them for cultivation under an "Amicable Settlement” in connection with an illegal dismissal case previously filed against the management of Hda. Dapdap I by its workers.- In addition, complainants charged Magdalena Fermin with unfair labor practice for trying to bust the National Federation of Sugar Workers Food and General Trades (NFSW-FGT) Union which forged the 1986 "Amicable Settlement."- Eight of the original complainants withdrew from the complaint and returned to work on the ground that their misunderstanding with management was already settled.- Pedro Barrientos Jr. was left as the sole complainant who amended the complaint by impleading Lumbia Agricultural and Development Corporation (LADCOR), the real owner of Hda. Dapdap I, as co-respondent with its President Magdalena Fermin.- LADCOR denied that complainant was terminated and alleged that complainant voluntarily abandoned his work to transfer to the adjacent farm of a certain Mr. Ramos. - In addition, LADCOR alleged that it had a personality separate and distinct from its president, Magdalena Fermin, hence the latter could not be held personally liable for the alleged illegal dismissal.- The Labor Arbiter ruled in favor of complainant.] While LADCOR was absolved from the charge of unfair labor practice it was held liable for illegal dismissal on the ground that its claim of voluntary abandonment by complainant of his work was not credible in view of the immediate institution of the case for illegal dismissal. - LADCOR appealed to the NLRC. - The NLRC affirmed the Labor Arbiter's decision in toto. The defense that complainant voluntarily abandoned his work was similarly rejected on the additional grounds that no notice of dismissal was sent by LADCOR to complainant as required by Sec. 2, Rule 14, Book V, of the Rules Implementing the Labor Code and no concurrence of the intention to abandon on the part of complainant and overt acts from which it could be inferred that

Labor Law 1 A2010 - 231 - Disinihe was no longer interested in working for LADCOR.

ISSUE WON petitioner was illegally dismissed

HELD YES- The Court is not a trier of facts. Whether respondent voluntarily abandoned his work issue of credibility best left to the determination of the Labor Arbiter. Great respect and even finality is accorded the conclusions of the Labor Arbiter and the NLRC in accordance with the well-settled rule that findings of fact of labor arbiters affirmed by the NLRC are binding on the Supreme Court. - Judicial review in such cases is limited only to issues of jurisdiction or grave abuse of discretion amounting to lack of jurisdiction.- No such grave abuse of discretion was committed by the NLRC as it correctly applied the consistent ruling in labor cases that a charge of abandonment is totally inconsistent with the immediate filing of a complaint for illegal dismissal. - It is indeed inconceivable that an employee like herein respondent who has been working at Hda. Dapdap I since 1977 and cultivating a substantial portion of a 6-hectare lot therein for himself would just abandon his work in 1992 for no apparent reason. - Nor could intent to abandon be presumed from private respondent's subsequent employment with another employer as petitioner alleges. The fact that the start of such employment, i.e., after 1 March 1992 as petitioner alleges, coincides with the date of the original complaint strongly indicates that such employment was only meant to help respondent and his family survive during the pendency of his case. - It has been said that abandonment of position cannot be lightly inferred, much less legally presumed from certain equivocal acts such as an interim employment.Disposition Petition was dismissed.

SPECIFIC ACTS

PREMIERE DEVT BANK V NLRC (LABANDA)293 SCRA 49

MARTINEZ; July 23, 1988

NATUREPetition for certiorari

FACTS- August 8, 1985: Ramon T. Ocampo, a depositor of Premiere Devt Bank (PDB), issued a check in the amount of P6,792.66 in favor of and for deposit to the account of Country Banker's Insurance Corporation (CBISCO), also a depositor of PDB. On the same day, after the check and the deposit slip were presented to respondent Teodora Labanda, who was employed as teller at PDB Taytay Branch, they were turned over to the Branch cashier for verification of the fund balance and signature of the drawer. There was a confirmation of the check and the same was accepted by Labanda for deposit to the current account of CBISCO.- The check was posted by Manuel S. Torio, the Taytay Branch bookkeeper. But instead of posting it to CBISCO's account, the same was posted to the account of Ocampo treating it as "On-Us Check," that is, drawn against the Taytay Branch where the check was deposited.- January 13, 1986: the wife of Ocampo, together with the auditor from CBISCO, went to PDB and complained to PDB Chairman Dr. Procopio C. Reyes that her husband was being held accountable for the amount. It was only then that PDB discovered the misposting of the check issued by Ocampo, resulting in the overstatement of his outstanding daily balance by P6,792.66. The overstatement remained undetected until Ocampo withdrew the money from PDB.- Due to this incident, PDB Asst VPres Pacita M. Araos sent a demand letter to Labanda requesting her to explain in writing the misposting and erroneous crediting of the subject check in issue as well as the circumstances surrounding the incident within three (3) days from receipt thereof, and in case she fails to do so, necessary action shall be taken against her.- PDB Exec VPres Renato G. Dionisio, upon instructions of Reyes, sent the internal auditors of the bank to investigate and make a detailed report about the incident. - January 22, 1986: the auditors came out with a report finding Labanda and bookkeeper Torio primarily liable for the incident. These findings prompted Dionisio to send a letter to Labanda requiring her to shoulder 20% of the amount lost via salary deduction. Labanda replied, objecting to such move, reasoning out that she is the breadwinner in the family. She further asked the bank to furnish her a copy of the audit report and requested for a full-dress investigation. For this reason, petitioners held in abeyance the salary deductions.- March 13, 1986: Labanda was placed under preventive suspension pending investigation of the incident. She was requested to report on April 4, 1986 so that she can present her side of the story. Labanda then wrote a letter to Reyes requesting information on the duration of her suspension and at the same time asking for an expeditious investigation. In response thereto, she was informed that the period of her suspension shall last until the investigation is completed and a decision is made thereon.- On the date of said inquiry, Labanda executed a statement. However, she manifested

before Atty. Revelo during the inquiry that she will not sign any of the preliminary statements she made unless the same is with the consent and advice of her husband. She also told the inquiring officer that she could not inform petitioners of the dates when she would be available for investigation.- April 8, 1986: another letter was sent to Labanda by Reyes informing the former that her refusal to sign or authenticate preliminary statements given on April 4, 1986 was a clear indication of her unwillingness to cooperate or an effort to hide something or suppress the truth.- The dates of the hearing were rescheduled by petitioners several times. The first rescheduled hearing was on April 14, 1986 where Labanda sent her lawyer bringing with him a letter asking that she be given time to confer with her counsel for which she was given until April 23. Notices were sent to inform her of the rescheduled dates with warning that failure to attend the same shall be taken as a tacit admission of her liability and the case shall be resolved based on the evidence available. In the meantime, Bookkeeper Torio admitted liability and was allowed to resign.- April 7, 1986: the bank officials received a letter from Labanda through her counsel demanding payment of actual damages in the amount of P50,000.00 for their alleged arbitrary, illegal and oppressive acts. Petitioners did not heed the demand.- May 23, 1986: Labanda filed a complaint for damages before the court. Petitioners’ motion to dismiss, and subsequent motion for reconsideration were both denied. The petition for certiorari was also dismissed by CA, without prejudice to the refiling of the complaint with the labor arbiter. The decision became final and executory on July 30, 1987.- Eight months from the finality of the CA decision and two years from the alleged termination of her employment, Labanda filed an illegal dismissal case before the Labor Arbiter on the ground that her dismissal was without lawful cause and without due process. After trial, the Labor Arbiter dismissed the labor case, ruling that Labanda was not illegally dismissed, and that she abandoned her job when she filed a complaint for compensatory damages with the regular court.- NLRC reversed the decision of the Labor Arbiter ruling that Labanda’s indefinite preventive suspension amounted to constructive dismissal. It ordered PDB to immediately reinstate Labanda to her former position with backwages and other benefits for a period not exceeding three (3) years without qualifications and deductions computed on the amount of P87,750.00. It denied the subsequent MFR.

ISSUES1. WON there was legal cause in placing Labanda under preventive suspension2. WON the filing of an action for damages against one's employer is tantamount to abandonment of job 3. WON PDB violated due process requirements in dismissing Labanda4. WON Labanda’s action is barred by laches

HELD1. NO- Labanda's preventive suspension is without valid cause since she was outrightly suspended by petitioner. As of the date of her preventive suspension on March 13, 1986 until the date when the last investigation was rescheduled on April 23, 1986, more than 30 days had expired. The preventive suspension beyond the maximum period amounted to constructive dismissal.- The question of whether or not an employee has abandoned his/her work is a factual issue, not reviewable by this Court.2. NO- Labanda did not abandon her job. To constitute abandonment, two elements must concur: (1) the failure to report for work or absence without valid or justifiable reason, and (2) a clear intention to sever the employer-employee relationship, with the second element as the more determinative factor and being manifested by some overt acts. Abandoning one's job means the deliberate, unjustified refusal of the employee to resume his employment and the burden of proof is on the employer to show a clear and deliberate intent on the part of the employee to discontinue employment.- The law, however, does not enumerate what specific overt acts can be considered as strong evidence of the intention to sever the employee-employer relationship. An employee who merely took steps to protest her indefinite suspension and to subsequently file an action for damages, cannot be said to have abandoned her work nor is it indicative of an intention to sever the employer-employee relationship. Her failure to report for work was due to her indefinite suspension. Petitioner's allegation of abandonment is further belied by the fact that Labanda filed a complaint for illegal dismissal. Abandonment of work is inconsistent with the filing of said complaint.3. YES- The twin requirements of notice and hearing constitute the essential elements of due process which are set out in Rule XIV, Book V of the Omnibus Rules Implementing the Labor Code.- Granting arguendo that there was abandonment in this case, it nonetheless cannot be denied that notice still has to be served upon the employee sought to be dismissed, as the second sentence of Section 2 of the pertinent implementing rules explicitly requires service thereof at the employee's last known address. While it is conceded that it is the employer's prerogative to terminate the services of an employee, especially when there is a just cause therefor, the requirements of due process cannot be taken lightly. The law does not countenance the arbitrary exercise of such a power or prerogative when it has the effect of undermining the fundamental guarantee of security of tenure in favor of the employee.

Labor Law 1 A2010 - 232 - Disini4. NO- Laches is the failure for an unreasonable and unexplained length of time to do that which in exercising due diligence, could or should have been done earlier. It is negligence or omission to assert a right within a reasonable time, warranting the presumption that the party entitled to assert it either has abandoned or has declined to assert it. A party cannot be held guilty of laches when he has not incurred undue delay in the assertion of his rights.- Under the law, an illegal dismissal case is an action predicated on the injury to the rights of the dismissed employee which prescribes in four (4) years. On April 4, 1988 or eight months from the finality of the Court of Appeals' decision and two years from the alleged termination of employment by respondent Labanda, she filed her complaint with the Labor Arbiter which is within the four-year reglementary period. She did not sleep on her rights for an unreasonable length of time.- SolGen: Labanda never intended to abandon her job. First, after her indefinite suspension, she requested that the "full-dressed" investigation be done at the quickest time possible, and appealed to petitioner Reyes to consider that she was the breadwinner in the family. Second, she actively fought for her right to security of tenure by filing first with the RTC an action for damages, and later with the Labor Arbiter a complaint for illegal dismissal. Moreover, Labanda's inability to report for work was not voluntary but was rather the result of her indefinite suspension, which in reality was a constructive dismissal. Petitioners never took the initiative to notify Labanda to report back to work or charge the latter with abandonment of work. These show that Labanda did not abandon her job but was illegally dismissed from employment without due process of law.Disposition Petition is DISMISSED. The challenged NLRC Resolution is AFFIRMED.

1. LOANS

BORROWING MONEY

MEDICAL DOCTORS INC V NLRC (MAGLAYA, ELOÑA)136 SCRA 1

MAKASIAR; April 24, 1985

NATUREAn appeal of the decision of the NLRC.

FACTS- Evelyn Eloña (complainant) was given a probationary appointment as Clerk by the Makati Medical Center from July 16, 1975 to January 15, 1976, and assigned at the Out-Patient Charity Department of said Medical Center.- Two of the conditions embodied in the appointment:

'Comply with all existing policies, rules and regulations and those that may be adopted or promulgated in the future deemed necessary in the internal affairs of the employer; 'If at anytime during the probationary employment of the employee her services are judged to be unsatisfactory, the employer may terminate such employment.'

- The termination or dismissal was and is predicated mainly on the fact that Evelyn Elona borrowed P50 from one of the patients, Mrs. Leticia Lavapiez, allegedly in violation of respondent's policies, rules and regulations against solicitation of any consideration from indigent patients. The borrowing took place at Mrs. Lavapiez’s house and after she was discharged from the OPCD. The amount of P50 that was borrowed was also returned, remitted or paid by complainant to Mrs. Lavapiez- Eloña worked in this capacity of clerk continuously until February 14, 1976 when she was dismissed or terminated.- NLRC: “Borrowing money and paying the same is not an act of dishonesty, of immorality, of illegality, or of omissions punishable by law as to be a ground for dismissal as in this case. We so hold that the Rules and Regulations & Policies of respondent Medical Center are whimsical, capricious, arbitrary and oppressive… The facts and the law point unerringly to her side. She has completed her probationary period. Her employment contract is not covered by an apprenticeship agreement stipulating a longer period.”

ISSUEWON Eloña was justly dismissed on sole reason of borrowing money from the patients

HELDNO, Eloña was not dismissed justly.Ratio Borrowing money is neither dishonest, nor immoral nor illegal, much less criminal.Reasoning- Private respondent paid the money she borrowed from the hospital patient. She was even recommended for permanent appointment from her probationary status, from clerk to secretary, by her immediate superior, Sis. Consolacion Briones.- It may be added that she must have been compelled to borrow P50.00 from her patient because of economic necessity, which circumstance should evoke sympathy from this Court, the very constitutional organ mandated by the fundamental law to implement the social justice guarantee for the protection of the lowly, efficient and honest employee, who is economically disadvantaged, like herein petitioner.”

Disposition Petition is dismissed, and decision of the labor arbiter is affirmed, with the modification that backwages should cover three (3) years.

SEPARATE OPINION

AQUINO [dissent]- Nicolas A. Zarate, the chief of the public information assistance division, apprised the Makati Medical Center of Evelyn's conduct. Zarate alleged that Evelyn "has the habit of borrowing money from OPD patients of that hospital." Evelyn allegedly borrowed P100 from Leticia Lavapiez after she delivered a baby. She attempted to borrow money from Teofila Luzon and tried to ask for lunch from another patient, Mrs. Fabian. A copy of the denunciation was furnished Mayor Nemesio Yabut.- To have more time for investigating the charge, Evelyn's probationary appointment was extended by one month or up to February 15, 1976. After due investigation, Consolacion Briones, the supervisor of the Outpatient Charity Department, submitted a report exonerating Evelyn. The Barangay Secretariat of Makati also recommended Evelyn's exoneration.- Eloña should not be reinstated or placed under permanent status because, as correctly observed by Commissioner Villatuya of NLRC, she was dismissed when she was still a probationary employee. It is true that the probationary status does not exceed six months but under the peculiar circumstances of this case Evelyn's probationary or temporary status was extended for one month due to the investigation. This may well be considered an exceptional case. Evelyn is not the kind of employee who can invoke security of tenure.

PEARL S. BUCK FOUNDATION V NLRC 182 SCRA 446

GUTIERREZ; February 21, 1990

NATUREAppeal from the decision of the NLRC as well as the resolution denying the motion for reconsideration

FACTS- Petitioner Pearl S. Buck Foundation, Inc. extends financial, education and medical assistance to indigent "Amerasian" youth through funds provided by individuals and church groups in the US. Private respondent Rubini Gosiaco Querimit was employed by the petitioner as a case worker in the Olongapo City branch. One of the wards assigned to Mrs. Querimit as such case worker was Richard Aliarte, Amerasian son of Andrea Aliarte.- It appears that Mrs. Querimit borrowed P300 from Andrea Aliarte. It is not clear from the records when she paid said debt but Mrs. Querimit once again borrowed P3,000.00 from Aliarte, who requested assistance from petitioner for the collection of the indebtedness. Mrs. Querimit paid the amount allegedly only after the petitioner had exerted incessant pressure on her. Thereafter, she received a letter dated from the petitioner's resident director informing her that her services would be terminated. Mrs. Querimit filed in the NLRC a complaint for illegal dismissal, underpayment, overtime pay and maternity benefits.- The labor arbiter dismissed the complaint for lack of merit. On appeal, the NLRC opined that borrowing money is not a ground for termination of employment under the Labor Code and that the loan is a "personal transaction" between Andrea Aliarte and Mrs. Querimit "the respondent not being a privy to (the) transaction and hence, had no cause to dismiss the complainant from her job more so that the loan had earlier been paid and settled." The petitioner filed an MFR. After it was denied, the petitioner filed the instant petition.

ISSUEWON private respondent was illegally dismissed

HELD1. NORatio Borrowing money is neither dishonest, nor immoral, nor illegal, much less criminal. However, said act becomes a serious misconduct that may justly be asserted as a ground for dismissal when reprehensible behavior such as the use of a trust relationship as a leverage for borrowing money is involved.Reasoning- The fact that Aliarte has retracted her complaint is of no moment. She loaned money to the respondent, not once but twice and there can be no other assumption where the money came from except from the trust funds intended for the ward. The NLRC should have considered that a higher degree of prudence is required of the foundation's employees especially when it comes to financial matters affecting the petitioner's wards. The petitioner solicits or "begs" for money from abroad to support its wards. It cannot be a third person where that money is involved.Disposition The petition is GRANTED. The decision of the NLRC is REVERSED and SET ASIDE. The decision of the Labor Arbiter is REINSTATED.

2. COURTESY RESIGNATION

Labor Law 1 A2010 - 233 - DisiniBATONGBACAL V ASSOCIATED BANK

168 SCRA 600FERNAN; December 21, 1988

NATUREReview of the decision of the NLRC

FACTS- Bienvenido Batongbacal, a lawyer, worked for Citizens Bank and Trust Company from 1961. On 1975, Citizens Bank and Trust Company merged with the Associated Banking Corporation. The merged corporate entity later became known as Associated Bank. In the new bank, petitioner resumed his position as assistant vice-president.- On March 1982, he learned that his salary was very much below compared to the other Asst. VPs of the bank. He wrote to the Board of Directors asking that he be paid the proper amount. Apparently, said letter fell on deaf ears.- On March 15, 1982, the board approved the following resolution:

“BE IT RESOLVED that the new management be given the necessary flexibility in streamlining the operations of the Bank and for the purpose it is hereby resolved that the Bank officers at the Head Office and the Branches with corporate rank of Manager and higher be required, as they hereby are required to submit IMMEDIATELY to the President their courtesy resignations.”

- Petitioner did not submit his courtesy resignation. On May 3, 1983, he received a letter from the Board saying that his resignation has been accepted. Petitioner wrote to the executive VP asking for reconsideration. He stated therein that he thought the call for the submission of courtesy resignations was only for erring "loathsome" officers and not those like him who had served the bank honestly and sincerely for sixteen years. - Starting May 4, 1983, he was not paid. He filed for illegal dismissal and damages with the NLRC. The NLRC ruled in favor of the petitioner. On MFR, the NLRC reversed.

ISSUEWON the bank may legally dismiss for refusal to tender the courtesy resignation which the bank required in line with its reorganization plan

HELDNO- While it may be said that the private respondent's call for courtesy resignations was prompted by its determination to survive, we cannot lend legality to the manner by which it pursued its goalBy directing its employees to submit letters of courtesy resignation, the bank in effect forced upon its employees an act which they themselves should voluntarily do. It should be emphasized that resignation per se means voluntary relinquishment of a position or office. 11 Adding the word "courtesy" did not change the essence of resignation. That courtesy resignations were utilized in government reorganization did not give private respondent the right to use it as well in its own reorganization and rehabilitation plan. There is no guarantee that all employers will not use it to rid themselves arbitrarily of employees they do not like, in the guise of "streamlining" its organization. On the other hand, employees would be unduly exposed to outright termination of employment which is anathema to the constitutional mandate of security of tenure- The record fails to show any valid reasons for terminating the employment of petitioner. There are no proofs of malfeasance or misfeasance committed by petitioner which jeopardized private respondent's interest.- However, we agree with the Solicitor General and the NLRC that petitioner is not entitled to an award of the difference between his actual salary and that received by the assistant vice-president who had been given the salary next higher to his. There is a semblance of discrimination in this aspect of the bank's organizational set-up but we are not prepared to preempt the employer's prerogative to grant salary increases to its employees. In this connection, we may point out that private respondent's claim that it needed to trim down its employees as a self-preservation measure is belied by the amount of salaries it was giving its other assistant vice-presidentsDisposition Remanded to the NLRC to determine WON the petitioner is a managerial employee

3. WORK ATTITUDE

ABSENCES

MANILA ELECTRIC CO V NLRC[PAGE 186]

GSP MANUFACTURING CORP V CABANBAN495 SCRA 123

CORONA; July 14, 2006

NATURE

Petition for review on certiorari from a decision and a resolution of the Court of Appeals.

FACTS- Cabanban worked with GSP Manufacturing Corporation (GSP) as a sewer from February 7, 1985 until her alleged termination on March 1, 1992. - On June 16, 1992, respondent filed with the National Labor Relations Commission (NLRC), National Capital Region Arbitration Branch, a complaint against petitioners for illegal dismissal, non-payment of holiday pay, service incentive leave pay and 13th month pay. She claimed she was terminated by petitioners because she failed to dissuade her daughter from continuing her employment at the Sylvia Santos Company, a business competitor of petitioners.In their defense, petitioners argued that respondent abandoned her work on March 14, 1992 and that they reported this to the Department of Labor and Employment on May 15, 1992.- Labor arbiter found petitioners guilty of illegal dismissal. Petitioners appealed to the NLRC, it was dismissed. Petition to CA was also dismissed. They claim that these findings, based solely on statements made by respondent in the affidavit attached to her position paper, were arrived at arbitrarily. ISSUEWON respondent is guilty of abandonment

HELDNO- Abandonment as a just ground for dismissal requires the deliberate, unjustified refusal of the employee to perform his employment responsibilities. Mere absence or failure to work, even after notice to return, is not tantamount to abandonment. The records are bereft of proof that petitioners even furnished respondent such notice. - Furthermore, it is a settled doctrine that the filing of a complaint for illegal dismissal is inconsistent with abandonment of employment. An employee who takes steps to protest his dismissal cannot logically be said to have abandoned his work. The filing of such complaint is proof enough of his desire to return to work, thus negating any suggestion of abandonment.- Clearly, petitioners’ claim that respondent’s complaint was “an afterthought,” having been filed a long time after the date of the supposed abandonment, was utterly without merit. As the Court of Appeals correctly pointed out, citing the case of Pare v. NLRC, respondent had four years within which to institute her action for illegal dismissal. Compared to the six months it took the aggrieved employee in that case to file his complaint for illegal dismissal, respondent’s 84 days was not unreasonably long at all.Disposition petition is hereby DENIED. The assailed decision and resolution of the Court of Appeals in CA-G.R. SP No. 51161 are hereby AFFIRMED.

4. TERM EMPLOYMENT

BRENT SCHOOL V ZAMORA[PAGE 94]

ROMARES V NLRC294 SCRA 411

MARTINEZ; August 19, 1998

NATUREAppeal from a decision of NLRC

FACTS- Complainant-petitioner Romares has been hired and employed by respondent PILMICO since Sept 1, ‘89 to Jan 15, ‘93, in a broken tenure but all in all totalled to over a year's service. Complainant's period of employment started on Sept 1, ‘89 up to Jan 31, ‘90 or for a period of 5 months. Then on Jan 16 ‘91, he was hired again up to June 15, ‘91, or for a period covering another 5 months. Then on Aug 16, ‘92, he was hired again up to Jan 15, ‘93 or for a period of another 5 months. Thus, from Sept 1, 1989 up to January 15, 1993, complainant has worked for 15 months more or less and has been hired and terminated 3 times. In all his engagements by respondent, he was assigned at respondent's Maintenance/Projects/Engineering Dept performing maintenance work, particularly the painting of company buildings, maintenance chores, like cleaning and sometimes operating company equipment and sometimes assisting the regulars in the Maintenance/ Engineering Dept. - Petitioner’s arguments: That having rendered a total service of more than 1 year and by operation of law, complainant has become a regular employee of respondent; That complainant has performed tasks and functions which were necessary and desirable in the operation of respondent's business which include painting, maintenance, repair and other related jobs; That complainant was never reprimanded nor subjected to any disciplinary action during his engagement with the respondent; That without any legal cause or justification and in the absence of any time to know of the charge or notice nor any opportunity to be heard, respondent terminated him; That his termination is violative of the security of tenure clause provided by law; That complainant be awarded damages

Labor Law 1 A2010 - 234 - Disiniand be reinstated to his former position, be awarded backwages, moral and exemplary damages and atty's fees. - Respondents’ arguments: That complainant was a former contractual employee of respondent and as such his employment was covered by contracts; That complainant was hired as mason in the Maintenance/Project Department and that he was engaged only for a specific project under such department; That when his last contract expired on Jan 15, 1993, it was no longer renewed and thereafter, complainant filed this instant complaint; and; That since petitioner's employment contracts were for fixed or temporary periods, as an exception to the general rule, he was validly terminated due to expiration of the contract of employment.- LA ruled in favor of petitioner finding him to be a regular employee and hence should be reinstated. NLRC reversed LA decision ruling that petitioner was engaged in a fixed term employment and as such, his termination was valid due to expiration of employment contract. Hence, this appeal.

ISSUEWON dismissal of complainant (under the just cause that such employment was of term employment) was justified

HELDNO[a] Petitioner was deemed a regular employee. Petitioner’s work with PILMICO as a mason was definitely necessary and desirable to its business. PILMICO cannot claim that petitioner's work as a mason was entirely irrelevant to its line of business in the production of flour yeast feeds and other flour products. During each rehiring, the summation of which exceeded 1 year, petitioner was assigned to PILMICO's Maintenance/Projects/Engineering Dept performing the same kind of maintenance work such as painting of company buildings cleaning and operating company equipment, and assisting the other regular employees in their maintenance works. Such a continuing need for the services of petitioner is sufficient evidence of the necessity and indispensability of his services to PILMICO's business or trade. [b] Even assuming arguendo that petitioner was temporary EE, he was converted to regular employee ff this rule: If the employee has been performing the job for at least one year, even if the performance is not continuous or merely intermittent, the law deems the repeated and continuing need for its performance as sufficient evidence of the necessity is not indispensability of that activity to the business. Hence, the employment is also considered regular but only with respect to such activity and while such activity exists. [c] In rehiring petitioner, employment contracts ranging from 2 to 3 months with an express statement that his temporary job/service as mason shall be terminated at the end of the said period or upon completion of the project was obtrusively a convenient subterfuge utilized to prevent his regularization. It was a clear circumvention of the employee's right to security of tenure and to other benefits. It likewise evidenced bad faith on the part of PILMICO.[d] NLRC erred in finding that the contract of employment of petitioner was for a fixed or specified period. From Brent v Zamora: The decisive determinant in "term employment" should not be the activities that the employee is called upon to perform but the day certain agreed upon by the parties for the commencement and termination of their employment relationship. But, if from the circumstances it is apparent that the periods have been imposed to preclude acquisition of tenurial security by the employee, they should be struck down or disregarded as contrary to public policy and morals.Note however that, "term employment" cannot be said to be in circumvention of the law on security of tenure if: (1) The fixed period or employment was knowingly and voluntarily agreed upon by the parties without any force, duress, or improper pressure being brought to bear upon the employee and absent any other circumstances vitiating his consent; or (2) It satisfactorily appears that the employer and the employee dealt with each other on more or less equal terms with no moral dominance exercised by the former or the latter None of these requisites were complied with.Disposition Petition GRANTED. NLRC decision SET ASIDE. LA decision REINSTATED

MEDENILLA V PHIL VETERANS BANKPURISIMA; March 13, 2000

FACTS- Petitioners were employees of the Philippine Veterans Bank (PVB). On June 15, 1985, their services were terminated as a result of the liquidation of PVB. On the same day of their termination, petitioners were rehired through PVB's Bank Liquidator.- All of them were required to sign employment contracts which provided that:(1) The employment shall be strictly on a temporary basis and only for the duration of the particular undertaking for which a particular employee is hired; (3) The Liquidator reserves the right to terminate the services of the employee at any time during the period of such employment if the employee is found not qualified, competent or, efficient in the performance of his job, or have violated any rules and regulations, or such circumstances and conditions recognized by law.- January 18, 1991 their employment was terminated. The reasons for which were "(a) To reduce costs and expenses in the liquidation of closed banks in order to protect the interests of the depositors, creditors and stockholders of Bank. (b) The employment were on strictly temporary basis."- Petitioners filed for illegal dismissal. Labor Arbiter found for employees. NLRC however reversed decision

ISSUES1. WON NLRC gravely abused its discretion in holding that the employment contract entered into by the complainants and the Liquidator of PVB was for a fixed-period2. WON NLRC act with grave abuse of discretion in finding that there was no illegal dismissal

HELD1. NO- Employment contract between parties states that:(1) The employment shall be on a strictly temporary basis and only for the duration of the particular undertaking for which you are hired and only for the particular days during which actual work is available as determined by the Liquidator or his representatives since the work requirements of the liquidation process merely demand intermittent and temporary rendition of services."- The Court has repeatedly upheld the validity of fixed-term employment. Philippine National Oil Company-Energy Development Corporation vs. NLRC gave two guidelines by which fixed contracts of employment can be said NOT to circumvent security of tenure:

1. The fixed period of employment was knowingly and voluntarily agreed upon by the parties, without any force, duress or improper pressure being brought to bear upon the employee and absent any other circumstances vitiating his consent;or:2. It satisfactorily appears that the employer and employee dealt with each other on more or less equal terms with no moral dominance whatever being exercised by the former on the latter."

- The employment contract entered into by the parties herein appears to have observed the said guidelines. Furthermore, it is evident from the records that the subsequent re-hiring of petitioners which was to continue during the period of liquidation and the process of liquidation ended prior to the enactment of RA 7169 entitled, "An Act to Rehabilitate Philippine Veterans Bank”2. YES- The reason given by the Liquidator for the termination of petitioners' employment was "in line with the need of the objective of the Supervision and Examination Sector, Department V, Central Bank of the Philippines, to reduce costs and expenses in the liquidation of closed banks in order to protect the interest of the depositors, creditors and stockholders- In cases of illegal dismissal, the burden is on the employer to prove that there was a valid ground for dismissal. Mere allegation of reduction of costs without any proof to substantiate the same cannot be given credence by the Court. As the respondents failed to rebut petitioners' evidence, the irresistible conclusion is that the dismissal in question was illegal.- the failure of respondent bank to dispute complainants' evidence pertinent to the various unnecessary and highly questionable expenses incurred renders the termination process as a mere subterfuge, as the same was not on the basis as it purports to see, for reason that immediately after the termination from their respective positions, the same were given to other employees who appear not qualified. What respondent's counsel did was merely to dispute by pleadings the jurisdiction of this Office and the claims for damages, which evidentiary matters respondent is required to prove to sustain the validity of such dismissals."- As held by this Court, if the contract is for a fixed term and the employee is dismissed without just cause, he is entitled to the payment of his salaries corresponding to the unexpired portion of the employment contract

MAGSALIN V NATIONAL ORGANIZATION OF WORKING MEN

[PAGE 77]

LABAYOG V MY SAN BISCUITS INC[PAGE 89]

5. PAST INFRACTIONS

PAST OFFENSES

STELLAR INDUSTRIAL SERVICE INC V NLRC (PEPITO)

252 SCRA 323REGALADO; January 24, 1996

NATURESpecial Civil Action for Certiorari

FACTS

Labor Law 1 A2010 - 235 - Disini- Stellar Industrial Services, Inc., an independent contractor engaged in the business of providing manpower services, employed private respondent Roberto H. Pepito as a janitor on January 27, 1975 and assigned the latter to work as such at the Maintenance Base Complex of the Philippine Airlines in Pasay City.- Pepito worked for 15 years.- According to petitioner, private respondent committed infractions of company rules ranging from tardiness to gambling, but he was nevertheless retained as a janitor out of humanitarian consideration and to afford him an opportunity to reform.- Stellar finally terminated private respondent's services on January 22, 1991 because of Absent Without Official Leave/Virtual Abandonment of Work Absent from November 2 - December 10, 1990.- Private respondent had insisted that during the period in question he was unable to report for work due to severe stomach pain and that, as he could hardly walk by reason thereof, he failed to file the corresponding official leave of absence. Attached was a medical certificate.- Petitioner filed a complaint for illegal dismissal, illegal deduction and underpayment of wages with prayer for moral and exemplary damages and attorney's fees.- LA was of the view that Pepito was not entitled to differential pay, or to moral and exemplary damages for lack of bad faith on the part of the company, he opined that private respondent had duly proved that his 39-day absence was justified on account of illness and that he was illegally dismissed without just cause. He ordered the respondent to immediately reinstate complainant to his former position as Utilityman, without loss of seniority rights and with full backwages and other rights and privileges appurtenant to his position until he is actually reinstated.- The respondent is further ordered to pay the complainant reasonable attorney's fees equivalent to 10% of the amount recoverable by the complainant.- LA’s decision was affirmed by NLRC

ISSUES1. WON serious misconduct for nonobservance of company rules and regulations may be attributed to Pepito2. WON the extreme penalty of dismissal meted to him by Stellar may be justified under the circumstances

HELD1. NO- Stellar's company rules and regulations on the matter could not be any clearer, to wit:

"Absence Without Leave"Any employee who fails to report for work without any prior approval from his superior(s) shall be considered absent without leave.In the case of an illness or emergency for an absence of not more than one (1) day, a telephone call or written note to the head office, during working hours, on the day of his absence, shall be sufficient to avoid being penalized.In the case of an Illness or an emergency for an absence of two (2) days or more, a telephone call to the head office, during regular working hours, on the first day of his absence, or a written note to the head office, (ex. telegram) within the first three (3) days of his absence, and the submission of the proper documents (ex. medical certificate) On the first day he reports after his absence shall be sufficient to avoid being penalized.1st offense- three (3) days suspension2nd offense- seven (7) days suspension3rdoffense- fifteen (15) days suspension4th offense- dismissal with a period of one (1) year

- There was substantial compliance with said company rule by private respondent. He immediately informed his supervisor of the fact that he could not report for work by reason of illness. At the hearing, it was also established without contradiction that Pepito was able to talk by telephone to one Tirso Pamplona, foreman, and he informed the latter that he would be out for two weeks as he was not feeling well. Added to this is his letter to the chief of personnel which states that, on November 2, 1990, he relayed to his supervisor his reason for not reporting for work and that, thereafter, he made follow-up calls to their office when he still could not render services. As earlier noted, these facts were never questioned nor rebutted by petitioner.- While there is no record to show that approval was obtained by Pepito with regard to his absences, the fact remains that he complied with the company rule that in case of illness necessitating absence of two days or more, the office should be informed beforehand about the same that is, on the first day of absence. Since the cause of his absence could not have been anticipated, to require prior approval would be unreasonable. On this score, then, no serious misconduct may be imputed to Pepito. Necessarily, his dismissal from work, tainted as it is by lack of just cause, was clearly illegal.2. NO- Petitioner's reliance on Pepito's past infractions as sufficient grounds for his eventual dismissal, in addition to his prolonged absences, is unavailing. The correct rule is that previous infractions may be used as justification for an employee's dismissal from work in connection with a subsequent similar offense.- In the present case, private respondent's absences, as already discussed, were incurred with due notice and compliance with company rules and fie had not thereby committed a "similar offense" as those lie had committed in the past. Furthermore, as

correctly observed by the labor arbiter, those past infractions had either been "satisfactorily explained, not proven, sufficiently penalized or condoned by the respondent." In fact, the termination notice furnished Pepito only indicated that he was being dismissed due to his absences from November 2. 1990 to December 10, 1990 supposedly without any acceptable excuse therefor. There was no allusion therein that his dismissal was due to his supposed unexplained absences on top of his past infractions of company rules. To refer to those earlier violations as added grounds for dismissing him is doubly unfair to private respondent. Significantly enough, no document or any other piece of evidence was adduced by petitioner showing previous absences of Pepito, whether with or without official leave.Disposition Petition dismissed

LA CARLOTA PLANTERS ASSN V NLRC (COMPACION)298 SCRA 252

VITUG; October 27, 1998

NATUREPetition for certiorari which seeks to set aside and nullify the decision of the NLRC promulgated on 25 September 1995 setting aside the LA’s decision and directing the respondent to pay complainant backwages and separation pay in lieu of reinstatement, computed at one (1) month per year of service.

FACTS - Compacion alleges that he was a regular employee of petitioner since 1988 hired as truck driver; that on December 14, 1992, at the instance of the petitioner, he drove the truck overloaded of sugarcane bound for La Carlota Sugar Central; that while driving through Sitio Bacus, Ma-ao, Bago City, the road was very slippery causing the truck to be outbalance (sic) resulting to the truck turning right side down; that he was not drunk when he drove the truck on December 14, 1992; that the Security Guards of Central La Carlota issued a clearance to the effect that he is cleared from whatever issues against him; that Rene Baylon reported the incident only on March 1993 when the incident happened on December 14, 1992 as shown by the Police Blotter; that because of his illegal dismissal, he sought the help of a legal counsel who helped him in filing this case for which he claims for payment of attorney's fees.- On the other hand, petitioner alleges that Compacion is a truck driver of Nature's Beauty Trucking Services; as such, he was assigned to Ma-ao Transloading Station, a loading station of sugarcanes bound for Central La Carlota located at Brgy. Ma-ao, Bago City, Negros Occidental; that on December 14, 1992, Compacion who was very drunk and with a knife entered the Ma-ao Transloading Station and harassed the office personnel to the extent of stabbing the person of Gerry Flores who fortunately was able to escape the said assault; that despite the repeated warning made by the Shifting In-charge Rene Baylon not to drive the truck, he drove the ten wheeler truck loaded with 18 tons of sugarcane bound for La Carlota Central in a reckless manner causing the truck to turn right side down resulting in a damage to property paid by the owner to Mr. Eulalio Pagunsan, owner of the bananas and pig pen hit and destroyed by the truck; that the said Mr. Eulalio Pagunsan observed that the driver Felix Compacion, at the time of the accident, was very drunk; that because of this accident which happened because of reckless driving, the truck underwent major repair; that after the accident, driver Felix Compacion was nowhere to be found, never reporting the accident to the police authorities or to the owner; that despite repeated calls, he refused to meet the owner nor did he report to the office thus prompting the latter to write him a letter dated January 4, 1993 suspending him for 30 days; further requiring him to report to the office and explain why he should not be terminated.- Petitioner further averred that during his employment, Compacion was paid wages and other benefits in accordance with law; that at the time of the accident, there was no rain and the road was not slippery; that at the time he stopped reporting, he has an outstanding account with respondent in the amount of P3,650.00; that prior to this accident on December 14, 1992, specifically on November 27, 1992, Felix Compacion was caught stealing diesel fuel from the drums owned by La Carlota Planters Association for which he was admonished and warned not to repeat the same.

ISSUEWON there was valid, legal and just cause for the dismissal of private respondent by petitioners

HELD NORatio The correct rule has always been that such previous offenses may be so used as valid justification for dismissal from work only if the infractions are related to the subsequent offense upon which basis the termination of employment is decreed. The previous infraction, in other words, may be used if it has a bearing to the proximate offense warranting dismissal. Reasoning - Petitioners contend that sufficient factual and legal bases exist to justify the dismissal of private respondent for misconduct. It cites various infractions allegedly committed in the past by private respondent; to wit:

Labor Law 1 A2010 - 236 - Disinia. Private respondent was caught twice stealing diesel fuel from the drum of the petitioner's association;b. He entered the transloading office on December 14, 1992 drunk, armed with bayonet knife, and harassed the personnel therein, even unsuccessfully stabbing one Gerry Flores for two (2) times; andc. Private respondent failed to report for work since December 14, 1992 which is an obvious sign of guilt.- The reliance by petitioners on the past offenses of private respondent supposedly dictating his eventual dismissal is unavailing. The complainant may have been at fault when he figured in a vehicular accident causing damage to the company truck; that fault, nevertheless, cannot be considered a just cause for dismissal. Indeed, it has once been held that the penalty of dismissal would be grossly disproportionate to the offense of driving through reckless imprudence resulting in damage to property. The claim of drunkenness on the part of private respondent has not been substantiated; the allegation is based solely on the uncorroborated statement made by one Rene Baylon in his affidavit executed on 24 April 1993, months after the accident had occurred in December of 1992. Disposition the Court is constrained to dismiss, as it hereby so DISMISSES, the instant petition for certiorari.

6. PROFESSIONAL TRAINING

RESIDENCY TRAINING

FELIX V BUENASEDA[PAGE 55]

7. LOVE AND MORALS

IMMORALITY

SANTOS V NLRC (HAGONOY INSTITUTE ET AL)287 SCRA 117

ROMERO; March 6, 1998

NATURE Petition for certiorari FACTS - Mrs. Martin and Petitioner Santos were both teachers at the Hagonoy Institute. Both were married to different people. During the course of their employment, they fell in love, and rumors about their relationship spread.- Private respondent advised Mrs. Martin to take a leave of absence, which she ignored. A week later, she was barred from reporting for work and was not allowed to enter Hagonoy’s premises, effectively dismissing her from employment - Mrs. Martin’s case for illegal dismissal was successful because the private respondent failed to accord her the necessary due process in her dismissal. - Meanwhile, HI set up a committee to investigate the veracity of the rumors. After 2 weeks, the committee confirmed the illicit relationship- in view of this finding, petitioner was charged administratively for immorality and was required to present his side of the controversy. 5 months later, he was informed of his dismissal. He thus filed a complaint for illegal dismissal.- After a full blown trial was conducted, the Labor Arbiter dismissed his complaint, but awarded him money as financial assistance.- petitioner filed an appeal with the NLRC which was dismissed for lack of merit

ISSUEWON the illicit relationship between the petitioner and Mrs. Martin could be considered immoral as to constitute just cause to terminate an employee under Article 282 of the Labor Code

HELD YESReasoning - Section 94 10 of the Manual of Regulations for Private Schools: Causes of Terminating Employment. In addition to the just cases enumerated in the Labor Code, the employment of school personnels, including faculty, may be terminated for any of the following causes:xxx xxx xxx E. Disgraceful or immoral conduct.

- To constitute immorality, the circumstances of each particular case must be holistically considered and evaluated in light of the prevailing norms of conduct and applicable laws. America jurisprudence has defined immorality as a course of conduct which offends the morals of the community and is a bad example to the youth whose ideals a teacher is supposed to foster and to elevate, the same including sexual misconduct. Thus, in petitioner's case, the gravity and seriousness of the charges against him stem from his being a married man and at the same time a teacher.- Having an extra-marital affair is an affront to the sanctity of marriage, which is a basic institution of society. Even our Family Code provides that husband and wife must live together, observe mutual love, respect and fidelity. This is rooted in the fact that both our Constitution and our laws cherish the validity of marriage and unity of the family. Our laws, in implementing this constitutional edict on marriage and the family underscore their permanence, inviolability and solidarity.

- As a teacher, petitioner serves as an example to his pupils, especially during their formative years and stands in loco parentis to them. To stress their importance in our society, teachers are given substitute and special parental authority under our laws.

- Teachers must adhere to the exacting standards of morality and decency. He must freely and willingly accept restrictions on his conduct that might be viewed irksome by ordinary citizens. The personal behavior of teachers, in and outside the classroom, must be beyond reproach.- Accordingly, teachers must abide by a standard of personal conduct which not only proscribes the commission of immoral acts, but also prohibits behavior creating a suspicion of immorality because of the harmful impression it might have on the students. - Likewise, they must observe a high standard of integrity and honesty.

- From the foregoing, it seems obvious that when a teacher engages in extra-marital relationship, especially when the parties are both married, such behavior amounts to immorality, justifying his termination from employment.

Disposition Petition DISMISSED

LOVE

CHUA-QUA V CLAVE189 SCRA 117

REGALADO; August 30, 1990

NATUREPetition for certiorari.

FACTS- This would have been just another illegal dismissal case were it not for the controversial and unique situation that the marriage of herein petitioner, then a classroom teacher, to her student who was fourteen (14) years her junior, was considered by the school authorities as sufficient basis for terminating her services.- Private respondent Tay Tung High School, Inc. is an educational institution in Bacolod City. Petitioner had been employed therein as a teacher since 1963 and, in 1976 when this dispute arose, was the class adviser in the sixth grade where one Bobby Qua was enrolled. Since it was the policy of the school to extend remedial instructions to its students, Bobby Qua was imparted such instructions in school by petitioner. In the course thereof, the couple fell in love and on December 24, 1975, they got married in a civil ceremony solemnized in lloilo City by Hon. Cornelio G. Lazaro, City Judge of Iloilo. - Petitioner was then thirty (30) years of age but Bobby Qua, being sixteen (16) years old, consent and advice to the marriage was given by his mother, Mrs. Concepcion Ong. Their marriage was ratified in accordance with the rites of their religion in a church wedding solemnized by Fr. Nick Melicor at Bacolod City on January 10, 1976. - On February 4, 1976, private respondent filed with the subregional office of the Department of Labor at Bacolod City an application for clearance to terminate the employment of petitioner on the following ground: "For abusive and unethical conduct unbecoming of a dignified school teacher and that her continued employment is inimical to the best interest, and would downgrade the high moral values, of the school." - Petitioner was placed under suspension without pay on March 12, 1976. - Executive Labor Arbiter rendered an "Award" in favor of private respondent granting the clearance to terminate the employment of petitioner. - NLRC unanimously reversed the Labor Arbiter's decision and ordered petitioner's reinstatement with backwages.- Minister of Labor reversed the decision of theNLRC. - Petitioner appealed the said decision to the Office of the President of the Philippines. Presidential Executive Assistant Jacobo C. Clave, rendered its decision reversing the appealed decision. - However, in a resolution dated December 6, 1978, public respondent, acting on a motion for reconsideration of herein private respondent and despite opposition thereto, reconsidered and modified the aforesaid decision, this time giving due course to the application of Tay Tung High School, Inc. to terminate the services of petitioner.

ISSUEWON there is substantial evidence to prove that the antecedent facts which culminated in the marriage between petitioner and her student constitute immorality and or grave misconduct

Labor Law 1 A2010 - 237 - DisiniHELDNORatio To constitute immorality, the circumstances of each particular case must be holistically considered and evaluated in the light of prevailing norms of conduct and the applicable law. Reasoning - Contrary to what petitioner had insisted on from the very start, what is before us is a factual question, the resolution of which is better left to the trier of facts.- Considering that there was no formal hearing conducted, we are constrained to review the factual conclusions arrived at by public respondent, and to nullify his decision through the extraordinary writ of certiorari if the same is tainted by absence or excess of jurisdiction or grave abuse of discretion. The findings of fact must be supported by substantial evidence; otherwise, this Court is not bound thereby. - We rule that public respondent acted with grave abuse of discretion. - As earlier stated, from the outset even the labor arbiter conceded that there was no direct evidence to show that immoral acts were committed. - Nonetheless, indulging in a patently unfair conjecture, he concluded that "it is however enough for a sane and credible mind to imagine and conclude what transpired during those times." In reversing his decision, the National Labor Relations Commission observed that the assertions of immoral acts or conducts are gratuitous and that there is no direct evidence to support such claim, a finding which herein public respondent himself shared.- What is revealing, however, is that the reversal of his original decision is inexplicably based on unsubstantiated surmises and non sequiturs which he incorporated in his assailed resolution in this wise:

". . . While admittedly, no one directly saw Evelyn Chua and Bobby Qua doing immoral acts inside the classroom, it seems obvious and this Office is convinced that such a happening indeed transpired within the solitude of the classroom after regular class hours. The marriage between Evelyn Chua and Bobby Qua is the best proof which confirms the suspicion that the two indulged in amorous relations in that place during those times of the day..."

- With the finding that there is no substantial evidence of the imputed immoral acts, it follows that the alleged violation of the Code of Ethics governing school teachers would have no basis. Private respondent utterly failed to show that petitioner took advantage of her position to court her student. If the two eventually fell in love, despite the disparity in their ages and academic levels, this only lends substance to the truism that the heart has reasons of its own which reason does not know. But, definitely, yielding to this gentle and universal emotion is not to be so casually equated with immorality. The deviation of the circumstances of their marriage from the usual societal pattern cannot be considered as a defiance of contemporary social mores.- It would seem quite obvious that the avowed policy of the school in rearing and educating children is being unnecessarily bannered to justify the dismissal of petitioner. This policy, however, is not at odds with and should not be capitalized on to defeat the security of tenure granted by the Constitution to labor. In termination cases, the burden of proving just and valid cause for dismissing an employee rests on the employer and his failure to do so would result in a finding that the dismissal is unjustified.Disposition Petition for certiorari granted. Decision of respondent annulled and set aside.

DUNCAN ASSOCIATION V GLAXO-WELLCOME[PAGE 43]

8. VIOLATION COMPANY RULES

APARENTE SR V NLRC (COCA-COLA BOTTLERS PHIL)

331 SCRA 82DE LEON JR; April 27, 2000

FACTS- Rolando Aparante, Sr. was first employed by private respondent Coca-Cola Bottlers Phils., Inc. (CCBPI), General Santos City Plant as assistant mechanic in April 1970. He rose through the ranks to eventually hold the position of advertising foreman until his termination on May 12, 1988 for alleged violation of company rules and regulations. His monthly salary at the time of his termination was P5,600.- On November 9, 1987, Aparante drove CCBPI's advertising truck to install a panel sign. He sideswiped Marilyn Tejero, a ten-year old girl. He brought Tejero to Heramil Clinic for first aid treatment. As the girl suffered a 2 cm fracture on her skull which was attributed to the protruding bolt on the truck's door, she was subsequently transferred to the General Santos City Doctor's Hospital where she underwent surgical operation. She stayed in the hospital for about a month.- Five days after the accident, he reported the incident to CCBPI. At about the same time, he submitted himself to the police authorities at Polomolok, South Cotabato for investigation where it was discovered that he had no driver's license at the time of the accident. In view thereof, FGU Insurance Corporation, an insurer of CCBPI's vehicles,

did not reimburse the latter for the expenses it incurred in connection with Tejero's hospitalization – a total amount of P19,534.45.- CCBPI conducted an investigation of the incident where Aparente was given the opportunity to explain his side and to defend himself.On May 12, 1988, Aparente was dismissed for having violated the company rules and regulations particularly Sec. 12 of Rule 005-858 for blatant disregard of established control procedures resulting in company damages.- The Labor Arbiter ordered his reinstatement without back wages. The NLRC affirmed but reversed its ruling upon motion of CCBPI. It declared the dismissal as one for just cause and effected after observance of due process.

ISSUES1. WON the NLRC erred in holding that CCBPI afforded petitioner due process2. WON the NLRC erred in upholding the dismissal despite its initial finding that the CCBPI had implicitly tolerated petitioner’s driving without a license3. WON the infraction committed by petitioner warrants the penalty of dismissal despite the fact that it was his first offense during his 18 long years of satisfactory and unblemished service

HELD1. NORatio The essence of due process does not necessarily mean or require a hearing but simply a reasonable opportunity or a right to be heard or as applied to administrative proceedings, an opportunity to explain one's side. In labor cases, the filing of position papers and supporting documents fulfill the requirements of due process.Reasoning - Aparente was fully aware that he was being investigated for his involvement in the vehicular accident that took place on November 9, 1987. It was also known to him that as a result of the accident, the victim suffered a 2 cm fracture on her skull which led to the latter's surgical operation and confinement in the hospital for which CCBPI incurred expenses amounting to P19,534.45 which FGU Insurance Corporation refused to reimburse upon finding that he was driving without a valid driver's license. Thus, being aware of all these circumstances and the imposable sanctions under CCBPI's Code of Disciplinary Rules and Regulations, he should have taken it upon himself to present evidence to lessen his culpability.2. NOReasoning - According to Aparente, he informed the company that he had lost his license five months before the accident. Notwithstanding such fact, the company allowed him to continue driving the vehicle assigned to him. Thus, he shifts the blame to the company, claiming that it should have simply ordered him to desist from driving the vehicle once it was informed of the loss of his license. His contention is belied by his very own admission in his position papers filed before the Labor Arbiter and the NLRC that the company had in fact prohibited him from driving immediately after he lost his license, and had requested him to secure a new license. However, through misrepresentations, he led CCBPI to believe that he had procured another driver's license. Thus, he was permitted to drive again.3. YESRatio The law warrants the dismissal of an employee without making any distinction between a first offender and a habitual delinquent where the totality of the evidence was sufficient to warrant his dismissal. In protecting the rights of the laborer, the law authorizes neither oppression nor self-destruction of the employer.Reasoning - Company policies and regulations, unless shown to be grossly oppressive or contrary to law, are generally valid and binding on the parties and must be complied with until finally revised or amended, unilaterally or preferably through negotiation, by competent authority. The Court has upheld a company's management prerogatives so long as they are exercised in good faith for the advancement of the employer's interest and not for the purpose of defeating or circumventing the rights of the employees under special laws or under valid agreements.- First, Aparente's dismissal is justified by Company rules and regulations. It is true that his violation of company rules is his first offense. Nonetheless, the damage caused to private respondent amounted to more than P5,000, thus, the penalty of discharge is properly imposable as provided by CCBPI's Code of Disciplinary Rules and Regulations.- Second, Article 282, in order that an employer may dismiss an employee on the ground of willful disobedience, there must be concurrence of at least two requisites: The employee's assailed conduct must have been willful or intentional, the willfulness being characterized by a wrongful and perverse attitude; and the order violated must have been reasonable, lawful, made known to the employee and must pertain to the duties which he had been engaged to discharge. We have found these requisites to be present in the case at bar. The evidence clearly reveals the willful act of Aparente in driving without a valid driver's license, a fact that he even tried to conceal during the investigation conducted by CCBPI. Such misconduct should not be rewarded with re-employment and back wages, for to do so would wreak havoc on the disciplinary rules that employees are required to observe.- In the instant case, we find the award to petitioner of separation pay by way of financial assistance equivalent to 1/2 month's pay for every year of service equitable. Although meriting termination of employment, petitioner's infraction is not as reprehensible or unscrupulous as to warrant complete disregard for the fact that this is his first offense in an employment that has spanned 18 long years.

Labor Law 1 A2010 - 238 - DisiniDisposition Decision of the NLRC is AFFIRMED.

9. CRIMINAL CASE

EFFECT OF ACQUITTAL

RAMOS V NLRC298 SCRA 225

PUNO; October 21, 1998

NATUREPetition for certiorari to annul NLRC decision

FACTS- In 1978, Elizabeth Ramos was employed by United States Embassy Filipino Employees Credit Cooperative (USECO)- In 1993, the USECO Board created an Audit and Inventory Committee to determine whether USECO has a sound financial management and control mechanism.- The committee found anomalies in USECO’s lending transactions. Petitioner and her co-employees, Luz Coronel and Nanette Legaspi, were called to shed light on some items in the Audit Committee Report, such as unrecorded loans, fabricated ledger, falsification of documents, accommodations of payroll checks, encashment of check/CPAs, resigned members, unrecorded loan of resigned members and withdrawal of more than the deposits.- During the meeting, Beth admitted her serious offense in regard to falsification of documents. When asked by the Board to explain how recently resigned members and other resigned employees in the past were able to secure loans, Beth replied that she ‘just wanted to help members without regard to existing policies.- In her written explanation, Beth said that the loans are approved based on prerogatives of individuals in authority. She said that, “it is unfortunate that the USECU Staff had to resort to creating dummy records. But since the loans are duly acknowledged by the borrowers in other legitimate documents, it is readily apparent that the records were made simply to accommodate those borrowers beyond the authorized limits, but never, never to defraud USECU.”- Ramos was preventively suspended for 30 days. Later, petitioner was placed on forced leave with pay, pending the completion of the investigation.- USECO commissioned an external auditing firm to examine the irregularities discovered in its lending practices. The auditor confirmed the irregularities and also discovered shortages in bank deposits.- USECO dismissed the petitioner for loss of trust and confidence. Petitioner countered with a complaint for illegal dismissal, illegal suspension, underpayment of salary, moral damages and attorney’s fees. - Labor Arbiter sustained the suspension and dismissal of petitioner but ordered the payment of her unpaid salary.

ISSUES1. WON there is just cause for petitioner’s suspension and dismissal2. WON the NLRC committed grave abuse of discretion in granting private respondent’s second motion for reconsideration

HELD1. YES- Position of petitioner as Management Assistant requires a high degree of trust and confidence. - Loss of confidence is a valid ground for dismissal of an employee. In the case at bar, USECO proved that its loss of confidence on petitioner has a rational basis. The findings of the labor arbiter on this factual issue are supported by the evidence.- Petitioner's explanation that the "loan practices" were made for the benefit of the borrowing members and not to defraud USECO cannot exonerate her. Her unsound practices endangered the financial condition of USECO because of the possibility that the loans could not be collected at all.- Petitioner was not denied due process before she was suspended and later dismissed. The records show that petitioner was called by the USECO Board of Directors and confronted with the findings of the Audit, and Inventory Committee showing the irregularities she committed. She was asked to explain in writing these irregularities. Petitioner submitted her written explanation. Thus, petitioner cannot complain that she did not understand the charges against her. She is educated and she immediately explained her side. Due process simply demands an opportunity to be heard and this opportunity was not denied her.2. NO- Section 14 of the Rules of the NLRC provides:

Section. 14. Motions for Reconsideration.--Motions for reconsideration of any order,

resolution or decision of the Commission shall not be entertained except when based on palpable or patent errors, provided that the motion is under oath and filed within ten (10) calendar days from receipt of the order, resolution or decision, with proof of service that a copy of the same has been furnished, within the reglementary period, the adverse party, and provided further that only one such motion from the same party shall be entertained.

- The NLRC initially reversed the ruling of the labor arbiter on the grounds that: (1) petitioner was denied procedural due process and (2) the criminal case for estafa filed against her has been dismissed by the Manila Prosecutor's Office for insufficiency of evidence, particularly, for lack of proof that the USECO was damaged by the acts attributed to petitioner.- As discussed above, petitioner was not denied due process. - Similarly, it is a well established rule that the dismissal of the criminal case against an employee shall not necessarily be a bar to his dismissal from employment on the ground of loss of trust and confidence. The NLRC corrected these patent errors when it granted private respondent's second motion for reconsideration. Disposition Petition dismissed for lack of merit.

CONVICTION

SAMPAGUITA GARMENTS CORP V NLRC (SANTOS)233 SCRA 260

CRUZ; June 17, 1994

NATUREPetition for review of a resolution of the NLRC

FACTS- Theft was claimed to have been done by Santos, employee of Sampaguita. It was alleged she attempted to bring out a piece of cloth w/o permission.- She was dismissed on this ground. She filed complaint for illegal dismissal. Labor Arbiter ruled in favor of Sampaguita. NLRC reversed and ordered reinstatement.- Sampaguita also filed criminal action against Santos. She was found guilty.- Santos moved for execution of NLRC decision. Sampaguita opposed and invoked her conviction in the criminal case.

ISSUEWON subsequent conviction in criminal prosecution for an offense will affect a previous administrative decision which absolved the employee of the same offense

HELDYES- Once judgment has become final and executory, it can no longer be disturbed except only for correction of clerical errors or where supervening events render its execution impossible or unjust. - Here, the decision of NLRC ordering reinstatement had become final and executory. Even so, we find that NLRC wasn’t correct in sustaining implementation - The affirmance by RTC and CA of private respondent's conviction is justification enough for NLRC to exercise this authority and suspend execution of its decision. Such conviction, also upheld by this Court is a supervening cause that rendered unjust and inequitable the decision mandating the private respondent's reinstatement.- Separation pay shall be allowed as a measure of social justice only in those instances where the employee is validly dismissed for causes other than serious misconduct or those reflecting on his moral character. A contrary rule would, as the petitioner correctly argues, have the effect of rewarding rather than punishing the erring employee for his offense. - The only award to which private respondent may be entitled is for the amount as a penalty for effecting her dismissal without complying with the procedural requirements.

DISMISSAL-CRIMINAL CASE

LACORTE V INCIONG (ESTRELLA, ASEAN FABRICATORS INC)

166 SCRA 1FERNAN; September 27, 1988

NATURECertiorari and Mandamus

FACTS- Salvador Lacorte was hired as a warehouseman whose duties were among others, to receive and store the raw and junk materials used by respondent in its business. - January 19, 1977: Lacorte offered to purchase some obsolete, defective and non-usable junk materials from AFI, who agreed and issued a cash invoice for the purchase of the scrap items.

Labor Law 1 A2010 - 239 - Disini- When Lacorte tried to bring out these items he was accosted by AFI' s security guard and in the course of the investigation, it was discovered that the items sought to be brought out by complainant weighed more than what he actually purchased. - Furthermore, it was found out that the items were not junk since some parts were brand new and usable. - As a consequence the respondent filed a case for qualified theft against complainant before the Provincial Fiscal of Bulacan. - The criminal complaint was however, dismissed for insufficiency of evidence. - The application of AFI to terminate LACORTE was granted as the latter was found by Labor Regional Director Estrella, to have committed certain acts in breach of the trust and confidence of his employer. - On appeal, Deputy Minister of Labor Amado Gat Inciong affirmed the aforementioned order. Hence, this present recourse.

ISSUEWON public respondents acted arbitrarily and/or with grave abuse of discretion (considering that the criminal complaint was dismissed) connection with the grant of the application for clearance to terminate the employment of petitioner filed by AFI

HELDYES- The purpose of the proceedings before the fiscal is to determine if there is sufficient evidence to warrant the prosecution and conviction of the accused. In assessing the evidence before him, the fiscal considers the basic rule that to successfully convict the accused the evidence must be beyond reasonable doubt and not merely substantial. - On the other hand, to support findings and conclusion of administrative bodies only substantial evidence is required.- The evidence presented before the two bodies may not be necessarily Identical. - The appreciation of the facts and evidence presented is an exercise of discretion on the part of administrative officials over which one cannot impose his conclusion on the other. - Sea-Land Service, Inc. v. NLRC: “The conviction of an employee in a criminal case is not indispensable to warrant his dismissal, and the fact that a criminal complaint against the employee has been dropped by the fiscal is not binding and conclusive upon a labor tribunal. - Also, the Court did not believe Lacorte’s claim that the real reason behind his termination was his union activities.- As regards Lacorte’s claim that there was no actual weighing and examination of the boxes containing the scrap materials he allegedly stole, the Court ruled that it was too late in the day for Lacorte to raise these matters of facts in this petition and that his evidence does not substantiate his claim. - The Court considered the records of this case as a whole, and was convinced that there is substantial basis for the Orders issued by respondent labor officials. Disposition Petition is dismissed for lack of merit.

GUILT OR INNOCENCE

CHUA V NLRC218 SCRA 545

FELICIANO; February 8, 1993

NATUREPetition for certiorari

FACTS- The Union of Filipro Employees, of which petitioner Chua was a member, declared a strike against the private respondent company, Nestle Philippines, Inc. During the strike, several of the striking employees threw stones at the trucks entering and leaving the company premises. One truck. whose driver was rendered unconscious by a stone hitting him on the head, rammed a private vehicle and crashed into a beauty parlor resulting in the death of three persons and extensive damage to private property. Consequently, a criminal complaint for multiple murder and frustrated murder was filed against petitioner and several other employees who were believed to be responsible for the stoning incident which resulted in the deaths and property damage. The criminal complaint was dismissed for insufficiency of evidence. The strike itself was, however, declared illegal in two decisions of the National Labor Relations Commission (NLRC) which were affirmed by the Supreme Court. - Subsequently, the union and its striking members offered to return to work and were readmitted by the company except 69 union officers and 33 union members, including petitioner. The union's counsel wrote to the private respondent requesting the reinstatement of five employees, including petitioner. The request, however, was denied. Petitioner received a notice of dismissal from private respondent for having participated in the illegal strike.- Two days later, petitioner initiated a complaint for illegal dismissal against private respondent company. The Labor Arbiter rendered a decision finding that petitioner had been validly dismissed. It was held that the evidence introduced by private respondent, in the form of the testimony of Maniego, Personnel Supervisor of its Cabuyao Plant, that he positively saw and identified petitioner as one of the union members who actively participated and manned the barricades during the strike is "a concrete manifestation of

an illegal act that is frowned upon by law." Wishing to be reinstated also, petitioner appealed the Labor Arbiter's decision to the NLRC which, however, affirmed in toto the decision of the Labor Arbiter. Hence, this petition.

ISSUEWON the NLRC committed grave abuse of discretion in affirming the decision of the Labor Arbiter

HELDNO- We find this contention to be without merit, Petitioner's participation in the illegal strike and his commission of illegal acts while the strike was in progress, i.e., he participated in the barricade which barred people from entering and/or leaving the employer's premises, had been sufficiently established by substantial evidence, including the testimony of Mr. Maniego, Personnel Supervisor at the Cabuyao Plant. Mr. Maniego testified, among other things, that he was not able to report to work because of the presence of the barricade. The law prohibits any person engaged in picketing from obstructing free ingress to or egress from the employer's premises for lawful purposes. - While the criminal complaint where petitioner was included as one of the accused was dismissed for insufficiency of evidence, the Court considers that the dismissal of the criminal complaint did not preclude a finding by the competent administrative authorities, that petitioner had indeed committed acts inimical to the interest of his employer.- Private respondent's guilt or innocence in the criminal case is not determinative of the existence of a just or authorized cause for his dismissal. This doctrine follows from the principle that the quantum and weight of evidence necessary to sustain conviction in criminal cases are quite different from the quantum of evidence necessary for affirmance of a decision of the Labor Arbiter and of the NLRC.- Since petitioner's participation in the unlawful and violent strike was amply shown by substantial evidence, the NLRC was correct in holding that the dismissal of petitioner was valid being based on lawful or authorized cause.Disposition Petition dismissed.

10. MOONLIGHTING

AGABON V NLRC[PAGE 35]

11. SUSPICION

EASTERN TELECOMMUNICATIONS PHILS INC V DIAMSE

491 SCRA 239YNAREZ-SANTIAGO; June 16, 2006

FACTS - Maria Charina Damse is the Head of Building Services of ETPI. She requested a cash advance of P150k for the renewal of ETPI’s business permits. The company’s policy is cash advances should be liquidated 15 days from the completion of the project or activity, or else it will be deducted from the employee’s salary, benefits or receivables. - She was able to use a total of P97,151. The last payment was made on Feb 26, 2001. She wasn’t able to liquidate the cash advance within 15 days. - On July 13, 2001, ETPI’s Finance Dept advised her to liquidate the amount. She submitted a liquidation report on August 13, 2001. This report was refused by the Fin Dept for being late. She was told that the entire amount would just be deducted from her monthly salary starting Sept 2001. By Dec 2001, a total of P23k had been deducted from her salary. She then requested for reimbursement for P97,151. This was reviewed by her supervisor and approved by HR and Fin Dept, and the amt was credited to her ATM payroll acct. - The Internal Audit Dept (IAD) apparently didn’t know what was going on. In Jan 2002, IAD found that her payroll acct had P86k. They required her to withdraw P52,533 for the unliquidated amt minus the deductions. She complied. The next day, they asked her again for P74,462.82, which is the difference bet the P97k++ credited to her acct minus the P23k deductions. She complied again. (I don’t understand how the computations were made.)- The next day, ETPI required Diamse to explain why she should not be disciplined for unauthorized diversion or application of company funds, and for acts of dishonesty, fraud, deceit and willful breach of trust. She explained what that the liquidation report wasn’t accepted by the Fin Dept and she was instead advised to do as she did. A month later, she was dismissed. - LA ruled in her favor. NLRC reversed. CA reversed NLRC and ordered separation pay etc instead of reinstatement because of the strained relations bet the parties.

ISSUEWON Diamse was illegally terminated

HELD

Labor Law 1 A2010 - 240 - DisiniNO- Employer wasn’t able to prove that the employee was terminated for valid and just cause. LOSS OF TRUST AND CONFIDENCE v. SUSPICION- To be a valid cause for dismissal, the loss of trust and confidence must be based on a willful breach and founded on clearly established facts. A breach is willful if it is done intentionally, knowingly and purposely, without justifiable excuse, as distinguished from an act done carelessly, thoughtlessly, heedlessly or inadvertently. Loss of trust and confidence must rest on substantial grounds and not on the employer's arbitrariness, whims, caprices or suspicion, otherwise, the employee would eternally remain at the mercy of the employer.- The SC held that the mere delay in the liquidation of the cash advance cannot sustain a finding of loss of trust and confidence. It was based on mere suspicion, without evidence to show that Diamse misappropriated funds. In fact, all documents submitted were found to be authentic. The evidence on record shows that Diamse was able to liquidate the cash advance and that the ensuing delay in its liquidation was attributable to ETPI.- It cannot be presumed that Diamse misappropriated the funds because to do so would do violence to her right to security of tenure and the well-settled rule that the burden of proof is on the employer to establish the ground for dismissal. Suspicion has never been a valid ground for dismissal and the employee's fate cannot, in justice, be hinged upon conjectures and surmises.- More suspicion with regard to the P86k in her ATM acct: The company suspected that it came from the P97k erroneously credited to her acct. They didn’t bother to prove it. They weren’t able to show any bank statements to that effect. Disposition Petition denied. CA decision affirmed and modified in that this case be REMANDED to the Labor Arbiter for the sole purpose of computing Diamse's full backwages, etc.

14.06 TRANSFERS – DISCHARGE AND SUSPENSION

LANZADERAS V AMETHYST SECURITY AND GENERAL SERVICES INC

404 SCRA 505QUISUMBING; June 20, 2003

FACTS - Amethyst has been supplying guard for Resin Industrial Chemical Corp (RICC) and its sister company Phil. Iron Construction and Marine Works (PICMW) since 1968. One condition was that Amethyst must supply guards between 25 – 45 years old. - In 1998, RICC/PICMW reminded Amethyst of this condition. Amethyst in turn required of the guards assigned to RICC/PICMW to submit copies of their birth certificates. Those beyond the limit were told to report to the office for reassignment.- Amethyst was able to renegotiate with RICC/PICMW to the effect that those beyond the age limit could be assigned as firewatch guards in the same company. (SO they had a choice of being assigned as firewatch guards in the same company or be transferred to Cagayan de Oro.)- The petitioners chose neither option and didn’t report for work. They filed illegal dismissal with the LA.- LA held Amethyst and RICC/PICMW solidarily liable for P1.25M to the petitioners. On appeal, NLRC reversed and set aside the LA’s decision on the ground that the relief of petitioners from their posts was a legitimate exercise of prerogative on RICC/PICMW’s part. - CA denied petitioners’ appeal on procedural grounds.

ISSUEWON petitioners were illegally dismissed

HELDNO- In the first place, the petitioners knew of the age limit and acted in bad faith when they weren’t honest about their ages.- The condition imposed by respondent RICC/PICMW, as a principal or client of the contractor Amethyst, regarding the age requirement of the security guards to be designated in its compound, is a valid contractual stipulation. It is an inherent right of RICC/PICMW, as the principal or client, to specify the qualifications of the guards who shall render service pursuant to a service contract. It stands to reason that in a service contract, the client may require from the service contractor that the personnel assigned to the client should meet certain standards and possess certain qualifications, conformably to the client's needs.- Security of tenure, although provided in the Constitution, does not give an employee an absolute vested right in a position as would deprive the company of its prerogative to change their assignment or transfer them where they will be most useful. When a

transfer is not unreasonable, nor inconvenient, nor prejudicial to an employee; and it does not involve a demotion in rank or diminution of his pay, benefits, and other privileges, the employee may not complain that it amounts to a constructive dismissal. - Case law recognizes the employer's right to transfer or assign employees from one area of operation to another, or one office to another or in pursuit of its legitimate business interest, provided there is no demotion in rank or diminution of salary, benefits and other privileges and not motivated by discrimination or made in bad faith, or effected as a form of punishment or demotion without sufficient cause. This matter is a prerogative inherent in the employer's right to effectively control and manage the enterprise.- The petitioners were given an option to stay at RICC/PICMW as firewatch guards or to be transferred to CDO as security guards. The petitioners didn’t report to the office to receive new deployment instructions. They have no excuse not to heed management’s exercise of management prerogative.Disposition Petition denied. CA affirmed. Note The SC also denied on procedural grounds but went into the issues to settle the matter completely.

WESTIN PHIL PLAZA HOTEL V NLRC (RODRIGUEZ)306 SCRA 631

QUISUMBING; May 3, 1999

NATUREPetition to review a decision of the NLRC

FACTS- Private respondent Len Rodriguez was continuously employed by petitioner in various capacities (pest controller, room attendant, bellman, and doorman) from July 1, 1977 until his dismissal on February 16, 1993.- On December 28, 1992, private respondent received a memorandum from the management transferring him from doorman to linen room attendant. The position of doorman is categorized as guest-contact position while linen room attendant is a non-guest contact position. - The transfer was allegedly taken because of the negative feedback on the manner of providing service to hotel guests by private respondent. - Instead of accepting his new assignment, private respondent went on vacation leave- The President of the National Union of Workers in Hotels, Restaurants and Allied Industries (NUWHRAIN) appealed to management concerning private respondent's transfer, but the director for human resources development, clarified that private respondent's transfer is merely a lateral movement. She explained that management believed that private respondent was no longer suited to be in a guest-contact position, but there was no demotion in rank or pay.- When private respondent reported back to work, he still did not assume his post at the linen room. - On February 11, 1993, private respondent was served with a memorandum asking him to explain in writing why no disciplinary action should be taken against him for insubordination. The memorandum noted that while private respondent regularly came to the hotel everyday, he just stayed at the union office. - In his reply private respondent merely questioned the validity of his transfer without giving the required explanation.- On February 16, 1993, petitioner terminated private respondent's employment on the ground of insubordination.- Private respondent filed with the Department of Labor and Employment a complaint for illegal dismissal against petitioner. - The labor arbiter declared that the dismissal was legal. Accordingly, the complaint was dismissed for lack of merit.- On appeal, public respondent reversed the judgment of the labor arbiter. It held that there was no just cause in dismissing private respondent.- Its motion for reconsideration having been denied, petitioner filed this instant petition.ISSUES1. WON private respondent was guilty of insubordination, thus giving petitioner just and valid cause for dismissal 2. WON the order of transfer was legal

HELD1. YES- Under Article 282 (a) of the Labor Code, as amended, an employer may terminate an employment for serious misconduct or willful disobedience by the employee of the lawful orders of his employer or representative in connection with his work. But disobedience to be a just cause for dismissal envisages the concurrence of at least two (2) requisites: (a) the employee's assailed conduct must have been willful or intentional, the willfulness being characterized by a wrongful and perverse attitude; and, (b) the order violated must have been reasonable, lawful, made known to the employee and must pertain to the duties which he has been engaged to discharge. - In the present case, the willfulness of private respondent's insubordination was shown by his continued refusal to report to his new work assignment: 1. Upon receipt of the order of transfer, private respondent simply took an extended vacation leave; 2. When he reported back to work, he did not discharge his duties as linen room attendant—while he came to the hotel everyday, he just went to the union office; 3. when he was asked to

Labor Law 1 A2010 - 241 - Disiniexplain why no disciplinary action should be taken against him, private respondent merely questioned the transfer order without submitting the required explanation.2. YES- It must be emphasized that this Court has recognized and upheld the prerogative of management to transfer an employee from one office to another within the business establishment, provided that there is no demotion in rank or a diminution of his salary, benefits and other privileges. - This is a privilege inherent in the employer's right to control and manage its enterprise effectively. - Besides, it is the employer's prerogative, based on its assessment and perception of its employee's qualifications, aptitudes and competence, to move him around in the various areas of its business operations in order to ascertain where the employee will function with utmost efficiency and maximum productivity or benefit to the company. - An employee's right to security of tenure does not give him such a vested right in his position as would deprive the company of its prerogative to change his assignment or transfer him where he will be most useful. - Petitioner is justified in reassigning private respondent to the linen room. Petitioner's right to transfer is expressly recognized in the collective bargaining agreement between the hotel management and the employees union as well as in the hotel employees handbook. The transfer order was issued in the exercise of petitioner's management prerogative in view of the several negative reports vis-a-vis the performance of private respondent as doorman. It was a lateral movement as the positions of doorman and linen room attendant are equivalent in rank and compensation. It was a reasonable relocation from a guest contact area to a non-guest contact area.Disposition Petition granted. NLRC decision reversed.

CASTILLO V NLRC (PCIB)308 SCRA 326

GONZAGA-REYES; June 1999

NATUREPetition for certiorari seeking to annul the NLRC Decision

FACTS- Castillo was an employee of Philippine Commercial & International Bank (PCIB) as Foreign Remittance Clerk. A Jordanian national, went to PCIB’s Ermita branch to claim a foreign remittance in the amount of US$2T. He paid P450 as commission charges as computed by petitioner. Upon re-computation, the correct amount of the charges amounted to only P248.75.- Because of this incident, Castillo received a Memorandum regarding her REASSIGNMENT. “In line with the Bank’s policy on flexibility employee development and internal control, effective immediately, you are hereby reassigned temporarily as Remittance Clerk-Inquiry.”- She then filed with the NCR Arbitration Branch a complaint-affidavit for illegal dismissal asking for her reinstatement as Foreign Remittance Clerk plus moral and exemplary damages.- She received another memorandum: “Relative to your reassignment as Remittance Clerk-Inquiry, for internal control purposes, you are hereby instructed that your specific duties and responsibilities will be confined to handling of inquiring by phone, by walk-in clients over the counter and to assist the FX Supervisor-Inquiry & Investigation in verifying inquiries of correspondent banks, agencies, other banks and branches.”- Castillo claimed that there was no legal basis for her transfer and demotion order. Also, PCIB immediately appointed another employee in her place and refused to allow petitioner to perform her usual functions as she became a mere fixture in the office premises to her gross humiliation. She was allegedly barred from the office premises and was thereby constructively dismissed without any legal ground and without due process.- Labor Arbiter ruled that Castillo was constructively dismissed , thus she was entitled to reinstatement with full backwages without loss of seniority rights, privileges and other rights granted by law.- NLRC reversed LA: there was no demotion because the position to which she was being reassigned belongs to the same job level as her former position and both positions have the same rate of compensation.

ISSUEWON Castillo was constructively and illegally dismissed

HELDNORatio The Court, as a rule, will not interfere with an employer’s prerogative to regulate all aspects of employment which includes among others, work assignment, working methods, and place and manner of work. It is the prerogative of the employer to transfer and reassign employees for valid reasons and according to the requirement of its business, provided that the transfer is not unreasonable, inconvenient, or prejudicial to the employee, and that there is no demotion in rank or a diminution of his salary, benefits and other privileges. An employee’s right to security of tenure does not give him such a vested right in his position as would deprive the company of its prerogative to change his assignment or transfer him where he will be most useful.- Constructive dismissal: The employer has the burden of proving that the transfer and

demotion of an employee are for valid and legitimate grounds. Where the employer fails to overcome this burden of proof, the employee’s demotion shall no doubt be tantamount to unlawful constructive dismissal.Reasoning - PCIB was acting within its management prerogative to protect its interest and that of its clients. NLRC upheld PCIB’s contention that the remittance clerk payment order/collection item is given the same weight in terms of duties and responsibilities as that of a remittance clerk inquiry. These positions are of “co-equal footing, co-important and of the same level of authority” and that the transfer did not entail any reduction of wages and other benefits. This is because both positions are in fact Remittance Clerks, which, in PCIB’s classification system, are both slotted at level S-III.- It is not true that Castillo has become a mere fixture in the office premises without any function and was given no responsibilities. As a matter of act, had she accepted her new position, she would have assumed a bigger responsibility, a big departure from her former position where she merely did routine processing work. Disposition Petition dismissed.

OSS SECURITY & ALLIED SERVICES INC V NLRC (LEGASPI)

325 SCRA 157DE LEON JR; February 9, 2000

NATUREPetition for certiorarii

FACTS- Private respondent Eden Legaspi worked as a security guard of OSS Security Agency from June 16, 1986. On January 17, 1986 petitioner Miguel and Victoria Vasquez acquired the assets and properties of OSS and absorbed some of its personnel, including Legaspi, who was assigned to render security services to the different clients of petitioner. She was last assigned at the Vicente Madrigal Condominium II located in Ayala Avenue, Makati. In a memorandum, to petitioner, the Building Administrator of VM Condominium II, complained of the laxity of the guards in enforcing security measures and requested that petitioner reorganize the men and women assigned to the building to instill more discipline and proper decorum by changing, if need be, some of the personnel, replacing, if possible, on a temporary basis, the women complement, to find out if it would improve the service.- In compliance therewith, petitioner issued Duty Detail Order, relieving Legaspi and another lady security guard of their assignment at VM, for reassignment to other units or detachments where vacancy exists. Thereafter, petitioner detailed Legaspi to the Minami International Corporation in Taytay, Rizal for 1 month to replace lady security guard who was on leave. However, Legaspi did not report for duty at her new assignment.- Legaspi filed her complaint for under payment and constructive dismissal. The Labor Arbiter upheld Legaspi’s position and ordered OSS to reinstate complainant to her former position and pay the latter backwages for 18 months. Upon appeal, the NLRC affirmed said decision. Hence, this petition.

ISSUEWON the transfer of Legaspi was illegal and tantamount to unjust dismissal

HELDNO- Service-oriented enterprises, such as petitioner's business of providing security services, generally adhere to the business adage that "the customer or client is always right". To satisfy the interests, conform to the needs, and cater to the whims and wishes of its clients, along with its zeal to gain substantial returns on its investments, employers adopt means designed towards these ends. These are called management prerogatives in which the free will of management to conduct its own affairs to achieve its purpose, takes from. Accordingly, an employer can regulate, generally without restraint, according to its own discretion and judgment, every aspect of business.- In the employment of personnel, the employer can prescribe the hiring, work assignments, working methods, time, place and manner of work, tools to be used, processes to be followed, supervision of workers, working regulations, transfer of employees, work supervision, lay-off of workers and the discipline, dismissal and recall of work, subject only to limitations imposed by laws.- Thus, the transfer of an employee ordinarily lies within the ambit of management prerogatives. However, a transfer amounts to constructive dismissal when the transfer is unreasonable, inconvenient, or prejudicial to the employee, and it involves a demotion in rank or diminution of salaries, benefits and other privileges. In the case at bench, nowhere in the record does it show that that the transfer of Legaspi was anything but done in good faith, without grave abuse of discretion, and in the best interest of the business enterprise.- No malice should be imputed from the fact that Legaspi was relieved of her assignment and, a day later, assigned a new post. We must bear in mind that, unlike other contracts of service, the availability of assignment for security guards is primarily at heart subservient to the contracts entered into by the security agency with its client-third parties. As such, being sidelined temporarily is a standard stipulation in employment

Labor Law 1 A2010 - 242 - Disinicontracts. When a security guard is placed "off detail" or on "floating" status, in security agency parlance, it means "waiting to be posted." Legaspi has not even been "off detail" for a week when she filed her complaint.- Evidence is wanting to support the Labor Arbiter's conclusion that petitioner discriminated against private respondent when it ordered her relief and transfer of assignment. Petitioner proved that such transfer was effected in good faith to comply with the reasonable request of its client, Madrigal Condominium Corporation Incorporated (MCCI), for a more disciplined service of the security guards on detail. The renewal of the contract of petitioner with MCCI hinged on the action taken by the former on the latter's request. Most contracts for security services stipulate that the client may request the replacement of the guards assigned to it. Besides, a relief and transfer order in itself does not sever employment relationship between a security guard and her agency.29

Neither was the transfer for any ulterior design, such as to rid itself of an undesirable worker or to penalize an employee for his union activities and thereby defeat his right to self-organization.- It appears that Legaspi declined the post assigned to her inasmuch as she considered it "a booby trap of crippling and dislocating her from her employment". She lived in V. Mapa, Sta. Mesa, Manila, and her new assigned post is in Taytay, Rizal, as against her previous post at VM Condominium II in Makati. Her new assigned post would entail changes in her routine, something that she was not agreeable with. But the mere fact that it would be inconvenient for her, as she has been assigned to VM Condominium II for a number of years, does not by itself make her transfer illegal. Even Legaspi admitted that she was assigned to render security service to the different clients of petitioner. An employee has a right to security of tenure, but this does not give her such a vested right in her position as would deprive petitioner of its prerogative to change her assignment or transfer her where her service, as security guard, will be most beneficial to the client. Thus, there was no basis to order reinstatement and back wages inasmuch as she was not constructively dismissed. Neither is private respondent entitled to the award of money claims for underpayment, absent evidence to substantiate the same.

MENDOZA V RURAL BANK OF LUCBAN433 SCRA 756

PANGANIBAN; July 7, 2004

NATUREPetition for Review under Rule 45, ROC

FACTS- April 1999, the Board of Directors of respondent bank (BANK) issued a Board Resolution announcing the reshuffling of assignments, without changes in compensation and other benefits, of several officers and employees of the bank (in line with the bank’s policy to familiarize bank employees with the various phases of bank operations and to further strengthen the existing internal control system). One of the employees assigned to a new position is petitioner Elmer Mendoza, who was transferred from his post as an appraiser to a Clerk for Meralco collection.- In May, Mendoza expressed his resentment on the reshuffling, saying that it was a demotion and that he heard intrigues that his “demotion” to a Clerk-Meralco collection was due to the malicious machination of a certain public official who is the friend of the Board chairman and with whom the relatives of Mendoza had filed a falsification case against. He also said that he had been working for 6 years in good standing in the bank and that the reshuffling is a blatant harassment on the part of the Board, an act which implicitly forces him to resign, and which constitutes an unfair labor practice. He requested to have his position as an appraiser retained. However, Mr. Daya (Board Chairman) explained in a reply that the reshuffling is not a demotion since his compensation as an appraiser is retained and no reductions were made. Also, Mr. Daya explained the objectives of the reshuffling, particularly the maintenance of an effective internal control system recommended by Bangko Sentral ng Pilipinas, and that it was the bank’s management prerogative to do so. Also, Mendoza could retain position upon a formal request to the board.- In June, petitioner submitted 2 applications for LOA, and during his 2nd LOA he filed a complaint before the Arbitration Branch of NLRC for illegal dismissal, underpayment, separation pay and damages. - LA: in favor of Mendoza (entitled to reinstatement + full backwages/ separation pay if reinstatement not possible…moral + exemplary + atty’s fees)- NLRC: in favor of Bank: no bad faith or malice on bank’s part; petitioner only feel inconvenienced; petitioner not only employee reshuffled; no clear, competent, convincing evidence that he holds a vested right to the position of Appraiser.- CA: affirm (Mendoza’s claims self-serving, no diminution, could retain title upon formal request, no bad faith/malice; no constructive dismissal – he was the one who separated himself from the bank’s employ)

ISSUES1. WON Mendoza was constructively dismissed2. WON the transfer of employees/ reshuffling was a valid exercise of the bank’s management prerogatives(Secondary Issues)3. WON Serrano v. NLRC is applicable4. WON NLRC and CA proceedings null

HELD1. NO- Findings of NLRC and CA were supported by substantial evidenceRatio Constructive dismissal is defined as an involuntary resignation resorted to when continued employment is rendered impossible, unreasonable or unlikely; when there is a demotion in rank or a diminution of pay; or when a clear discrimination, insensibility or disdain by an employer becomes unbearable to the employee.Reasoning- Petitioner presented no evidence to support his claims. (More on the second issue)2. YESRatio In the pursuit of its legitimate business interest, management has the prerogative to transfer or assign employees from one office or area of operation to another -- provided there is no demotion in rank or diminution of salary, benefits, and other privileges; and the action is not motivated by discrimination, made in bad faith, or effected as a form of punishment or demotion without sufficient cause. This privilege is inherent in the right of employers to control and manage their enterprise effectively. The right of employees to security of tenure does not give them vested rights to their positions to the extent of depriving management of its prerogative to change their assignments or to transfer them.-Managerial prerogatives, however, are subject to limitations provided by law, collective bargaining agreements, and general principles of fair play and justice.-TEST of validity of transfer of employees (Blue Dairy Corporation v. NLRC): "The managerial prerogative to transfer personnel must be exercised without grave abuse of discretion, bearing in mind the basic elements of justice and fair play. Having the right should not be confused with the manner in which that right is exercised. Thus, it cannot be used as a subterfuge by the employer to rid himself of an undesirable worker. In particular, the employer must be able to show that the transfer is not unreasonable, inconvenient or prejudicial to the employee; nor does it involve a demotion in rank or a diminution of his salaries, privileges and other benefits. Should the employer fail to overcome this burden of proof, the employee’s transfer shall be tantamount to constructive dismissal, which has been defined as a quitting because continued employment is rendered impossible, unreasonable or unlikely; as an offer involving a demotion in rank and diminution in pay. Likewise, constructive dismissal exists when an act of clear discrimination, insensibility or disdain by an employer has become so unbearable to the employee leaving him with no option but to forego with his continued employment."- Employees may be transferred – based on their qualifications, aptitudes and competencies – to positions in which they can function with maximum benefit to the company.Reasoning- Mendoza’s transfer complied with the test. Transfer made in pursuit of valid objectives (see above, 1st paragraph – inside parenthesis); Mendoza was not singled out; no diminution of salary, privileges, and other benefits.3. NO- No constructive dismissal, not entitled to monetary benefits as awarded in the Serrano case.4. NO- Petitioner’s arguments regarding the Bank’s appeal before the NLRC filed beyond the reglementary period was not raised in CA, thus cannot be entertained if raised for the 1st

time.Disposition Petition is DENIED, and the June 14, 2002 Decision and the September 25, 2002 Resolution of the Court of Appeals are AFFIRMED.

12. RESIGNATION AND EFFECTIVITY

EMCO PLYWOOD CORP V ABELGAS[PAGE 14]

SHIE JIE CORP/SEASTER EX-IM CORP V NATIONAL FEDERATION OF LABOR

463 SCRA 569SANDOVAL-GUTIERREZ; July 15, 2005

FACTS- Respondents were employed by petitioner as fish processors. Respondents staged a walk-out and abandoned their work, bringing operations to a standstill. They were suspended for a week. Petitioner claims that instead of coming to work, some of the respondents submitted resignation letters and quitclaims. Petitioner then sent the rest a notice terminating their services for abandonment of work.- the Labor Arbiter found petitioners guilty of unfair labor practice for illegally dismissing respondents and awarding the latter claims. On appeal, the NLRC reversed the decision. The CA later reversed the decision again based on Article 277 which requires that the employer prove that the termination was for a valid or just cause. Hence this petition.

ISSUEWON respondents made valid resignations and were thus not illegally dismissed

Labor Law 1 A2010 - 243 - DisiniHELDNO- Voluntary resignation is defined as the act of an employee, who finds himself in a situation in which he believes that personal reasons cannot be sacrificed in favor of the exigency of the service; thus, he has no other choice but to disassociate himself from his employment. Acceptance of a resignation tendered by an employee is necessary to make the resignation effective, which was not shown in the instant case. - To constitute a resignation, it must be unconditional and with the intent to operate as such. There must be an intention to relinquish a portion of the term of office accompanied by an act of relinquishment.- It is illogical that respondents would file complaints of illegal dismissal 17 days after filing their resignation letters. Such acts negate any intention on their part to relinquish their jobs. It was held in Molave Tours Corp. vs NLRC, “By vigorously pursuing the litigation of his action against petitioner, private respondent clearly manifested that he has no intention of relinquishing his employment, which act is wholly incompatible to petitioner’s assertion that he voluntarily resigned.”Disposition WHEREFORE the petition is DENIED

13. ABOLITION OF POSITION

BENGUET ELECTRIC COOPERATIVE V FIANZA425 SCRA 41

YNARES-SANTIAGO; March 9, 2004

NATUREReview on certiorari

FACTS- Josephine Fianza had been employed with petitioner Benguet Electric Cooperative (BENECO) as Property Custodian under the Office of the General Manager.- BENECO’s General Manager, Versoza, issued Office Order No. 42 addressed to Fianza communicating that she is temporarily detailed to the Finance Department to assume the duties of a Bill Distributor without any change in salary rate. This is line with their efforts to reduce the cost of operation.- Fianza acknowledged receipt of the letter under protest. She avers that it amounts to a demotion because there are significant differences in the educational qualifications, work experience, skills and job description and the working conditions of a Bill Distributor are totally different and more strenuous and expose her to unfavourable and dangerous circumstances, and therefore not similarly situated as that of Property Custodian.- In response, Versoza issued a Memorandum informing her that the position of Property Custodian may eventually phased out upon approval of the already proposed Table of Organization as part of business decision.- Still, Fianza refused to heed the order of the General Manager and continued to work as Property Custodian despite successive issuance of Memorandum until the management no longer authorized her to perform the duties and functions of a Property Custodian.

ISSUES1. WON Fianza’s transfer from Property Custodian to Bill Distributor is valid2. WON the position of Property Custodian is abolished and WON the abolition is valid

HELD1. YES Ratio The management has a wide latitude to regulate, according to his own discretion and judgment, all aspects of employment, including the freedom to transfer and reassign employees according to the requirements of its business. However, the transfer of an employee may constitute constructive dismissal when it amounts to “an involuntary resignation resorted to when continued employment is rendered impossible, unreasonable or unlikely; when there is a demotion in rank and/or a diminution in pay, or when a clear discrimination, insensibility or disdain by an employer becomes unbearable to the employee.2. YES Ratio The abolition of a position deemed no longer necessary is a management prerogative, and this Court, absent any findings of malice and arbitrariness on the part of management, will not efface such privilege if only to protect the person holding that office.Reasoning- There was no showing that the position of Property Custodian was abolished in order to single out Fianza, or that malice and ill-will attended the phasing out of the position.

14. DISHONESTY

NAGUIT V NLRC (MANILA ELECTRIC)408 SCRA 617

CARPIO-MORALES; August 12, 2003

NATURE Petition for certiorari seeking to annul and set aside the decision and resolution of the NLRC.

FACTS - Petitioner Aniceto W. Naguit, Jr., an employee of respondent Manila Electric Company (MERALCO) was dismissed after 32 years of service. At the time of his dismissal, he was Administrative Officer of MERALCO.- On June 5, 1987, petitioner informed his Supervisor-Branch head Sofronio Ortega, Jr. that he would render overtime work on June 6, 1987, a Saturday, and that after concluding his field work on that day, he would proceed to Pagbilao, Quezon to accompany his wife who was a principal sponsor to a kin’s wedding.- On June 6 Naguit proceeded to his field assignment to conduct “supervisory survey on re-sequence of customer’s account numbers”, and to supervise MERALCO’s “Operation FC” (apprehension of customers with illegally connected service). At 12:00 noon, he, along with his co-employee Accounts Representative Fidel Cabuhat who drove his (petitioner’s) jeep, proceeded to Pagbilao, Quezon.- On June 8, the timekeeper prepared an Overtime Notice and the corresponding Timesheet[9] wherein it was reflected that petitioner worked from 8:00 a.m. to 5:00 p.m. on June 6 and 7. Petitioner corrected the documents by erasing the entries made for June 7. The documents were approved by petitioner’s supervisor Ortega. Petitioner was thereafter paid for overtime work on June 6.- Documents including petty cash voucher covering Cabuhat’s alleged overtime work on June 6 were also prepared on account of which petitioner, as custodian of petty cash, released to Cabuhat the amount of P192.00 representing meal allowance and rental for a jeep.- More than two years later, petitioner received from the Legal and Investigation Staffs Head of MERALCO a letter stating that the Special Presidential Committee (SPC) is in receipt of information that he caused reimbursement of transportation expenses for the work of Cabuhat not actually rendered. It requested that he report to the Ortigas office Feb 27, 1990 for the administrative proceedings.- During the administrative proceedings, Naguit wiaved his right to counsel and gave sworn statements denying the charges. - Evidence against petitioner consisted primarily of the sworn statements of Cabuhat who was charged along with petitioner with falsification of time card; Olivia Borda, billings clerk; and five customers of MERALCO. The statements tried to establish that, petitioner induced Cabuhat to prepare a petty cash voucher covering expenses for meal and rental of a jeep for the June 6 alleged conduct by the latter of field verification of “Bill Omissions;” that on petitioner’s invitation, Cabuhat also repaired to Pagbilao, Quezon on June 6; and that petitioner gave the petty cash payable to Cabuhat making it appear that some collections for “bill omissions” were received from customers on June 6 when in fact no such collections were ever received from the customers in whose name official receipts were issued.- SPC found Naguit and Cabuhat guilty of falsification of time cards under Sec. 7, par. 7 of the Company Code on Employee Discipline. Additionally, petitioner was found guilty under Sec. 6, par. 24 of the Code for encouraging Cabuhat to commit an act constituting a violation of the Code.- MERALCO thus informed petitioner that he was, for falsification of time card and encouraging and inducing another employee to perform an act constituting a violation of the Company Code on Employee Discipline, dismissed from the service with forfeiture of all rights and privileges. - Naguit filed a complaint with the NLRC against MERALCO for illegal dismissal, he praying for reinstatement, backwages, damages, attorney’s fees and other awards he is entitled to. Labor Arbiter found for Naguit.- Meralco appealed which reversed the Labor Arbiter’s decision. Petitioner’s claims > the factual findings of the Labor Arbiter clearly show that he, as an Administrative Officer, is covered by respondent MERALCO’s policy pertaining to field personnel, particularly when he is designated to perform field assignments. As such, he did not bother to correct the Overtime Notice which indicated that he worked from 8 a.m. to 5 p.m., albeit he actually worked until 12 noon, the company policy being that even if an employee who had a field assignment did not actually render 8 hours of work, he is deemed to have worked for such duration provided he had completed the assigned task as he claims he did.

ISSUES1. WON Naguit is guilty of falsification2. WON NLRC committed grave abuse of discretion when they gave full credence to Cabuhat’s affidavits that he was induced to claim overtime pat despite Cabuhat’s failure to affirm such in the arbitral proceedings 3. WON there was valid ground for dismissal

HELD1. NO- The petitioner was in good faith when he did not correct the entry in the Notice of Overtime and Timesheet reflecting that he worked up to 5pm on June 6. - Petitioner advised his superior Ortega about his rendering overtime work the following day, June 6, 1987, after which he would head for Pagbilao after concluding his work. If petitioner had intended to do overtime work up to 5:00 p.m., there would have been no need for him to advise Ortega that he would thereafter go to Pagbilao. Since Ortega

Labor Law 1 A2010 - 244 - Disininever refuted petitioner’s claim about his advising him of his proceeding to Pagbilao and in fact the grant and release of petitioner’s overtime pay was approved by Ortega, who had the discretion to “judge the number of hours that can be foregone” in light of his (Ortega’s) explanation that office personnel on field assignment “forego the convenience of the office, they [being] exposed to the heat of the sun” and the like, this Court would not, as the Labor Arbiter did not, attribute malice to petitioner. Thus, the Labor Arbiter held that Ortega opined that half day would not be allowed. But, the fact remains that such discretion is exercised, the limit of which was not shown to have been disseminated to the employees, the qualifying factor being whether the job was satisfactory or not. If on the contrary, there was indeed no such practice or, that complainant, being an office personnel, is removed from coverage thereof and governed strictly by the time-rule such that he would have been off at the actual completion of the assigned task, he would not have bothered to inform his branch head - in effect a request for permission of his planned trip to Pagbilao, Quezon thereafter. That would have been meaningless gesture on the part of the complainant.- With the incentive scheme or tolerance of Naguit, there is no resulting prejudice to Meralco so to speak of nor intention on the part of complainant to cause it. What was done was consistent with management policy on covering the overtime work in the branch.2. YES- In labor cases, where the adverse party is deprived of the opportunity to cross-examine affiants, affidavits are considered hearsay unless the affiants are placed on the witness stand to testify thereon. Cabuhat’s affidavits are inadmissible as evidence.

3. YES - Naguit, despite his knowledge that Cabuhat did not hire any jeep nor conduct field verification on June 6, released the petty cash representing Cabuhat’s meal allowance and rental fee for a jeep. As custodian of the petty cash fund, he had the duty to ascertain that the circumstances which brought about any claim therefrom were in order. He cannot now shirk from this responsibility by indirectly pinning the blame on the approving officer and asserting that the transgression was the result of mere inadvertence, given his admission that he very well knew that Cabuhat did not conduct any field work on June 6, 1987, he (Cabuhat) having merely driven for him to Pagbilao.- Petitioner thus committed dishonesty and breached MERALCO’s trust, which dishonesty calls for reprimand to dismissal under MERALCO’s rules.- Dismissal is, however, too severe as a penalty in petitioner’s case, given his 32 years of service during which he had no derogatory record.At the time petitioner was dismissed, he was still below the retirement age of employees of MERALCO at 60. However, he is now about 65. Imposing a penalty less harsh than dismissal and ordering his reinstatement are thus functus oficio, the Labor Arbiter’s order for his reinstatement not having been executed.Disposition Decision and Resolution of the NLRC are hereby SET ASIDE. Respondent MERALCO is, in light of the foregoing discussions, hereby ORDERED to pay petitioner Aniceto W. Naguit, Jr. his retirement benefits to be computed from the inception of his service up to the time he reached 60 years of age, in accordance with its retirement plan.

CONSTRUCTIVE DISCHARGE

DEFINED

PHIL JAPAN ACTIVE CARBON CORP V NLRC (QUINANOLA)171 SCRA 164

GRINO-AQUINO; March 8, 1989

NATUREA petition for review

FACTS- Quinanola had been employed in Phil. Japan since January 19, 1982, as Assistant Secretary/Export Coordinator. He was promoted to the position of Executive Sec. to the Executive Vice President and General Manager. On May 31, 1986, for no apparent reason at all and without prior notice to her, she was transferred to the Production Department as Production Secretary, swapping positions with Ester Tamayo. Although the transfer did not amount to a demotion because her salary and workload remained the same, she believed otherwise so she rejected the assignment and filed a complaint for illegal dismissal. LA found that the transfer would amount to constructive dismissal and her refusal to obey the order was justified. Upon appeal to the NLRC, the Commission approved the Labor Arbiter's decision but reduced to P10,000 the award of moral damages and the attorney's fees to 10% of the judgment.

ISSUEWON Quinanola was constructively and illegally dismissed as a result of her transfer or assignment to the Office of the Production Manager even if she would have received the same salary rank, rights and privileges

HELD

NO- A constructive discharge is defined as: "A quitting because continued employment is rendered impossible, unreasonable or unlikely; as, an offer involving a demotion in rank and a diminution in pay." In this case, Quinanola’s assignment as Production Secretary of the Production Department was not unreasonable as it did not involve a demotion in rank (her rank was still that of a department secretary) nor a change in her place of work (the office is in the same building), nor a diminution in pay, benefits, and privileges. It did not constitute a constructive dismissal. - It is the employer's prerogative, based on its assessment and perception of its employees' qualifications, aptitudes, and competence, to move them around in the various areas of its business operations in order to "ascertain where they will function with maximum benefit to the company. An employee's right to security of tenure does not give him such a vested right in his position as would deprive the company of its prerogative to change his assignment or transfer him where he will be most useful. When his transfer is not unreasonable, nor inconvenient, nor prejudicial to him, and it does not involve a demotion in rank or a diminution of his salaries, benefits, and other privileges, the employee may not complain that it amounts to a constructive dismissal.- On the other hand, we reject the petitioner's contention that the private respondent's absence from work on June 2 to June 3, 1986 constituted an abandonment of her job in the company resulting in the forfeiture of the benefits due her. While she was guilty of insubordination for having refused to move out of her position as Executive Secretary to the Executive Vice-President and General Manager of the company, dismissal from the service would be a draconian punishment for it, as her complaint for illegal dismissal was filed in good faith.Disposition the decision of the NLRC insofar as it orders the petitioner to reinstate the private respondent is affirmed, but she shall be reinstated to her position as Production Secretary of the Production Department of petitioner's corporation without loss of seniority rights and other privileges. The awards of backwages, moral damages and attorney's fees to the private respondent are hereby set aside. No pronouncement as to costs.

DUSIT HOTEL NIKKO V NUWHRAIN466 SCRA 374

CALLEJO; August 9, 2005

NATUREPetition for review on certiorari of the Decision of the Court of Appeals

FACTSThe Case for Rowena Agoncillo- Agoncillo was employed by the Hotel. After some time, she was promoted as Supervisor of Outlet Cashiers and later promoted as Senior Front Office Cashier. - The Hotel decided to trim down the number of its employees from the original count of 820 to 750. - The Hotel offered a Special Early Retirement Program (SERP) to all its employees. It was stated therein that the program was intended to “provide employees financial benefits prior to prolonged renovation period and, at the same time, to enable management to streamline the organization by eliminating redundant positions and having a more efficient and productive manpower complement.”- Union president Rasing, sought “a commitment from the management that the employees terminated due to redundancy will not be replaced by new employees; nor will their positions be given to subcontractors, agencies or casual employees.”- A total of 243 employees, including Agoncillo, 161 of whom were Union officers and members, were separated from the Hotel’s employment. As a result, the membership of the Union was substantially reduced.- The Hotel wrote DOLE saying that the Hotel terminated the employment of 243 employees due to redundancy. On the same day, Agoncillo was summoned by Hotel Comptroller Reynaldo Casacop, who gave her a letter of even date informing the latter of her “separation from service due to redundancy effective close of office hours of April 30, 1996.” - Casacop advised Agoncillo to just avail of the Hotel's SERP, as embodied in the inter-office memorandum of Masuda. He informed her that she had the option to avail of the program and that, in the meantime, he will defer the processing of her termination papers to give her time to decide. On April 3, 1996, Agoncillo finally told Casacop that she would not avail of the SERP benefits. By then, she had decided to file a complaint for illegal dismissal against the Hotel.- Meanwhile, the Hotel temporarily closed operations because of the renovation thereof.- When news spread among the hotel employees that Agoncillo would contest her termination before the NLRC, she was summoned by Personnel Manager Leticia Delarmente to a conference. Delarmente and Dizon repeatedly asked Agoncillo to give back the original copy of the April 1, 1996 termination letter. Agoncillo told them that the letter was already in the possession of her counsel. Agoncillo was relieved when she was given another letter of even date stating that, by reason of her non-availment of the SERP, she was still considered an employee but on temporary lay-off due to the ongoing renovation of the Hotel and that she will just be advised accordingly of her work schedule when the Hotel reopens. - Delarmente and Dizon offered to reinstate Agoncillo but not to her former position as Senior Front Office Cashier. Agoncillo objected but informed them that she could accept the position of Reservation Clerk. However, no response was received.

Labor Law 1 A2010 - 245 - Disini- She was told by Dizon that the Hotel was willing to reinstate her but as an Outlet Cashier. Dizon explained that the Hotel had already hired new employees for the positions of Reservation Clerks. Agoncillo, however, pointed out that she was already an Outlet Cashier Supervisor before her promotion as Senior Front Office Cashier and that if she accepted the position, it would be an unjustified demotion on her part. After Agoncillo’s meeting with Dizon, the latter kept on promising to find a suitable position for her. In those meetings, Dizon always offered reinstatement to positions that do not require guest exposure like Linen Dispatcher at the hotel basement or Secretary of Roomskeeping. When Agoncillo refused, Dizon just instructed her to return. Agoncillo had no specific position or assigned task to perform.- When the Hotel resumed operations, the Union filed a Notice of Strike for unfair labor practice with the DOLE.

The Case for the Hotel- Pursuant to the reorganization program, a reclassification of positions ensued upon resumption of the Hotel’s operation. Consequently, the position of Agoncillo as Senior Front Office Cashier was abolished and a new position of Guest Services Agent absorbing its functions was created. Considering that the new position requires skills in both reception and cashiering operations, respondent Hotel deemed it necessary to transfer Agoncillo to another position as Outlet Cashier, which does not require other skills aside from cashiering.- The transfer of Agoncillo from Senior Front Office Cashier to Outlet Cashier does not entail any diminution of salary or rank. Despite which, she vehemently refused the transfer and insisted that she be reinstated to her former position. Since Agoncillo was not amenable to the said transfer, she did not assume her new position and since then had stopped reporting for work despite the Hotel’s patient reminder to act on the contrary. Instead, she filed a complaint to question the prerogative of the management to validly transfer her to another position as she considers the transfer an act of constructive dismissal amounting to illegal termination and unfair labor practice in the form of union busting.

ISSUE WON Agoncillo was illegally dismissed

HELDYES- We agree with the contention of the petitioners that it is the prerogative of management to transfer an employee from one office to another within the business establishment based on its assessment and perception of the employee’s qualification, aptitude and competence, and in order to ascertain where he can function with the maximum benefit to the company. But, like other rights, there are limits thereto. The managerial prerogative to transfer personnel must be exercised without grave abuse of discretion, bearing in mind the basic elements of justice and fair play. Having the right should not be confused with the manner in which that right is exercised. - There is constructive dismissal when there is a demotion in rank and/or diminution in pay; or when a clear discrimination, insensibility or disdain by an employer becomes unbearable to the employee. - In the present case, the Hotel recalled the termination of respondent Agoncillo when they learned that she was going to file a complaint against them with the NLRC for illegal dismissal. However, instead of reinstating her to her former position, she was offered the position of Linen Dispatcher in the hotel basement or Secretary of the Roomskeeping Section, positions much lower than that of a Supervisor of Outlet Cashiers which the respondent held before she was promoted as Senior Front Office Cashier. With the said positions, the respondent would not certainly be receiving the same salary and other benefits as Senior Front Office Cashier.- The offers by the petitioners to transfer Agoncillo to other positions were made in bad faith, a ploy to stave off a suit for illegal dismissal. In fact, Agoncillo had not been transferred to another position at all.- Even assuming, for the sake of argument, that the hotel had a valid ground for dismissing [the] complainant and that it had merely spared her such fate, the hotel is still guilty of illegal dismissal. Had the hotel made the transfer of complainant in good faith and in the normal course of its operation, it would have been justified. In this case, however, the supposed transfer was made only after complainant had been earlier terminated. Complainant’s statement in her affidavit that she was summoned by the hotel after news of her plan to contest her dismissal circulated remains unrefuted. Furthermore, the hotel has not explained why there was no official memorandum issued to complainant formally informing her of her “transfer”. All these lead to only one conclusion – that the alleged transfer was not made in good faith as a valid exercise of management prerogative but was intended as a settlement offer to complainant to prevent her from filing a case.Disposition Petition is DENIED for lack of merit. Costs against the petitioners.

MOBILE PROTECTIVE AND DETECTIVE AGENCY V OMPAD

458 SCRA 308PUNO; May 9, 2005

NATURE Petition for review on certiorari of the decision of the CA

FACTS- Private respondent, Alberto Ompad, was employed by the petitioner as a security guard in 1990. He was assigned to the various clients of Mobile. In June 1997, respondent was assigned as a security guard at Manila Southwoods when he inquired from the project manager of Southwoods if they have already paid their backwages to the security agency. Ompad claims that when the Agency found out about his query, he was relieved from his post and never given another assignment.- The petitioner on the other hand claims that Ompad was assigned to another client, Valle Verde Country Club from August 29 to October 31, 1997 after he was relieved from his post at the Manila Southwoods. Petitioner further claims that one of the guards at Valle Verde attested that Ompad had told her that he would earn better if he just drives his tricycle full time. On October 15, 1997, Ompad reported for work but he was limping due to an accident he suffered while driving his tricycle. Petitioner claims that he stopped reporting for work after that date. On September 23, 1998, Domingo Alonzo, operations manager of Mobile saw respondent and inquired as to whether he was still interested in reporting for work. The petitioner allegedly answered in the negative and it was at that time that Alonzo advised him to resign. Ompad, he claims, submitted his hand written resignation which also was a quit claim.- Petitioner contended that the letter of resignation was forced on him in return for monies owed him. As he needed the money, he had no choice but to comply. He however was only being given Pesos 5,000 which he rejected. He filed this case the following day.- Ompad alleged that he was illegal terminated and claimed underpayment or non-payment of wages, overtime pay, premium pay for holiday and rest day, separation pay, etc.- Labor Arbiter dismissed the complaint for lack of merit. The NLRC reversed the decision. The CA also dismissed the action for reconsideration, noting that there was no voluntariness in the acts of Ompad in submitting the resignation letters. Hence this action.

ISSUEWON Ompad was illegally dismissed

HELDYES- The resignation letters of Ompad are dubious as they were written in a language obviously not his and “lopsidedly worded” to free the Agency from liabilities. The affidavits issued by the witnesses of Mobile are suspect considering that these witnesses were/are in fact employed by the petitioner. Reasoning- All the documentary evidence proves that respondent was assigned to Valle Verde from September 29 to October 31, 1997 and that he stopped reporting for work on October 16, 1997. After this period, respondent did not seem to have be given any further assignment. - The SC ruled that while it is true that security guards may be put on floating status the same should last for only six months. In the case at bar, there was no showing that Mobile lacked engagements to which they can post their guards. Absent any dire exigency justifying their failure to give respondent further assignment, the only logical conclusion is that respondent was constructively dismissed. - Even assuming that Mobile was justified in not immediately giving Ompad any assignment after October, the length of time that he was put on floating status is tantamount to constructive dismissal. - In an illegal dismissal case, the onus probandi is on the employer to prove that the dismissal was in fact for valid cause. It was in this case also the burden of Mobile to submit evidence that the resignation was voluntary on the part of Ompad. Disposition Petition dismissed.

DUNCAN ASSOCIATION V GLAXO WELLCOME[PAGE 43]

R.P. DINGLASAN CONSTRUCTION INC V ATIENZA433 SCRA 263

PUNO; June 29, 2004

NATUREThis is an appeal from the decision and resolution of the Court of Appeals, dated January 17, 2001 and October 30, 2002, respectively, upholding the finding of constructive dismissal against petitioner.

FACTS- Petitioner R.P. Dinglasan Construction, Inc. provided janitorial services to Pilipinas Shell Refinery Corporation (Shell Corporation) in Batangas City. Private respondents Mariano Atienza and Santiago Asi served as petitioner’s janitors assigned with Shell Corporation since 1962 and 1973, respectively. - July 7, 1994 - Dinglasan called for a meeting and informed them and 3 other employees that their employment with Shell Corporation would be terminated effective July 15, 1994.

Labor Law 1 A2010 - 246 - DisiniThey were told that Dinglasan lost the bidding for janitorial services with Shell. Dinglasan notified them that they may reapply as helpers and redeployed in other companies where DInglasan had subsisting contracts but they would receive only a minimum wage. Atienza refused as the offer would be a form of demotion --- they would lose their seniority status and would not be guaranteed to work at regular hours.- December 1994 – Atienza et al filed a complaint against Dinglasan for non-payment of salary with the DOLE district office in Batangas City. - February 1995 - during the conciliation proceedings with the DOLE, Dinglasan sent notices to Atienza et al informing them that they would be reinstated with Shell Corporation as soon as they submit their barangay clearance, medical certificate, picture and information sheet as per the new identification badge requirements of Shell Corporation. Barangay officials met with Dinglasan to signify Atienza et al’s willingness to to be reinstated bringing with them said requirements- May 1995 – Atienza et al demanded the payment of their backwages starting from July 15, 1994. - June 1, 1995 – Dinglasan notified Atienza et al they have been declared absent without leave (AWOL) as they allegedly failed to signify their intention to return to work and submit the badge requirements for their reinstatement. - June 13, 1995 – Atienza et al wrote Dinglasan and insisted that they had complied with the badge requirements. Accompanied by the barangay officials, Atienza et al attempted to meet with the officers of Dinglasan but the latter refused to dialogue with them. As proof of their compliance with the Shell requirements, Atienza et al submitted to the DOLE their x-ray results, dated May 17 and 19, 1995 and their barangay certification, dated May 13, 1995.- NLRC > Atienza et al charged DInglasan with illegal dismissal and non-payment of 13th month pay, with a claim for payment of attorney’s fees and litigation expenses, and a prayer for reinstatement with payment of full backwages from July 15, 1994. > Dinglasan gave a different version of the incident. It allegedly informed Atienza et al and the other affected employees that they would be deployed to petitioner’s other principal companies but that their work would be different. Except for Atienza et al, all the affected employees accepted its offer of redeployment and reported back to work. Atienza et al failed to submit a resignation letter to signify their intention not to return to work. - during the pendency of the labor case – Dinglasan in 2 separate notices informed Atienza et al that they could be reinstated at Shell Corporation with no diminution in their salary provided that they submit the documents for the new identification badge requirement of Shell Corporation. Atienza et al, however, refused to return to work until they were paid their backwages. Consequently, Dinglasan was constrained to consider them as having abandoned their work and to terminate their employment on September 19, 1995. Dinglasan, thus, justified the dismissal of Atienza et al on the grounds of gross and habitual neglect of duties and abandonment of work. - LABOR ARBITER > September 3, 1998, LA Andres Zavalla rendered a decision finding that private respondents were illegally dismissed from service and ordering their reinstatement. - NLRC > the decision of the labor arbiter was affirmed - CA > PROCEDURAL: petition could not prosper as petitioner failed to move for a reconsideration of the NLRC decision; SUBSTANTIVE: upheld the findings of the labor arbiter and the NLRC that: (1) private respondents were constructively dismissed as petitioner’s offer of reassignment involved a diminution in pay and demotion in rank that made their continued employment unacceptable; and, (2) private respondents could not be considered to have abandoned their work; DInglasan’s motion for reconsideration was denied

ISSUES1. WON there is valid dismissal on the ground that they failed to report back to the office and this abandoned their work 2. WON there was constructive dismissal

HELD1. Ratio In an illegal dismissal case, the onus probandi rests on the employer to prove that its dismissal of an employee is for a valid cause. In the case at bar, Dinglasan failed to discharge its burden. It failed to establish that Atienza et al deliberately and unjustifiably refused to resume their employment without any intention of returning to work. - To constitute abandonment of work, two (2) requisites must concur: first, the employee must have failed to report for work or must have been absent without justifiable reason; and second, there must have been a clear intention on the part of the employee to sever the employer-employee relationship as manifested by overt acts. Abandonment as a just ground for dismissal requires deliberate, unjustified refusal of the employee to resume his employment. Mere absence or failure to report for work, after notice to return, is not enough to amount to abandonment.Reasoning- the evidence negates the theory that they abandoned their work. (1) Atienza et al reported back to Dinglasan’s office a number of times expressing their desire to continue working without demotion in rank or diminution of salary. This fact was established by the corroborating testimony of barangay officials, accompanied Atienza et al to Dinglasan’s office at least ten (10) times to negotiate their redeployment on more acceptable terms. (2) in seeking reinstatement, Atienza et al also sought the intervention of the DOLE to arbitrate the labor issue between the parties.

(3) Atienza et al submitted the barangay clearances and x-ray results required from them by petitioner for their reinstatement as witnessed by the barangay officials. (4) the records would bear that Atienza et al lost no time and sought their reinstatement by filing an illegal dismissal case against Dinglasan, which act is clearly inconsistent with a desire to sever employer-employee relations and abandon their work. - All these overt acts on the part of Atienza et al negate Dinglasan’s claim of abandonment of work and prove beyond doubt their steadfast desire to continue their employment with petitioner and be reinstated to their former position. 2. YESRatio Constructive dismissal is defined as quitting when continued employment is rendered impossible, unreasonable or unlikely as the offer of employment involves a demotion in rank and diminution of pay. Reasoning- Dinglasan committed constructive dismissal when it offered to reassign Atienza et al to another company but with no guaranteed working hours and payment of only the minimum wage. The terms of the redeployment thus became unacceptable for private respondents and foreclosed any choice but to reject petitioner’s offer, involving as it does a demotion in status and diminution in pay. Thereafter, for six (6) months, Atienza et al were in a floating status. Interestingly, it was only after Atienza et al filed a complaint with the DOLE that Dinglasan backtracked in its position and offered to reinstate Atienza et al to their former job in Shell Corporation with no diminution in salary. Eventually, however, Dinglasan unilaterally withdrew its offer of reinstatement, refused to meet with the Atienza et al and instead decided to dismiss them from service. Disposition petition is DISMISSED and the impugned decision and resolution of the Court of Appeals, dated January 17, 2001 and October 30, 2002, respectively, are AFFIRMED in toto.

GO V CA (MOLDEX PRODUCTS INC)430 SCRA 358

YNARES-SANTIAGO; May 28, 2004

NATUREPetition for review decision of CA (which set aside resolutions of NLRC)

FACTS- Fernando Go was hired by Moldex Products Inc. in 1986 as a salesman, then, over the years, was promoted to a Senior Sales Manager. As such officer, he was responsible for overseeing and managing the sales force of the company such as dealing with clients, getting orders, entering into an agreement with clients (subject to approval of higher management).- Sometime in 1998, the EVP o Moldex called the attention of Go regarding the discovery of alleged anomalies purportedly committed by the sales people under Go’s control. Such anomalies stemmed from the disbursement of funds by Moldex to gov’t officials to secure big supplpy contracts from the gov’t.- Because of the issue, a number of employees were dismissed, including those under Go’s supervision. Go himself was terminated, allegedly “on account of command responsibility”. Moldex claimed that Go, “obviously feeling guilty for not exercising effective supervision over his subordinates, submitted a letter of resignation dated October 12, 1998 but effective on November 16, 1998.” Moldex added that Go went on leave from Oct 12, 1998 to Nov 16, 1998. While on leave, he worked for the release of his clearance and the payment of 13th month pay and leave pay benefits. On the other hand, Go contends that he was not investigated. The investigation only involved other sales people. He filed a complaint for constructive dismissal. LA ruled for Go (there was illegal dismissal), NLRC affirmed, but CA set aside the decisions, relying on evidence that Go was actively performing his normal duties and functions during the months immediately prior to his resignation, contrary to the finding of constructive dismissal.

ISSUEWON there was constructive dismissal

HELDNORatio Constructive dismissal exists where there is a cessation of work because continued employment is rendered impossible, unreasonable or unlikely. It is present when an employee’s functions, which were originally supervisory in nature, were reduced, and such reduction is not grounded on valid grounds such as genuine business necessity.Reasoning - Apparently, Go still fully exercised the prerogatives and the responsibilities of his office as the Senior Sales Manager during the time that the said functions were supposedly removed from him. Therefore, there can be no constructive dismissal to speak of.- Go claims that his separation from employment with Moldex was a case of constructive dismissal, an allegation which the company refutes with its own set of evidence pointing to the Go’s voluntary resignation.- It should be remembered that Go has submitted a letter of resignation. It is thus incumbent upon him to substantiate his claim that his resignation was not voluntary but in truth was actually a constructive dismissal. This the petitioner failed to do. His bare allegations, when uncorroborated by evidence, cannot be given credence.

Labor Law 1 A2010 - 247 - Disini- on the other hand, Moldex presented confidential sales evaluation forms that prove that Go was still performing his duties and responsibilities one month prior to his resignation. - While on leave, he worked for the release of his clearance and the payment of his 13 th

month pay and leave pay benefits. In doing so, he in fact performed all that an employee normally does after he resigns. Resignation is the formal pronouncement or relinquishment of an office. The voluntary nature of Go’s acts has manifested itself clearly belie his claim of constructive dismissal.The totality of the evidence indubitably shows that Go resigned from employment without any coercion or compulsion from respondent. His resignation was voluntary. Disposition: Petition denied, and decision of CA AFFIRMED.

ACUNA V CA[PAGE 12]

POSEIDON FISHING V NLRC[PAGE 98]

CONSTRUCTIVE DISCHARGE AND ILLEGAL DISMISSAL

MARK ROCHE V NLRC313 SCRA 356

BELLOSILLO; August 31, 1999

FACTS- On different dates, private respondents filed separate complaints for underpayment of wages and non-payment of overtime pay against petitioners Mark Roche International (MRI), Eduardo Dayot and Susan Dayot. Private respondents sought the assistance of a labor organization which helped them organize the Mark Roche Workers Union (MRWU). Apparently irked by the idea of a union within the company, petitioners ordered private respondents to withdraw the petition and further threatened them that should they insist in the organization of a union they would be dismissed. Unfazed, private respondents refused. As expected, private respondents were discharged from work. Petitioners disclaimed knowledge of any deficiency owing to private respondents since all the benefits due them as required by law were fully paid, except overtime pay which they were not entitled to on account of their being piece-rate workers. The Labor Arbiter rendered his decision declaring as illegal the constructive dismissal of private respondents and ordered their reinstatement, payment of backwages, salary differentials and proportionate 13th month pay and service incentive leave pay. On appeal, the National Labor Relations Commission (NLRC) affirmed the decision of the Labor Arbiter, but set aside the award of service incentive leave on the ground that private respondents were not entitled thereto as they were piece-rate workers. Petitioners moved for reconsideration, but it was denied. Hence, the present petition.

ISSUEWON the dismissal of private respondents was a constructive dismissal or an illegal dismissal

HELD- Constructive dismissal or a constructive discharge has been defined as a quitting because continued employment is rendered impossible, unreasonable or unlikely, as an offer involving a demotion in rank and a diminution in pay. In the instant case, private respondents were not demoted in rank nor their pay diminished considerably. They were simply told without prior warning or notice that there was no more work for them. After receiving the notice of hearing of the petition for certification election on 27 October 1992, petitioners immediately told private respondents that they were no longer employed. Evidently it was the filing of the petition for certification election and organization of a union within the company which led petitioners to dismiss private respondents and not petitioners' allegations of absence or abandonment by private respondents. The formation of a labor union has never been a ground for valid termination, and where there is an absence of clear, valid and legal cause, the law considers the termination illegal.

GLOBE TELECOM INC V FLORENDO390 SCRA 201

September 27, 2002

NATUREPetition for review on certiorari of a decision of CA.

FACTS- FLORENDO, a Senior Account Manager of Globe, filed a complaint for constructive dismissal against Globe with some key officials [GLOBE et al., for brevity] and FLORENDO’s immediate superior Cacholo Santos [SANTOS, for brevity]. FLORENDO

complained that SANTOS never submitted her performance evaluation report thereby depriving her of salary increases and incentives which other employees of the same rank had been receiving; reduced her to a house-to-house selling agent (i.e. a direct sales agent) of company products ("handyphone") despite her rank as supervisor of company dealers and agents; never supported her in the sales programs she presented; and, withheld all her other benefits.- GLOBE et al., on the other hand, claimed that FLORENDO abandoned her work; that her complaint rested on a purely private disagreement with her immediate superior, and that she filed the complaint without consulting the company’s grievance process.

ISSUEWON FLORENDO can be constructively dismissed from service

HELDYESRatio Constructive dismissal exists where there is cessation of work because "continued employment is rendered impossible, unreasonable or unlikely, as an offer involving a demotion in rank and a diminution in pay." All these are discernible in FLORENDO’s situation. She was singularly edged out of employment by the undesirable treatment she received from her superior , who discriminated against her without reason . ( See above for SANTOS’ acts against FLORENDO.) And although FLORENDO continued to have the rank of a supervisor, her functions were reduced to a mere direct sales agent. This was tantamount to a demotion . She might not have suffered any diminution in her basic salary but GLOBE et al. did not dispute her allegation that she was deprived of all benefits due to another of her rank and position, benefits which she apparently used to receive . - Far from blaming SANTOS alone, FLORENDO also attributes her degraded state to GLOBE et al. She cited GLOBE et al.'s indifference to her plight as she was twice left out in a salary increase, without GLOBE et al. giving her any reason. It eludes belief that GLOBE et al. were entirely in the dark as the salary increases were granted across-the-board to all employees except FLORENDO. It is highly improbable that the exclusion of FLORENDO had escaped GLOBE et al.'s notice. The absence of an evaluation report from SANTOS should have been looked into by GLOBE et al. for proper action. If a salary increase was unwarranted, then it should have been sufficiently explained by GLOBE et al. to FLORENDO. And despite GLOBE et al.’s claim that FLORENDO did not brought her problem against SANTOS to the company's grievance machinery, it remains uncontroverted that FLORENDO had inquired from GLOBE et al. why her other benefits had been withheld and sought clarification for her undeserved treatment but GLOBE and SANTOS remained mum.- Thus, the dispute was not a mere private spat between FLORENDO and her superior; the case overflowed into the realm of FLORENDO's employment. And at the very least, GLOBE et al. were negligent in supervising all of their employees.- In constructive dismissal, the employer has the burden of proving that the transfer and demotion of an employee are for just and valid grounds such as genuine business necessity. The transfer must not involve a demotion in rank or a diminution of salary and other benefits. If the employer cannot overcome this burden of proof, the employee's demotion shall be tantamount to unlawful constructive dismissal. The award of back wages in the instant case is justified upon the finding of illegal dismissal.Disposition CA decision that FLORENDO abandoned her work, SET ASIDE. GLOBE et al. to pay FLORENDO full back wages from the time she was constructively dismissed until her reinstatement, and to cause immediate reinstatement of FLORENDO to her former position, without loss of seniority rights and other benefits.

PREVENTIVE SUSPENSION

GLOBE-MACKAY CABLE AND RADIO CORP V NLRC (SALAZAR)

206 SCRA 702ROMERO; March 3, 1992

NATUREAppeal from a decision of NLRC

FACTS- Imelda L. Salazar was employed by Globe-Mackay Cable and Radio Corporation (GMCR) as general systems analyst. Also employed by petitioner as manager for technical operations' support was Delfin Saldivar with whom private respondent was allegedly very close.

Labor Law 1 A2010 - 248 - Disini- GMCR, prompted by reports that company equipment and spare parts worth thousands of dollars under the custody of Saldivar were missing, caused the investigation of the latter's activities.- The report prepared by the company's internal auditor indicated that Saldivar had entered into a partnership styled Concave Commercial and Industrial Company with Richard A. Yambao, owner and manager of Elecon Engineering Services (Elecon), a supplier of petitioner often recommended by Saldivar; that Saldivar had taken petitioner's missing Fedders airconditioning unit for his own personal use without authorization and also connived with Yambao to defraud petitioner of its property; that Imelda Salazar violated company regulations by involving herself in transactions conflicting with the company's interests. Evidence showed that she signed as a witness to the articles of partnership between Yambao and Saldivar. It also appeared that she had full knowledge of the loss and whereabouts of the Fedders airconditioner but failed to inform her employer.- GMCR placed Salazar under preventive suspension for 1 month, thus giving her 30 days within which to, explain her side. But instead of submitting an explanation, private respondent filed a complaint against petitioner for illegal suspension, which she subsequently amended to include illegal dismissal, vacation and sick leave benefits, 13th month pay and damages, after petitioner notified her in writing that she was considered dismissed in view of her inability to refute and disprove the findings. - Labor Arbiter ordered petitioner company to reinstate private respondent to her former or equivalent position and to pay her full backwages and other benefits - NLRC affirmed the aforesaid decision

ISSUES1. WON the suspension was illegal2. WON Art.2794 of the Labor Code should apply

HELD1. NORatio By itself, preventive suspension does, not signify that the company has adjudged the employee guilty of the charges she was asked to answer and explain. Such disciplinary measure is resorted to for the protection of the company's property pending investigation any alleged malfeasance or misfeasance committed by the employee.Reasoning - The investigative findings of GMCR which pointed to Saldivar's acts in conflict with his position as technical operations manager, necessitated immediate and decisive action on any employee closely, associated with Saldivar. The suspension of Salazar was further impelled by the discovery of the missing Fedders airconditioning unit inside the apartment private respondent shared with Saldivar. Under such circumstances, preventive suspension was the proper remedial recourse available to the company pending Salazar's investigation. - If at all, the fault, lay with Salazar when she ignored petitioner's memo giving her ample opportunity to present her side. Instead, she filed her complaint for illegal suspension without giving her employer a chance to evaluate her side of the controversy.2. YESRatio Where a case of unlawful or unauthorized dismissal has been proved by the aggrieved employee, or on the other hand, the employer whose duty it is to prove the lawfulness or justness of his act of dismissal has failed to do so, then the remedies provided in Article 279 should find, application.

Reasoning- It must be recalled that the present Constitution has gone further than the 1973 Charter in guaranteeing vital social and economic rights to marginalized groups of society, including labor. To be sure, both Charters recognize "security of tenure" as one of the rights of labor which the State is mandated to protect. But there is no gainsaying the fact that the intent of the framers of the present Constitution was to give primacy to the rights of labor and afford the sector "full protection," at least greater protection than heretofore accorded them, regardless of the geographical location of the workers and whether they are organized or not.- that the right of an employee not to be dismissed from his job except for a just or authorized cause provided by law has assumed greater importance under the 1987 Constitution with the singular prominence labor enjoys under the article on Social Justice. And this transcendent policy has been translated into law in the Labor Code- The intendment of the law in prescribing the twin remedies of reinstatement and payment of backwages is, in the former, to restore the dismissed employee to her status before she lost her job, for the dictionary meaning of the word "reinstate" is "to restore to a state from which one had been removed" and in the latter, to give her back the income lost during the period of unemployment. - Over time, the following reasons have been advanced by the Court for denying reinstatement under the facts of the case and the law applicable thereto; that reinstatement can no longer be effected in view of the long passage of time or because of the realities of the situation; or that it would be "inimical to the employer's interest; " or that reinstatement may no longer be feasible; or that it will not serve the best interests of

4 The following provision on security of tenure is embodied in Article 279 reproduced herein but with the amendments

inserted by RA 6715:In cases of regular employment, the employer shall not terminate the services of-an employee except for a just cause or when authorized by this Title. An employee who is unjustly dismissed from work shall be entitled to reinstatement without loss of seniority rights AND OTHER PRIVILEGES and to his FULL backwages, inclusive of allowances, and to his other benefits or their monetary equivalent computed from the time his compensation was withheld from him up to the time of his ACTUAL reinstatement.

the parties involved; or that the company would be prejudiced by the workers' continued employment; or that it will not serve any prudent purpose as when supervening facts have transpired which make execution on that score unjust or inequitable or, to an increasing extent, due to the resultant atmosphere of "antipathy and antagonism" or "strained relations" or "irretrievable estrangement" between the employer and the employee. In lieu of reinstatement, the Court has variously ordered the payment of backwages and separation pay or solely separation pay.- If in the wisdom of the Court, there may be a ground or grounds for non-application of the Art.279, this should be by way of exception, such as when the reinstatement may be inadmissible due to ensuing strained relations between the employer and the employee.- Here, it has not been proved that the position of private respondent as systems analyst is one that may be characterized as a position of trust and confidence such that if reinstated, it may well lead to strained relations between employer and employee. Hence, this does not constitute an exception to the general rule mandating reinstatement for an employee who has been unlawfully dismissed.- To rely on the Maramara report as a basis for Salazar's dismissal would be most inequitous because the bulk of the findings centered principally oh her friend's alleged thievery and anomalous transactions as technical operations' support manager. Said report merely insinuated that in view of Salazar's special relationship with Saldivar, Salazar might have had direct knowledge of Saldivar's questionable activities. Direct evidence implicating private respondent is wanting from the records.Disposition The assailed resolution of NLRC is AFFIRMED. Petitioner GMCR is ordered to REINSTATE Salazar and to pay her backwages equivalent to her salary for a period of 2 years only. The decision is immediately executory.

SEPARATE OPINIION

MELENCIO-HERRERA [dissent]- I believe there is just cause for dismissal per investigative findings.

PHIL AIRLINES INC V NLRC (CASTRO)292 SCRA 40

ROMERO; July 8, 1998

NATUREAppeal from a decision of the NLRC affirming the decision of the LA

FACTS- Private Respondent Edilberto Castro, an employee (manifesting clerk) of PAL was apprehended by govt. authorities while about to board a flight to H.K. Castro and co-employee Arnaldo Olfindo were found to be in possession of P39,850 and P6,000 respectively, in violation of Central Bank (CB) Circular 265, as amended by CB Circular 383, 1 in relation to Section 34 of R.A. 265, as amended. - Upon knowledge of this incident, PAL required respondent to explain within 24 hrs why he should not be charged administratively. He failed to comply and was placed on preventive suspension effective March 27, 1984 for grave misconduct. An investigation was later conducted wherein respondent admitted ownership of the confiscated money but denied any knowledge of CB Circular 265. Respondent, through the PAL Employees Association (PALEA) then sought not only the dismissal of his case but also prayed for his reinstatement. - 3 years and 6 months after his suspension, PAL issued a resolution finding respondent guilty of the offense charged but nonetheless reinstated the latter explaining that the period within which he was out of work shall serve as penalty for suspension. Upon reinstatement, respondent filed a claim against PAL for backwages and salary increases granted under the CBA covering the period of his suspension which the latter, however, denied on account that under the existing CBA, “an employee under suspension is not entitled to CBA salary increases granted during the period covered by his penalty.”- Labor Arbiter De Vera rendered a decision in favor of Castro; limiting his suspension to 1 month; ordering PAL to pay his salaries, benefits, and other privileges from April 26, 1984 up to Sept. 18, 1987 and to pay his salary increases accruing during the period aforesaid. Moral damages and exemplary damages were likewise awarded. On appeal, the NLRC affirmed the LA decision but deleted the award of moral and exemplary damages, hence, this petition.

ISSUEWON an employee who has been preventively suspended beyond the maximum 30-day period is entitled to backwages and salary increases granted under the CBA during his period of suspension

HELDYES- The rules are rather clear under Secs. 3 and 4, Rule XIV of the Omnibus Rules Implementing the Labor Code:

Sec.3. Preventive suspension. The employer can place the worker concerned under preventive suspension if his continued employment poses a serious and imminent threat to the life or property of the employer or of his co-workersSec.4. Period of suspension. No preventive suspension shall last longer than 30 days. The employer shall thereafter reinstate the worker in his former or in a substantially

Labor Law 1 A2010 - 249 - Disiniequivalent position or the employer may extend the period of suspension provided that during the period of extension, he pays the wages and other benefits due to the workers. In such case, the worker, shall not be bound to reimburse the amount paid to him during the extension if the employer decides, after completion of the hearing, to dismiss the worker.

Reasoning - It is undisputed that the period of suspension of respondent lasted for 3 years and 6 months. PAL, therefore, committed a serious transgression when it manifestly delayed the determination of respondent’s culpability in the offense charged. The provisions of the rules are explicit and direct; hence, there is no reason to further elaborate on the same.- PAL faults the LA and the NLRC for allegedly equating preventive suspension as remedial measure with suspension as penalty for administrative offenses. This argument is inaccurate. As held in Beja Sr. v CA: “Imposed during the pendency of an administrative investigation, preventive suspension is not a penalty in itself. It is merely a measure of precaution so that the employee who is charged may be separated, for obvious reasons, from the scene of his alleged misfeasance while the same is being investigated. While the former may be imposed on a respondent during the investigation of the charges against him, the latter is the penalty which may only be meted upon him at the termination of the investigation or the final disposition of the case.” A cursory reading of the records reveals no reason to ascribe grave abuse of discretion against the NLRC; its decision was grounded upon petitioner’s manifest indifference to the plight of its suspended employee and its consequent violation of the Implementing Rules of the Labor Code. - As the NLRC correctly ruled: “The long period of preventive suspension could even be considered constructive dismissal because were it not for his letters demanding his reinstatement, PAL by its inaction appeared to have no plan to employ respondent back to work.” The manifest inaction of PAL over the pendency of the administrative charge is indeed violative of Castro’s security of tenure because without any justifiable cause and due process, his employment would have gone into oblivion. - PAL contends that when respondent consented to the resolution that the entire period of suspension shall constitute his penalty for the offense charged, the latter is thereby estopped to question the validity of said suspension. We concur with the labor arbiter when he ruled that the ensuing conformity by respondent does not cure petitioner's blatant violation of the law, and the same is therefore null and void- We do not question the right of the petitioner to discipline its erring employees and to impose reasonable penalties pursuant to law and company rules and regulations. “Having this right, however, should not be confused with the manner in which that right must be exercised.” Thus, the exercise by an employer of its rights to regulate all aspects of employment must be in keeping with good faith and not be used as a pretext for defeating the rights of employees under the laws and applicable contracts. Petitioner utterly failed in this respect.Disposition Petition is DISMISSED for lack of merit. Assailed decision is AFFIRMED.

VALIAO V CA[PAGE 11]

CADIZ V CA[PAGE 224]

MARICALUM MINING CORP V DECORION487 SCRA 182

TINGA; April 12, 2006

NATUREAppeal from decision of CA

FACTS- Decorion was a regular employee of Maricalum Mining who started out as a Mill Mechanic and was later promoted to Foreman I. - On April 11, 1996, the Concentrator Maintenance Supervisor called a meeting which Decorion failed to attend as he was then supervising the workers under him. Because of his alleged insubordination for failure to attend the meeting, he was placed under preventive suspension on the same day. He was also not allowed to report for work the following day.- May 12, 1996, Decorion was served a Notice of Infraction and Proposed Dismissal to enable him to present his side. On May 15, he submitted to the Personnel Department his written reply. A grievance meeting was held upon Decorion's request on June 5, during which he manifested that he failed to attend the meeting on April 11 because he was then still assigning work to his men. He maintained that he has not committed any offense and that his service record would show his efficiency.- July 23, 1996, Decorion filed before the Labor Arbiter a complaint for illegal dismissal and payment of moral and exemplary damages and attorney's fees.

- In the meantime, the matter of Decorion's suspension and proposed dismissal was referred to Atty. Roman G. Pacia, Jr., Maricalum Mining's Chief and Head of Legal and Industrial Relations, and recommended that Decorion's indefinite suspension be made definite with a warning that a repetition of the same conduct would be punished with dismissal. Maricalum Mining's Resident Manager issued a memorandum on August 2 placing Decorion under definite disciplinary suspension of 6 months which would include the period of his preventive suspension which was made to take effect retroactively.

- On September 4, 1996, Decorion was served a memorandum informing him of his temporary lay-off due to Maricalum Mining's temporary suspension of operations and shut down of its mining operations for 6 months, with the assurance that in the event of resumption of operations, he would be reinstated to his former position without loss of seniority rights. - Decorion, through counsel, wrote to Maricalum Mining on October 8, 1996, requesting that he be reinstated to his former position. The request was denied with the explanation that priority for retention and inclusion in the skeleton force was given to employees who are efficient and whose services are necessary during the shutdown.- Labor Arbiter found Decorion's dismissal illegal and ordered his reinstatement with payment of backwages and attorney's fees. According to the labor arbiter, Decorion's failure to attend the meeting called by his supervisor did not justify his preventive suspension. Further, no preventive suspension should last longer than 30 days.- The NLRC, however, reversed the labor arbiter's decision and dismissed Decorion's complaint. The reversal was premised on the finding that the case was litigated solely on Decorion's allegation that he was dismissed on April 11, 1996. However, during the grievance meeting held on June 5, 1996, Decorion left it up to management to decide his fate, indicating that as of that time, there was no decision to terminate his services yet. According to the NLRC, to consider the events that transpired after April 11, 1996 and make the same the basis for the finding of illegal dismissal would violate Maricalum Mining's right to due process.- CA reinstated decision of labor arbiter. It held that Decorion was placed under preventive suspension immediately after he failed to attend the meeting called by his supervisor on April 11, 1996. At the time he filed the complaint for illegal dismissal on July 23, 1996, he had already been under preventive suspension for more than 100 days in violation of Sec. 9, Rule XXIII, Book V of the Omnibus Rules Implementing the Labor Code (Implementing Rules) which provides that no preventive suspension shall last longer than 30 days.

- Maricalum Mining’s MFR was denied.

ISSUEWON Decorion was dismissed or merely under preventive suspension

HELD- Decorion’s preventive suspension has already ripened into a constructive dismissal.- Sections 8 and 9 of Rule XXIII, Book V of the Implementing Rules provide:

Section 8. Preventive suspension. — The employer may place the worker concerned under preventive suspension if his continued employment poses a serious and imminent threat to the life or property of the employer or his co-workers. Section 9. Period of Suspension — No preventive suspension shall last longer than thirty (30) days. The employer shall thereafter reinstate the worker in his former or in a substantially equivalent position or the employer may extend the period of suspension provided that during the period of extension, he pays the wages and other benefits due to the worker. In such case, the worker shall not be bound to reimburse the amount paid to him during the extension if the employer decides, after completion of the hearing, to dismiss the worker.

- Preventive suspension is justified where the employee's continued employment poses a serious and imminent threat to the life or property of the employer or of the employee's co-workers. Without this kind of threat, preventive suspension is not proper.- Decorion was suspended only because he failed to attend a meeting called by his supervisor. There is no evidence to indicate that his failure to attend the meeting prejudiced his employer or that his presence in the company's premises posed a serious threat to his employer and co-workers. The preventive suspension was unjustified. - Decorion's suspension persisted beyond the 30-day period allowed by the Implementing Rules. In Premiere Development Bank v. NLRC the Court ruled that preventive suspension which lasts beyond the maximum period allowed by the Implementing Rules amounts to constructive dismissal. - At the time Decorion filed a complaint for illegal dismissal, he had already been suspended for 103 days. Decorion's preventive suspension had already ripened into constructive dismissal at. While actual dismissal and constructive dismissal do take place in different fashion, the legal consequences they generate are identical. His employment may not have been actually terminated in the sense that he was not served walking papers but there is no doubt that he was constructively dismissed as he was forced to quit because continued employment was rendered impossible, unreasonable or unlikely by Maricalum Mining's act of

preventing him from reporting for work. - Article 286 of the Labor Code, which provides that the bona fide suspension of the operation of a business or undertaking for a period not exceeding six (6) months shall not terminate employment, may not be applied in this case. The

Labor Law 1 A2010 - 250 - Disiniinstant case involves the preventive suspension of an employee not by reason of the suspension of the business operations of the employer but because of the employee's failure to attend a meeting. The allowable period of suspension in such a case is only 30 days as provided by the Implementing Rules. Disposition Petition denied. CA’s decision affirmed.

RATIONALE

KWIKWAY ENG’G WORKS V NLRC (VARGAS)195 SCRA 526

MEDIALDEA; March 22, 1991

FACTS- Respondent Vargas was formerly employed by Kwikway as bookkeeper and secretary. As bookkeeper, it was her duty to fill up the check vouchers and indicate therein the name of the customer agent and the amount payable to each before they are presented to the agents for signing.- The new branch manager (BM) discovered that several blank vouchers already contained the signatures of the mechanic agents. BM confronted the branch cashier in charge of the vouchers, Marina Corpus, concerning the irregularity. Corpus explained that Vargas was aware of this practice. When asked for an explanation, Vargas stated that the procedure has been the practice in that office since the time of the former branch manager who had knowledge thereof. - BM informed the head office with his discovery. Kwikway’s VP conducted an investigation. On the following day, Vargas and Corpus were placed under preventive suspension for an indefinite period of time on the ground of loss of trust and confidence.- Vargas was informed of the result of the investigation. Kwikway offered her a chance to resign with separation pay, which she accepted.- The Labor Arbiter rendered a decision directing the reinstatement of respondent Vargas to her former position with backwages. NLRC affirmed the decision of the labor arbiter.- Petitioner: the nature of the position of Vargas involves trust and confidence. That private respondent's acts of dishonesty as well as her active participation in violating and infringing company accounting procedure which allowed the cashier to personally misappropriate sums of money provide sufficient basis for dismissing respondent. That Vargas was aware that her cashier Corpus was committing acts of dishonesty and misappropriation of company funds but she did not report the matter to her superiors in the company. That the actuations of Vargas were in violation of the company's code of conduct, which is punishable by dismissal.

ISSUES1. WON the dismissal of respondent Vargas was for a just and valid cause2. WON respondent Vargas was deprived of her constitutional right to due process

HELD1. YES- The rule is settled that if there is sufficient evidence to show that the employee has been guilty of breach of trust or that his employer has ample reason to distrust him, the labor tribunal cannot justly deny to the employer the authority to dismiss such employee.- Jurisprudence abounds with cases recognizing the right of the employer to dismiss the employee on loss of confidence. More so in the case of supervisors or personnel occupying positions of responsibility, loss of trust justifies termination - The mere existence of basis for believing that the employee has breached the trust of employer is sufficient and does not require proof beyond reasonable doubt.- Clearly, respondent Vargas's position involves a high degree of responsibility requiring trust and confidence. Her position carries with it the duty to observe proper company procedures in the fulfillment of her job as it relates closely to the financial interests of the company.2. YESNotice and Hearing- The twin requirements of notice and hearing constitute essential elements of due process in cases of employee dismissal: the requirement of notice is intended to inform the employee concerned of the employer's intent to dismiss and the reason for the proposed dismissal; upon the other hand, the requirement of hearing affords the employee an opportunity to answer his employer's charges against him accordingly to defend himself therefrom before dismissal is effected. Neither of these two requirements can be dispensed with without running afoul of the due process requirement of the 1987 Constitution.- In the instant case, the records are bereft of any indication that a formal notice of the charge was given to the respondent prior to the suspension or that the said investigation gave adequate opportunity to the respondent to defend herself. It is important to stress that an employee whose services are sought to be terminated, has the right to be informed beforehand of his proposed dismissal or suspension as well as of the reasons therefor and to be afforded an adequate opportunity to defend himself from the charges leveled against him. We give respect to the following conclusions of the labor arbiter and respondent Commission:

“It is patent from the respondent's submission that written notice specifying the causes for termination was never furnished to complainant. Neither does it appear that she

was given enough opportunity to explain her side and defend herself with the assistance of a representative of her choice is she so desires. “

Preventive Suspension- Further, the preventive suspension of respondent Vargas for an indefinite period amounted to a dismissal and is violative of Section 4, Rule XIV of the Implementing Rules of the Labor Code which limits the preventive suspension to thirty (30) days. The said rule also provides that "the employer shall thereafter reinstate the worker in his former or in a substantially equivalent position or the employer may extend the period of suspension provided that during the period of extension, he pays the wages and other benefits due to the worker." (Pacific Cement Company Inc. v. NLRCDisposition the petition GRANTED. The questioned decision of the respondent NLRC insofar as it ordered the reinstatement of respondent Rosalinda Vargas with payment of three (3) years backwages is REVERSED and SET ASIDE. - Petitioner company is ordered to pay an indemnity of P1,000.00 to respondent Vargas (For failure of the employer to comply with the requirements of due process in terminating the employees service, it shall be liable to indemnify the employee in the sum of P1,000.00 as damages)

GATBONTON V NLRC (MIT, CALDERON)479 SCRA 416

AUSTRIA-MARTINEZ; January 23, 2006

NATUREPetition for review on certiorari

FACTS- Gatbonton is a professor at the Mapua Institute of Technology, a member of the Faculty of Civil Engineering. In November 1998, a student filed a complaint of sexual harassment against Gatbonton. He was then placed under preventive suspension for 30 days pending investigation. - Gatbonton filed a complaint against MIT for illegal suspension. - Petitioner questioned the validity of the administrative proceedings with the Manila RTC but the case was terminated on May 21, 1999 when the parties entered into a compromise agreement wherein MIT agreed to publish in the school organ the rules and regulations implementing the Anti-Sexual Harassment Act. The Labor Arbiter later declared that the preventive suspension imposed was illegal. - Both respondents and petitioner filed their appeal from the Labor Arbiter’s Decision, with Gatbonton questioning the dismissal of his claim for damages. The NLRC favorable granted the appeal of respondent MIT. Gatbonton.

ISSUEWON the NLRC erred in dismissing Gatbonton’s claim for damages stemming from an alleged illegal suspension

HELDYESRatio Preventive suspension is a disciplinary measure for the protection of the company’s property pending investigation of any alleged malfeasance or misfeasance committed by the employee. The employer may place the worker concerned under preventive suspension if his continued employment poses a serious and imminent threat to the life or property of the employer or of his co-workers. However, when it is determined that there is no sufficient basis to justify an employee’s preventive suspension, the latter is entitled to the payment of salaries during the time of preventive suspension.Reasoning- R.A. No. 7877 imposed the duty on educational or training institutions to “promulgate rules and regulations in consultation with and jointly approved by the employees or students or trainees, through their duly designated representatives, prescribing the procedures for the investigation of sexual harassment cases and the administrative sanctions therefor.” - Petitioner’s preventive suspension was based on respondent MIT’s Rules and Regulations for the Implemention of the Anti-Sexual Harassment Act which provides that “any member of the educational community may be placed immediately under preventive suspension during the pendency of the hearing of the charges of grave sexual harassment against him if the evidence of his guilt is strong and the school head is morally convinced that the continued stay of the accused during the period of investigation constitutes a distraction to the normal operations of the institution or poses a risk or danger to the life or property of the other members of the educational community.”- But the said rules and regulations were published only on February 23, 1999. - The Mapua Rules is one of those issuances that should be published for its effectivity, since its purpose is to enforce and implement R.A. No. 7877, which is a law of general application. Thus, at the time of the imposition of petitioner’s preventive suspension on January 11, 1999, the Mapua Rules were not yet legally effective, and therefore the suspension had no legal basis.- The Court also finds that there is insufficient legal basis to justify the preventive suspension because of the absence of the required requisites. Under the Mapua Rules, an accused may be placed under preventive suspension during pendency of the hearing under any of the following circumstances:

Labor Law 1 A2010 - 251 - Disini(a) if the evidence of his guilt is strong and the school head is morally convinced that the continued stay of the accused during the period of investigation constitutes a distraction to the normal operations of the institution; or (b) the accused poses a risk or danger to the life or property of the other members of the educational community.Disposition petition is PARTIALLY GRANTED. The decision of Labor Arbiter is reinstated while the decisions of the CA and the NLRC are set aside.

NUMBER OFFENSES

APARENTE V NLRC[PAGE 240]